fwk bauer tif organizational behavior all chapters

516
Organizational Behavior, Version 1.1 Bauer & Erdogan FWK Test Item File Chapter 1 TRUE/FALSE 1. Flat World Knowledge was started by two textbook publishing industry veterans. (True) 2. Flat World Knowledge is improving on the traditional publishing model but is not innovating much. (False) 3. So far, Flat World Knowledge is a start up without any external funding. (False) 4. Most successful CEOs believe, “A company is only as good as the innovation it creates.” (False) 5. Current research suggests that the average employee today will change jobs 10 times in 20 years. (True) 6. Organizational behavior is the systematic study and application of knowledge about how individuals and groups act within the organizations where they work. (True) 7. Emile is seated in a lecture where the instructor is discussing personality. He is most likely in a sociology class. (False) 8. The study of organizational behavior analyzes the three levels of individual, organization, and society. (False)

Upload: ricky-dhaliwal

Post on 04-Jan-2016

335 views

Category:

Documents


14 download

DESCRIPTION

org behavior

TRANSCRIPT

Page 1: Fwk Bauer Tif Organizational Behavior All Chapters

Organizational Behavior, Version 1.1Bauer & Erdogan

FWK Test Item FileChapter 1

TRUE/FALSE

1. Flat World Knowledge was started by two textbook publishing industry veterans.(True)

2. Flat World Knowledge is improving on the traditional publishing model but is not innovating much.

(False)

3. So far, Flat World Knowledge is a start up without any external funding.(False)

4. Most successful CEOs believe, “A company is only as good as the innovation it creates.”

(False)

5. Current research suggests that the average employee today will change jobs 10 times in 20 years.

(True)

6. Organizational behavior is the systematic study and application of knowledge about how individuals and groups act within the organizations where they work.

(True)

7. Emile is seated in a lecture where the instructor is discussing personality. He is most likely in a sociology class.

(False)

8. The study of organizational behavior analyzes the three levels of individual, organization, and society.

(False)

9. If I am examining how my manager’s behavior influences my work group, I am looking at the organizational level of analysis in the organizational behavior discipline.

(False)

10. Organizational behavior matters at three critical levels because it analyzes what you care about, what employers care about, and what organizations care about.

(True)

11. Successful organizations tend to limit the amount of information shared by maintaining a centralized structure.

Page 2: Fwk Bauer Tif Organizational Behavior All Chapters

(False)

12. Research shows that those organizations that are more effective limit the amount of feedback they provide employees, limit the amount of information shared, and allow roles to be ambiguous.

(False)

13. Those firms that are categorized as successful tend to provide employment security, train employees, and reduce status differences.

(True)

14. Financially there is little incentive to attain a higher level degree.(False)

15. The unemployment rate decreases as the level of education attained increases.(True)

16. In general, the more training you have, the more financial success you will achieve.

(True)

17. Journaling is a technique that helps you chart your progress as you learn new skills.

(True)

18. The discipline of organizational behavior is simply common sense.(False)

19. Michael is an auditory learner. He learns best by actually doing things and learning from trial and error.

(False)

20. A kinesthetic learner is very likely to avoid taking once a week classes as they require too much sitting and listening time.

(True)

21. Close-ended survey questions can be very time consuming to summarize and very hard to interpret.

(False)

22. Glen is interested in finding out how effective a new incentive program might be in an organization. He chooses two departments to focus upon in a company. One department works under the incentive program, one does not. This is an experimental design study.

(True)

23. Case studies suffer from questions as to whether the results of one organization are generalizable to other situations and organizations.

(True)

Page 3: Fwk Bauer Tif Organizational Behavior All Chapters

24. Meta-analysis shows that the relationship between job satisfaction and job performance is moderately strong.

(True)

25. Reliability refers to consistency of measurement, while validity assesses whether the measurement does, indeed, measure what it is supposed to measure.

(True)

26. Most of management research addresses causation.(False)

27. The federal government passed the Sarbanes-Oxley Act as a long-term solution to dealing with unethical behavior.

(False)

28. Employee engagement has a significant impact on the corporate bottom line.(True)

29. Research by Gallup indicates that only 20% of employees are disengaged from their organization.

(False)

30. Moore’s Law suggests that computing power doubles every two years.(True)

31. Because of information overload, some organizations, like Intel, have introduced “No e-mail Fridays” as a way to address the issue.

(True)

32. Access to information has led to an increase in innovation and a “flattening of the world.”

(True)

33. The triple bottom line refers to evaluating organizations against the three performance criteria of social viability, economic viability, and demographic viability.

(False)

34. One of the major challenges facing organizations today is how to reconcile the accountability publicly owned firms have in generating wealth for shareholders while attending to the triple bottom line.

(True)

35. In general terms, the American workforce is growing younger.(False)

36. Outsourcing is becoming an increasingly common practice in many organizations.

(True)

Page 4: Fwk Bauer Tif Organizational Behavior All Chapters

37. A “shamrock organization” is comprised of one-third regular employees, one-third temporary employees, and one-third consultants and contractors.

(True)

38. Offshoring and the development of a shamrock organization structure challenges organizational behavior to manage teams made up of employees of different nationalities separated by culture, language, time and space.

(True)

39. The Millennial Generation has technology embedded in their lives so organizational behavior has the challenge of managing people of different generations who have different values with regard to teamwork, rewards, and work-life balance.

(True)

40. Organizational behavior provides organizations the tools to address the technological, societal, and cultural issues arising today to create an environment that is mutually beneficial to the firm and its employees.

(True)

MULTIPLE CHOICE

Opening Section: College Textbook Revolution: The case of Flat World Knowledge

41. The CEO of Flat World Knowledge believes:a. the textbook industry is fine as it is.b. textbook costs are not a major concern.c. the traditional textbook industry model no longer makes sense.d. that charging a high price for textbooks will lead to company growth.

(c) Medium/Analysis

42. The success of Flat World Knowledge is surprising because the textbook industry is characterized by a. Low profit margins.b. Expensive textbooks.c. Harmonious customer interactions.d. Interesting and challenging textbooks.

(b) Easy/Comprehension

Page 5: Fwk Bauer Tif Organizational Behavior All Chapters

Section I: Understanding Organizational Behavior

43. According to the authors of Organizational Behavior, a company’s greatest asset isa. its product.b. its financial resources.c. its people.d. its location.

(c) Easy/Knowledge

44. Maurice is enrolled in a career development class at his university. Which of the following statements best reflects important themes in the course?a. All students have common experiences and capabilities so an

organization’s career development activities can be standardized.b. Learn as much as you can in as many areas as you can in college

because you will not learn much more throughout your worklife.c. Once individuals choose their first job and the organization in which they

work, they make very few changes in either throughout the rest of their worklives; so choose carefully.

d. Uniqueness in a career path is the norm, so the best prepared students are able to diagnose situations, ask tough questions, evaluate answers and act in effective and ethical manners.

(d) Difficult/Synthesis

45. Which of the following is a key level of analysis investigated in Organizational Behaviora. industry.b. individual.c. meta.d. society.

(b) Easy/Knowledge

46. The systematic study and application of knowledge about how individuals and groups act within the organizations where they work isa. clinical psychology.b. organizational behavior.c. sociology.d. economics.

(b) Easy/Knowledge

47. Sociology is a. the systematic study of group and team processes.b. the systematic study of the production, distribution and consumption of

goods and services.c. the systematic study of the processes by which groups of people make

decisions.d. the systematic study of the processes in the organization and workplace

that improve the performance and well being of people.(a) Difficult/Synthesis

Page 6: Fwk Bauer Tif Organizational Behavior All Chapters

48. Manuel is studying both psychology and sociology this term. He is having a difficult time dealing with the overlap of some of the topics in the course. To help him focused on the relevant aspects of each course, he should keep in mind thata. sociology is primarily focused on the individual; psychology on society.b. psychology primarily looks at how individuals are influenced in groups;

sociology looks at aspects of the individual.c. sociology focuses on mental processes and behavior of the individual in

society; psychology focuses on how individuals are influenced by groups.d. psychology primarily focuses on mental process and behavior in the

individual; sociology on how individuals are influenced in groups.(d) Difficult/Evaluation

49. Psychology isa. the scientific study of mental processes and behaviors of the individual.b. the systematic study of individual behavior in society.c. the systematic study of the processes by which groups of people make

decisions.d. the systematic study of how to improve the performance and well being of

individuals in the workplace.(a) Difficult/Synthesis

50. Organizational behavior draws heavily on personality and motivation studies from what other discipline?a. psychologyb. physiologyc. economicsd. political science

(a) Easy/Comprehension

51. Studies on decision making have been carried out in all of these disciplines EXCEPT a. anthropologyb. political sciencec. economicsd. psychology

(a) Easy/Comprehension

52. Work on team processes has been carried out in what area?a. sociologyb. financec. economicsd. physics

(a) Easy/Comprehension

53. During the first day of an organizational behavior class at your university, the instructor tells the students that during the term they will learn all of the following EXCEPTa. how to work effectively on team projects.b. how to make more effective decisions about their academic and social

lives.

Page 7: Fwk Bauer Tif Organizational Behavior All Chapters

c. how different organizations interact with each other.d. how to cope with the stress of finals week.

(c) Easy/Comprehension

54. A recent National Association of Colleges and Employers survey indicated all of the following are important skills for evaluating job candidates EXCEPTa. communication skills.b. demographics.c. honesty/integrity.d. strong work ethic.

(b) Easy/Knowledge

55. Which of the following is accurate according to organizational behavior research?a. successful companies are more centralized.b. successful companies emphasize the status differences between their

employees.c. successful companies have very simple and informal hiring systems.d. successful companies treat their employees well.

(d) Easy/Knowledge

56. Which of the following is likely to produce an effective organization?a. A firm that controls its labor costs by limiting training opportunities to only

a few employees.b. A firm that maintains two separate cafeterias for employees; one for

executives and managers and one for everyone else.c. A firm where information is shared on a “need-only” basis.d. A firm that employs selective hiring practices.

(d) Difficult/Evaluation

57. Research indicates that all of the following characteristics produce healthy organizations EXCEPTa. role clarity.b. participative decision making.c. information sharing.d. limited feedback.

(d) Medium/Analysis

58. Rosario is examining a summary chart on four firms’ organizational characteristics. She feels which of the organizations below is most likely to have the greatest chance to be successful in the next year?a. Organization A uses self-managed teams, pays its workers a little above

the average for the industry and has regular “state of the business meetings.”

b. Organization B has a pay policy that matches the industry average for each job category, maintains special parking, cafeteria, and restroom facilities for its managers, and conducts performance appraisals every two years.

c. Organization C has gone through regular quarterly lay-offs the last few quarters, established a policy last month where all decisions involving the expenditure of fifty dollars or more must be approved by two managers and has temporarily suspended all performance evaluations for the year.

Page 8: Fwk Bauer Tif Organizational Behavior All Chapters

d. Organization D maintains a training program that is very selective with regard to employee involvement. They recently instituted a large-scale job flexibility program that “blurs” the lines between job duties and passed a strict pay secrecy policy last month.

(a) Difficult/Synthesis

59. Research indicates that effective organizations tend to a. centralize their entire decision making.b. have a very large differential in pay between the lowest paid employee

and the CEO.c. hire the first person available for any job opening.d. provide as much employment security as possible to their employees.

(d) Medium/Application

Unemployment and Earnings for Workers 25 and older by Educational Attainment, 2006Unemployment Rate (Percent)

Educational Degree Median Weekly Earnings (U.S. Dollars)

1.4 Doctoral Degree 1,4411.7 Master’s Degree 1,1402.3 Bachelor’s Degree 9623.0 Associate Degree 7213.9 Some College 6744.3 High School Graduate 5956.8 Some High School 419

60. The information on the preceding table indicatesa. the higher the educational level, the higher the pay.b. there is a negative return to investment in education.c. the level of educational degree does not matter, it is whether you have a

degree or not.d. unemployment rates increases as you increase educational attainment

level.(a) Difficult/Evaluation

Unemployment and Earnings for Workers 25 and older by Educational Attainment, 2006Unemployment Rate (Percent)

Educational Degree Median Weekly Earnings (U.S. Dollars)

1.4 Doctoral Degree 1,4411.7 Master’s Degree 1,1402.3 Bachelor’s Degree 9623.0 Associate Degree 7213.9 Some College 6744.3 High School Graduate 5956.8 Some High School 419

Page 9: Fwk Bauer Tif Organizational Behavior All Chapters

61. Which is likely, given the data in the preceding table?a. The more training an individual has, the more financial success he

enjoys.b. There is a negative financial return to higher-level degree attainment.c. Unemployment levels increase as educational level increases.d. Financially, there is little incentive to achieve a higher-level degree.

(a) Difficult/Synthesis

62. Tom Peters, management expert, suggests individuals manage themselves as if they were “the brand that is you.” Such brand management might include all of the following suggestions EXCEPTa. appreciate new technology.b. avoid uncertainty.c. master something.d. network.

(b) Medium/Comprehension

63. To be effective in keeping up your skill set in the future, make sure toa. purchase and use every new technological gadget that comes out on the

market.b. make as many connections as you can on Facebook, MySpace or

LinkedIn.c. avoid change as much as you can.d. excel at something so it sets you apart.

(d) Difficult/Synthesis

64. Journaling is an effective developmental technique. Which of the following is true about the process of journaling?a. Writing your thoughts down increases your stress level as you re-live the

events you write about.b. Thinking about your day is as effective as writing it down.c. You must write down your thoughts a minimum of 30 minutes a day to

derive any benefits from the process.d. Tracking changes in yourself can be motivating.

(d) Medium/Analysis

65. Which of the following is accurate according to OB research?a. We all have excellent perception and readily attend to those things we

should and ignore those we should not.b. We all understand and use the logical approach to decision making.c. People perform better if goals are difficult rather than easy.d. Brainstorming in a group is more effective than brainstorming alone.

(c) Difficult/Synthesis

Section II: Understanding Your Learning Style

66. A kinesthetic learnera. has a preference for doing things and learning by trial and error.b. retains information by reading and seeing diagrams and flow charts.c. learns primarily by listening to others.

Page 10: Fwk Bauer Tif Organizational Behavior All Chapters

d. learns best by attending videotaped lectures.(a) Easy/Knowledge

67. A visual learnera. learns by retaining information from reading and seeing diagrams.b. has a preference for doing things and learning by trial and error.c. learns best based upon the circumstances of the situation.d. learns primarily by listening to others.

(a) Easy/Knowledge

68. An auditory learnera. learns primarily by taking tests.b. learns primarily by listening to others.c. learns by reading and seeing diagrams and graphs.d. learns best by trial and error.

(b) Easy/Knowledge

69. To facilitate his learning in a college course, a visual learner would likelya. join a study group.b. tape his professor’s lectures.c. take notes during class.d. avoid scheduling classes that meet for long time periods.

(c) Medium/Comprehension

70. A student who needs to take breaks during his study sessions and who needs to move around after reading a chapter in his textbook is likely a(n) ___________ learner.a. visualb. auditoryc. kinestheticd. emotional

(c) Medium/Comprehension

71. Auditory learnersa.             draw pictures to help them understand material.b.             take notes during lectures to force them to pay attention.c.             use charts to summarize the main points of class material.d.             record lectures to refer back to them later.

(d)                       Medium/Comprehension

Section III: Understanding How OB Research is Done

72. Hypotheses area. entities that take on different values.b. research tools that elicit respondents’ reactions to specific questions.c. the process of writing out thoughts and emotions on a regular basis.d. tentative guesses for expected observations that can be tested.

(d) Easy/Knowledge

73. An entity that takes on different values is

Page 11: Fwk Bauer Tif Organizational Behavior All Chapters

a. a hypothesis.b. a variable.c. a survey.d. a field study.

(b) Easy/Knowledge

74. Research conducted in an actual organization is a. a field study.b. a survey.c. a lab study.d. a meta-analytic study.

(a) Easy/Knowledge

75. Which of the following statements about surveys is INCORRECT?a. Either open-ended or close-ended questions can be used in surveys.b. Close-ended question responses are difficult to summarize.c. Surveys involve asking individuals to respond to questions.d. Many effective online survey tools are available free of charge.

(b) Medium/Comprehension

76. Research that is performed under very controlled conditions and can include observation, interviews, or experiments is calleda. a case study.b. a survey.c. a field study.d. a lab study.

(d) Easy/Comprehension

77. A control groupa. receives experimental manipulation.b. is an entity that can take on different values.c. receives no experimental manipulation.d. is a tentative guess on an observation that can be tested.

(c) Easy/Comprehension

78. Which of the following statements about case studies is true?a. They involve a study conducted under controlled conditions.b. They offer the researcher a very simple and effective way to generalize

what worked in a single situation to many other situations.c. They are a technique employed by researchers to summarize what other

researchers found on a topic.d. They are a way to gather data and explain an event or situation in detail.

(d) Medium/Analysis

79. Marcus wants to thoroughly understand the issues that created the severe economic straits General Motors currently finds itself experiencing. He should consider what kind of a research study?a. A case study.b. A survey.c. A field study.d. A laboratory study.

Page 12: Fwk Bauer Tif Organizational Behavior All Chapters

(a) Medium/Application

Before instituting a new health benefit program for its employees, one Big Ten University offered all its employees in the Liberal Arts and Engineering Schools an opportunity to participate in a Wellness Program that focused on walking to enhance employee health and cut down on those employees using health benefits. Those liberal arts employees agreeing to participate would receive a $25 gift card to Best Buy. Those in the engineering department were not offered the gift card for signing up.

80. The scenario described above represents which type of research study?a. A case study.b. A survey.c. An experimental field study.d. A laboratory study.

(c) Medium/Application

81. In the scenario described above, the liberal arts school employees area. the treatment group.b. the control group.c. the survey group.d. the case group.

(a) Medium/Application

82. The engineering school employees, described in the scenario above, could be labeled thea. control group.b. treatment group.c. case group.d. survey group.

(a) Medium/Application

83. Meta-analysis isa. a study with a group that receives a treatment and a comparison group

that receives no treatment.b. a study conducted in an actual organization.c. the process of summarizing research findings from studies on related

topics.d. an in-depth analysis of a single industry or company.

(c) Easy/Knowledge

84. Validity is a. an entity that can take on different values.b. the consistency in measurement.c. the strength of the relationship between two variables.d. the truth of a measurement.

(d) Easy/Knowledge

Page 13: Fwk Bauer Tif Organizational Behavior All Chapters

85. Natalia administered the test to the job candidate three different times. She is looking at the scores and sees that they are 85, 87 and 85 out of 100. She notes that the scores are fairly consistent or a. valid.b. reliable.c. strong.d. relevant.

(b) Medium/Application

86. In analyzing the data, the researcher notes that as employee absences increase, scores on performance evaluations decrease. This suggestsa. there is a correlation between employee absenteeism and employee

performance.b. employee absence causes poor performance.c. employee absence is a reliable measure.d. employee performance is a valid measure.

(a) Difficult/Application

87. Datum is a. multiple observations.b. the consistency of a measure.c. the truth of a measure.d. a single observation.

(d) Easy/Knowledge

88. The likelihood that findings in a given study would be found in another setting or study isa. reliability.b. validity.c. meta-analysis.d. generalizability.

(d) Easy/Knowledge

Section IV: Trends and Challenges

89. Research indicates that the most important determinant of ethical behavior in an organization isa. the establishment of policies prohibiting unethical behavior.b. the passage of laws requiring ethical behavior.c. manager commitment to ethical behavior.d. the establishment of rules requiring greater accountability.

(c) Medium/Analysis

Page 14: Fwk Bauer Tif Organizational Behavior All Chapters

90. An ethics audit would ask all the following questions of a firm EXCEPT:a. Do you make sure to hide any examples of unethical behavior so others

do not repeat the behavior?b. Do you ensure that pay is equitable for all employees?c. Do you question whether employees are seeing any examples of

unethical behavior occurring in the workplace?d. Do you ensure that daily decisions are assessed for ethical compliance?

(a) Medium/Application

91. Trends in all of the following areas represent challenges for organizational behavior EXCEPTa. increasing technological innovation.b. increasing employee engagement.c. increasing access to information.d. increasing attention to sustainable business practices.

(b) Medium/Application

92. The triple bottom line refers to evaluating an organization on all of the following performance criteria buta. demographic.b. economic.c. social.d. environmental.

(a) Easy/Knowledge

93. Which of the following statements is INCORRECT with regard to trends that organizational behavior is addressing?a. The workforce is getting younger and thus their integration into most

organizations will be seamless.b. Organizations will likely become “shamrock-shaped” in the future.c. The world is “flattening” so information access is increasing.d. Outsourcing is becoming a way of life in many organizations.

(a) Medium/Application

94. A shamrock organizationa. prevents flexible staffing in an organization.b. is comprised of one-third regular employees, one-third temporary

employees, and one-third offshore employees.c. is the current structure of most organizations in the United States.d. increases full-time employment rates in the overall economy.

(b) Difficult/Analysis

95. Offshoring isa. asking an outside organization to perform functions that could have been

performed within the organization.b. the movement of a business practice from one country to another country.c. purchasing a firm from a foreign country.d. most prevalent in the auto industry.

(b) Easy/Comprehension

Page 15: Fwk Bauer Tif Organizational Behavior All Chapters

96. In analyzing trends that present challenges for organizational behavior, which of the following is INCORRECT?a. Outsourcing offshore presents a challenge managing teams of different

nationalities separated by culture.b. A flattening world creates an uneven playing field with regard to

information access.c. Sustainable business practices are those that meet present needs without

compromising the needs of future generations.d. “Moore’s Law” suggests that computing power doubles every two years.

(b) Difficult/Synthesis

97. Which of the following statements regarding trends that represent challenges for organizational behavior is correct?a. The number of workers aged 25 to 39 will represent over 51% of the

overall workforce by 2010.b. Gallup measured employee engagement and found that it is highly

unlikely that when today’s employees have choices, they will act to further their organization’s efforts.

c. The rate of technological innovation is projected to slow dramatically by 2010.

d. Sustainable business practices present a challenge to reconciling accountability to shareholder wealth generation with attention to social responsibility.

(d) Difficult/Application

Closing Section: Maintaining core values: The case of Nau

98. The apparel industry uses ______________:a. very little water.b. only recycled water.c. more water than any industry except agriculture.d. the same amount of water as the agriculture industry.

(c) Medium/Comprehension

99. Nau was founded on the idea of ___________.a. maintaining the status quo.b. generating record profits in the apparel industry.c. using business as a vehicle for change.d. using plastics to make clothing.

(c) Medium/Comprehension

FILL IN THE BLANK

100. Flat World Knowledge is a company that creates ___________.(textbooks)

101. Flat World Knowledge proves that firms can be successful by offering high quality content for _______ online.

(free)

Page 16: Fwk Bauer Tif Organizational Behavior All Chapters

102. Because it is likely that U.S. employees will change jobs 10 times in 20 years, those employees should be ___________ learners and continually upgrade their skill set.

(life-long)

103. The systematic study and application of knowledge about how individuals and groups act within the organizations where they work is called ________ _____________.

(organizational behavior)

104. The three key levels of analysis in organizational behavior are ________, _______________ and _________________.

(individuals, groups, organizations)

105. Maurice is trying to understand why Mary agrees with him about the cost of the manufacturing program when he talks with her on the phone, but is vehemently opposed when they are in a department meeting. Maurice needs to analyze this issue at the ____________ level.

(group)

106. Nancy is the new employee and Maureen has decided to talk to her about the dress policy in the firm, the manner in which expense reports are filled out, the employees that tend to sit together at lunch, and various other related issues. Maureen is offering a look at the company’s culture that is at the ______________ level of analysis.

(organization)

107. Effective organizations feature role __________, information sharing and performance feedback.

(clarity)

108. In general, as level of degree attainment increases, the unemployment rate __________.

(decreases)

109. In general, the higher the ________, the more money you will make.(degree you attain )

110. _____________ is the process of writing out thoughts and emotions on a regular basis.

(Journaling)

111. If you learn best by lectures, conversations and videos, you are a(n) __________ learner.

(auditory)

112. A(n) ________ learner draws pictures and diagrams and summarizes main points by using charts to help him understand material.

(visual)

113. ________ are tentative guesses on an expected observation that can be tested.

Page 17: Fwk Bauer Tif Organizational Behavior All Chapters

(Hypotheses)

114. In an experimental design, the group that receives no experimental manipulation is called the __________ group.

(control)

115. The technique used by researchers to summarize what other researchers have found on a given topic is called __________.

(meta-analysis)

116. __________ is to truth in measurement, as ___________ is to consistency in measurement.

(Validity, reliability)

117. ______________ is the strength of a relationship between two variables.(Correlation)

118. A single observation is ________, multiple observations are ________.(datum, data)

119. Maria wonders what the emotional impact of all the downsizing in the auto industry is on the employees. She has made arrangements to interview a dozen employees who have just been downsized from Ford Motor Company. Maria is conducting a ______ _______.

(field study)

120. The downside of a ______ __________ is that it may not be generalizable from the single situation to other situations and organizations.

(case study)

121. As job satisfaction increases, so does job performance. This is an example of the ________ between two variables.

(correlation)

122. Applying ethical principles to situations that arise at work is _________ ___________.

(business ethics)

123. Alexander is enthused about his firm and consistently uses his talents to further the organization’s success. Alexander is an example of a(n) _____________ employee.

(engaged)

124. ___________________ says computing power doubles every two years.(Moore’s law)

125. The triple bottom line refers to evaluating organizations against the three performance criteria of __________, ___________, and ___________ viability.

(economic, social and environmental)

Page 18: Fwk Bauer Tif Organizational Behavior All Chapters

126. Because Wal-Mart is concerned about _________ ________ practices, it is pressuring suppliers to produce compact fluorescent light bulbs and has required all suppliers to make packages smaller.

(sustainable business)

127. The American workforce is _______, which will create opportunities for industries like health care but, at the same time, will create challenges for industries dedicated to basic infrastructure.

(aging)

128. Dell Computer has a number of customer service representatives located in India who take calls from U.S. computer owners experiencing computer problems. This arrangement is an example of ______________.

(offshoring)

129. The ______ ________ is an organization comprised of one-third regular employees, one-third temporary employees and one-third consultants or contractors.

(shamrock organization)

SHORT ANSWERS

130. Briefly indicate the manner in which organizational behavior can help you deal with the fact that, on average, you are likely to change jobs 10 times in 20 years.

The number of job changes the average student will go through makes it necessary for all employees to be life-long learners. Organizational behavior will help develop critical thinking skills that will help you diagnose situations, ask tough questions, evaluate the resulting answers and act in an effective and ethical manner regardless of situational characteristics.

131. Identify each of the levels of analysis in organizational behavior and give an example of each.

The first level is that of the individual. When I am examining my personality to assess how I handle ambiguity and how flexible I am, I am using individual analysis.

The second level is the group level. When I look at how my decision on a topic changes because of who I am working with on a project, I am performing group analysis.

Finally, the third level is the organization level. When I look at the dress policy of my company and how it impacts what I wear to work on a daily basis, I am performing organization analysis.

132. Kunal will be serving as a student teacher in a local school district next term. You are meeting with him this afternoon and want to make certain he recognizes that there are three general learning styles that students exhibit. Name and describe those three types of learners.

Page 19: Fwk Bauer Tif Organizational Behavior All Chapters

A visual learner is one who retains more information by reading and seeing diagrams and flow charts.

An auditory learner is one who learns by listening to others such as in lectures, conversations and videos.

A kinesthetic learner is one who has a preference for doing things and learning from trial and error.

133. Discuss the kinds of questions researchers utilize on surveys.

Researchers use both open- and close-ended questions. Close-ended questions are those that have a series of responses supplied by the researcher. The study participant chooses the response that most closely matches his position.

Open-ended questions have no responses offered, leaving it up to the survey participant to supply responses.

134. Differentiate between a control group and a treatment group.

In a study employing an experimental design, the control group is the group that does not receive any experimental manipulation, while a treatment group is the group that does receive the experimental manipulation. A comparison between the two groups indicates the impact the treatment had in the study.

135. What is a meta-analysis?

A meta-analysis is a technique used by researchers to summarize what other researchers have found on a given topic. The analysis is based on taking observed correlations from multiple studies, weighing them by the number of observations in each study, and finding out if the overall effect holds or not.

136. Differentiate between reliability and validity.

Reliability is the consistency in a measurement.

Validity is the truth of the measurement or whether you are measuring what you intended to measure.

137. Differentiate between a causal and a correlational relationship.

A causal relationship is one in which one variable, event or situation causes a second variable, event or situation to occur. For example, individuals in asbestos-insulated buildings often contract cancer, and asbestos has been shown to be a carcinogen. Thus, asbestos causes cancer.

A correlational relationship is one in which the strength of the relationship between two variables is measured. For example, research indicates that as job satisfaction increases, so does job performance. Job satisfaction does not cause job performance, it simply increases it.

Page 20: Fwk Bauer Tif Organizational Behavior All Chapters

138. Define the concept of triple bottom line.

A triple bottom line is evaluating organizations against three performance criteria including economic, social and environmental viability. Wal-Mart, for example, looked at the impact packaging for its products had on its costs, on the environment and on its customers. Its findings led to the chain’s requirements for all suppliers to make smaller packages.

139. What is a “shamrock organization”?

A shamrock organization is a term coined by Charles Handy in his book, The Age of Paradox. The organization is comprised of one-third regular employees, one-third temporary employees and one-third consultants and contractors. Handy feels that this structure is the direction in which organizations are heading in the future.

ESSAY

140. Define organizational behavior and discuss some of the other disciplines from which it draws to create its theories and models.

Organizational behavior is the systematic study and application of knowledge about how individuals and groups act within the organizations where they work.

OB is rooted in such disciplines as political science (power and influence), medical science (stress and its impact), sociology (team processes), and psychology (personality and motivation).

141. Renaldo is choosing his classes for next term at State University. His advisor suggests he take Organizational Behavior. Renaldo says, “Why would I take that class? It’s all common sense anyway.” Take the role of his advisor and sell Renaldo on the value of the course.

Organizational behavior matters because of all the things individuals, employers and organizations care about. You care about getting a good job, making better decisions and keeping your stress level as low as possible. Each of these topics is discussed in the course.

Employers care about OB because it prepares employees to be better communicators, enhances their interpersonal skills and enhances a strong work ethic.

Organizations care about OB because time and time again it has been found that people make the organization. To learn how to help yourself, your group and your organization, take OB.

Page 21: Fwk Bauer Tif Organizational Behavior All Chapters

Now you may say that OB is all common sense, but research continually indicates to us that individuals have faulty memories and that their perceptions do not always match reality. The study of OB addresses all these concerns and assists us in being better employees who are more satisfied and productive. If a course assists you in all these ways, doesn’t it seem like it’s worth taking?

142. Counsel a newly hired, 22-year-old man on keeping up with his skill set.

The world is a much more dynamic place today than it has ever been before. In fact, the one constant in life is change. With change come new demands, so the best way to meet that challenge is to become a life-long learner.

Life-long learners keep their skill set fresh. It is suggested that every 6 years your portfolio of skills should be revolutionized.

With so many accomplished individuals with whom you are competing, it is imperative that you master some skill that will set you apart from the others.

Recognize that with change comes uncertainty and ambiguity. Don’t run from either. Approach the novel situation as an opportunity.

Network, network, network. But, recognize that effective networkers are not those who simply have the most “friends” on Facebook or MySpace. Quality trumps quantity any day.

Finally, appreciate the new technology. You don’t have to buy every new gadget that comes out, but do keep up with what is new and how it might impact you.

143. You are an instructor in an organizational behavior course. Discuss how you would prepare a lesson on motivation that would be equally effective for all students in the class whether they are auditory, kinesthetic or visual learners.

A motivation lesson might begin with a PowerPoint presentation where the concept of motivation is defined and the types of motivation theories are presented. The PowerPoint presentation might be handed out to the students prior to the presentation so that students can take notes on those handouts. The use of PowerPoint slides, which feature diagrams and notes on motivation concepts and theories, satisfies the visual learner and to some extent, the kinesthetic learners. Since the instructor is talking about each slide and the information on it, and perhaps giving examples of events from actual firms that underscore that information, an auditory learner will also be satisfied.

Following the presentation of basic background information on the subject through the use of the PowerPoint slides, an experiential exercise could be completed wherein participants would role-play scenarios between a boss and his subordinates whom he is working to motivate. Again, the kinesthetic learner will be learning because of the opportunity to participate in the exercise and actually carry out the motivational approaches.

Page 22: Fwk Bauer Tif Organizational Behavior All Chapters

144. Describe a field study that Meredith could carry out in a 20-person customer service department of a firm to assess the impact a monetary bonus has on the speed with which a task is completed.

The researcher could conduct a field study using an experimental design to address this scenario.

First divide the department into two groups. One group will be the control group and will not be subjected to any experimental manipulation and will continue to perform its job exactly as it has in the past. The second group will be the treatment group. This group will receive the monetary bonus.

Next decide on the monetary bonus. For purposes of this experiment, we will use a $5 gift certificate as the bonus. Next, determine the standards in place that will measure performance. In this scenario, the job is to handle incoming customer phone calls, some positive, some negative. The former standard of performance was to complete each phone call in 12 minutes. The new standard is to complete each call in 10 minutes.

All customer service representatives are required to meet the new standard. For the treatment group however, every time a representative does complete a phone call in ten minutes or less, he will receive a gift card. Those in the control group have the new standard but they have no card provided them for calls less than or equal to the 10-minute mark.

When comparing the results of the experiment, see how many more calls make the standard established in the treatment group compared to the control group. If there are a higher number of calls in the treatment group than the control group,you have established that the bonus card system is an effective incentive.

145. What are some of the key trends that produce challenges and opportunities for organizational behavior?

In light of the scandals of Enron, Tyco and WorldCom, a judgment has to be made on the ethical behavior exhibited in the American workplace. The seemingly endless announcements of unethical behavior represent a challenge to firms as to how to encourage and sustain ethical behavior.

Technology has transformed the way that work is completed and represents a challenge because of the speed with which that technology advances. The opportunity from this trend, however, is the fact that access to information is very open now and the playing field has been leveled.

Companies are beginning to examine their practices not just for greater profitability, but also for greater sustainability. It is of great interest to many firms that the environment is turned over to our future generations in a condition that will not only sustain our children but allow them to grow and prosper.

The workforce in America is an aging one. While that represents many work opportunities for young people who will replace retirees, it also threatens many

Page 23: Fwk Bauer Tif Organizational Behavior All Chapters

firms as the retirees take with them knowledge which is so vital to the continued growth and development of the firm.

The increasing globalization of business, with the regular practice of outsourcing and offshoring to address labor costs, represents challenges of culture integration and acceptance.

Page 24: Fwk Bauer Tif Organizational Behavior All Chapters

Organizational Behavior, Version 1.1Bauer & Erdogan

FWK Test Item FileChapter 2

TRUE/FALSE

1. While the American workforce is becoming more diverse, the world’s workforce is not.

(False)

2. Title VII of the Civil Rights Act prohibits discrimination in employment with regard to race, color, sex, religion and national origin.

(True)

3. With the passage of a number of laws in the United States prohibiting discrimination against race, color, gender, religion, national origin and age, complaints filed charging discrimination have largely been eliminated.

(False)

4. A more diverse work team produces higher quality decisions because more alternatives are considered.

(True)

5. Employees who are discriminated against experience more stress at work but are no more likely to leave their jobs than those who have not faced discrimination.

(False)

6. When discrimination complaints are filed, the Equal Employment Opportunity Commission (EEOC) acts as a mediator between the company and the complainant.

(True)

7. In firms pursuing a growth strategy, there is a negative relationship between racial diversity in the firm and company performance.

(False)

8. The similarity/attraction phenomenon suggests that individuals are attracted to those who are similar to them.

(True)

9. Race, age, and attitudes are examples of surface level diversity.(False)

10. Deep-level diversity traits are most important for early interactions in the workplace, but as time goes on, surface traits become more important.

Page 25: Fwk Bauer Tif Organizational Behavior All Chapters

(False)

11. A group with three older males and three younger women will have a strong faultline.

(True)

12. Research indicates that groups with strong faultlines can perform well if certain norms are established.

(True)

13. It is a commonly held stereotype that men are more relationship-oriented than women.

(False)

14. One of the explanations for the wage gap between men and women is that women pursue occupations that are lower-paying than those pursued by men.

(True)

15. Though the glass ceiling was a legitimate issue a few years ago, today the number of women in executive positions of organizations is roughly the same as the number of men at that level.

(False)

16. One rationale provided for the existence of the glass ceiling for women is that women are viewed as having personality characteristics that are not associated with successful leaders.

(True)

17. Ethnic minorities experience a wage gap and a glass ceiling even more severe than that faced by women.

(True)

18. Denny’s Restaurants were able to successfully institute diversity training efforts that increased the numbers of minority members of its board and management team as well as the number of minority franchise owners.

(True)

19. Research indicates that older employees exhibit higher absenteeism than their younger counterparts.

(False)

20. The Age Discrimination Act makes it illegal to discriminate against employees age 40 or older.

(True)

21. Research suggests that age diversity in a work team can lead to higher performance.

(True)

22. Reasonable accommodation for religious reasons does not require schedule modifications although it may require modifications to a firm’s dress code.

Page 26: Fwk Bauer Tif Organizational Behavior All Chapters

(False)

23. Reasonable accommodation for employees with disabilities may require the firm’s purchase of some equipment for the disabled employees’ use but does not require any schedule or job duty modifications.

(False)

24. Disclosing sexual orientation is the key to explaining work attitudes of lesbian, bisexual, gay and transgender employees.

(False)

25. To break the ice in an interview situation, an interviewer could legally ask, “What are some of the traditions your family practices for Christmas”?

(False)

26. One suggestion for building a culture that respects diversity is that managers should be accountable for diversity-related goals.

(True)

27. Affirmative action programs are among the most controversial diversity management methods available.

(True)

28. Preferential treatment programs for minorities can be illegal in some cases.(True)

29. Research suggests that some employees have a stigma attached to them because they were hired under an affirmative action program.

(True)

30. In the United States, the workforce is becoming increasingly multicultural.(True)

31. An expatriate is an employee who is temporarily assigned to work in a foreign country.

(True)

32. Low power distance societies view an unequal distribution of power as relatively acceptable.

(False)

33. Companies high in uncertainty avoidance prefer predictable situations and have low tolerance for ambiguity.

(True)

34. In collectivist cultures, the importance of family is critical to understanding the nature of the society.

(True)

35. In high power distance companies, participative decision making is a regular process.

Page 27: Fwk Bauer Tif Organizational Behavior All Chapters

(False)

36. Feminine cultures are more likely to be characterized by a separation of gender roles than masculine cultures.

(False)

37. Developing an openness to different experiences is a very viable method to prepare oneself for a global career.

(True)

38. To enhance their cultural intelligence, employees should become experts in one culture.

(False)

39. Ethnocentrism is the belief that one’s own culture is superior to other cultures.(True)

40. Ethical standards held in different societies may emphasize different behaviors as ethical or unethical.

(True)

41. Managing diversity effectively may have different meanings in different cultures.(True)

42. IBM is a pioneer in programming for a diverse workforce.(True)

43. IBM’s diversity management task forces acted on issues that were either based in reality or perception.

(True)

44. IBM, like other firms today, has little difficulty finding minority representation in the computer science and engineering fields.

(False)

45. IBM’s culture of respecting diversity seems to be creating a competitive advantage for the firm.

(True)

MULTIPLE CHOICE

Opening Section: Doing Good as a Core Business Strategy: The Case of Goodwill Industries

46. Goodwill Industries is an advocate for ________.a. survival of the fittestb. sales taxc. using interviews to weed out diverse applicantsd. diversity

(d) Easy/Comprehension

Page 28: Fwk Bauer Tif Organizational Behavior All Chapters

47. A total of $3.23 _______ was invested into employment programs in 2008 by Goodwill Industries.a. billionb. millionc. thousandd. dollars

(a) Easy/Comprehension

Section I: Demographic Diversity

48. The ways in which people are similar or different from one another isa. culture.b. diversity.c. stereotypes.d. faultlines.

(b) Easy/Knowledge

49. Which of the following laws prohibits discrimination based on race, color, sex or national origin?a. Age Discrimination Actb. Americans with Disabilities Actc. Family and Medical Leave Actd. Title VII of the Civil Rights Act

(d) Easy/Comprehension

50. Which of the following is NOT a benefit of effective management of diversity?a. greater creativity in decision makingb. better service to customersc. higher productivity of operations d. more satisfied workers

(c) Medium/Comprehension

51. Which of the following statements on the benefits of effective management of diversity is INCORRECT?a. Diverse workforces lead to higher-quality decisions.b. Diverse workforces create cheaper products.c. Diverse workforces lead to lower litigation costs.d. Diverse workforces have led to higher stock prices for firms.

(b) Difficult/Synthesis

52. Research on diversity in the workplace has not substantiated which of the following benefit to firms?a. Diversity, and the presence of faultlines, increases cohesiveness at work. b. Award-winning diversity management programs lead to higher stock

prices for their companies.c. In companies with a growth strategy, there is a positive correlation

between racial diversity in the firm and company performance.d. Diverse teams make higher quality decisions.

(a) Difficult/Comprehension

Page 29: Fwk Bauer Tif Organizational Behavior All Chapters

53. The similarity/attraction phenomenon may explain which of the following?a. Minorities are hired as often as their Caucasian counterparts.b. Disabled individuals are hired as often as those who are not disabled.c. Women do not earn salaries as high as that of their male counterparts.d. Women and men have equal access to mentoring opportunities.

(c) Medium/Evaluation

54. Demographic traits make upa. deep-level diversity.b. cultural diversity.c. collective diversity.d. surface diversity.

(d) Easy/Knowledge

55. Surface diversity traits includea. values.b. gender.c. attitudes.d. beliefs.

(b) Easy/Knowledge

56. Deep-level diversity traits includea. gender.b. age.c. attitudes.d. race.

(c) Easy/Knowledge

57. Which of the following is a surface-level diversity trait?a. aggressivenessb. Type A personalityc. being smartd. being Asian

(d) Easy/Application

58. Which of the following is a deep-level diversity trait?a. middle-agedb. Caucasianc. aggressivenessd. amputee

(c) Medium/Application

59. Our first interaction with individuals makes us aware ofa. surface diversity.b. collective diversity.c. deep-level diversity.d. value-based diversity.

(a) Medium/Comprehension

60. Which of the following statements regarding the similarity/attraction phenomenon is INCORRECT?

Page 30: Fwk Bauer Tif Organizational Behavior All Chapters

a. Surface-level traits affect interpersonal interactions only after people have known each other for a long time.

b. Age, race and gender dissimilarity are strong predictors of employee turnover in that employee’s early tenure with a firm.

c. Those employees who feel different from other employees in a firm may feel isolated in their early tenure with a firm.

d. Surface traits are attended to because they are assumed to be related to deep-level diversity traits.

(a) Medium/Evaluation

61. Which of the following approaches is NOT a viable option to manage the similarity/attraction phenomenon?a. Develop a formal mentoring program that assigns new employees a

mentor in the firm.b. Provide management training programs that make managers aware of

the similarity/attraction phenomenon.c. Develop a recruiting program based upon peer referrals.d. Attend to new employees as they enter the organization and during their

early stays in the firm.(c) Difficult/Evaluation

62. A faultline isa. an attribute along which a group is split into subgroups.b. a generalization about a particular group of people.c. the way in which people are similar or different from each other.d. the values, beliefs and customs that exist in a society.

(a) Easy/knowledge

63. A workgroup has three Caucasian males and three African American females. How would you categorize the faultline in this group?a. There is no faultline in the group.b. There will be a strong faultline in the group.c. There will be a weak faultline in the group.d. There will be a faultline of medium strength in the group.

(b) Medium/Application

64. If a workgroup contained three males, one over 55, one 20 and another 30 years of age, and three females, one 50 years of age, one 30 and one 20 years of age, how would you categorize the faultlines in the group?a. There will be a strong faultline in this group that aids performance. b. There will be a strong faultline in the group that weakens performance.c. There will be a weak faultline within the group.d. There will be a faultline of medium strength in the group.

(c) Difficult/Application

65. Research indicates that groups with faultlines exhibit which of the following characteristics?a. teams with faultlines make more effective decisions.b. teams with faultlines are more creative.c. teams with strong faultlines have the potential to perform well if they can

establish certain norms.

Page 31: Fwk Bauer Tif Organizational Behavior All Chapters

d. all diverse teams exhibit faultlines.(c) Medium/Analysis

66. A stereotype isa. an attribute along which a group is split into subgroups.b. a generalization about a particular group of people.c. a way in which people are similar or different from each other.d. a difference in values, attitudes or beliefs.

(b) Easy/Knowledge

67. “Women are bad drivers,” is an example of aa. surface diversity trait.b. faultline.c. stereotype.d. deep-level diversity trait.

(c) Medium/Application

68. Stereotypesa. often lead to unfair decisions being made.b. are never accurate descriptions of particular groups.c. are assumptions held by only one individual.d. are often verified prior to making decisions based upon them.

(a) Medium/Analysis

69. The faculty of a Midwestern business school is comprised of 33 males and 4 females. The past five candidates hired have all been male. There seems to be a (an)__________ diversity issue in this workplace.a. ageb. racec. religiond. gender

(d) Easy/Application

70. Gender discrimination is prohibited by which of the following laws?a. Age Discrimination Actb. Americans with Disabilities Actc. Title VII of the Civil Rights Actd. Occupational Safety and Health Act

(c) Easy/Knowledge

71. Which of the following is not an explanation that has been offered to account for the earnings gap of 21% between the pay of females and males?a. Women lack the high-level skill sets that men possess.b. Women have résumé gaps due to time off for childcare.c. Women pursue lower-paying occupations than men pursue.d. Women tend to negotiate less than men and when they do negotiate,

women attain lower levels of pay than men do.(a) Medium/Analysis

72. The glass ceilinga. is based on a faultline in the employee workgroup.

Page 32: Fwk Bauer Tif Organizational Behavior All Chapters

b. is the underrepresentation of women in executive and upper level management positions of companies.

c. is an issue that was a problem until a decade ago.d. is the result of the less effective leadership styles practiced by women.

(b) Medium/Comprehension

73. Which of the following statements regarding the explanations for the glass ceiling is INCORRECT?a. The glass ceiling may exist for women because they lack the education

levels necessary to assume high-level managerial positions.b. The glass ceiling may be based on the stereotype that the male

characteristic of assertiveness is necessary to manage, but women tend to be passive.

c. The glass ceiling may exist for women because their accomplishments are often not valued as highly as are those of their male counterparts.

d. The glass ceiling may exist for women because they are viewed as displaying submissive characteristics in contrast to their male counterparts who are viewed as possessing more confidence.

(a) Medium/Evaluation

74. Acme Corporation sales associates are generally attractive Caucasian college age men and women. Individuals who work in the stock room unpacking and readying clothing for displays are young, Asian and African American employees. This scenario suggests that Acme may need to examine their recruiting efforts to address __________ diversity in their stores.a. genderb. agec. raced. age and race

(d) Medium/Application

75. Which of the following statements regarding race diversity in the American workplace is correct?a. Highly-paid minority managers indicate that they have experienced very

few situations in the workplace where race was an issue.b. A majority of American employees surveyed indicate that their firms are

quite effective at hiring and promoting minorities.c. Ethnic minorities experience a slight wage gap but no glass ceiling.d. Overt discrimination against ethnic minorities still occurs in many

workplaces.(d) Medium/Analysis

76. Which of the following is a research finding about race diversity? According to prior research findings,

a. African Americans are more likely to turnover than their Caucasian counterparts.

b. the sales performance of Hispanics is equal to their Caucasian counterparts.

c. Caucasian Americans are more likely to be absent from work than their African American counterparts.

Page 33: Fwk Bauer Tif Organizational Behavior All Chapters

d. ethnic minorities are more likely to experience a satisfying work environment than their Caucasian counterparts.

(a) Medium/Evaluation

77. Based on employment statistics, age diversity at work will continue into the future. Thus, the findings from research examining age diversity will become increasingly useful in managing the workforce. Which of the following statements about those research findings is accurate?a. Older workers are generally stereotyped as more qualified and having

higher potential.b. Older employees are more likely to be absent from their jobs. c. Older workers are less likely to quit their jobs when dissatisfied at work.d. Older workers are stereotyped as being more able to handle stress.

(c) Difficult/Evaluation

78. Age is positively correlated with all of the following behaviors EXCEPTa. citizenship behaviors.b. compliance with safety rules.c. performance.d. work injuries.

(d) Medium/Analysis

79. The Age Discrimination in Employment Act a. prohibits discrimination against employees older than 65.b. prohibits discrimination against employees age 40 or older.c. prohibits discrimination against employees only with regard to pay.d. is seldom cited in lawsuits as age discrimination is largely absent from the

American workplace.(b) Easy/Knowledge

80. All of the following are challenges of managing age diversity in the workplace EXCEPTa. an age-diverse team has differing opinions on fairness than one not age-

diverse.b. an age-diverse team has different learning styles that vary by generation.c. an age-diverse team has higher team performance than one not age-

diverse.d. an-age diverse team offers fewer alternatives to a problem than a team

where there is limited age diversity.(d) Medium/Analysis

81. Accommodating religious diversity in the workplace does not includea. work schedule modifications.b. dress code modifications.c. work duty modifications.d. pay level modifications.

(d) Easy/Comprehension

82. The Americans with Disabilities Act a. requires a firm to hire individuals with disabilities.b. requires a firm to interview individuals with disabilities.

Page 34: Fwk Bauer Tif Organizational Behavior All Chapters

c. prohibits discrimination in employment against individuals with physical disabilities.

d. prohibits discrimination in employment against individuals with physical or mental disabilities if these employees are otherwise qualified to do their jobs with or without reasonable accommodation.

(d) Medium/Comprehension

83. Discrimination charges filed against employers based on disabilities cover which of the following impairments?a. cancerb. hearing impairmentsc. manic-depressive disorderd. all of the above

(d) Medium/Knowledge

84. Research suggests all of the following reasons why individuals with long-term illnesses or those requiring ongoing care seem to be discriminated against EXCEPTa. stereotyping as incapable of performing a job.b. more likely to be locked into a dead end job.c. more likely to be employed in jobs with high skill and qualification

requirements they do not possess.d. more likely quit their jobs.

(c) Medium/Knowledge

85. Reasonable accommodations for employees with disabilities may include all of the following EXCEPTa. modifications in employee schedules.b. providing auxiliary equipment like telephone amplifiers.c. physical changes to the company facilities (e.g. wheelchair ramp).d. assignment of their essential job functions to other employees.

(d) Medium/Application

86. Which of the following federal laws prohibits discrimination in employment against lesbian, bisexual, gay and transgender individuals?a. Title VII of the Civil Rights Actb. Family and Medical Leave Actc. Americans with Disabilities Actd. There is currently no federal law protecting lesbian, bisexual, gay and

transgender individuals.(d) Easy/Knowledge

87. Research findings indicate which of the following statements about issues related to sexual orientation in the workplace is INCORRECT?a. Whether they disclose their sexual orientation or not is the key to the work

attitudes of lesbian, bisexual, gay and transgender individuals.b. Fear of disclosing sexual orientation and its possible negative

consequences leads to lower job satisfaction among lesbian, bisexual, gay and transgender individuals.

Page 35: Fwk Bauer Tif Organizational Behavior All Chapters

c. Respect for diversity in sexual orientation can be shown by written company policies prohibiting discrimination based upon sexual orientation.

d. In firms with benefits programs extended to lesbians, bisexuals, gays and transgender individuals, discrimination based on sexual orientation is less frequent and job satisfaction is higher.

(a) Medium/Analysis

88. Which of the following is an illegal interview question?a. What was your grade point average at XYZ University?b. Tell me about a time when you experienced conflict in a past job.c. What are your childcare arrangements?d. What was your salary at your last place of employment?

(c) Difficult/Application

89. Which of the following statements regarding diversity in the workplace is correct?a. The wage gap between Hispanic and Caucasian workers is smaller than

the wage gap between male and female workers.b. While female workers face a glass ceiling, other minorities face no such

issue to attainment of upper level managerial jobs.c. Company policies requiring disclosure of sexual orientation lead to higher

job satisfaction and organization commitment among lesbian, bisexual, gay and transgender employees.

d. Federal law requires reasonable accommodation be considered when an employee brings up a disability.

(d) Difficult/Synthesis

90. Which of the following statements regarding diversity in the workplace is accurate?a. Older employees are more prone workplace injury than their younger

counterparts.b. Women who negotiate achieve higher salaries than men who negotiate.c. More than half of the Fortune 500 companies offer same sex health

benefits to their employees.d. While employees with physical disabilities are protected by federal law,

employees with mental disabilities remain unprotected, and face routine discrimination.

(c) Difficult/Synthesis

91. Which of the following is not a suggestion for managing demographic diversity?a. Review recruitment practices.b. Make managers accountable for diversity.c. Institute affirmative action programs with quotas.d. Build a culture of respect for diversity.

(c) Medium/Comprehension

92. Which of the following statements regarding the management of demographic diversity is INCORRECT?a. Diversity training programs are most successful in those firms where

there are explicit rewards for increasing company diversity.

Page 36: Fwk Bauer Tif Organizational Behavior All Chapters

b. Companies can support diversity by reviewing their employment advertising to ensure that diversity is being advocated at all job levels.

c. In the most successful companies, diversity is viewed as human resource management department’s responsibility.

d. Many people see affirmative action programming as a controversial way to manage diversity because it provides an unfair advantage to minority members.

(c) Difficult/Synthesis

93. Which of the following groups of programs is not viewed as being part of an overall affirmative action program?a. broad-based recruitmentb. simple elimination of discriminationc. preferential treatmentd. tie-breakers such that all things being equal preference is given to

minority candidates(a) Medium/Analysis

94. The affirmative action program about which the most negative perceptions are held is a. simple elimination of discrimination.b. tiebreakers.c. preferential treatment.d. targeted recruitment.

(c) Difficult/Evaluation

95. At some time in your career, you may be different from others in your workplace. Which of the following is not a viable piece of advice to offer someone who is “different”?a. Know your rights; you are protected from discrimination in employment on

the basis of race, color, sex, religion or national origin.b. Choose a mentor who is similar to you and at your same level.c. Build effective relationships; create opportunities to get to know others in

the workplace.d. Examine company resources; see if your firm offers networking

opportunities or interest groups.(b) Medium/Comprehension

Section II: Cultural Diversity

96. The values, beliefs, and customs that exist in a society isa. culture.b. diversity.c. power distance.d. affirmative action.

(a) Easy/Knowledge

97. Which of the following is not one of Hofstede’s four dimensions that explain variation among cultures?a. individualism/collectivismb. introversion/extraversion

Page 37: Fwk Bauer Tif Organizational Behavior All Chapters

c. low uncertainty avoidance/high uncertainty avoidanced. masculinity/femininity

(b) Medium/Knowledge

98. Collectivistic cultures are a. cultures where people define themselves as individuals and form looser

ties with their groups.b. cultures that value achievement and competitiveness as well as

acquisition of money and other material objects.c. cultures where people have strong bonds to their groups and group

membership forms a person’s self-identity.d. cultures that value maintaining good relationships, caring for the weak

and quality of life.(c) Easy/Knowledge

99. Cultures that are comfortable in unpredictable situations and have high tolerance for ambiguity area. low uncertainty avoidance cultures.b. collectivistic cultures.c. high uncertainty avoidance cultures.d. individualistic cultures.

(a) Medium/Comprehension

100. A culture that values maintaining good relationships, caring for the weak and quality of life isa. a low power distance culture.b. a masculine culture.c. a high power distance culture.d. a feminine culture.

(d) Easy/Comprehension

101. High power distance culturesa. view an unequal distribution of power as relatively acceptable.b. prefer predictable situations and have low tolerance for ambiguity.c. emphasize caring for the weak and quality of life. d. are comfortable in unpredictable situations and have high tolerance for

ambiguity.(a) Easy/Comprehension

102. A culture categorized by high uncertainty avoidance isa. Denmark.b. Jamaica.c. Greece.d. China.

(c) Medium/Comprehension

103. Which of the following is not a culture that is characterized as feminine in its orientation?a. Norwayb. Japanc. Sweden

Page 38: Fwk Bauer Tif Organizational Behavior All Chapters

d. Costa Rica(b) Medium/Comprehension

104. All of the following cultures are categorized as individualistic EXCEPTa. USA.b. United Kingdom.c. Pakistan.d. Australia.

(c) Medium/Comprehension

105. If a collectivist was told, “Tell me about yourself.”, he might saya. I want to be a doctor when I graduate.b. I like playing video games.c. I am a black belt in karate.d. I am the third child of a doctor and a lawyer.

(d) Medium/Application

106. Which of the following statements is not illustrative of actions of an individual from a collectivist culture?a. “My brother caused the accident, we will cover the costs.”b. “My parents and I live with my grandparents and an aunt and uncle.”c. “I must decide which of the two jobs I am going to accept.”d. “My mother and father must approve of my girlfriend before I can ask her

to marry me.”(c) Difficult/Application

107. Which of the following statements about collectivists and individualists is accurate?a. Collectivists are more generous in the assessment of their group

members while individualists are relatively more accurate in their evaluation of group members.

b. Collectivists are friendly with both in-group and out-group members.c. Collectivists use direct communication with their in-groups at all times. d. Collectivists are used to changing groups over the course of their lives

and are highly adaptable to new groups.(a) Medium/Analysis

108. In cultures high in power distance a. there is a higher degree of egalitarianism. b. managers are believed to be more powerful and deserving of respect.c. managers and subordinates use first names among each other in the

workplace.d. there is higher utilization of participative decision making practices.

(b) Medium/Analysis

109. Companies in cultures high in uncertainty avoidancea. have a very short, simple recruiting and selection process for new

employees.b. tend to have written employment contracts for employees.c. have limited numbers of policies and procedures in employee handbooks

and other manuals.

Page 39: Fwk Bauer Tif Organizational Behavior All Chapters

d. tend to enter a large number of foreign markets.(b) Medium/Analysis

110. Which of the following statements regarding the masculinity/femininity dimension of cultures is INCORRECT?a. In feminine cultures, only women value maintaining good relationships,

but in masculine cultures, both males and females value maintaining good relationships.

b. In masculine cultures, the ratio of CEO pay to other management level employees is higher.

c. In cultures high in femininity, work/life balance programming like telecommuting is more plentiful than in masculine cultures.

d. There is greater separation of gender roles in masculine cultures than in feminine cultures.

(a) Medium/Analysis

111. Angela is planning on an international business career following graduation. She is meeting with a career counselor to ensure that she is preparing herself as fully as she might for such a career. Which of the following pieces of advice will the counselor offer her?a. There is no need to learn a foreign language; English has become the global business language.b. Become familiar with different cultures by visiting several countries in a short amount of time. c. It is a good idea to plan your return, as well as planning your departure. d. Do not show your limited understanding to locals to avoid losing

credibility. (c) Medium/Analysis

112. Cultural intelligence isa. the ways in which people are similar or different from one another.b. the values, beliefs and customs that exist in a society.c. a person’s capability to understand how a person’s cultural background

influences one’s behavior.d. the degree to which the society views an unequal distribution of power as

acceptable.(c) Easy/Knowledge

113. The belief that one’s own culture is superior to other cultures isa. geocentrism.b. ethnocentrism.c. monocentrism.d. polycentrism.

(b) Easy/Knowledge

114. Which of the following statements is a good suggestion for managing cultural diversity?a. For the most effective international experiences, train individual

employees thoroughly in one culture before moving on to another.

Page 40: Fwk Bauer Tif Organizational Behavior All Chapters

b. Keep in mind that local employees in the foreign country are too immersed in the culture to help you understand their culture. For a more objective viewpoint, other expatriates are more useful.

c. There is a very high probability that problems experienced on an expatriate assignment are due to cultural differences and misunderstandings.

d. Culture changes, sometimes quickly; do not assume that what was true a year ago is true today.

(d) Medium/Analysis

Section III: The Role of Ethics and National Culture

115. Which of the following statements regarding demographic diversity and ethics is correct?a. Younger employees are more upset by unethical behaviors than older

employees.b. When judging the ethicality of hypothetical situations, women and men

use similar standards when judging the ethicality of monetary issues.c. Younger employees tend to find negotiation tactics like bluffing to be

more unethical than their older counterparts.d. Men and women have similar standards when judging the ethicality of

breaking organizational rules.(b) Medium/Evaluation

116. Which of the following statements regarding cultural diversity and ethics is correct?a. In a study, Koreans suggested that being ethical was compatible with

being profitable.b. In a study, American subjects viewed environmental damage as more

ethical than their Korean or Indian counterparts.c. In a study, Americans viewed an auditing company sharing a client’s

auditing information with another client as more ethical than Russians subjects did.

d. In a study, Korean and Indian subjects found software piracy and nepotism less ethical than American subjects did.

(b) Medium/Evaluation

117. Which of the following statements about demographic diversity around the world is INCORRECT?a. In Japan, women face less of a glass ceiling than in the United States.b. In France, a study indicated that resumes with French-sounding names

were more likely to get interviews than those with North-African–sounding names.

c. Japan is a relatively homogeneous society.d. Overt sexism against women is rampant in Japan

(a) Medium/Evaluation

Closing Section: Managing Diversity for Success: The Case of IBM

Page 41: Fwk Bauer Tif Organizational Behavior All Chapters

118. IBM has always been a pioneer in valuing and appreciating diversity in its workforce. Which of the following is NOT a program IBM instituted before legislation or public policy dictated it?a. gender pay equityb. women in executive positionsc. family leave for all female employeesd. executive training programs for women

(d) Medium/Evaluation

119. Prior to 1993, IBM managed diversity by a. ignoring differences and providing equal employment opportunities.b. minimizing differences and emphasizing similarities.c. emphasizing differences and opportunities.d. minimizing differences as deficiency and highlighting differences as

opportunities.(a) Difficult/Evaluation

120. Which of the following was NOT a challenge that the IBM diversity task force on women found to be a concern?a. lack of networking opportunitiesb. lack of equal pay opportunitiesc. presence of a male-dominated cultured. work/life management challenges

(b) Medium/Comprehension

121. Which of the following was NOT a challenge that the IBM diversity task force on Asian employees found to be a concern?a. stereotypingb. limited employee development plansc. work life management challengesd. lack of networking

(c) Medium/Comprehension

122. Which of the following was a challenge that the IBM diversity task force on African American employees found to be a concern?a. work/life management challengesb. lack of networkingc. insufficient health plan coveraged. lack of participation in decision making

(b) Medium/Comprehension

123. Which of the following statements is INCORRECT about IBM’s approach to diversity management?a. IBM is interested in measuring the success of its diversity management

programs as they believe that presence of these programs, rather than an emphasis on results, shows its commitment to diversity.

b. IBM has an excellent recruiting program for increasing its pool of diversity candidates.

c. IBM’s diversity recruiting often faces major hurdles due to the limited minority representation in the computer science and engineering fields.

Page 42: Fwk Bauer Tif Organizational Behavior All Chapters

d. IBM’s diversity task forces consist of senior and higher level management charged with understanding the eight identified diversity constituencies.

(a) Difficult/Comprehension

124. Which of the following statements regarding diversity in the American workforce is INCORRECT?a. Individuals of African American, Hispanic and Asian backgrounds

constitute roughly thirty percent of the American workforce.b. Women constitute almost half of the American workforce.c. Increasing numbers of young people are flooding the labor markets.d. Sixteen percent of the American workforce is foreign-born.

(c) Medium/Comprehension

FILL IN THE BLANK

125. ____________ refers to the ways in which people are similar or different form each other.

(Diversity)

126. ______ ________ _ ___ _______ _________ ______ prohibits discrimination in employment on the basis of race, color, sex, religion or national origin.

(Title VII of the Civil Rights Act)

127. The tendency for individuals to be attracted to similar individuals is called the ____________-_____________ phenomenon.

(similarity/attraction)

128. Demographic traits like race, gender and age are part of _________ diversity.(surface)

129. Diversity in values, beliefs and attitudes is called _________ ____________ diversity.

(deep-level)

130. An attribute along which a group is split into subgroups is called a ___________.(faultline)

131. A generalization about a particular group of people, like “men never ask for directions” is an example of a ________________.

(stereotype)

132. The fact that women earn 79% of what men earn demonstrates the _________ _________.

(earnings or wage gap)

133. The ______ __________ is the invisible barrier faced by women and minorities where they are represented in lower level occupations in the firm but not at the higher and executive levels.

(glass ceiling)

Page 43: Fwk Bauer Tif Organizational Behavior All Chapters

134. The Age Discrimination in Employment Act prohibits employment discrimination against individuals age __________ and over.

(40)

135. The Americans with Disabilities Act prohibits discrimination in employment against individuals with physical or mental disabilities if they are otherwise qualified to do their jobs with or without _________ _____________.

(reasonable accommodation)

136. __________ __________ programs are those designed to recruit, promote, train and retain employees belonging to a protected class.

(Affirmative Action)

137. The values, beliefs and customs that exist in a society are the _________.(culture)

138. An American citizen working in an American company located in Japan would be called a(n) ____________.

(expatriate)

139. ___________ ______________ are cultures where people have stronger bonds to their groups and group membership forms a person’s self-identity and ________ ___________are cultures where people define themselves as individuals and form looser ties with their groups.

(Collectivistic cultures, individualistic cultures)

140. A society that views an unequal distribution of power as relatively acceptable would be categorized as a _______ _________ ___________culture.

(high power distance)

141. Cultures that are comfortable in unpredictable situations and have high tolerance for ambiguity are called ________ ____________ _________ cultures.

(low uncertainty avoidance)

142. A ________ culture values achievement, competitiveness and the acquisition of money and other material objects.

(masculine)

143. _____________ _____________ is a person’s capability to understand how a person’s cultural background influences one’s behavior.

(Cultural intelligence)

144. ______________ is the belief that one’s own culture is superior to other cultures.(Ethnocentrism)

SHORT ANSWERS

145. What are some of the benefits of diversity?

Page 44: Fwk Bauer Tif Organizational Behavior All Chapters

Higher creativity in decision making, better understanding and service to customers, more satisfied workforce, higher stock prices, lower litigation expenses, and higher company performance.

146. Briefly describe the similarity/attraction phenomenon.

The similarity/attraction phenomenon is the tendency for individuals to be attracted to similar individuals. So, individuals are more likely to have conflict with those who differ from them with regard to demographic characteristics like race, color, gender, etc. The phenomenon may explain some of the unfair treatment individuals experience. For example, women tend not to be hired into higher-level positions in the firm. To progress to those levels, they may need to have a mentor to “show them the ropes”. But if only men are in those positions, they would, according to the phenomenon, pick others like them, so they would choose men. Hence, women never get the chance to be promoted.

147. Define and give an example of surface diversity.

Surface diversity includes traits that are visible to those around us and include such demographic traits as gender, race and age.

148. Define and give an example of deep-level diversity.

Deep-level diversity includes values, beliefs mental abilities, and attitudes.

149. Define and provide an example of a faultline.

A faultline is an attribute along which a group is split into subgroups. An example is a group of six, three older women and three younger men. This group has a strong faultline because the group would be divided by gender and age.

150. Define the term stereotype and give an example.

A stereotype is a generalization about a particular group of people. An example of a stereotype is that women are poor drivers.

151. What is the earnings gap?

The earnings gap is the difference in salary level between men and women or between minorities and Caucasian males (generally). Currently, the wage gap between men and women is that women earn 79% of that earned by men. Minorities differ slightly from that figure with Hispanics making about 64% of what Caucasian males earn.

152. What is the glass ceiling?

The glass ceiling is an invisible barrier that tends to keep women and minorities at lower levels in the firm. In 2008, only twelve of the Fortune 500 firms had female CEOs and only three had African American CEOs.

Page 45: Fwk Bauer Tif Organizational Behavior All Chapters

153. A number of research studies have focused on older workers in the workplace. What were some of the findings that impact the management of the workplace?

Age is correlated with higher levels of citizenship behaviors like volunteering, higher safety rule compliance, lower work injuries, lower counterproductive behaviors and lower rates of tardiness and absenteeism. In addition, older people are less likely to quit their jobs when they are dissatisfied, are able to handle stress effectively and may perform better than their younger counterparts.

154. What are some accommodations that must be made for those with religious issues in the employment relationship?

Title VII of the Civil Rights Act prohibits firms from discriminating in employment on the basis of religion. Employers are required to make reasonable accommodation to ensure the practice of their religious beliefs unless it poses an unreasonable hardship on the employer. Generally the accommodations that the firm makes have to do with schedule and dress code modifications.

155. What is an example of a reasonable accommodation for a disabled individual that a firm might make?

One reasonable accommodation has to do with access to the place of employment. Wheelchair ramps, for example, are such an accommodation. Other accommodations, depending on the disability, might include some equipment that the disabled individual could use to assist him in the performance of his job such as a telephone amplifier for those with hearing impairment, schedule modifications, and reassignment of some non-essential job functions.

156. What is a program that can be a part of a larger affirmative action initiative?

Any of the following are possibilities: simple elimination of discrimination, targeted recruitment, tie-breakers and preferential treatment.

157. Define Hofstede’s individualism-collectivism dimension and give an example of each type of culture.

Individualism refers to cultures where people define themselves as an individual and form looser ties with their groups. An example of an individualist culture is the United States where personal independence is valued.

Collectivism refers to cultures where people have stronger bonds to their groups and group membership forms a person’s self-identity. An example of this type of culture is Mexico where the family is very important.

158. Define Hofstede’s low-high power distance dimension and give an example of each type of culture.

Low power distance is a society that views an unequal distribution of power as relatively unacceptable. In a low power distance culture, egalitarianism is the norm. A high power distance society views an unequal distribution of power as

Page 46: Fwk Bauer Tif Organizational Behavior All Chapters

relatively acceptable. An example is Turkey, where elementary and high school students stand to greet their teachers when they enter the room.

159. Define Hofstede’s low-high uncertainty avoidance dimension and give an example of each type of culture.

A low uncertainty avoidance culture is comfortable in unpredictable situations and has high tolerance for ambiguity. An example is Russia, where companies do not have rule-oriented cultures.

A high uncertainty avoidance culture prefers predictable situations and has low tolerance for ambiguity. An example of this culture is Germany, where people prefer structure in their lives and like rules and procedures that manage situations.

160. Define Hofstede’s masculinity/femininity dimension and give an example of each type of culture.

Masculinity is a culture that values achievement, competitiveness, and the acquisition of money and other material objects. An example of a masculine culture is Japan where there is a separation of gender roles.

Femininity is a culture that values maintaining good relationships, caring for the weak and quality of life. An example of a feminine culture is Sweden where work/life balance is important and work arrangements like telecommuting are popular.

161. IBM was a pioneer in creating programs to value and develop a diverse workforce prior to federal legislation requiring such programs. Briefly discuss one of the programs IBM instituted that values diversity.

In 1935, IBM President Thomas Watson promised equal pay between the genders employed by the firm. In 1943, IBM had its first female vice president. The firm granted women unpaid leave for the birth of a child thirty years before the Family and Medical Leave Act; it then extended that benefit to female employees, raising the leave to one year in the 1960s and three years in 1988.

162. Briefly discuss the task forces that IBM established under Louis Gerstner to promote diversity in the firm, and address the masculine culture of the firm at the time.

The eight task forces established were women and men, Asians, African Americans, lesbian/bisexual/gay/transgender, Hispanic, Native American and employees with disabilities. Each task force had senior and higher level employees on it and each task force was charged with understanding how each constituency felt so they could be made to feel welcome at IBM. Each task force conducted meetings and surveyed employees to arrive at key factors related to each group. The key factors did not have to be based in reality; if employees perceived the issues to exist, IBM designed a program to address it.

ESSAY

Page 47: Fwk Bauer Tif Organizational Behavior All Chapters

163. What lessons do the IBM diversity management task forces offer to other firms?

Answers will vary but some general thoughts include: IBM’s success with diversity management suggests that other firms should consider implementing similar programs. IBM has tripled the number of female executives, doubled minority executives and increased those categorized as lesbian, bisexual, gay and transgender seven-fold. IBM has seen this program to be a competitive advantage. Using the information from the task forces, IBM introduced programs to enhance networking activities for women and Asians, and helped retain African Americans.

164. Why would companies consider implementing diversity programs?

There are a number of different reasons for such programming, but some of the benefits of diversity initiatives include:

Higher creativity in decision making: The more diverse a group, the more options a group considers, which leads to higher-quality decisions. This increased quality could lead to more creative decisions that could positively impact the company’s bottom line.

Better understanding and service to customers: The more a firm understands about the breadth of its customer base, the more opportunity is offered for growing a market or tapping a new market segment.

More satisfied workforce: When employees feel fairly treated, they are more satisfied. More satisfied workers turnover less.

Higher stock prices: Companies with a more diverse workforce find that they have a higher stock price in the days following the announcement of diversity initiatives.

Lower litigation costs: If a firm is being fair to its employees, there is no need for employees to file complaints against the firm for discriminating against them with regard to federal law. Obviously a lack of complaints saves substantial litigation fees.

Higher company performance: Those firms managing diversity effectively tend to perform better.

165. Michael is a new manager for a Fortune 500 firm. He is preparing for his first assignment, which is to complete a diversity audit of his firm. What are some of the areas he should consider for assessment?

A number of demographic groups can be investigated in the firm. However, some of the major groups include: gender (what is the number of women in executive or higher level management positions, are there pay differences between women and other groups in the firm); race (again what is the number of minorities in upper management positions in the firm and what is their pay rate); age (how are older workers treated, are they stereotyped); religious (what accommodations are

Page 48: Fwk Bauer Tif Organizational Behavior All Chapters

made for religious beliefs); disabled (what accommodations need to be made) and sexual orientation (are same sex benefits offered).

166. You are being interviewed for a position in a financial services firm. The last three questions that the interviewer asked of you have been illegal questions. What can you do?

Some options:

Continue to refuse to answer and maybe even point out the illegality of the question.

Answer shortly and then change the subject.

Answer the intent of the question, not the actual question.

Walk away from the interview.

167. You are an organizational behavior consultant who has been brought into a firm to present some best practices with regard to managing demographic diversity. What are some of those best practices which you could present in a seminar?

Build a culture that respects diversity. All levels of the firm must support diversity.

Make managers accountable for diversity.

Institute diversity training programs.

Review recruitment practices and ensure the firm is targeting a diverse candidate pool.

Consider affirmative action programs to enhance diversity.

168. You are an advisor for international business students. You are meeting with freshmen business students this afternoon. What are some recommendations you could make to these students to prepare them for global careers?

Answers will vary but some general responses are:

Learn a language.Immerse yourself in different cultures.Develop an openness to different experiences.Develop a strong social support network.Develop a sense of humor.Plan your return from a foreign assignment to avoid reverse culture shock.

169. You are an organizational behavior consultant who has been hired to present a training seminar to managers in a Fortune 500 firm about suggestions for managing cultural diversity in the firm. What are some of the topics you will consider presenting during the seminar?

Page 49: Fwk Bauer Tif Organizational Behavior All Chapters

Help employees build cultural intelligence; companies are very diverse now and the more cultures you can understand the better prepared your firm will be.

Avoid ethnocentrism.

Listen to locals.

Recognize that culture changes.

Do not always assume that culture is the problem.

Page 50: Fwk Bauer Tif Organizational Behavior All Chapters

Organizational Behavior, Version 1.1Bauer & Erdogan

FWK Test Item FileChapter 3

TRUE/FALSE1. Applicant-matching software reduces the time required to hire candidates.(True)

2. A risk-averse individual is generally a poor fit for a high-tech firm in a dynamic environment.

(True)

3. Behavior is a function of the person and the situation interacting with each other.(True)

4. Person/job fit is related to job satisfaction.(True)

5. When people fit their jobs, they experience higher levels of stress.(False)

6. There is usually a strong relationship between good person/organization fit and high job performance.

(False)

7. Value orientations change dramatically as individuals age and mature.(False)

8. Instrumental values are end states people desire in life.(False)

9. Family security is a terminal value.(True)

10. Behavior may be affected as much by what is expected of us as how we want to behave.

(True)

11. Conscientiousness, more than any other personality trait, predicts how successfully a person performs a variety of jobs.

(True)

12. People high in openness are more likely to start their own business than those low in openness.

(True)

Page 51: Fwk Bauer Tif Organizational Behavior All Chapters

13. Extraverts are happier in their work situations and perform much more effectively than introverts.

(False)

14. Extraverts are always model employees.(False)

15. High agreeableness is always a personality trait sought in job candidates.(False)

16. The Myers-Briggs Type Indicator is a very effective employee selection tool.(False)

17. The MBTI classifies individuals on the basis of types, not traits.(True)

18. Positive affective people are absent from work less often.(True)

19. One tip to remember in working with people with negative affectivity is that you can change someone’s personality with relative ease.

(False)

20. People who are labeled as high social monitors can experience high levels of stress.

(True)

21. Social monitors are especially good at evaluating the performance of other employees.

(False)

22. Proactive people understand the political environment in organizations, so they adjust quickly to new jobs.

(True)

23. Mary said, “I am quite confident in the plan I developed, as I worked quite diligently on it and I have implemented similar programs in similar settings.” Mary seems to have high self-efficacy.

(True)

24. Self-efficacy is the degree to which people have overall positive feelings about themselves.

(False)

25. “It was really nothing. These things happen to me because I happen to be in the right place at the right time.” This statement suggests an external locus of control.

(True)

26. Research indicates that personality tests are an excellent predictor of performance.

Page 52: Fwk Bauer Tif Organizational Behavior All Chapters

(False)

27. One of the challenges of using personality tests in employee selection is that the rankings of the candidates who take the test may be affected by their ability to fake.

(True)

28. Our visual perception is biased because we do not perceive objects in isolation.(True)

29. Research suggests that when we do self-evaluations, our scores are consistently higher than the scores given us by our peers and superiors. This is an example of self-enhancement bias.

(True)

30. Jacqueline is a college freshman. She loves to sleep late on weekends and assumes that college students in general prefer to sleep late. Jacqueline is making a false consensus error.

(True)

31. A neat, professional appearance and a firm handshake are two important aspects of an interview because of the lasting impact first impressions make.

(True)

32. If Dan complains about his instructors being boring every time they lecture, he is demonstrating high consistency.

(True)

33. The USA, as a country, is considered to have high self-esteem.(True)

34. Individuals with an economic value orientation tend to make more unethical choices.

(True)

MULTIPLE CHOICE

Opening Section: Advice for Hiring Successful Employees: The Case of Guy Kawasaki

35. Guy Kawasaki is known for _________.a. Proving that replacing an employee is costly.b. believing that high turnover endangers customer service.c. being an entrepreneur.d. replacing employees is a time-consuming process.

(c) Easy/Comprehension

36. Which of the following statements about Guy Kawasaki is true?a. He is a best selling author.

Page 53: Fwk Bauer Tif Organizational Behavior All Chapters

b. He declared bankruptcy in 2010.c. He invented the first Apple computer.d. He thinks Organizational Behavior is unimportant.

(a) Easy/Comprehension

Section I: The Interactionist Perspective: The Role of Fit

37. The interactionist perspective suggests a. behavior is a function of the person and the situation interacting with each other.b. behavior is solely a function of the person’s personality.c. behavior is solely a function of the situation.d. behavior is not predictable and can not be profiled.

(a) Easy/Knowledge

38. Person/job fit is thea. degree to which a person’s skill, knowledge, abilities and other

characteristics match the job demands.b. degree to which a person likes an organization.c. degree to which a person’s values, personality, goals and other

characteristics match those of the organization.d. degree to which a person’s attitude matches the culture of the work group.

(a) Easy/Knowledge

39. Person/organization fit is the a. degree to which a person’s skill, knowledge, abilities and other

characteristics match the job demands.b. degree to which a person likes an organization.c. degree to which a person’s values, personality, goals and other

characteristics match those of the organization.d. degree to which a person’s attitude matches the culture of the work group.

(c) Easy/Knowledge

40. Sherri is creative, warm and caring. Person/job fit suggests one job she might be particularly effective at isa. elementary school teacher.b. accountant.c. researcher.d. pharmaceutical sales.

(a) Medium/Application

41. An applicant-matching software program would suggest which of the following personality traits would provide the best person/organization fit with a Wall Street firm?a. risk aversionb. risk-takingc. flexibilityd. stability

Page 54: Fwk Bauer Tif Organizational Behavior All Chapters

(b) Medium/Application

42. Research on the interactionist perspective suggestsa. there is a consistent positive relationship between person/organization fit

and job performance.b. if there is a misfit between an individual and the organization, there are no

extenuating circumstances that eliminate its impact.c. organization culture is equally important to all individuals.d. people who fit into their organization tend to be more satisfied with their jobs.

(d) Difficult/Evaluation

43. Findings of person/organization fit studies indicatea. when people fit their organization, they consistently perform better.b. when people fit their organization, they are less influential in the firm.c. when people fit their organization, they tend to leave an organization at

about the same rate as those who are organization misfits.d. when people fit their organization, they are more committed to the organization.

(d) Medium/Analysis

Section II: Individual Differences: Values and Personality

44. Values a. encompass the relatively stable feelings, thoughts, and behavioral

patterns a person has.b. are initial thoughts and perceptions we form about people, which tend to

be stable and resilient to contrary information.c. are generalizations based upon a perceived group characteristic.d. refer to stable life goals people have, reflecting what is most important to them.

(d) Easy/Knowledge

45. According to Rokeach, which of the following is not accurate about values?a. Values are grouped into terminal and instrumental categories.b. Values are hierarchically ordered. c. Values are shaped late in life.d. Values are relatively stable over time.

(c) Easy/Comprehension

46. __________ is a terminal value.a. Broad-minded.b. Inner harmony.c. Forgiving.d. Obedient.

(b) Easy/Knowledge

47. Which of the following is an instrumental value?a. cleanb. self-respectc. family security

Page 55: Fwk Bauer Tif Organizational Behavior All Chapters

d. a world of beauty(a) Easy/Knowledge

48. Removing your hat during the playing of The National Anthem in the United States is an example of the value of ________________.a. benevolence.b. security. c. tradition.d. universalism.

(c) Difficult/Application

49. Choosing a job that offers health care as part of its benefit package over a position that pays more but has no such health care might reflect the job seeker’s concern abouta. inner harmony.b. imaginative.c. family security.d. self-respect.

(c) Medium/Application

50. Which instrumental value might surveys find to be prevalent in Wall Street brokers?a. forgivenessb. ambitionc. obedienced. imagination

(b) Difficult/Application

51. Values change over the generations, evolving in response to historical contexts. Which of the following statements is true regarding values of different generations?a. Generation Xers (born between the mid-1960s and 1980s) are very

group-oriented.b. Generation Xers (born between the mid-1960s and 1980s) see work as

central in their lives.c. Generation Xers (born between the mid-1960s and 1980s) don’t care

what they do for work, they just want money. d. Generation Xers (born between the mid-1960s and 1980s) have to see a

congruence between personal goals and organizational ones before they work toward company goals.

(d) Medium/Evaluation

52. Which of the following statements about values is true?a. Values impact the individual, but not how they work.b. Values are not very stable; they change over short periods of time. c. Values affect the decisions people make, but have little impact on their behavior.d. Values are the result of accumulation of life experiences.

(d) Medium/Synthesis

Page 56: Fwk Bauer Tif Organizational Behavior All Chapters

53. Early experiences are important influences over dominant values. If Sharon was raised by an aunt because her parents had both died when she was ten, she most likely would see what value as central to her life?a. an exciting lifeb. securityc. self-respectd. forgiveness

(b) Medium/Evaluation

54. The relatively stable feelings, thoughts, and behavioral patterns a person has isa. values.b. personality.c. self-esteem.d. conscientiousness.

(b) Easy/Knowledge

55. Study findings on personality indicatea. we are more emotionally stable and more organized between the ages of

20 and 40.b. personality is the primary predictor of work behavior.c. although personality impacts us during our youth it has no lasting

consequences for us.d. although it is interesting information, employee personality has relatively

little impact on placing individuals in jobs.(a) Medium/Analysis

56. Maurice is described as “very curious. He has very original thoughts. He is a bright young man with a very creative flair.” Maurice exhibits the personality trait ofa. extraversion.b. agreeableness.c. conscientiousness.d. openness.

(d) Medium/Application

57. Lauren takes a personality survey and finds she strongly exhibits the trait of agreeableness. She might be described as beinga. organized, punctual, systematic and dependable.b. nice, tolerant, sensitive and kind.c. outgoing, talkative, and sociable.d. curious, creative and original.

(b) Easy/Comprehension

58. Jennifer complains to her friends every Friday night about her job. Each week she tells them she is going to leave her company even though she has been working there for five years. Jennifer is likely to bea. high in conscientiousness.b. high in neuroticism.c. low in agreeableness.d. high in extraversion.

(b) Difficult/Evaluation

Page 57: Fwk Bauer Tif Organizational Behavior All Chapters

59. __________ is the degree to which a person is anxious, irritable, aggressive and moody.a. Neuroticismb. Extraversionc. Conscientiousnessd. Openness

(a) Easy/Knowledge

60. People who exhibit the Big 5 personality characteristic of openness would be most likely to work in which of the following firms?a. an entrepreneurial start-up in the high tech industryb. a large discount retailer like Wal-Martc. a large fast food franchised. a small business that makes cardboard boxes

(a) Difficult/Synthesis

61. Across a variety of occupations and jobs, what is the one personality trait that most uniformly predicts how high a person’s performance will be?a. conscientiousnessb. extraversionc. agreeablenessd. openness

(a) Medium/Analysis

62. Which personality characteristic tends to lead to effectiveness in jobs with a lot of opportunities to interact with others?a. opennessb. conscientiousness.c. extraversiond. agreeableness

(c) Medium/Analysis

63. Which of the following statements about the personality trait agreeableness is FALSE?a. People high in agreeableness tend to exhibit strong helping behavior.b. All occupations look to hire people who are high in agreeableness.c. People high in agreeableness tend to be less effective at providing

constructive criticism.d. People high in agreeableness are very likeable and get along well with others.

(b) Medium/Analysis

64. Empathy is to agreeableness as sociability is to ______________.a. neuroticism.b. conscientiousness.c. openness.d. extraversion.

(d) Medium/Comprehension

Page 58: Fwk Bauer Tif Organizational Behavior All Chapters

65. Which of the following statements regarding the Big 5 personality traits is NOT true?a. Individuals high in extraversion are model employees.b. Individuals high in openness seek feedback on what they are doing and

thus adjust to their jobs more quickly.c. Conscientiousness is related to career success.d. Individuals high in neuroticism tend to create unfair work climates if they

become managers.(a) Difficult/Synthesis

66. Which of the following is NOT a dimension by which individuals are grouped in the Myers-Briggs Type Indicator (MBTI)?a. seeing/hearingb. thinking/feelingc. extraversion/intraversiond. judgment/perception

(a) Easy/Knowledge

67. In the Myers-Briggs Type Indicator, what dimension describes those who derive their energy from inside?a. intuitionb. perceptionc. introversiond. judgment

(c) Easy/Knowledge

68. According to the Myers-Briggs Type Indicator, what dimension describes those who would like to have clarity and closure?a. thinkingb. feelingc. intuitiond. judgment

(d) Easy/Knowledge

69. Which of the following statements about the Myers-Briggs Type Indicator is INCORRECT?a. The original objective of the MBTI was to aid World War II veterans in

identifying the occupation that would suit their personalities.b. The MBTI relies on types, not traits.c. The MBTI is a very effective tool to use in making selection decisions.d. The MBTI classifies people into one of sixteen types.

(c) Medium/Analysis

70. Which of the following statements is true with regard to research on individual differences?a. Teams dominated by positive affective people experience higher levels of

absenteeism.b. People with a low openness trait are more likely to start their own

business.c. Agreeable people are less likely to engage in change-oriented

communication.

Page 59: Fwk Bauer Tif Organizational Behavior All Chapters

d. Social monitors are rated as lower in performance.(c) Medium/Analysis

71. Negative affective peoplea. see “the glass as half full.”b. experience anxiety.c. are tolerant of others’ opinions.d. help their teams have lower rates of absenteeism.

(b) Medium/Comprehension

72. Which of the following statements about positive and negative affectivity is correct?a. Positive affective people see the glass as half empty.b. Negative affective people in power tend to find mutually agreeable

solutions to problems.c. Positive affective people experience nervousness.d. When negative affective people comprise a team, there tends to be less

cooperation among members.(d) Medium/Analysis

73. To work effectively with an individual with a high level of negative affectivity, a. recognize that you can change someone’s personality if you give feedback.b. let them vent without limits.c. ask them for specific examples of what the problem is.d. challenge them; they are seldom right about anything on which they voice

a complaint.(c) Medium/Evaluation

74. Self-monitoring isa. the degree to which a person has overall positive feelings about himself.b. the extent to which a person is capable of monitoring his/her actions and

appearance in social situations.c. the belief that one can perform a specific task successfully.d. a person’s inclination to fix what is perceived as wrong, change the status

quo, and use initiative to solve problems.(b) Easy/Knowledge

75. High social monitorsa. tend to be more successful in their careers.b. tend to experience lower levels of stress.c. are very accurate at conducting performance evaluations.d. are more committed to their companies.

(a) Medium/Comprehension

76. All of the following are accurate about those with a proactive personality EXCEPTa. they have higher levels of performance in their firms.b. they adjust to new jobs quickly.c. they may try to change things others are not willing to let go.d. they often have difficulty making friends.

Page 60: Fwk Bauer Tif Organizational Behavior All Chapters

(d) Medium/Analysis

77. Self-esteem isa. the belief that one can perform a specific task successfully.b. the degree to which a person has overall positive feelings about him or herself.c. the degree to which people feel accountable for their own behaviors.d. the inclination to change the status quo.

(b) Easy/Knowledge

78. Ned keeps saying to his coach, “I am really concerned that I won’t be able to remember the plays when you put me in. Maybe you should play someone else.” Ned seems to be demonstratinga. low openness to experience.b. low self-monitoring.c. a proactive personality.d. low self-efficacy.

(d) Medium/Evaluation

79. John said, “If I just hadn’t had that argument with my girlfriend last night, I wouldn’t have been so upset and I would have done much better on my finance exam.” John exhibits a. high internal locus of control.b. high self-esteem.c. high self-monitoring.d. high external locus of control.

(d) Medium/Synthesis

80. Individuals with an internal locus of controla. have an inclination to change the status quo.b. believe that what happens to them is their own doing.c. understand what the situation demands and act accordingly.d. feel that things happen to them because of luck.

(b) Medium/Synthesis

81. Which of the following statements about self-efficacy is INCORRECT?a. Self-efficacy is job specific.b. People with high self-efficacy tend to procrastinate.c. Self-efficacy is related to job performance.d. Training people to increase their self-efficacy can be effective.

(b) Difficult/Analysis

82. Marissa always seems to know who to talk to in her organization to get things done. When her immediate supervisor seemed less than enthusiastic about her proposal, she made sure to mention it to the manager in another department who shared it with his boss. Marissa was just approached about implementing her proposal. Marissa seems to have a. low self-efficacy.b. a proactive personality.c. low self-esteem.d. an internal locus of control.

Page 61: Fwk Bauer Tif Organizational Behavior All Chapters

(b) Medium/Synthesis

83. Which of the following is NOT a sound piece of advice to help build your self-confidence?a. Set challenging, not impossible goals.b. Ignore negative advice, especially when it comes from a negative person.c. Take a self-inventory.d. Act confident only when you feel confident.

(d) Medium/Evaluation

84. All of the following statements regarding personality testing in employee selection are correct EXCEPTa. job interviewers are very adept at assessing conscientiousness in

candidates, so the personality test is not really needed in employee selection.

b. there are mixed feelings as to whether candidates’ faking responses on personality tests is a serious problem.

c. self-reporting measures of personality may not be the best way to measure personality.

d. personality is a better predictor of job satisfaction than job performance.(a) Difficult/Synthesis

85. Among the limitations associated with using personality tests in employee selection isa. candidate inability to understand the purpose of the personality test.b. candidates faking their answers.c. candidates refusing to take personality tests.d. finding that performance cannot be predicted by personality testing.

(b) Difficult/Synthesis

Section III: Perception

86. When an incomplete figure appears on a sheet of paper, we are able to discern the entire figure because wea. have selective perception.b. extrapolate from the information available to us.c. visualize items in isolation.d. perceive our environment objectively.

(b) Difficult/Synthesis

87. Mike went to his friend Abe’s house with Nick. When the pair returned to Mike’s house, Nick said, “Mike, isn’t Abe’s cat the brightest orange cat you’ve ever seen?” Mike replied, “Nick to be honest, I didn’t notice anything but Abe’s dog. I’m afraid of dogs and wasn’t sure if he would bother me or not.” Mike exhibiteda. selective visual perception at Abe’s house.b. visual extrapolation at Abe’s house.c. focal contrast at Abe’s house.d. visual objectivity at Abe’s house.

(a) Difficult/Synthesis

Page 62: Fwk Bauer Tif Organizational Behavior All Chapters

88. Beth, Ramon and Nandini just walked out of the office where they delivered a mock commercial for a proposed client’s cereal. Beth says to her co-presenters, “I’m so sorry I messed up during the presentation. I really did not mean to put up the slide with last year’s cereal box on it instead of the one for this year. I know we won’t get the account now, and it’s all my fault.” Beth is exhibitinga. self-enhancement bias.b. false consensus error.c. self-effacement bias.d. stereotypes.

(c) Medium/Evaluation

89. Self-enhancement bias isa. the tendency for people to underestimate their performance, undervalue

capabilities, and see events in a way that puts them in a more negative light.

b. the tendency to overestimate our performance and capabilities and see ourselves in a more positive light than others see us.

c. the overestimation of how similar we are to other people.d. the process with which individuals detect and interpret environmental stimuli.

(b) Easy/Knowledge

90. The car was traveling very slowly on the highway in front of Ned. He turned to Sally and said, “This is ridiculous, we are never going to get there on time. That must be a woman driver in front of us!” Ned is engaging ina. false consensus error.b. self-fulfilling prophecy.c. stereotyping.d. first impressions.

(c) Easy/Analysis

91. The professor announced the first day of class: “Students are not good at meeting deadlines. I guarantee you that someone will email me the day after a due date and ask if I will accept a paper for partial credit.” Yesterday the first paper was due in the professor’s class. He just checked his email and a student inquired whether the paper could be turned in for partial credit today. This is an example ofa. self-fulfilling prophecy.b. self-enhancement bias.c. self-effacement bias.d. selective perception.

(a) Medium/Synthesis

92. Which of the following statements regarding first impressions is INCORRECT?a. First impressions are fleeting impressions.b. First impressions are resilient even in the face of contrary evidence.c. First impressions become independent of the evidence that created them.d. First impressions are stable impressions.

(a) Medium/Synthesis

93. When interviewing keep all of these pieces of advice in mind EXCEPT

Page 63: Fwk Bauer Tif Organizational Behavior All Chapters

a. it is not how you dress, but how confident you are that determines your success.b. have a professional-looking résumé.c. be on time for the interview.d. practice for the interview.

(a) Medium/Comprehension

94. Internal attributiona. is the belief that a behavior is caused by the internal characteristics of a person.b. is the causal explanation given for an observed behavior.c. explains someone’s behavior by referring to the situation.d. is the tendency to attribute failures to the situation while attributing

successes to internal causes.(a) Easy/Knowledge

95. Which of the following is not a key factor in understanding what kind of attributions we make?a. consistencyb. distinctivenessc. perceptiond. consensus

(c) Easy/Knowledge

96. Low distinctiveness is characterized bya. a person in a situation, acting the same way he always does in that situation.b. a person behaving differently than he usually does in different situations.c. a person behaving this way in different situations.d. everyone else behaving in the same way.

(c) Easy/Knowledge

Bill is the quarterback of his college football team. The team was 10-0, but lost this week after what Bill says was a very bad call by the referee. Bill seldom says anything about how the referees called any game, but he is very vocal about this one. You know that Bill works hard both on the field and off. You talk to other players on the team and they complain about the referee’s call also.

97. In the preceding story, both Bill and the other players expressed concern about the referee’s call. This demonstratesa. low distinctiveness.b. low consistency.c. high consistencyd. high consensus.

(d) Medium/Analysis

98. The fact that Bill seldom complains about the referees suggests he hasa. low distinctiveness.b. high distinctiveness.c. high consistency.d. low consensus.

Page 64: Fwk Bauer Tif Organizational Behavior All Chapters

(b) Medium/Evaluation

99. In this scenario, given what you know about the referee’s call, you could explain Bill’s behavior as a. internal attribution.b. external attribution.c. self-serving bias.d. self-fulfilling prophecy.

(b) Difficult/Synthesis

100. Which of the following statements about attribution is correct?a. Attributions depend only on consensus, distinctiveness and consistency.b. When faced with poor performance, one punishes the person more if we

make an external attribution.c. If someone’s failure is due to internal causes, we feel empathy and offer help.d. If we make an external attribution to someone’s goal achievement, we are

less likely to reward the individual.(d) Difficult/Synthesis

101. Internal attribution occurs witha. low consensus, high distinctiveness, and high consistency.b. high consensus, high distinctiveness, and high consistency.c. low consensus, low distinctiveness, and high consistency.d. high consensus, low distinctiveness, and high consistency.

(c) Medium/Analysis

Section IV: The Role of Ethics and National Culture

102. Which of the following statements regarding individual differences and ethics is true?a. People with an economic value orientation tend to make more ethical choices.b. Employees with internal locus of control make more unethical choices.c. We tend to underestimate how ethical we are.d. How we make attributions in a given situation will determine how we

respond to others’ actions.(d) Medium/Analysis

105. Hofstede found that countries differ on the equality or hierarchy among people or itsa. uncertainty avoidance.b. power distance.c. masculinity.d. individualism

(b) Easy/Knowledge

106. People from Europe and the United States have higher levels of ___________ than that of Asia and Africaa. introversionb. extraversion

Page 65: Fwk Bauer Tif Organizational Behavior All Chapters

c. consensusd. agreeableness

(b) Medium/Comprehension

107. Which of the following statements regarding the universality of personality traits is correct?a. There is no difference in the dominant personality trait in countries the

world over.b. In regions that suffered from infectious diseases, openness to

experiences is less dominant.c. The five-factor model of personality traits is not universally useful.d. The presence of democratic values in a culture is associated with

introversion.(b) Difficult/Evaluation

Closing Section: Using Science to Match Candidates to Jobs: The Case of Kronos

103. Firms in the retail industry like Kronos have an incentive to find strong candidates for all of the following reasons EXCEPTa. replacing an employee is costly.b. high turnover endangers customer service.c. happy workers are productive workers.d. replacing employees is a time-consuming process.

(c) Medium/Comprehension

104. Which of the following statements about applicant-matching software is true?a. The software indicates that the best profile for salespeople includes

personality traits like extraversion and sociability.b. Faking answers on the software program is fairly easy so the candidates

must be closely monitored as they respond to questions.c. The software makes inquiries as to applicants’ backgrounds and salary histories.d. Companies using the software report increases in turnover rates, but

substantially higher performance in the workplace.(c) Difficult/Analysis

FILL IN THE BLANK109. Matching candidates to jobs is a key way of ensuring ________ performance and

____________ turnover in the workplace.(high, low)

110. Experts suggest that ____________ answers to questions in applicant-matching software is difficult because the candidate can not predict the desired profile.

(faking)

111. The __________ perspective suggests behavior is a function of the person and the situation interacting with each other.

(interactionist)

Page 66: Fwk Bauer Tif Organizational Behavior All Chapters

112. When hiring individuals, companies are interested in two types of fit: __________________ and __________________.

(person/job, person/organization)

113. Maggie loves routine, repetitive tasks and is very risk averse. She loves working as a cashier. This is an example of person/_________ fit.

(job)

114. Mark is interviewing with a Wall Street firm. He notes that employees dress in business suits on a daily basis and address each other by Ms. and Mr. He decides that the firm’s culture is too conservative for him. Mark is concerned about person/_______________ fit.

(organization)

115. Important __________ affect the decisions people make, how they perceive their environment and their behavior.

(values)

116. ________ are stable life goals that people have, reflecting what is most important to them.

(Values)

117. Broad mindedness, obedience, forgiveness and imagination are examples of _________ values.

(instrumental)

118. End states that people desire in life such as a prosperous life are examples of ___________ values.

(terminal)

119. ___________ suggests that values are arranged in hierarchical order.(Rokeach)

120. ______________ is defined as relatively stable feelings, thoughts, and behavioral patterns a person has.

(Personality)

121. A person who is very curious, original, creative and very adaptable to change is high in ________________.

(openness)

122. ______________ is the one personality trait that uniformly predicts how high a person’s performance will be across a variety of jobs.

(Conscientiousness)

123. People with high __________ are often effective sales representatives.(extraversion)

124. Individuals high in ____________ are often effective managers and tend to demonstrate inspirational leadership behaviors.

Page 67: Fwk Bauer Tif Organizational Behavior All Chapters

(extraversion)

125. Individuals high in ___________ are nice, tolerant, kind and warm people who get along well with others and are valuable additions to any team.

(agreeableness)

126. A person who is high in ____________ are anxious, irritable, aggressive, temperamental, and moody.

(neuroticism)

127. The __________________ is an instrument that groups individuals by type, not trait, and is useful for team-building and training purposes.

(Myers-Briggs Type Indicator)

128. Stanley always sees “the glass as half empty.” Stanley is a ___________ person.(negative affective)

129. Gabrielle is a high _________ _________. She can walk into a meeting and quickly identify the mood and the key interactions between attendees and adapt her behavior accordingly.

(self-monitor)

130. Raul is highly respected by his colleagues. When there is a problem, he quickly takes the initiative to find a solution and tries to remove all barriers to success. Raul has a _____________ personality.

(proactive)

131. “I won’t get this job either. I’ve been on twenty interviews already and I haven’t received one callback. I don’t think I’ll ever get a job. I’m such a failure,” says the graduating college senior. The student is exhibiting low ______ ________.

(self-esteem)

132. People with high ____________set higher goals for themselves.(self-efficacy)

133. The basketball coach says, “This team has to control its own destiny. You cannot be looking to other teams to lose so you can get into the play-offs. Only we can get ourselves into the tournament. We have to take care of our own house.” The coach likely has ______________________.

(internal locus of control)

134. When people believe that things that happen to them happen because of luck, other people or a higher being, they are classified as high in _________ ___________ ____ ___________.

(external locus of control)

135. _______________ could be a serious problem when using personality tests in employee selection because it can change the rank ordering of candidates.

(Faking)

Page 68: Fwk Bauer Tif Organizational Behavior All Chapters

136. Harrison was glancing over the list of runners who finished the Boston Marathon and the names of the two runners from his hometown of Richmond, Virginia “jumped” out at him. The phenomenon is an example of the process of ______________.

(selective perception)

137. You are asked to predict how quickly you will run a mile track. You run on a regular basis to stay in shape. You say you will run the track in five minutes. It takes you eight minutes. Your prediction was an example of ___________ __________ bias.

(self-enhancement)

138. The assumption that whatever quirks we have are shared by many other people is ____________ __________ ___________.

(false consensus error)

139. You are a manager who believes that all young employees “goof off at work.” You therefore limit the number of assignments you give to your young employees. Lately you have noticed that the young employees in your department are texting on their cell phones, playing computer games and just talking to each other quite a bit. This situation is an example of a __________ ____________.

(self-fulfilling prophecy)

140. The dramatic drop in sales was discussed at the weekly company meeting. The manufacturing manager kept looking at the production-related aspects of the problem while the marketing manager concentrated on the promotional aspects. __________ ___________ explains why these two managers were focusing on their specific areas and ignoring others.

(Selective perception)

141. Since ____________ _____________ are lasting, make sure your résumé is neat in appearance and error free. When interviewing, dress appropriately and greet the interviewer with a firm handshake.

(first impressions)

142. Three key factors in understanding attribution are ___________, _____________, and ______________.

(consensus, distinctiveness, consistency)

143. Agnes failed her accounting exam and complains that the exam was too difficult. Marta just ignores her because Agnes did not study at all in the three weeks prior to the exam. Marta is making an __________ for Agnes’s failure.

(internal attribution)

SHORT ANSWERS

144. Name and describe the two types of fit companies are interested in assessing when hiring employees, and describe the relationship between each type of fit and work behaviors.

Page 69: Fwk Bauer Tif Organizational Behavior All Chapters

The first type of fit is person/job fit. This is the degree to which a person’s skills, knowledge, abilities and other characteristics match the job demands. Person/organization, the second type of fit, is the degree to which a person’s values, personality, goals and other characteristics match those of the organization. When a person fits his job, there is greater satisfaction with the work environment and better job performance. Further, when there is fit between a person and the organization where he works, he tends to be more committed to his organization and remains at the firm longer. Clearly turnover is costly, so anything that helps avoid it is beneficial to a firm.

145. What is the difference between a terminal value and an instrumental value? What impact does an assessment of employee values have upon a firm?

A terminal value is an end state people desire in life, such as leading a prosperous life. An instrumental value, in contrast, is a view on an acceptable mode of conduct such as being ethical. Values that are important to people impact their decisions and behaviors. Further, values are stable aspects of a person’s life that influence their perspective on an ongoing basis. Thus, it would be very beneficial for a firm to recognize its employees’ core values to discern the nature of their behavior in the workplace and to determine whether the job and/or organization provides an opportunity to satisfy the dominant values of the employees.

146. Briefly discuss each of the five personality traits and the usefulness each has for personnel selection.

Openness is the degree to which a person is curious, original, intellectual, creative and open to new ideas. People high in openness thrive in jobs and positions where they can be flexible and learn new things.

Conscientiousness is the degree to which a person is organized, systematic, punctual, achievement-oriented and dependable. High conscientiousness is the trait most desired by recruiters as it uniformly predicts how high a person’s performance on the job will be.

Extraversion is the degree to which a person is outgoing, talkative, sociable, and enjoys being in social situations. These individuals are particularly effective in jobs involving sales.

Agreeableness is the degree to which a person is nice, tolerant, sensitive, trusting, kind and warm. Agreeable people help others work consistently; thus they are good team members.

Neuroticism is the degree to which a person is anxious, irritable, aggressive, temperamental and moody. These individuals experience a variety of problems at work so identification of them in the interview will help a firm avoid the relationship issues they create as employees.

147. You work with Ben, one of the most negative individuals you have ever met. How can you work more effectively with him?

Page 70: Fwk Bauer Tif Organizational Behavior All Chapters

Begin by recognizing the fact that you are not going to change him. A person’s personality is rather stable so criticizing his negativity will not change it.

Be open-minded. Just because someone is negative does not mean he is not right.

Set a time limit for your conversations with the negative person. Don’t let them consume you.

Empower the person to act on the things he complains about. Ask him what he would do to fix the problematic situation.

Ask for specifics. Let him provide details of the problem.

148. Explain the difference between self-esteem and self -efficacy.

Self-esteem is the overall positive feelings a person holds about himself or herself. Self-efficacy, in contrast, is the belief that a person can perform a task. Self-esteem is a more universal or overall assessment of one’s beliefs while self-efficacy focuses on beliefs related to performance.

149. Horace completed a locus of control survey in his organizational behavior class. The results indicate he has a high internal locus of control. What might he read about the trait in the profile of high locus of control individuals?

High internal locus of control individuals believe they control their own destiny and what happens to them is their own doing. Internals tend to act in ways that increase their chances of success. Thus, they initiate relationships with mentors, become more involved at work and demonstrate higher levels of motivation.

150. Differentiate between self-enhancement bias and self-effacement bias.

Self-enhancement bias is the tendency for people to overestimate their performance and capabilities and see themselves in a more positive light than others see them. This scenario occurs quite frequently in self-assessment in performance evaluations as individuals, in general, rate themselves higher than either their peers or superiors do.

Self-effacement bias is the tendency for people to underestimate their performance and see events that put them in a more negative light.

151. Define the concept of stereotypes and give an example.

A stereotype is a generalization based upon group characteristics. The stereotype can be positive, negative or neutral. An example of a stereotype is that women talk too much and that men never ask for directions. Our stereotypes can influence the nature of our communication, decision making and other processes.

152. What is a self-fulfilling prophecy?

Page 71: Fwk Bauer Tif Organizational Behavior All Chapters

A self-fulfilling prophecy is when an established stereotype causes one to behave in a certain way, which leads another party to behave in a way that makes the stereotype come true. In a classic example of the self-fulfilling prophecy, an third-grade elementary school teacher is told by a second-grade teacher that the students she is sending into his third-grade class next year are very poor readers. The following year, the third-grade teacher teaches reading to the students from the standpoint of being poor readers. When the scores for their first reading test of third-grade are examined, they indicate the students are poor readers. The self-fulfilling prophecy has been fulfilled.

ESSAY

153. You are a counselor in the career placement office at your university. Jackie, a business student, recently completed a personality survey. You are reviewing the results and find she has low self-esteem and low- to mid-range self-efficacy. Sketch out some possible steps Jackie can begin to undertake to improve both of these personality dimensions.

Individuals with low self-esteem have high levels of self-doubt and frequently question their overall self-worth. Often those with low self-esteem try to become invisible. Thus, Jackie may largely keep to herself at the university, not being part of any clubs or social groups. As a counselor, you might want to encourage her to find a club related to her major and join it, initially attending meetings as a listener and working up to a more participative role. Also, Jackie should attend events on campus where she can meet others and begin to forge ties. Low self-esteem individuals need positive feedback to begin the process of enhancing their self-worth. Putting herself in the aforementioned situations will help Jackie begin to get such feedback. Initial positive feedback from club meetings may be things as simple as smiles of recognition from other club members for regular attendance or acknowledgements of Jackie by those club members when they see her on campus. Later, short conversations will develop both in meetings and on campus as the concept of self-worth will build.

Low self-efficacy is the belief that you are unable to perform a specific task successfully. Improving one’s level of self-efficacy begins by placing yourself in the situation where you will have the opportunity to perform the task. Then, set a simple achievable goal for that task. Congratulate yourself on its achievement and set the goal higher the next time you encounter the situation. In Jackie’s case, she may find that she is struggling with finance, as the low grade on her first test indicates. Instead of dropping the course, she might consider a change in study habits, committing herself to reviewing material each day after class and visiting her professor for assistance during office hours. The professor will encourage her as she successfully completes problem sets. Jackie could then set a goal of an average grade on the next exam; as that is achieved, she can continue to place herself in situations where her skills can be honed and tested.

154. You are a member of a human resource department in a large corporation. One of your colleagues has recently developed a personality test he wants to use along with the cognitive abilities test you have been using to select employees for

Page 72: Fwk Bauer Tif Organizational Behavior All Chapters

positions in the firm. He has come to you for help in deciding how to validate the test. What recommendations would you give him?

Validation involves ensuring that the test measures what it is intended to measure and looks for the relationship (correlation) between the predictor (the personality test) and some standard or measures of success on the job (criteria). You might, therefore, suggest that the personality test be given to current employees. From the group of current employees tested, you would assess who are your best performers and who are not quite as effective. You then would examine the results of the personality test and look for relationships between the personality profiles and the individual employee’s performance level. For example, do you have an employee who has been rated a very high performer over the last few years in the firm and whose personality profile indicates him to be high in conscientiousness, agreeableness and low in neuroticism? If the answer is yes, then you could assume the personality test has some validity because the research suggests that the traits uncovered tend to produce better performers. Similarly, if the personality test indicated a second worker, a manager, was high in neuroticism and you note that his performance appraisal results suggest his subordinates complain that he does not treat them fairly, you can again assume the personality test is valid. It predicted what it was supposed to predict.

You also must examine the personality test to ensure it is not discriminating in any manner against any of the groups who are legally protected by their characteristics including race, color, sex, religion, national origin, age, and disabilities.

155. Should personality tests be used in the employee selection process?

Though turnover in the job is expensive, and enhancing the person/organization and person/job fit is important, companies should exercise caution in using personality tests in selection, as the research on their validity and reliability is mixed. Some experts believe that personality tests predict performance and other criteria such as job satisfaction and thus advocate their usage. Others believe that the nature of the personality tests themselves allow for the job candidate to fake his/her answers and that personality may not actually be such a good predictor of performance anyway. Some studies indicate that personality predicts only 10-15% of the variation in job performance. In short, companies should avoid relying upon personality tests alone in making their selection decisions, but if used in concert with other selection techniques including cognitive ability tests, personality tests may improve the selection decision.

156. You are a career development counselor at your university. You have been instructed by your supervisor to develop a program called, “Tips for Your First Interview.” What are some of the tips you could provide students as they prepare for that first interview?

It is imperative that students recognize the power of first impressions. It must be emphasized multiple times in the presentation that first impressions are lasting impressions.

Page 73: Fwk Bauer Tif Organizational Behavior All Chapters

Before ever meeting with a recruiter, students must recognize that their résumés are extensions of themselves and make impressions just as physical meetings make impressions. Therefore, the program should feature a section on resumes focusing on the necessity of a neat appearance to the document and the criticality of ensuring no errors are found on that resume. Further, cover letters should be addressed to a specific individual, not “To Whom It May Concern.”

The program should also highlight the pre-interview preparation as important for establishing a positive first impression. Time should be spent talking about how to dress for success, including mention of jewelry, piercings and hair style.

The interviewee must be punctual to the interview. In point of fact, being 10 minutes early is not problematic. If the interviewee is unfamiliar with the location of the firm at which he is interviewing or is unfamiliar with the nature of the traffic in that area, a “dry run” is advised to ensure making the interview in a timely manner when scheduled.

Finally, the initial moments of the actual interview should be addressed. It is imperative that students begin their session with a firm handshake and a smile for the interviewer. The program should feature practice in shaking hands. Though it may seem to be a silly exercise to some students, practice is important to avoid awkward moments.

The more aspects of the initial interactions between interviewer and interviewee the student is prepared for, the more successful the interview will be.

157. Mario is a ten-year employee of a large manufacturing firm where he is the cost accountant. Over his ten-year tenure, Mario has been an outstanding employee. In fact, twice during that time period, Mario was named “Employee of the Year.” However, over the past four months, Mario’s manager, Marielle, has begun to notice changes in his behavior. Interactions with Mario are often unpleasant. Last week Marielle told Mario about a new project that will begin in one month for which he was named team leader. He did not take the news well, complaining bitterly to Marielle about the amount of work that he had, the extra hours that he is expected to put in with no addition in pay, the nature of the individuals on the teams of which he is a part, and just about everything else happening in the firm. Yesterday, Marielle asked him a question about a project that is to be completed by Friday of this week and received the same lengthy complaints. She are stunned by this behavior since no one else voiced such opinions before, and is attempting to discern its root cause so that Mario can return to his former levels of performance. Marielle overheard two junior level accountants talking about Mario’s son being in trouble at school and also heard in the cafeteria last week that Mario’s wife is suffering from a prolonged illness. Using attribution theory, help Marielle provide a preliminary explanation for the behavior as she prepares to counsel Mario.

Mario is behaving in a manner that is very different from his norm, and in fact, is very different than most of the employees in the firm. The constant complaining in the face of relative calm from the other employees suggests Mario is exhibiting low consensus.

Page 74: Fwk Bauer Tif Organizational Behavior All Chapters

In ten years, Mario has always been an outstanding performer and never complained about anything. The fact that it is occurring now suggests that high distinctiveness is being exhibited.

Finally, every time Marielle speaks to Mario about his work, he complains. Since the two have always had a very pleasant relationship before and since Marielle is beginning to hear of potential issues at home, she may make a preliminary assessment that the personal family issues are beginning to spill over into the workplace for Mario. This is an external attribution and Marielle can now decide exactly how to address these delicate issues in a meeting to offer some support and counseling.

158. What value is added to a firm in knowing the nature of personality traits in other cultures?

Diversity in the workforce is a competitive advantage in today’s dynamic, global marketplace. If a firm has an appreciation for market niches available due to the differences in culture, language, sexual orientation, gender, etc., they can address those customers much more effectively by drawing on the diverse nature of its own workforce. Further, since securing a person/job and person/organization fit facilitates performance, and personality helps determine that fit, understanding the nature of personality traits in other cultures is very important.

For example, while some countries seem to have a majority of extraverts, as in the United States and Europe, Asia and Africa do not share the same traits. Clearly the nature of sales and marketing will differ in these countries based upon these traits. Similarly, being cognizant of a country’s culture will assist you in competing more effectively in that market.

Page 75: Fwk Bauer Tif Organizational Behavior All Chapters

Organizational Behavior, Version 1.1Bauer & Erdogan

FWK Test Item FileChapter 4

TRUE/FALSE

1. SAS Institute advocates its employees create a work/life balance by suggesting, “If you are working more than 8 hours, you are just adding bugs.”

(True)

2. SAS Institute is a pioneer in perks given by the company with the philosophy that the firm should “offer everything but the kitchen sink” to maintain employee satisfaction.

(False)

3. Attitudes are more strongly related to behaviors than to intentions to behave.(False)

4. Behavior is influenced by attitudes and situations.(True)

5. Based on the number of studies conducted, job satisfaction is the most important job attitude.

(True)

6. There is a high overlap between job satisfaction and organizational commitment.(True)

7. Job satisfaction can be fully explained by the work environment.(False)

8. Maria just completed a series of personality assessments and is found to have the traits of external locus of control and introversion. Given the results of studies on attitude, it is likely she also has positive work attitudes.

(False)

9. Person/job fit and person/organization fit are positively related to job satisfaction.(True)

10. People with positive affective dispositions tend to be more satisfied with their jobs.

(True)

11. The job characteristics of autonomy and high skill variety are related to organizational commitment.

Page 76: Fwk Bauer Tif Organizational Behavior All Chapters

(True)

12. Realistic job previews help prevent breaches of the psychological contract.(True)

13. Employees are happier at work if they are socially accepted in their work groups.(True)

14. All stress on the job is bad.(False)

15. The concept of always putting work life first is outdated.(True)

16. A happy worker is always a productive worker.(False)

17. The relationship between positive work attitude and behavior is clear.(False)

18. Natalia is assisting new employee Jane by showing her around the facility. This is an example of organizational citizenship behavior.

(True)

19. If you don’t like your job, it is likely you will reduce your performance.(False)

20. A departing employee’s manager is the best person to conduct the exit interview.(False)

21. The success of work attitude surveys depends on the perceived credibility of management by employees.

(True)

22. The most powerful influence on job performance is general mental ability.(True)

23. The importance of higher general mental ability for high performance is stronger for manual labor positions than management positions.

(False)

24. When an employee is treated well, he wants to reciprocate, so he performs his job more effectively.

(True)

25. General mental ability is likely the most important factor explaining employee citizenship behaviors.

(False)

26. Young people exhibit higher citizenship behaviors than older people.(False)

Page 77: Fwk Bauer Tif Organizational Behavior All Chapters

27. Firms with “sick leave” policies tend to have more individuals with unscheduled absences than those with “personal day off” policies.

(True)

28. Age is positively related to both frequency and duration of absenteeism.(False)

29. Personality is the main reason why individuals turnover in a firm.(False)

30. In organizations that have a pay for performance system, overall turnover level will be lower.

(False)

31. Whistleblowers, or people who report wrongdoing, tend to be those who have very high levels of commitment to a company.

(False)

32. Work/family conflict lowers job satisfaction in collectivistic cultures.(False)

33. In China, absence from work due to illness is considered relatively unacceptable.(True)

MULTIPLE CHOICE

Opening Section: People Come First: The Case of SAS

34. SAS Institute has been ranked annually as “one of the best places to work.” Among the reasons for such a ranking is the “perks” SAS offers its employees including all of the following EXCEPTa. on-site health care for employees and covered dependents.b. a full gym and swimming pool.c. five-hour work days.d. racquetball and basketball courts on-site.

(c) Easy/Knowledge

35. SAS Institute’s commitment to its employees is demonstrated through all of the following programs EXCEPTa. employee opportunity to work on interesting and challenging projects.b. regular administration of employee satisfaction surveys.c. pioneering of unique and effective perks for employees.d. paying employees below market salaries but offering additional perks.

(d) Medium/Application

Section I: Work Attitudes

36. An attitude is

Page 78: Fwk Bauer Tif Organizational Behavior All Chapters

a. our opinions, beliefs and feelings about our environment.b. an outcome of our efforts.c. the drive that energizes an individual.d. the degree to which people have freedom to decide how to act.

(a) Easy/Knowledge

37. If one looks at the number of studies conducted on attitudes, which of the following seems to be the most important?a. organizational commitmentb. job satisfactionc. satisfaction with supervisorsd. cognitive ability

(b) Easy/Comprehension

38. Job satisfaction isa. the feelings people have toward their jobs.b. the emotional attachment people have toward the company they work for.c. an unwritten understanding about what the employee will bring to the

work environment.d. also known as emotional intelligence.

(a) Easy/Knowledge

39. Organizational commitment isa. the feelings people have toward their jobs.b. the emotional attachment people have toward the company they work for.c. an unwritten understanding about what the employee will bring to the

work environment.d. also known as emotional intelligence.

(b) Easy/Knowledge

40. Which of the following statements is true?a. There is an exceptionally strong correlation between attitudes and

behavior such that attitudes will almost perfectly predict behavior.b. Intention to behave and actual behavior are almost always the same.c. Attitude provides clues to behavior, but situational factors also impact

behavior.d. Attitude is weakly related to intention to behave and actual behavior.

(c) Difficult/Evaluation

41. Which of the following statements regarding employee happiness at work is FALSE?a. Personality is related to one’s happiness at work.b. Develop good relationships at work.c. A fit between you and the company is important.d. Pay is the most important aspect of job satisfaction.

(d) Medium/Analysis

42. Happiness at work can be enhanced bya. pay at all times; pay is always the key aspect to job satisfaction.b. reaction to organizational life. Work is stressful, just “suck it up.”c. pulling yourself away from your co-workers and concentrating on the job.

Page 79: Fwk Bauer Tif Organizational Behavior All Chapters

d. knowing yourself and what you want from the job.(d) Medium/Evaluation

43. Research shows that all of the following impact the employee’s satisfaction with the job and commitment to the organization EXCEPTa. how he or she is treated in the firm.b. the actual work the employee performs.c. the firm’s competitiveness.d. the relationships formed with colleagues and managers.

(c) Medium/Knowledge

44. All of the following factors contribute to job satisfaction EXCEPTa. personality.b. job characteristics.c. work/life balance.d. firm competitiveness in the market.

(d) Easy/Knowledge

45. All of the following factors contribute to organizational commitment EXCEPTa. personality.b. job characteristics.c. firm competitiveness in the market.d. work/life balance.

(c) Easy/Knowledge

46. Employees with which of the following personality traits would experience high job satisfaction?a. those with positive affective dispositionsb. those with neurotic personalitiesc. those with low self-esteemd. those who are introverted

(a) Difficult/Synthesis

47. Which of the following statements is true?a. External locus of control and introversion result in positive work attitudes.b. Low self-esteem results in more positive work attitudes.c. People who are critical of themselves experience less job satisfaction.d. People with positive affective dispositions are less committed to their

work organizations.(c) Difficult/Synthesis

Page 80: Fwk Bauer Tif Organizational Behavior All Chapters

48. Research on job characteristics indicate that a. people with a minimal variety of skills are always less satisfied with their

job.b. people with autonomy on their job are always more satisfied with their job.c. all people want feedback on their jobs.d. people who have high growth need tend to expect their jobs to help them

build new skills.(d) Difficult/Evaluation

49. Research on relationships at work indicates all of the following EXCEPTa. whether we are treated with respect is related to our commitment to the

company.b. the development of a trust-based relationship with the manager is related

to organizational commitment.c. how considerate our manager is impacts our job satisfaction, but not our

organizational commitment.d. our level of social acceptance by our coworkers impacts our

organizational commitment.(c) Difficult/Evaluation

50. All of the following stressors lead to lower job satisfaction and organizational commitment EXCEPT:a. role conflict.b. high responsibility at work.c. organizational politics.d. job insecurity.

(b) Easy/Knowledge

51. Which of the following statements about work attitude causes is accurate?a. Job satisfaction is purely environmental.b. Person/job fit is positively related to job satisfaction.c. Psychological contract breach leads to high job satisfaction.d. Organizational justice results in psychological contract breach.

(b) Difficult/Synthesis

52. Which of the following statements about work attitude causes is true?a. role ambiguity can lead to job dissatisfaction.b. negative affective disposition leads to higher organizational commitment.c. stress always leads to job dissatisfaction.d. using too many skills on the job usually leads to job satisfaction because

it increases stress.(a) Difficult/Synthesis

53. Which of the following is related to lower levels of organizational commitment?a. negative affective dispositionb. maintenance of the psychological contract between worker and firmc. perceived fairness of company policiesd. manager consideration of our opinions and suggestions

(a) Difficult/Synthesis

Page 81: Fwk Bauer Tif Organizational Behavior All Chapters

54. Which of the following statements is true regarding the causes of positive work attitudes?a. The notion of work/life balance is outdated.b. All stressors are bad.c. Job/person fit is positively related to job satisfaction.d. The level of coworker acceptance of an employee has no impact on the

employee’s commitment to the organization.(c) Difficult/Synthesis

55. Which of the following statements regarding work attitude and performance is FALSE?a. Those with the most positive work attitudes are always strong performers.b. In engineering, there tends to be a stronger link between work attitude

and performance than one finds in an assembly line job.c. The correlation between commitment and performance is not strong.d. Your skill level in performing a job matters.

(a) Medium/Analysis

56. Indicate which of the following statements is true.a. A happy worker is always a productive worker.b. Work attitude is related to organizational citizenship behaviors.c. Attitude is the most important factor in performance.d. Work attitude has no impact on customer satisfaction.

(b) Difficult/Synthesis

57. Organizational citizenship behaviors include all of the following EXCEPTa. working voluntary overtime.b. assisting new employees.c. speaking positively about the organization.d. performing the duties in one’s job description.

(d) Medium/Comprehension

58. Which of the following is accurate for the relationship between attitudes and performance?a. though you want to perform better, your skill set may prevent that from

occurring.b. though you want to perform better, the equipment used at work may not

be functioning properly.c. though you want to perform better, too much may be outside your control.d. All of the above.

(d) Medium/Comprehension

Page 82: Fwk Bauer Tif Organizational Behavior All Chapters

59. Which of the following statements regarding work attitudes and job performance is accurate?a. Work attitudes are more strongly related to job performance than they are

to organizational citizenship behaviors.b. If you want to perform better, you will actually perform better.c. Workforce satisfaction has an impact on profitability.d. People who are happy at work are more satisfied in their overall life too.

(d) Difficult/Synthesis

60. Workforce satisfaction is related to all the following outcomes EXCEPTa. greater customer loyalty.b. more absenteeism.c. higher profitability.d. fewer accidents in the workplace.

(b) Difficult/Analysis

61. Companies track employee work attitudes througha. performance evaluation interviews.b. attitude surveys.c. employee suggestions.d. overhearing workplace conversations.

(b) Easy/Knowledge

62. Attitude surveys conducted by companies produce more useful information ifa. surveys are completed electronically. b. the management is perceived as credible by employees.c. past surveys were conducted but no action has resulted from them.d. individual survey responses are made public.

(b) Easy/Knowledge

63. Which of the following statements about exit interviews is true?a. A departing employee’s manager should always conduct the exit

interview because the immediate supervisor knew the employee best.b. An exit interview always indicates why an employee is departing a firm.c. An exit interview always indicates how management can improve so other

employees will not leave.d. An exit interview should be conducted by a member of the human

resource department for best results.(d) Medium/Comprehension

Page 83: Fwk Bauer Tif Organizational Behavior All Chapters

64. When analyzing the information on work attitudes and aspects of the work environment, which of the following statements is true?a. Attitudes always predict behavior in the work environment.b. People with negative work attitudes never perform well.c. Personality and organizational citizenship are the two key attitudes that

are the most relevant to important outcomes in the work environment.d. Maintenance of the psychological contract helps develop organizational

commitment.(d) Difficult/Synthesis

Section II: Work Behaviors

65. All of the following are general mental abilities EXCEPTa. reasoning skills.b. analytical skills.c. emotional intelligence.d. verbal skills.

(c) Easy/Knowledge

66. Which of the following factors is a major predictor of job performance?a. work/life balance issuesb. employee age c. employee personalityd. how we are treated at work

(d) Medium/Analysis

67. Which of the following factors is NOT a major predictor of organizational citizenship behaviors?a. general mental abilitiesb. positive work attitudesc. how we are treated at workd. personality

(a) Easy/Knowledge

68. Which of the following statements regarding job performance is INCORRECT?a. Measures of job performance include the speed and accuracy with which

the job is performed.b. One of the powerful influences on job performance is how we are treated

at work.c. Contrary to what most people would think, stress has very little impact on

job performance.d. Work attitudes, like job satisfaction, are moderate correlates of job

performance.(c) Medium/Analysis

69. Research indicates which of the following factor is the strongest predictor of job performance?a. general mental abilityb. work attitudes

Page 84: Fwk Bauer Tif Organizational Behavior All Chapters

c. stressd. positive work attitudes

(a) Medium/Analysis

70. Research indicates which of the following statements regarding factors that predict job performance is FALSE?a. When we are treated well, we want to reciprocate, thus when the

company treats us well, we perform more effectively.b. Positive work attitudes and performance are unrelated.c. Conscientious employees perform better than those who are not

conscientious.d. High role ambiguity, and the resulting stress, can eventually lead to lower

performance.(b) Medium/Analysis

71. Research on the relationship between personality and job performance indicatesa. people who are extraverted perform slightly less well than those who are

intraverted.b. dependable people are better performers than those less dependable.c. there is little relationship between reliability and job performance.d. achievement-oriented people are so goal-oriented that they often fall

short in their performance.(b) Easy/Comprehension

72. Research on work attitudes and performance indicates a. work attitudes are strong correlates of job performance.b. work attitudes are a stronger correlate of performance in complex jobs

than in simpler jobs.c. dissatisfied employees always reduce performance level.d. work attitudes are a moderate correlate of performance.

(d) Medium/Analysis

73. Which of the following factors is not a predictor of organizational citizenship behaviors?a. abilitiesb. personalityc. positive work attitudesd. age of the employee

(a) Easy/Knowledge

74. Which of the following statements regarding the predictors of organizational citizenship behaviors is FALSE?a. The link between ability and citizenship behavior is stronger than the link

between motivation and citizenship behavior.b. The most important factor in determining employee citizenship behavior is

how employees are treated by those around them.c. People with positive affectivity tend to exhibit stronger citizenship

behaviors.d. Older people tend to exhibit stronger citizenship behaviors than younger

people.(a) Difficult/Analysis

Page 85: Fwk Bauer Tif Organizational Behavior All Chapters

75. Which of the following scenarios is likely to produce low levels of citizenship behaviors?a. Maria is an agreeable, conscientious employee of a large conglomerate.b. Randall’s boss is extremely family-oriented and recently allowed him to

leave work early to attend his son’s after-school soccer game.c. Horatio is a young engineer with negative affectivity.d. Jennifer is a devoted employee who is happy at her work.

(c) Difficult/Synthesis

76. Which statement regarding citizenship behaviors is accurate?a. People who are older demonstrate higher levels of citizenship.b. People who are unhappy still perform citizenship behaviors to a very high

degree.c. Job attitudes are weakly related to citizenship behaviors.d. People who are treated poorly tend to perform higher levels of citizenship

behaviors.(a) Medium/Comprehension

77. Which of the following factors is not a cause of high levels of absenteeism in a firm?a. positive work attitudesb. health problemsc. age of the workerd. work/life balance issues

(a) Easy/Comprehension

78. Which of the following statements regarding absenteeism is INCORRECT?a. Instituting organizational policies that penalize absenteeism are

reasonable approaches to controlling the problem.b. Wellness programs are a viable way to address employee absenteeism.c. Flexibility in work scheduling addresses absenteeism concerns.d. There is no consistent link between personality and absenteeism.

(a) Medium/Analysis

79. Research on absenteeism indicates which of the following statements to be accurate?a. Problems in job design can result in absenteeism.b. Older individuals, because they are more likely to have health problems,

are more likely to be absent from work.c. Employees who are permitted to manage their work schedules with

regard to time at the company are more likely to be absent.d. Absenteeism saves organizations money because the company does not

have to pay employees who are absent from their jobs. (a) Medium/Analysis

80. Which of the following statements regarding ways to deal with late coworkers is INCORRECT?a. Try ignoring lateness and not imposing negative consequences.b. Be on time yourself!c. Schedule meetings around everyone else’s meetings.

Page 86: Fwk Bauer Tif Organizational Behavior All Chapters

d. Find out exactly why employees are late.(a) Easy/Comprehension

81. Which of the following employees is more likely to turnover?a. those who have been in the company for a long time. b. those who have negative work attitudesc. those who have a low deal of stressd. those who are high performers.

(b) Easy/Knowledge

82. Research on turnover indicates which of the following statements is FALSE?a. Unhappy employees are more likely to leave than those who are satisfied

with the organization.b. Poor work attitudes cause intentions to leave, which often causes actually

leaving the company. c. All turnover is bad for an organization.d. Agreeable and conscientious people are less likely to quit their jobs.

(c) Medium/Comprehension

83. Research suggests which of the following relationships between performance and turnover?a. Pay for performance systems have no impact on an employee’s likelihood

to turnover.b. The establishment of a pay for performance system in a company

guarantees that high performers will stay in the firm.c. In general, good performers are more likely to leave a firm because other

firms readily recognize their skills and actively seek their employment.d. Poor performers are more likely to quit their jobs compared to high

performers.(d) Medium/Comprehension

84. The primary reason for employee turnover isa. gender.b. work attitudes.c. age.d. general mental abilities.

(b) Medium/Knowledge

85. Which of the following statements regarding age and turnover is FALSE?a. Programming opportunities offered to new hires, such as orientation, can

reduce the turnover of this group.b. Younger individuals are more likely to turnover than older individuals.c. Individuals who have been on the job only a short period of time are

unlikely to turnover because they just got the job.d. Younger individuals have an easier time, overall, leaving a job than older

individuals.(c) Medium/Analysis

86. Which of the following statements regarding turnover is true?a. The link between work attitudes and turnover is direct.

Page 87: Fwk Bauer Tif Organizational Behavior All Chapters

b. Even though they are very unhappy with a job, employees may not leave an organization.

c. Employees will quit their jobs voluntarily only when they feel they will get fired.

d. Manager absenteeism causes employee turnover.(b) Medium/Analysis

87. Which of the following is NOT a tip for leaving your job gracefully?a. The world is a small place; don’t let tales of your unprofessional behavior

travel.b. Don’t badmouth your employer; why make others feel bad?c. When you decide to leave, leave. Don’t hang around doing work you will

not be doing a very good job on.d. Return all office supplies, keys, and anything else that belongs to the

company.(c) Medium/Comprehension

Section III: The Role of Ethics and National Culture

88. When an ethical climate exists at a firm, studies suggest all of the following EXCEPTa. people are less likely to want to leave the firm.b. people are likely to only perform the duties of their job description

because they do not want to engage in anything that will upset the balance of the ethical situation in place.

c. people are happier at work.d. people are more committed to the firm.

(b) Medium/Comprehension

89. Which of the following is an issue to keep in mind with regard to maintaining an ethical environment in the firm?a. Monitoring employee online activity is a good way of creating an ethical

climate. b. Use of video cameras in the workplace will make employees feel the firm

cares about ethics.c. High levels of employee monitoring increases citizenship behaviors.d. Monitoring employees may increase ethical compliance but may create a

culture of mistrust. (d) Difficult/Analysis

90. Which of the following statements regarding ethical behaviors in the workplace is true?a. High levels of employee monitoring increase employee commitment to

the organization.b. High levels of employee monitoring increase work performance.c. High levels of employee monitoring increase citizenship behaviors.d. High levels of employee monitoring may reduce the frequency of

unethical behaviors.(d) Difficult/Analysis

Page 88: Fwk Bauer Tif Organizational Behavior All Chapters

91. Which of the following statements regarding job attitudes and cultural differences is true?a. Work/family conflict lowers job satisfaction in collectivistic cultures.b. High levels of empowerment are related to high job satisfaction in India.c. Work/family conflict lowers job satisfaction in individualistic cultures.d. Low levels of empowerment are related to high job satisfaction in the

USA.(c) Difficult/Analysis

Closing Section: Rebounding from Defeat: The Case of Jeffrey Katzenberg

92. Jeffrey Katzenberg has ________.a. been successful at everything he has tried.b. been CEO of Starbucks.c. been CEO of DreamWorks Animation.d. is the CEO of Disney.

(c) Easy/Comprehension

FILL IN THE BLANK

93. An _______ is an opinion, belief or feeling about an aspect of the environment.(attitude)

94. The feelings people have toward their job is called ______ ___________.(job satisfaction)

95. __________ ____________ is the emotional attachment people have toward the company they work for.

(Organizational commitment)

96. The tendency to experience positive moods more often than negative ones is _________ _____________ ______________.

(positive affective disposition)

97. When __________ match job demands, we tend to be more satisfied with our job.

(abilities)

98. An unwritten understanding about what the employee will bring to the work environment and what the company will provide in exchange is called a ______________ ______________.

(psychological contract)

99. _______ ____________ is vagueness as to what job responsibilities are in the job.

(Role ambiguity)

100. Helping new employees and working voluntary overtime are examples of ______________ _____________ ____________.

Page 89: Fwk Bauer Tif Organizational Behavior All Chapters

(organizational citizenship behaviors)

101. _______ ___________ are meetings with departing employees.(Exit interviews)

102. The worst person to conduct an exit interview is the _________ __________ __________.

(departing employee’s manager)

103. ___________ ______________ and _____________ _____________ are two key attitudes that are the most relevant to important outcomes in the work environment.

(Job satisfaction, organizational commitment)

104. ____ ___________ is the performance level on factors included in the job description.

(Job performance)

105. The most powerful influence over job performance is ______ __________ _________.

(general mental ability)

106. _________________ include reasoning abilities, verbal and numerical skills, and analytical skills.

(General mental abilities)

107. Jobs with high complexity need to have employees with high ___________.(general mental abilities)

108. Predictors of _____________ include general mental abilities, how we are treated at work, stress, and positive work attitudes.

(job performance)

109. Predictors of _______________ include how we are treated at work, personality, positive work attitudes and age of the employee.

(citizenship behaviors)

110. Predictors of __________include health problems, work/life balance issues, poor work attitudes and age of the employee.

(absenteeism)

111. Predictors of _____________include poor performance, poor work attitudes, stress, personality, and age of the employee.

(turnover)

112. With regard to age, ___________ individuals are better citizens.(older)

113. .__________ is unscheduled absences from work.(Absenteeism)

Page 90: Fwk Bauer Tif Organizational Behavior All Chapters

114. With regard to age, ________ employees are less likely to be absent from work.(older)

115. ___________ is an employee leaving an organization.(Turnover)

SHORT ANSWERS

116. Briefly discuss some of the reasons why SAS Institute is ranked on Fortune’s “Best Places to Work” list.

SAS Institute successfully manages its employees. The company shows its employees that it cares, and in return, the employees are devoted to the firm. Turnover at SAS is less than 5%.

The firm maintains its positive relationship with its employees by providing them opportunities to work on interesting and challenging projects. In working on those projects, barriers are removed by the firm so all the employee has to do is work on the project; no worries about equipment, rules, etc.

In addition, the perks offered to the employees are innovative and exciting. There are company athletic fields, a pool, gym memberships, tennis and basketball courts, on-site health care and the demand that employees go home to their families after a full 8-hour day.

117. Briefly describe which two job attitudes have the greatest potential to influence behavior.

The two attitudes with the greatest behavioral influence are job satisfaction and organization commitment. Job satisfaction is the feelings people have toward their job and organization commitment is the emotional attachment people have toward the company. These two work attitudes are closely monitored by companies because they are associated with performance, citizenship, absenteeism, and turnover outcomes.

118. Name and define two work attitudes.

Job satisfaction is one work attitude. Job satisfaction is the feelings people have toward their job.

Organizational commitment is a second work attitude. Organizational commitment is the emotional attachment people have toward the company they work for.

119. What is the relationship between attitude and behavior?

Much work has been done on the attitude to behavior link. The strength of this link depends on the particular attitude focused upon. Attitude can influence behavior but not necessarily predict it. In fact, research indicates that attitudes are more strongly related to intentions to behave than actual behaviors. Thus

Page 91: Fwk Bauer Tif Organizational Behavior All Chapters

attitudes provide hints as to how employees will behave, but behavior is also strongly influenced by situational constraints.

120. List three factors that contribute to job satisfaction.

The following factors have been shown to contribute to job satisfaction: personality, person/environment fit, job characteristics, psychological contract, organizational justice, work relationships, stress and work/life balance.

121. List three factors which contribute to organizational commitment.

The following factors have been shown to contribute to organizational commitment: personality, person/environment fit, job characteristics, psychological contract, organizational justice, work relationships, stress and work/life balance.

122. Differentiate between positive and negative affective disposition.

Positive affective disposition is when an individual experiences positive moods more often than negative moods. Negative affective disposition is when an individual experiences more negative moods than positive moods.

123. Describe two specific personality traits related to positive work attitudes.

Research has examined a number of personality traits in relationship to positive work attitudes. They include: affective disposition, neurotic personality, emotional stability, conscientiousness, self-esteem, locus of control and extraversion.

124. What is meant by person/job fit?

Person/job fit is the fit between what the employee brings to the work environment and the environmental demands of that workplace. For example, when employee abilities fit the job demands, employee satisfaction can result.

125. Give an example of how a psychological contract between an employee and a firm could be breached.

When employees do not get what they expect from a company, a psychological contract can be breached. A psychological contract is an unwritten understanding about what the employee will bring to the work environment and what the company will provide in exchange. For example, if you were told during your initial selection interviews with a firm that the firm was family friendly, but now the boss is wanting you to be available 24/7 and is calling you on your company-issued cell phone at odd hours of the evening, you feel like the contract has been breached and you are sometimes, even often, dissatisfied.

126. Briefly describe how stress can be related to high levels of satisfaction.

Stress can be a two-way street. If role ambiguity is experienced (vagueness in relation to what one’s responsibilities are), role conflict (facing contradictory

Page 92: Fwk Bauer Tif Organizational Behavior All Chapters

demands at work), organizational politics or lack of job security, satisfaction can be severely reduced. However, some stress can make us happy. Some people work best under deadlines, even tight deadlines. Further some people like having a great deal of responsibility and the stress which accompanies that simply makes them better at meeting the challenge.

127. Amanda is a new mother. Briefly describe two programs that Alcoa, her employer, might implement to attempt to create some work/life balance for Amanda and others like her.

Alcoa might be able to offer Amanda a flextime schedule where she would need to be in the office for certain hours or on certain days, but could work her schedule out around those core hours as her personal needs dictate as long as she fulfills the hour requirements for the position as a whole.

Again, depending on the specific job she performs, Amanda may be able to work out a telecommuting arrangement with the firm.

Work sharing is another option where Amanda and some other employee will share the one position, dividing up hours, pay, or benefits according to the number of hours they work each year.

128. What is the value of an exit interview for a firm?

Exit interviews are conducted with employees departing a firm. The intent behind them is to find out why the employee is leaving. The challenge is to make sure the employee is being “honest” with his reason for leaving. For example, if the interview is conducted by the employee’s immediate manager, the reason for leaving could actually involve that individual and to have him asking the questions would not probably gather the kinds of information needed to make appropriate changes. Turnover is very expensive for a firm, both in lost productivity and in the need to engage in another recruiting effort, so every opportunity a firm has to keep turnover low should be addressed.

129. What influence does general mental ability have over job performance?

Research indicates that general mental ability is the most powerful influence on our job performance. Reasoning abilities, verbal and numerical skills, analytical skills and overall intelligence seems to be important across most situations. From the time we are young throughout our later life, the correlation between mental ability and performance is strong.

130. Why does how an employee is treated in a firm predict citizenship behavior?

The motivation/behavior link with regard to citizenship behavior is very clear. We tend to do for others as they do for us. So, if we are treated well by those around us, we, in turn, what to return the favor and thus engage in citizenship behaviors.

Page 93: Fwk Bauer Tif Organizational Behavior All Chapters

131. John is a 57-year-old employee at XYZ, Inc. Describe the expectations we might have with regard to the likelihood that he will be absent quite frequently from work.

Since John is older, the expectation is that he will be sick more often and thus will be absent more often. Surprisingly, however, research suggests that age and absenteeism are inversely related. That is, older individuals tend to be absent less than younger employees. Some of the reasons given for this finding include the loyalty older individuals tend to hold for their firms, and their stronger work ethic.

132. Give an example of when turnover is a problem and when turnover is beneficial to a firm.

Turnover is a problem when there is no one to perform a job and the job is critical. Of course, this issue can be further compounded by the difficulty experienced by the firm in filling the open position. Add to that the need to train an employee and wait until he “gets up to speed” on the job and the impact can be quite negative.

On the other hand, if you have an employee who simply is not performing well and may, in fact, be reducing the level of performance of those around him, then the turnover of the poor performer becomes a positive opportunity for the firm.

133. Describe the impact pay for performance systems could have on the employees in a firm.

Pay for performance systems can have a positive impact on a firm. First, the opportunity to enhance pay is afforded in these systems to those who perform well. Therefore, high performers will be paid well. Those who are not good performers may hasten their leave-taking from the firm because they are being paid less. However, in the pay for performance system, you only get paid less if you perform poorly. So poor performers are leaving, and that may not be all bad!

134. Describe the link between work attitudes and turnover.

The link between work attitudes and turnover is actually fairly simple. If workers are unhappy, they will leave. That said, the linkage between work attitudes and turnover is not direct. Instead, those who are unhappy with a firm have an intention to leave, but whether they will actually act on that intention is a separate issue.

135. Discuss the impact stress has on turnover in a firm.

Stress can cause employees to leave a firm. If the position held by an employee seems to have role ambiguity and role conflict as part of it, then employees are more likely to leave the firm.

136. Which personality traits tend to be exhibited by those employees who are less likely to turnover?

Page 94: Fwk Bauer Tif Organizational Behavior All Chapters

Employees who are conscientious, agreeable and emotionally stable are less likely to quit their jobs. Why? Some suggest that these personality traits may assist the employee in performing better at work and thus, they quit less. Further, these characteristics may lead them to develop better relationships with coworkers, which again is a factor in retention.

137. Tyler is a 22-year-old, recent graduate of a large university. Given the research on age and turnover, what are some expectations a manager may have with regard to Tyler’s likelihood of remaining with the firm?

Research tends to suggest that younger employees are more likely to leave organizations. Young employees have fewer responsibilities and may be much more able to quit a job they don’t like. Since young employees experience much more stress there is little bonding them to the firm and they again elect to leave the company. Anything the manager can do to address these issues might help to retain his young recruit.

138. Discuss the findings of studies examining job attitudes and ethical work climates.

Studies examining ethical work climates find that when organizations value doing the right thing, employees tend to be happier at work, are more committed to the firm, and are less likely to leave.

139. Discuss the dilemma companies face in trying to prevent their employees from behaving unethically by monitoring employees.

Preventing unethical behavior in employees can take many forms. One such option is to monitor employees through the use of video cameras and e-mail scanning. Engaging in such monitoring activities, however, leads employees to feel like they are being treated unfairly and are being disrespected. The catch-22 arises that if you increase the level of monitoring in the firm, you do decrease the incidence of unethical behavior, but you will likely decrease job satisfaction and commitment to the firm as well as job performance.

140. Briefly describe differences in job attitudes held around the world.

A study looking at 20 countries found that only in individualistic cultures did work/family conflict lower job satisfaction. In collectivist cultures, the bonds created between coworkers counter the sacrifices they make to work, so they are not dissatisfied.

Autonomy and empowerment are clearly valued in the United States, but empowerment, when instituted in India, led to lower job satisfaction.

141. Briefly discuss how cultures around the world influence work behaviors.

In the United States, helping an individual learn his new job is regarded as a citizenship behavior. In other cultures, it is simply seen as part of the job itself.

Page 95: Fwk Bauer Tif Organizational Behavior All Chapters

Similarly, to be absent from work in China for illness, stress or depression is unacceptable. In Canada, unacceptable reasons for absence from work include family reasons.

ESSAY

142. Is a happy worker a productive worker?

This question has been the focus of substantial amounts of research over the years and the answer is still not clear. Clearly happy workers seem to want to be better performers as their happiness makes them more engaged workers. However, wanting to perform better and actually performing better are not necessarily the same.

One aspect which could prevent a happy worker from being a more productive worker may simply be that the worker lacks the necessary skills to perform better.

And sometimes, too, the happy worker is so busy being happy that he fails to perform. While many programs instituted by firms use this statement as their guiding principle, there is no hard evidence to prove that a fruitful endeavor.

143. Rosaline is the superintendent of a school district. A new board of directors, to whom she is responsible, has just been elected and the philosophical direction the board wishes to take varies from the direction Rosaline has taken the district over the past five years. Rosaline is going to tender her resignation. How can she do so gracefully?

After working for five years to make your vision a reality, the likely urge is to vent your frustrations as loudly as you can. However, such an action, while refreshing for maybe five minutes, simply does not serve either party well in the future. Thus, Rosaline should consider the following:

Don’t engage in a knee-jerk reaction. Everyone has bad days and an entirely new philosophy may appear to be a mountain that you are unwilling to climb. That said, think of your career. Before tendering the resignation, look to see if there are opportunities for job change. Current economic conditions do not bode well for rash decisions. Find an alternative and then carefully plan your subsequent steps toward it.

When you do choose to go, inform your boss (the board president) of your decision first. While dramatic announcements get attention, again they serve few down the road.

Don’t speak poorly of the organization you are departing. The world is a small place and getting smaller. You have no clue what the future will hold or who you might need to contact in the future.

Finish what you have started. Don’t leave anyone scrambling to figure out what you are doing or how to do it. Just finish it.

Page 96: Fwk Bauer Tif Organizational Behavior All Chapters

Return what is the firm’s, keep what is yours.

Be professional in everything you do.

144. Agnes is so frustrated. This morning she called a meeting for 9 o’clock sharp to discuss the new product launch. Though most of the key players were on time, Kim and Nelson were both 25 minutes late, making some excuses about how they had to make an important call before either could attend. That made four meetings in a row when they were late. What can you do? Other members at the meeting are getting frustrated too as you have to repeat information and re-explain key points.

First, find out why they are late. Are they late because they are unhappy or dissatisfied? Or, are they simply late because it is their nature to be so?

Don’t let the continued tardiness go unnoticed and “unpunished.” Ignoring the behavior may be saying to them that you somehow approve of what they are doing.

Find out what everyone’s schedules are. Maybe you really are scheduling meetings when they have standing phone appointments.

If they are late, require them to make up for their lateness by taking on some extra duties or responsibilities related to the project.

Remind people of the meeting. Sometimes we are just so busy in today’s dynamic work environment that we do forget commitments.

Reward those who do come on time. Food works well. Barring that, just thank everyone for being there on time!

Make sure you are on time. If you have a tendency to call a meeting for 9 but it never gets started to 9:10 because you are out copying the agenda for everyone, then they will arrive late to compensate for your tardiness. Before you know it, a 9 o’clock meeting begins at 9:30.

Page 97: Fwk Bauer Tif Organizational Behavior All Chapters

145. Matthew is a 22-year-old accountant working in his first job at a major accounting firm in Philadelphia. When he is home in Virginia for a visit, he sits down for a chat with his father, who is a human resource director at a large manufacturing firm. Matthew tells his father how unhappy he is that he chose to work for the Philadelphia firm. As he does his job, he realizes this is not what he wants to do for the rest of his life. What advice can dad provide him to help him be a happier worker?

First, be positive. Any experience is a learning experience. You are never stuck forever anywhere.

If, indeed, the fit between you and the firm or you and the job is not what it should be, then find out what is wrong with the fit. Use this information as you begin a job search. Don’t however, have a knee-jerk reaction and just quit.

Get accurate information about the firm and the job. Maybe you are in a job that you just don’t fit, but maybe that is only a temporary assignment with better things to come.

Develop good relationships at work. The work relationships are part of your network and you never know when your network will come in handy.

Know when to leave. Gather information and weigh it carefully. Make sure the next organization or job is a good fit.

Page 98: Fwk Bauer Tif Organizational Behavior All Chapters

Organizational Behavior, Version 1.1Bauer & Erdogan

FWK Test Item FileChapter 5

TRUE/FALSE

1. Motivation is a function of the interaction between performance, ability and environment.

(False)

2. Job security is an example of Maslow’s safety needs.(True)

3. Alderfer’s existence needs correspond to Maslow’s physiological and safety needs.

(True)

4. The frustration-regression hypothesis offered by Alderfer suggests individuals frustrated in their attempts to satisfy one need regress to another.

(True)

5. Salary is a motivator in Herzberg’s two-factor theory.(False)

6. Hygiene factors are part of the context of the job.(True)

7. The absence of hygiene factors in the work environment leads to worker dissatisfaction.

(True)

8. Individuals with a high need for achievement may neglect managerial activities like coaching as a waste of time.

(True)

9. A high need for power is always a destructive element in the workplace.(False)

10. Process-based theories view motivation as a rational process.(True)

11. One way to deal with perceived inequity is to decrease your own inputs.(True)

12. Equity theory suggests that those perceiving inequity can react to the situation by having the referent decrease inputs.

Page 99: Fwk Bauer Tif Organizational Behavior All Chapters

(False)

13. Over-rewarded individuals experience a great deal of guilt and substantially increase their effort to restore their feelings of equity.

(False)

14. Benevolents expect to receive substantial compensation for relatively little input.(False)

15. You worked really hard on your paper and were very proud to get an A on it. Then you found out that everyone who wrote ten pages or more got an A and now you are upset. You are experiencing high distributive justice.

(False)

16. One effective way of achieving procedural justice is to allow employees a voice in decision making.

(True)

17. High levels of justice create higher levels of employee commitment.(True)

18. People perceived as fair recognize that what they believe is fair may differ from what others perceive as fair.

(True)

19. Expectancy theory’s instrumentality dimension refers to the degree to which one believes that effort leads to performance.

(False)

20. To influence instrumentality, make sure rewards are seen as desirable.(False)

21. Expectancy theory is a need-based theory.(False)

22. The phenomenon of “the folly of rewarding A while hoping for B” suggests that at times, people are rewarded for the wrong kind of behavior.

(True)

23. Reinforcement theory is based on behavioral and operant conditioning.(True)

24. To reduce the frequency of negative behaviors, remove the rewards that followed those unwanted behaviors.

(True)

25. Extinction is increasing the frequency of desirable behaviors.(False)

26. Using the extinction approach to get a coworker to quit using inappropriate jokes during his conversations means you ignore the jokes instead of laughing at them.

Page 100: Fwk Bauer Tif Organizational Behavior All Chapters

(True)

27. Giving an employee a sales commission every time he makes a sale is an example of a continuous reinforcement schedule.

(True)

28. Michael needs to discipline one of his employees. Before doing so, he should make sure the punishment he is administering fits the crime.

(True)

29. Motivation is culturally based.(True)

30. Being among the best paid workers in the retail industry is the sole reason why employees at Trader Joe’s are so motivated.

(False)

MULTIPLE CHOICE

Opening Case: A Motivating Place to Work: The Case of Zappos

31. Analyzing Zappos success using Herzberg’s two-factor theory of motivation would suggest thata. contextual factors, like being among the best paid employees in the retail

industry, are the only reason for the firm’s success.b. content factors, like the collaborative atmosphere each employee creates

are the only reason for the firm’s success.c. growth needs, like the autonomy in customer service, are the only reason

for the firm’s success.d. Contextual, content, and growth needs factors all help to contribute to

Trader Joe’s success. (d) Easy/Comprehension

Section I: Need-Based Theories of Motivation

32. Maslow’s physiological needs may be satisfied by:a. food.b. feeling danger-free.c. feelings of importance.d. human bonds.

(a) Easy/Knowledge

33. How could a firm best satisfy an employee’s safety needs?a. Enroll the employee in a Microsoft Office training course.b. Provide an employee appreciation luncheon.c. Develop a weekly employee bowling league.d. Create a job security clause in the employment contract.

(d) Medium/Application

34. Esteem needs are

Page 101: Fwk Bauer Tif Organizational Behavior All Chapters

a. the need to bond with other human beings.b. the desire to form lasting attachments.c. the desire to feel important.d. the freedom from an uncertain future.

(c) Easy/Knowledge

35. One way to satisfy self-actualization needs is to a. pursue a master’s degree through a company’s tuition reimbursement

program.b. go out to dinner once a month with coworkers from your department.c. get a $100 suggestion bonus from your firm for a labor cost saving tip you

placed in your department’s suggestion box.d. receive an employee of the week award.

(a) Difficult/Synthesis

36. Maslow’s theory of motivation suggestsa. employee needs are ordered.b. employee needs are grouped in three categories.c. employee needs are divided into motivator and hygiene categories.d. employee needs are acquired as the result of life experiences.

(a) Easy/Comprehension

37. Maslow’s hierarchy of needs theory of motivationa. suggests once basic needs are satisfied, higher order needs become

motivators.b. enjoys strong research support.c. is a systematic approach managers can utilize to explain the similar

needs employees have at any given time and the similar reactions of employees to similar treatment.

d. was developed based on Maslow’s observations of individual employees in a variety of workplace settings.

(a) Difficult/Comprehension

38. The collaborative work environment at Trader Joe’s satisfies which of Maslow’s hierarchy of needs?a. safetyb. self-actualizationc. sociald. physiological

(c) Medium/Analysis

39. The simple premise of Maslow’s hierarchy of needs theory isa. more than one human need operates at one time.b. work environment aspects that satisfy an individual are very different than

those aspects which dissatisfy an individual.c. individuals acquire three types of needs as a result of their life

experiences.d. human beings have needs that are hierarchically ranked.

(d) Medium/Comprehension

40. Maslow’s theory of motivation is criticized because

Page 102: Fwk Bauer Tif Organizational Behavior All Chapters

a. research does not support the argument that satisfied needs no longer motivate behavior.

b. its “frustration-regression” hypothesis has no practical implications for the workplace.

c. the research methodology employed does not fully account for the attribution of cause to the individual or the situation.

d. substantial empirical research has indicated little practical application of the theory.

(a) Difficult/Evaluation

41. Maslow’s social needs could be satisfied bya. receiving an employee of the month award.b. getting promoted to assistant manager from a supervisor position.c. attending a department picnic after work.d. taking a class on Microsoft Office.

(c) Medium/Application

42. Your paycheck could likely satisfy which of the following needs identified by Maslow?a. physiological needs.b. esteem needs.c. safety needs.d. all of the above.

(d) Medium/Analysis

43. Alderfer’s existence needs correspond to Maslow’s a. social needs.b. esteem needs.c. self-actualization needs.d. safety needs.

(d) Medium/Synthesis

44. Alderfer’s growth needs correspond to Maslow’sa. social needs.b. safety needs.c. esteem needs.d. physiological needs.

(c) Medium/Synthesis

45. In comparing Maslow’s hierarchy of needs theory to Alderfer’s ERG theorya. ERG theory recognizes more than one need may operate at one time.b. Maslow’s theory recognizes more than one need may operate at one

time.c. ERG theory is based upon observations of individuals in clinical settings.d. Maslow’s theory suggests that individuals who are frustrated in their

attempts to satisfy one need may regress to another one.(a) Difficult/Evaluation

46. Alderfer’s theory of motivation does all of the following EXCEPTa. relax the basic assumptions of Maslow’s theory of needsb. assume needs are hierarchically-based

Page 103: Fwk Bauer Tif Organizational Behavior All Chapters

c. propose a “frustration-regression” hypothesisd. assume needs are grouped into three categories

(b) Medium/Comprehension

47. According to ERG theory, if an individual is frustrated in the growth opportunities of his job,a. he may regress only to relatedness needs.b. he may regress only to existence needs.c. he may regress to either relatedness or existence needs.d. he will be motivated by the next higher need in his need hierarchy.

(c) Difficult/Analysis

48. Which of the following is not a hygiene factor in Herzberg’s theory of motivation?a. working conditionsb. salaryc. company policyd. recognition

(d) Easy/Knowledge

49. Which of the following is a hygiene factor in Herzberg’s theory of motivation?a. promotion opportunities.b. interesting work.c. salary.d. achievement.

(c) Easy/Knowledge

50. All of the following are criticisms of Herzberg’s theory of motivation EXCEPTa. the primary research methodology of the study is flawed.b. the classification of factors as hygiene or motivator is not simple.c. pay may have symbolic value as communication that the employee is

advancing within the company.d. the theory suggests that improving the environment only goes so far in

motivating employees.(d) Difficult/Evaluation

51. Which of the following statements is true?a. A need for achievement can be a destructive element in relationships with

colleagues.b. A high need for affiliation serves as a real advantage in managerial

positions especially with regard to conducting performance appraisals.c. A high need for power is a trait of those successful in the sales area.d. A need for power can have an altruistic form.

(d) Difficult/Analysis

52. Individuals with a high need for affiliationa. want to influence others.b. have a strong need to be successful.c. desire to make an impact on the organization.d. want to be accepted by others.

(d) Easy/Comprehension

Page 104: Fwk Bauer Tif Organizational Behavior All Chapters

53. Abdul tends to emphasize harmonious interpersonal relationships, prefers to be with friends, and enjoys family events. Abdul would probably be categorized as beinga. low in need for achievement.b. high in need for affiliation.c. high in need for power.d. low in need for affiliation.

(b) Difficult/Synthesis

54. What job would be best suited for someone with a high need for affiliation?a. salesb. teacherc. engineerd. entrepreneur

(b) Medium/Comprehension

55. Michele is being considered for a managerial position at Halliburton. She completes a TAT as part of an assessment center exercise for prospective managers. Facilitators will examine the results carefully for their manager choices knowing thata. individuals high in need for achievement may find it difficult to delegate

authority to subordinates.b. individuals high in need for affiliation are especially effective at providing

critical feedback to employees.c. individuals high in need for power have difficulties disciplining poor

performers.d. individuals high in need for achievement are especially effective at

coaching and meeting with subordinates.(a) Difficult/Evaluation

56. Which of the following is a process based theory of motivation?a. Alderfer’s ERG theory.b. Herzberg’s two-factor theory.c. Skinner’s reinforcement theory.d. McClelland’s acquired needs theory.

(c) Easy/Comprehension

57. Which of the following need-based theories has received the greatest amount of support?a. Herzberg’s two-factor theoryb. Maslow’s hierarchy of needsc. Alderfer’s ERG theoryd. McClelland’s acquired needs theory

(d) Easy/Comprehension

Section II: Process-Based Theories

58. Chloe and Francois are graphic designers at Highland Lithographics. Both have the same job description. Earlier this month, Chloe found out that Francois makes $12.50 an hour while she makes $11.00. What theory of motivation best explains why Chloe went to her boss to complain about the pay disparity?

Page 105: Fwk Bauer Tif Organizational Behavior All Chapters

a. ERG theoryb. Equity theoryc. Expectancy theoryd. Reinforcement theory

(b) Medium/Comprehension

59. Ramone feels that he was unfairly passed over for a promotion at his firm. How might he deal with the perceived inequity?a. Have the referent decrease inputs.b. Maintain current perceptions of the referent’s inputs.c. Increase his inputs.d. Increase his outcomes.

(d) Medium/Application

60. All of the following are actions those who perceive inequity can undertake to “balance the scales” EXCEPTa. steal.b. begin to arrive at work late.c. quit.d. produce higher quality work.

(d) Difficult/Synthesis

61. Research on equity theory suggestsa. those who perceive inequity increase their work performance.b. those who feel over-rewarded experience little distress over the situation.c. those who feel over-rewarded experience guilt over the situation.d. those who perceive inequity encourage their fellow coworkers to work

less hard.(b) Difficult/Analysis

62. Individuals who expect yearly wage increases without exhibiting increased performance may be labeleda. benevolents.b. entitleds.c. equitably insensitive.d. expectancy sensitive.

(b) Medium/Application

63. Research on procedural justice suggestsa. it is best to exclude employees from commenting on potential changes in

company procedures like performance appraisals because they do not understand the decision-making process anyway.

b. it is best to unexpectedly announce department-wide layoffs because when employees suspect changes in their job status are upcoming, they tend to become much less productive.

c. it is best to be consistent in the manner in which employees are disciplined for violating company policy for situations like cell phone usage.

d. it is best to send an announcement of changes to the company benefit package for the next fiscal year without providing explanatory details because employees do not understand the financial implications anyway.

Page 106: Fwk Bauer Tif Organizational Behavior All Chapters

(c) Difficult/Application

64. Interactional justice is a. the degree to which outcomes received from the organization are

perceived to be fair.b. the degree to which fair decision-making procedures are used to arrive at

a decision.c. the degree to which people are treated with respect, kindness and dignity

in interpersonal interactions.d. the belief that high levels of effort will lead to high performance.

(c) Easy/Knowledge

65. Nejah is reading a report on employee justice perception. The article notes all of the following points EXCEPTa. high levels of justice create higher levels of organizational citizenship.b. high levels of justice create higher levels of employee commitment to the

organization.c. high levels of justice create higher levels of customer satisfaction.d. high levels of justice lead to support of unionization.

(d) Medium/Comprehension

66. Joaquin received the Chancellor’s Award and an $8,000 scholarship for a research paper and formal presentation he created. He believes that he worked hard for the paper and the presentation. Joaquin would feel a sense ofa. high interactional justice.b. high interpersonal justice.c. high distributive justice.d. high procedural justice.

(c) Medium/Application

67. If you wanted to be perceived as a fair person, take this piece of advice:a. “People are selfish, they care only about their own justice level.”b. “Treat everyone equally when distributing rewards. Differentiation in

rewards often lead to unfairness.”c. “Make your own decisions, you are the boss. Since you are accountable

for those decisions, you do not need to waste time listening to everyone’s opinions.”

d. “Think before you speak. Be considerate of others.”(d) Difficult/Comprehension

68. Michael has two finals, accounting and organizational behavior, on the same day of finals week. Based on expectancy theory, which of the following statements suggest how he should allocate his limited study time?a. For the previous three OB exams Michael studied for hours and received

below average grades. He spent fewer hours studying for accounting and earned above average grades. He therefore decides to focus his efforts equally on OB and accounting.

b. Accounting is Michael’s major, OB is a general business requirement in his academic program. Michael aspires to be a CPA, so he decides to focus his study time on Accounting.

Page 107: Fwk Bauer Tif Organizational Behavior All Chapters

c. Michael is switching to another college for a communication major after this semester. His business courses will not count toward his total credits at the new college. His grades on these exams will not matter, but he decides to focus on his accounting anyway.

d. Michael loved the group work format that his OB instructor used to teach the course and disliked the lecture format utilized by his accounting professor. He decides to focus his attention on OB.

(b) Difficult/Synthesis

69. You work for your college’s annual alumni fund. You have averaged 25 phone calls in a three-hour shift throughout your employment. Your supervisor now demands that you increase that number to 40 calls per three-hour shift. You feel that you do not have the skills or the time to make 40 calls; this is an unrealistic goal. According to expectancy theory, you are experiencinga. Low expectancyb. Low instrumentalityc. Low valenced. Low equity

(a) Difficult/Evaluation

70. Which of the following statements is true about the manner in which managers can influence the perceptions of expectancy theory?a. Individuals with an internal locus of control and high self-esteem believe

there is little connection between their effort and performance so a great deal of feedback is needed to motivate them.

b. Regardless of the performance rating given on a performance evaluation, make certain that everyone gets a small raise annually.

c. Recognize good performance of individuals with cost effective items like coffee mugs and t-shirts.

d. Conduct employee surveys periodically to ascertain what they consider to be rewards for performance and integrate that in some manner into the performance evaluation program.

(d) Medium/Analysis

71. In expectancy theory, valence isa. the perception that high levels of effort will lead to outcomes of interest

like performance.b. the perception that performance is related to subsequent outcomes such

as rewards.c. the anticipated satisfaction that will result from an outcome.d. the degree to which people are treated with respect, kindness, and

dignity.(c) Easy/Knowledge

72. To influence instrumentality, managers cana. ensure that environment facilitates performance.b. give employees choice over rewards.c. consistently reward high performers.d. encourage people to believe their effort makes a difference.

(c) Medium/Application

Page 108: Fwk Bauer Tif Organizational Behavior All Chapters

73. Steve Kerr’s phenomenon, “the folly of rewarding A while hoping for B” is demonstrated bya. a manager failing to acknowledge an employee who stayed after hours

for one week to complete a report for an ill coworker.b. a manager who fears his top sales person will leave if he criticizes him for

being 15 minutes late every day.c. a manager who promotes a subordinate known for his propensity to argue

with his coworkers so that subordinate can be transferred to another department where an individual just retired.

d. rewarding a manager who orders the release of a shipment of manufactured parts full of defects to meet a deadline in spite of the company’s highly advertised quality commitment.

(d) Difficult/Application

74. Negative reinforcement a. decreases the desired behavior.b. decreases the frequency of negative behavior.c. increases the desired behavior.d. reduces the frequency of undesired behavior.

(c) Easy/Knowledge

75. Positive reinforcementa. decreases the desired behavior.b. decreases the frequency of negative behavior.c. increases the desired behavior.d. reduces the frequency of undesired behavior.

(c) Easy/Knowledge

76. An example of extinction isa. nagging an employee to call a customer.b. suspending an employee for being late three days in a row.c. praising an employee for a well written report.d. ignoring a coworker who is telling dirty jokes.

(d) Medium/Comprehension

77. When reinforcers follow all examples of positive behavior you are using a __________ ___________ of reinforcement.a. variable ratiob. equity ratioc. continuous scheduled. fixed ratio schedule

(c) Medium/Comprehension

78. Praising the employee occasionally when the person shows up on time is an example of __________________ of reinforcement.a. variable ratiob. equity ratioc. continuous scheduled. fixed ratio schedule

(a) Medium/Application

Page 109: Fwk Bauer Tif Organizational Behavior All Chapters

79. All of the following are tips to make disciplining an employee more effective EXCEPTa. discipline when you have time.b. be consistent in your treatment of employees.c. determine if punishment is the effective way to modify behavior.d. document behavior.

(a) Medium/Synthesis

80. Which of the following statements is NOT an effective approach to disciplining employees?a. Carrots can be more effective than sticks.b. Unfair punishment may not change unwanted behavior.c. No two employees or incidents are the same, so no two employee should

be disciplined the same way.d. Discipline is not the most effective way for changing behavior in the long

term.(c) Medium/Synthesis

81. The first step of an organizational behavior modification program isa. evaluate and maintain.b. identify behavior to be modified.c. analyze behavior antecedents and outcomes.d. measure the baseline level.

(b) Easy/Knowledge

82. When organizational behavior modification is used to examine employee absenteeism, which step looks at why employees are absent?a. Step 5: Evaluate and maintain.b. Step 1: Identify behavior to modify.c. Step 3: Analyze behavior antecedents and outcomes.d. Step 2: Measure the baseline level.

(c) Easy/Comprehension

83. Which of the following is a need based theory of motivation?a. expectancy theory.b. ERG theory.c. reinforcement theory.d. equity theory.

(b) Easy/Knowledge

Section III: The Role of Ethics and National Culture

84. In simulations examining whether reinforcement theory explains ethical behavior, all of the following are findings EXCEPTa. subjects were less likely to cut kickback payments if there was a threat of

punishment.b. subjects were more likely to continue kickback payments if they made a

profit after providing the kickbacks.c. the perceived likelihood of punishment is a major influence of ethical

behavior.

Page 110: Fwk Bauer Tif Organizational Behavior All Chapters

d. the severity of expected punishment was the primary predictor of whether subjects were inclined to behave unethically.

(a) Difficult/Evaluation

85. Research on motivation suggests the following:a. Ethical behavior is learned behavior.b. Satisfaction with social needs is a powerful motivator in industrialized

countries.c. Subjects in different cultures do not vary in their distribution of rewards

regardless of the subject’s age.d. Valuing justice and fairness is a distinctive American-only value.

(a) Medium/Comprehension

86. Which of the following summary statements is true regarding motivation theories?a. Need-based motivation theories use the mental processes of employees

to understand employee motivation.b. Research indicates that what is perceived as fair and unfair is culturally

defined.c. Research indicates that because of the unfairness perceived in their

environments, individuals no longer repeat behavior that leads to positive results.

d. Process-based theories suggest managers should identify what employees need and make the work environment a means of satisfying those needs.

(b) Medium/Synthesis

87. Research on motivation in different cultures indicates thata. high power distance cultures value having a voice in the justice process.b. Chinese study participants place the highest value on procedural justice.c. equity is not culture specific; all cultures utilize this method to achieve

fairness.d. in industrialized nations, satisfaction with esteem needs was a more

powerful motivator than satisfaction of physiological or security needs.(d) Medium/Synthesis

Opening Case: Motivation in Action: The Case of Trader Joe’s

88. Analyzing Trader Joe’s success using Herzberg’s two-factor theory of motivation would suggest thata. contextual factors, like being among the best paid employees in the retail

industry, are the only reason for the firm’s success.b. content factors, like the collaborative atmosphere in each supermarket,

are the only reason for the firm’s success.c. growth needs, like the autonomy of opening products for customer

samples, are the only reason for the firm’s success.d. Contextual, content, and growth needs factors all help to contribute to

Trader Joe’s success. (d) Difficult/Synthesis

Page 111: Fwk Bauer Tif Organizational Behavior All Chapters

89. Ability isa. having the resources, information and support one needs to perform well.b. having the skills and knowledge required to perform the job.c. the intention of achieving a goal, leading to goal-directed behavior.d. the function of performance, motivation, and environment.

(b) Easy/Knowledge

90. Which of the following is true about motivation?a. Being motivated is the same as being a high performer.b. Motivation is the sole reason why people perform well.c. Motivation is a key influence over an employee’s performance level.d. Motivation is usually the most critical factor in a person’s performance.

(c) Medium/Comprehension

91. Motivation isa. having the skills and knowledge required to perform the job.b. a function of performance, ability and environment.c. the intention of achieving a goal, leading to goal-directed behavior.d. the only key to high performance.

(c) Easy/Knowledge

92. Which of the following statements is correct?a. Performance is a function of the interaction between motivation, ability

and environment.b. Motivation is a function of the interaction between ability, environment

and performance.c. Ability is a function of the interaction between motivation, performance

and environment.d. Performance is a major influence on motivation, ability and environment.

(a) Easy/Comprehension

FILL IN THE BLANK

93. Job performance is a function of the interaction between a person’s _______, ability, and ___________.

(motivation, environment)

94. Marcus eagerly uses his company’s tuition reimbursement program for an undergraduate degree. Marcus is fulfilling his ____________ need.

(self-actualization)

95. Antonio feels strongly about satisfying his subordinates’ __________needs. Therefore, each time one of his employees does an outstanding job on some project, he calls the entire department together to formally recognize the individual’s accomplishment.

(esteem)

96. For most employees, ___________________satisfies their physiological needs.(pay)

Page 112: Fwk Bauer Tif Organizational Behavior All Chapters

97. Social needs is to Maslow, as _______________ is to Alderfer.(relatedness)

98. _______________arranges needs in a hierarchy. (Maslow’s theory of motivation)

99. Herzberg’s two-factor theory of motivation suggests factors that are part of the context in which the job is performed are called _______factors.

(hygiene)

100. Herzberg’s _______ factors include company policy, supervision, working conditions, security and salary.

(hygiene)

101. Li Cho is participating in a study in which after looking at a picture, she will write a story describing the person in the picture, what he is doing and why. Li Cho is using a __________ _________ _________ to assess motivation themes.

(Thematic Apperception Test)

102. Under McClelland’s theory, individuals who focus on goals and deadlines have a _________ _____ ___________.

(need for achievement)

103. Clarissa emphasizes frequent harmonious interpersonal relationships. Clarissa has a high _________ _________ ______.

(need for affiliation)

104. ______________ motivation theories see motivation as a rational process.(Process-based)

105. Mary and Jan are two graphic designers at Master Graphics. They were hired at the same time with the same basic qualifications. Under equity theory, Jan is Mary’s ________ or comparison person.

(referent)

106. In equity theory, an individual’s _______/______ is compared to the same ratio of a referent.

(outcome/input)

107. Nina is experiencing feelings of inequity; she can address the situation by ____________ outcomes.

(increasing)

108. _________ ________ is a personality trait that explains different reactions to inequity.

(Equity sensitivity)

109. Monica was hired as an entry level accountant. She found out that the firm decided to hire her after learning that her mother was also an accountant. Monica likely feels low _________________ justice.

Page 113: Fwk Bauer Tif Organizational Behavior All Chapters

(procedural)

110. ___________ ________ is the degree to which the outcomes received from the organization are perceived to be fair.

(Distributive justice)

111. You just got a promotion, one which you feel you deserve after all your hard work. You perceive high __________ _____________.

(distributive justice)

112. ___________ are those who expect substantial compensation for little input.(Entitleds)

113. The perception that high levels of effort will lead to performance is called __________.

(expectancy)

114. Mario is making sure that his subordinates view rewards offered by the firm as desirable. Mario is attempting to influence the _________ element of expectancy theory.

(valence)

115. The degree to which employees believe that their performance will result in rewards is called _____________.

(instrumentality)

116. ____________ is based on the work of Ivan Pavlov on behavioral conditioning and B. F. Skinner on operant conditioning.

(Reinforcement theory)

117. The professor is praising the student for an outstanding PowerPoint presentation. The professor is practicing _________ ___________ to get the student to do as well the next time he presents.

(positive reinforcement)

118. The XYZ Corporation just announced a new bonus program where managers can provide “on the spot” bonuses for an exceptional job done that day. XYZ is proposing to offer bonuses on a _____________ _____________ schedule.

(variable ratio)

119. Equity and expectancy theories are examples of ________-based theories of motivation.

(process)

120. ERG theory is to _______ ________ motivation theories as equity theory is to ________ ______ motivation theories.

(need-based, process-based)

121. Of the need-based theories mentioned in the text, _________ ________ __________ theory has received the most support.

(McClelland’s acquired needs)

Page 114: Fwk Bauer Tif Organizational Behavior All Chapters

122. Research suggests that low power distance cultures value __________more than high power/distance cultures.

(voice)

SHORT ANSWERS

123. Describe how motivation determines employee performance.

Job performance is a function of the interaction between a person’s motivation, ability and environment. Motivation, the intention of achieving a goal, helps a person try hard to accomplish a certain task. Motivation, is not, however, sufficient for high performance and not the key reason why people perform well, but is a key influence on performance.

124. Compare and contrast Maslow’s hierarchy of needs to Alderfer’s ERG theory.

Both Maslow’s hierarchy of needs and Alderfer’s ERG theory are need-based theories.

Maslow suggests there are five basic human needs: physiological, safety, social, esteem and self-actualization, which are arranged in a hierarchy. As one need is satisfied, the next need in the hierarchy drives behavior.

Alderfer modifies Maslow’s theory by proposing that there are three needs, existence, relatedness and growth. These needs are not arranged hierarchically, but grouped as categories. Consequently Alderfer is proposing that more than one need may operate at any time. In addition, Alderfer proposes a “frustration-regression hypothesis” in his theory. The hypothesis suggests that individuals who are frustrated when trying to satisfy one need may regress to another.

125. List each of the needs in Maslow’s hierarchy and provide an example of how a firm could try to satisfy each.

Physiological needs are those related to food, water, and shelter. A firm could satisfy each of these needs by paying an employee.

Safety needs are satisfied when one is free from danger, pain or uncertainty. Again, pay would be an example of how to satisfy these needs as the paycheck would enable the employee to purchase items to keep him free from danger (a car instead of walking on a busy highway).

Social needs are those of belonging to a group or being loved. Firms could create a golf league, take their employees out for dinner for the holidays or hold summer picnics to satisfy this need.

Esteem needs are those where one desires respect from others and wants to feel important. A program where employees are formally recognized for a job well done, like an “employee of the month” award would satisfy this need.

Page 115: Fwk Bauer Tif Organizational Behavior All Chapters

Self-actualization needs are where an employee desires to be “all he can be.” Offering your employees a tuition reimbursement program would help satisfy this need.

126. Describe how Herzberg’s hygiene and motivator factors differ and give an example of each.

Hygiene factors are part of the context of a workplace. Their presence is often overlooked by an employee, but their absence leads to feelings of dissatisfaction in the employee. For example, if the working conditions are adequate at a place of business (relatively clean, warm and dry), employees tend not to notice. However, if the air conditioning breaks in the summer, employees will complain a great deal about the heat. Other examples of hygiene factors include: company policy, supervision and relationships, salary and security.

Motivators are part of the job’s content and are intrinsic to the job. Examples include: achievement, recognition, interesting work, increased responsibility, and advancement. If these conditions are not found in the job, the worker is somewhat neutral; however, when the conditions are created by a manager, for example, they can motivate an employee to high performance levels.

127. What are some of the key characteristics of individuals identified by a thematic apperception test as high need for achievement? In what occupations do high need for achievement individuals perform well?

Individuals with a high need for achievement meet deadlines, come up with great ideas, and generally aim for success in their jobs. Consequently they find success in jobs such as scientists, entrepreneurs, sales and engineers.

128. You perceive inequity in a situation. Describe reactions that equity theory suggests to alter the perception.

If inequity is perceived, an individual can alter his perceptions of his own or the referent’s inputs and outcomes. For example, you could downplay your own inputs, value them more highly, and distort the other’s inputs or his outcomes. You might also get the referent to increase his inputs. Yet a third alternative is for you to reduce your inputs or increase your outcomes. Finally, you could change your comparison person (the referent) or leave the situation completely.

129. Define distributive, procedural and interactional justice and note how they are related.

Distributive justice is the degree to which the outcomes received from the organization are perceived to be fair. Procedural justice refers to the degree to which fair decision-making procedures are used to arrive at a decision. Finally, interactional justice is the degree to which people are treated with respect, kindness, and dignity in interpersonal interactions. The three concepts interact with each other and together. In short, the three types of justice are providing an opportunity for an employee to receive a fair outcome that was provided based

Page 116: Fwk Bauer Tif Organizational Behavior All Chapters

upon a fair decision and awarded to the employee in a respectful, kind manner. Employers benefit from attending to all three types of justice.

130. Describe how research suggests an employee achieves procedural justice.

Research suggests the following ways to achieve procedural justice: provide employees advance notice, allow them a voice in decision making, provide explanations for all decisions, and above all be consistent in the manner in which you treat all employees.

131. According to expectancy theory, what are the three questions individuals ask themselves when evaluating a situation?

Will my effort lead to high performance?Will performance lead to outcomes?Do I find outcomes desirable?

132. Name two ways in which managers can influence employees’ expectancy perceptions.

If employees do not believe they have sufficient skills to handle a job, the manager could ensure they are trained.

If employees do not believe the effort leads to performance because of, for example, political behavior in the workplace, the manager must ensure that the tie between the two is clearly established.

Finally, many employees have low self-esteem or external loci of control that make them believe their effort will never make a difference.

133. Name two ways in which managers can influence employees’ instrumentality perceptions.

First, the connection between pay and performance must be made clear. Announcing the rewards such as bonuses and merit pay is a first step to this understanding.

The manager must also make it clear that performance, and only performance, is being rewarded.

134. Name two ways in which managers can influence employees’ valence perceptions.

First, the manager must make sure the rewards are attractive to employees. Therefore, the manager must know employees’ needs, wants and desires. Then the manager must follow up on the information on a regular basis to see if there have been changes. Finally, the manager must ensure that employees have some choice between rewards.

135. What does the phenomenon labeled, “the folly of rewarding A while hoping for B” mean?

Page 117: Fwk Bauer Tif Organizational Behavior All Chapters

The statement means that sometimes people are rewarded for the wrong kind of behavior. For example, a company can make public statements about its high quality products. Pressured to get products to the customers however, they often ship items that lack quality. Managers must be cognizant of the consequences of actions such as these.

136. Name and define each of the four reinforcement methods and provide an example of each.

Positive reinforcement is a method of increasing desired behaviors. An example of this would be to praise an employee for treating a customer nicely during a complaint call by that customer.

Negative reinforcement is also a method for increasing desired behaviors. Here removing unpleasant consequences cause the desired behavior to result. For example, teenagers often clean their rooms simply to have their parents quit nagging them about it.

Extinction is used to decrease the frequency of negative behaviors. When rewards are removed following negative behavior, persistent removal leads to cessation of the behavior. Employees who continually mock their co-workers cease the behavior when these comments are ignored.

Punishment is presenting negative consequences following unwanted behaviors. If you are repeatedly late for work, you could have a disciplinary suspension of three days brought against you.

137. How would a fixed ratio schedule and a variable ratio schedule be used in a workplace?

A fixed ratio schedule is when rewards are provided every set time the right behavior is demonstrated. For example, a salesman may receive a bonus for every tenth sale he makes.

A variable ratio schedule reinforces on a random pattern. For example, a manager may praise his employees on a very infrequent, occasional schedule.

138. List the stages of organization behavior modification.

1. Identify behavior to be modified.2. Measure the baseline level.3. Analyze its antecedents and outcomes.4. Intervene.5. Evaluate and maintain.

139. Briefly describe how process and need-based theories of motivation are useful to firms.

Need-based theories help a manager identify what people need to be motivated and will help make the work environment a place for satisfying those needs.

Page 118: Fwk Bauer Tif Organizational Behavior All Chapters

Maslow’s hierarchy, Alderfer’s ERG and McClelland’s acquired needs theories are key in helping to identify such needs.

Process theories look at the mental processes of employees that are key in driving behavior. Equity, expectancy and reinforcement theories are useful in designing reward systems for a firm. Together the two groups of theories help make a workplace productive for firm and employees alike.

140. Describe how reinforcement theory has been successful in explaining ethical behavior.

The theories help managers understand how employees behave unethically. Process theories like reinforcement suggest that people will behave unethically if those unethical behaviors are rewarded. The need-based theories help the manager understand what the basic needs and desires of employees are that drive the behavior.

141. Give an example of how motivation is culturally bound.

Maslow’s hierarchy of needs is one such theory that is culturally bound. Financial satisfaction, for example, is a stronger predictor of overall life satisfaction in developing nations. In contrast, esteem needs are a more powerful motivator for industrialized countries.

142. List two tips a manager could use to make the discipline process more effective.

Consider whether the punishment is the most effective way of modifying the behavior.Be sure that the punishment fits the crime.Be consistent in your treatment of employees.Document the behavior in question.Be timely in discipline.

ESSSAY

143. Using either Maslow’s hierarchy of needs or Alderfer’s ERG theory, describe why Trader Joe’s employees provide such excellent service to customers.

Maslow’s hierarchy of needs presents needs in a ranked order beginning with physiological and then moving upward to safety, social, esteem and self-actualization. The theory suggests that once a need is satisfied, it no longer motivates behavior and the next higher order need becomes the driving force for behavior. Trader Joe’s, as one of the best paying retail organizations in that sector with an average full-time employee income of $40,150 annually, satisfies the physiological and safety needs of its employees. The atmosphere found in the store where employees happily cover for each other when one is missing and enthusiastically serve the needs of the customers, goes a long way to satisfying the social and esteem needs of employees. The quarterly performance evaluations facilitate career opportunities for employees, as does the autonomy afforded by many positions. Finally, the training opportunities provided by Trader

Page 119: Fwk Bauer Tif Organizational Behavior All Chapters

Joe’s supports self-actualization. The specific program elements when viewed as a whole demonstrate the manner in which firms can use motivational theories to support employees as well as their own competitive goals.

The same argument can be made for how Alderfer’s model applies to Trader Joe’s. The only changes would be that Alderfer’s existence needs would replace the physiological and safety needs of Maslow; relatedness needs would replace social; and growth needs would replace the esteem and self actualization dimensions.

144. Maslow’s hierarchy of needs and Alderfer’s ERG theory are need-based theories of motivation. What are some key criticisms of Maslow’s theory and how does Alderfer’s theory modify that of Maslow?

Maslow’s research was done in the clinical setting, which raises questions as to its applicability in the workplace, especially since it lacks empirical support. Further, the model proposes that needs are hierarchically ranked and that as one need is satisfied, the next higher order need becomes the focus of effort. Yet one can observe individuals who, while pursuing a higher order need, seem to stay motivated not only by that need but aspects of the previous one. For example, if an individual were ever so poor that he had no food and was homeless, even after becoming somewhat successful, the fear of a return to homelessness might still drive him to some degree. Consistently, research has not supported the argument that only one need is dominant at any time or that once satisfied, a need is not motivating.

Alderfer’s model addresses some of these concerns, first by offering needs in a categorical framework (existence, relatedness and growth) rather than a hierarchical mode. This representation suggests that more than one need can be motivating at any one time. In addition, Alderfer offers a “frustration-regression” hypothesis in his model, suggesting that if individuals become frustrated in pursuing a specific need, they may regress to another need for motivating behavior. In short, Alderfer suggests that more than one need may be motivating at any time.

145. Herzberg has been strongly criticized for his research finding that salary is a hygiene factor. Describe the nature of hygiene factors and how salary fits this category. What are the implications of this finding for managers in the workplace?

Hygiene factors are ones that are found in the context of the job environment. Herzberg argues, based upon his study, that these factors are those one would expect to have in his environment. For example, the physical environment should be one free from hazards, where it is not too hot or too cold in which to work. The worker, is, in essence, neutral when these environmental factors are as expected. Only when the factors are unexpected and problematic does the employee react and then he expresses dissatisfaction. For example, if the air conditioning in an office is broken during the summer months, sweltering employees will complain, likely very loudly. Herzberg contends that salary fits this profile. Clearly employees expect a certain level of salary, at least minimum

Page 120: Fwk Bauer Tif Organizational Behavior All Chapters

wage, only when their pay is not at the level of the market, for example, will they complain about it and begin to pursue other options.

The important implications of Herzberg’s theory are that while managers need to be aware of environmental factors and their improvement, they are not sufficient to motivate an employee. Context factors matter because when they are absent, employees are dissatisfied. The job and its content also must be addressed.

146. McClelland’s acquired needs theory has been assessed in various cultures. Choose a country with whose culture you have some familiarity. Describe some key aspects of the culture and which of the three needs might be most prevalent among residents of the country. Provide a rationale for your suggestions.

Answers will vary. One general example: Latin American cultures tend to focus on close-knit work groups with strong interpersonal interactions between individuals. One professor’s personal experience is that Latin American workers are extremely attuned to each other’s likes and dislikes. Mentioning that you like an individual’s shirt could very likely result in your receipt of a gift box the next day, containing an identical shirt. Clearly these specific examples reflect a need for affiliation orientation.

147. You are a trained expert in interpreting the themes of responses to the TAT. You get the following story based on the respondent’s viewing the TAT picture in your book: Jacqueline is a recent graduate of The University of Michigan, now employed as a marketing specialist at a Fortune 500 firm. She has just completed her first marketing campaign for the firm. The campaign was a major success. She is looking once more at the plaque the vice president of marketing awarded her to commemorate the campaign’s success. Analyze this story to determine the key themes, then indicate which of McClelland’s needs seem most prevalent and what the implications are for managing this employee.

Themes: success: Graduate of major research university, primary force behind outstanding marketing campaign and recipient of plaque commemorating the event.

Goal-oriented: graduate, coordinator of marketing campaign

Feedback-oriented: enjoying plaque received.

The McClelland theme which best describes Jacqueline is need for achievement. The best way to manage Jacqueline is to continue to provide her with situations in which she can create and achieve goals and then provide her the feedback that recognizes the successes achieved. Those high in need for achievement meet deadlines, come up with innovative ideas and achieve success. Managers simply need to support these dimensions.

148. You are the new dean of the college of business administration at a major university. One of the first tasks you have undertaken is a review of the salaries of your non-tenured faculty members. You note that the OB instructor has similar credentials to the marketing instructor, but is paid $5,000 less per year for similar teaching duties. You are also aware that faculty members recently found out

Page 121: Fwk Bauer Tif Organizational Behavior All Chapters

salaries of all instructors when the payroll office erroneously sent an email with that information. Knowing equity theory, what reactions to the inequity might you expect, and how would you address them?

Equity theory suggests that two ratios are assessed by an individual: first, a self-assessment ratio comparing one’s input to outcomes. If the outcome is favorable given the input invested, an individual feels equity. That self-assessed ratio is then compared against a similar ratio “mentally” prepared on a referent. Again the comparison leads to feelings of equity or inequity. In this scenario, a comparison of the OB instructor to that of the marketing instructor, all things being equal, is going to make the OB instructor feel inequitably treated by the college.

Knowing the inequity felt by the OB instructor, the dean could pursue a number of different options to address feelings of unfairness in the university system. For example, the dean might attempt to help the OB instructor alter his perception of his own or the referent’s inputs and outcomes. Specifically, the dean could help the OB instructor see that he really did not work as hard on his classes, nor teach them as successfully as he thinks he did (perhaps by providing student evaluation of such). Other options to address the inequity felt include: having the referent increase his inputs, having the OB instructor reduce his inputs or attempt to increase outcomes, change the comparison person (perhaps by comparing himself to another OB instructor) and having the OB instructor leave the university.

149. General Motors, Ford and Daimler Chrysler are closing numerous factories and laying off thousands of employees. Based on the research on procedural justice, what recommendations could you make to the firms to be fair with those individuals?

Again, answers will vary. Some options the auto companies might utilize include providing the laid-off employees with advance notice of the job action, thus providing them an opportunity to prepare themselves for the major life impact. Other options include: allowing employees a voice in decision making, providing full explanations for the need for such job action, and treating all employees in a consistent manner (having a set procedure for determining who will be laid off and not deviating from the plan).

150. Using expectancy theory, describe the thought process a pharmaceutical salesman might go through if his firm has developed a new drug and provides him the option of staying with his previous products or adding the new product to his account.

Expectancy theory suggests that three factors should be examined. First is the notion of expectancy itself. Will my effort lead to high performance? In this case, does the salesman believe he can sell the new drug? Will his current selling techniques allow him to maintain his high performance level?

The second issue to be considered is instrumentality. Instrumentality is the degree to which the person believes performance is related to rewards. Will the sale of this drug produce good commissions?

Page 122: Fwk Bauer Tif Organizational Behavior All Chapters

Finally, the notion of valence must be assessed. Is the size of the commissions sufficient to facilitate such effort? When these issues are considered in concert with one another, the motivation to pursue the action is determined. The role of the manager of this pharmaceutical sales person is to try to influence all these perceptions.

151. You have been hired to replace a very unpopular manager of a marketing department. Department members were especially vocal about how unfair the previous manager was in conducting performance appraisals and allocating merit bonuses. What ideas do you have about being a fair person?

Being fair involves looking at a few key issues:

The prior manager may not have done so, but when distributing rewards, you must make sure you are paying attention to the different contribution levels of your employees. Not all employees contribute equally and to reward them all equally is to be unfair to those with greater participation.

That said, there are occasions when the nature of people’s contributions should be ignored and the rewards distributed equally. For example, when holding a luncheon to celebrate the attainment of a production goal, all employees should be invited, not just those most directly responsible for the production of the actual product.

You must also pay attention to how you actually make decisions. Though you are accountable for your decisions, you should not always make those decisions unilaterally. Talk to your employees and get some perspective by those who will be affected by the decisions.

Pay attention to how you talk to people. Use the old adage: Treat people as you yourself would like to be treated.

Justice is also in the eye of the beholder. Even though you may feel that you are being fair, everyone has different perceptions. Recognize that and accept it.

Finally, people care about how they are treated, but they also care about how others are treated.

With these thoughts in mind, you will have a head start on addressing the issues created by your predecessor.

152. Many firms are instituting wellness programs for their employees as a means to improve those employees’ health and well-being and lower healthcare premiums. Use the stages of organizational behavior modification to explain how to modify employee health in the workplace.

Stage 1: Identify the behavior to be modified. Overweight individuals, particularly those who are established as being clinically obese, represent real medical concerns both now and in the future. Identifying those individuals whose BMI

Page 123: Fwk Bauer Tif Organizational Behavior All Chapters

(body mass index) suggest they are candidates for medical problems down the road is an important first step.

Stage 2: Measure the baseline level. A firm can decide to institute a wellness program whereby they publicize the opportunity to privately find out your BMI. Many firms institute a team competition to get department members to be measured. The department with the most individuals measured, for example, may get $5 gift cards to a local eatery.

Stage 3: Analyze its antecedents and outcomes: Once the BMI is assessed and the potential level of obesity in the firm determined, the firm can begin to determine the potential causes of the weight problem among its employees. Often the problem is a lack of exercise.

Stage 4: Intervene. If lack of activity is one contributing problem for the firm’s employees, a program addressing the issue can be created. Many firms are taking part in a walking program called “America on the Move.” The program distributes pedometers to all participants, and baseline numbers of steps in a daily routine are determined. Once that baseline is assessed, the firm attempts to encourage its employees to increase the number of steps daily. One goal might be to take 2,000 extra steps each day.

Stage 5: Evaluate and maintain. Teams are formed in departments of the firm to compete against one another as each individual and group attempts to reach the 2,000 steps per day goal. After a set time period, progress toward the goal is assessed and those teams and individuals achieving the goal are rewarded for their efforts. At one university instituting this program, goal achievers have their monthly insurance premiums reduced by an established formula for every month that goal is achieved. Clearly the overall effect is that employees become healthier and overall company healthcare premiums are reduced.

Page 124: Fwk Bauer Tif Organizational Behavior All Chapters

Organizational Behavior, Version 1.1Bauer & Erdogan

FWK Test Item FileChapter 6

TRUE/FALSE

1. The Nucor Steel Company’s incentive system penalizes low performers, but sets no upper limit on the amount high performers can earn annually.

(True)

2. Evidence of the success of Nucor Steel is the low turnover rate and nonunion status of the firm.

(True)

3. Nucor Steel’s formula for success is to centralize decision making and rigidly define employee and manager job duties.

(False)

4. Leaving employees free to choose job performance methods is an important element of scientific management techniques.

(False)

5. Job specialization entails breaking down jobs into their simplest components.(True)

6. Job enlargement involves moving employees from job to job at different intervals.(False)

7. You are a Walt Disney Company management trainee. The first three months of your job you will be working in the restaurant and hospitality area. The next three months you will be in the merchandising area, followed by three months in lodging and finally, three months in park operations. The Walt Disney management trainee program is designed using the job rotation philosophy.

(True)

8. In a supermarket study, cashiers rotated through different departments have lower stress levels and less pain in necks and shoulders.

(True)

9. Research shows that the psychological state of meaningfulness is the most important such state for employee attitudes and behaviors.

(True)

10. The job characteristics model is an attempt to design jobs for increased motivational potential.

(True)

Page 125: Fwk Bauer Tif Organizational Behavior All Chapters

11. Skill variety and task significance are the most important elements in deciding motivational potential.

(False)

12. Access to information is a key factor in empowering employees.(True)

13. Goal commitment is higher when employees have trust-based relationships with managers.

(True)

14. Goal setting is one of the most influential and practical theories of motivation.(True)

15. The most effective goals are easy ones.(False)

16. The performance appraisal meeting is the most important component of a performance appraisal.

(True)

17. In the most effective performance appraisal meetings, criticism of the individual personality traits is very important.

(False)

18. Quality expert Edward Deming advocates abolishing performance appraisals in the workplace.

(True)

19. Adequate notice ensures that there is two-way communication during the performance appraisal process.

(False)

20. Relative ranking appraisals may help an organization become more performance-oriented if they are used for a few years because they tend to weed out employees with persistent performance problems.

(True)

21. To increase the effectiveness of performance meetings, increase employee participation.

(True)

22. Attractive women are rated lower if they are employed in nonmanagement jobs and higher if they are in management jobs.

(False)

23. Leniency bias makes it harder for employees to change their behaviors.(True)

24. Merit pay is a permanent pay raise based on past performance.(True)

Page 126: Fwk Bauer Tif Organizational Behavior All Chapters

25. In a pay system based upon commission, rewarding only sales volume can lead salesmen to heavily discount merchandise.

(True)

26. The notion of using incentives to increase performance is a very old idea, actually going back to the time of Napoleon.

(True)

27. Gift cards as awards to employees are not appreciated because they are often purchased for stores and restaurants that employees do not frequent.

(False)

28. The use of stock options remains a popular incentive technique in start-up organizations.

(True)

29. Even though goal setting is a good motivational tool, there is strong evidence that goal setting can also lead to unethical behavior.

(True)

30. One technique to reduce the likelihood of unethical behavior resulting from goal setting is to create multiple levels of goals and distribute rewards according to the goals achieved rather than rewarding only those who reach the highest goal and not giving anything to anyone else, including those who were very close to that high goal.

(True)

31. Indian employees are more satisfied when they are empowered in the workplace than their counterparts in the United States or Poland.

(False)

32. Chinese employees are more motivated than American employees when the goals set for them are very difficult.

(True)

33. Countries high in power distance respond positively to appraisal systems where lower level employees give feedback to their managers.

(False)

34. Research in Western countries suggests that empowerment is an effective tool to motivate employees.

(True)

35. In China, goals high in specificity are more motivational in contrast to the low specificity goals preferred by employees in Western cultures, including the United States.

(False)

Page 127: Fwk Bauer Tif Organizational Behavior All Chapters

MULTIPLE CHOICE

Opening Section: Motivating Steel Workers Works: The Case of Nucor

36. All of the following are reasons why Nucor Steel is so effective at motivating its employees EXCEPTa. the incentive system penalizes lower performers but offers no limits on

how much can be made by those that perform well.b. the firm has a centralized structure that retains decision making at upper

levels in the organization.c. employees have the opportunity to fix problems that they see occurring

during production.d. there is a great deal of task flexibility as some duties of managers have

been pushed down to lower level line employees.(b) Medium/Comprehension

Section I: Motivating Employees through Job Design

37. Which of the following factors has the most influence on worker motivation?a. payb. growth opportunitiesc. job designd. recognition

(c) Easy/Knowledge

38. Scientific management’s primary focus isa. goal setting.b. efficiency.c. flexibility.d. motivation.

(b) Medium/Analysis

39. Key elements of the scientific management philosophy include all of the following EXCEPTa. using training and specific instructions to minimize the number of workers

needed on jobs.b. minimizing waste by identifying the most efficient method to perform the

job.c. expanding the nature of tasks performed by employees to add more

variety to the job.d. breaking down jobs into their simplest components to facilitate

specialization.(c) Medium/Analysis

Page 128: Fwk Bauer Tif Organizational Behavior All Chapters

40. Job specialization a. is breaking down jobs into their simplest components and assigning them

to employees so that each person will perform a few tasks in a repetitive manner.

b. refers to expanding the tasks performed by employees to add more variety.

c. involves moving employees from job to job at regular intervals.d. allows workers more control over how they perform their own tasks.

(a) Easy/Knowledge

41. Which of the following is an advantage to job specialization?

a. Training costs are higher, but because of the specialized nature of tasks, they are learned more fully.

b. The nature of the jobs leads to lower absenteeism.c. Staffing costs are lower because the repetitive nature of tasks makes skill

requirements lower.d. The nature of the jobs is especially effective in rapidly changing

environments.(c) Difficult/Evaluation

42. Job rotation a. is breaking down jobs into their simplest components and assigning them

to employees so that each person will perform a few tasks in a repetitive manner.

b. refers to expanding the tasks performed by employees to add more variety.

c. involves moving employees from job to job at regular intervals.d. allows workers more control over how they perform their own tasks.

(c) Easy/Knowledge

43. Job enlargementa. is breaking down jobs into their simplest components and assigning them

to employees so that each person will perform few tasks in a repetitive manner.

b. refers to expanding the tasks performed by employees to add more variety.

c. involves moving employees from job to job at regular intervals.d. allows workers more control over how they perform their own tasks.

(b) Easy/Knowledge

44. Job enrichmenta. is breaking down jobs into their simplest components and assigning them

to employees so that each person will perform few tasks in a repetitive manner.

b. refers to expanding the tasks performed by employees to add more variety.

c. involves moving employees from job to job at regular intervals.d. allows workers more control over how they perform their own tasks.

(d) Easy/Knowledge

Page 129: Fwk Bauer Tif Organizational Behavior All Chapters

45. Frederick Taylor’s work led to a fundamental change in management philosophy. The change a. was understanding that managers could not influence the output levels of

employees.b. was understanding that repetitive activities are the direct result of job

generalization.c. paved the way for today’s automation and standardization.d. was increasing the complexity of jobs in the workplace for greater

productivity.(c) Difficult/Analysis

46. The local high school band needs to raise money to pay for a trip to New York City. It decides to sell sandwiches. Early Saturday morning, 20 volunteers report to the school cafeteria to make sandwiches. Stations are set up to make the 500 sandwiches ordered for that day. Station one has four individuals cutting sandwich buns, station two has four individuals placing the meats on the rolls, station three, etc. The individual who devised the sandwich-making production plan is operating under a a. job specialization approach.b. job rotation approach.c. job enlargement approach.d. job enrichment approach.

(a) Difficult/Synthesis

47. Job specialization cana. increase the number of employees needed to complete the job.b. minimize training costs.c. lead to greater motivation among employees.d. be very effective in rapidly changing environments.

(b) Difficult/Evaluation

48. Which of the following statements is INACCURATE about job rotation?a. Rotation is the regular movement of employees through firm jobs.b. Rotation leads to increased likelihood that turnover will be reduced in the

firm.c. Rotation is an opportunity for employees to acquire new skills.d. Rotation leads to slightly elevated blood pressure measured in employees

using the process due to their movement between jobs.(d) Medium/Comprehension

49. All of the following are advantages of job rotation EXCEPTa. employees become cross-trained due to their movement between jobs.b. employees become less marketable after rotating because their skill sets

become very company-specific.c. managers have greater scheduling flexibility in a job rotation system.d. employees experience less boredom in a job rotation system.

(b) Medium/Comprehension

50. Stuart works on an auto assembly line. Last year he was responsible for welding the upper panel of the wheel well onto the left rear wheel area. This year, he is

Page 130: Fwk Bauer Tif Organizational Behavior All Chapters

responsible for welding all parts of the entire left rear wheel area. This auto assembly line underwent a job re-design using the ________ ________technique.a. job specialization.b. job enrichment.c. job rotation.d. job enlargement.

(d) Medium/Application

51. Which of the following statements is true about job design approaches?a. Job rotation takes place only with lower level jobs in an organization.b. Job enrichment may lead to employee frustration or dissatisfaction with

pay.c. Job specialization is effective in rapidly changing work environments.d. Job enlargement implementation leads employees to feel capable of

performing job tasks.(b) Difficult/Analysis

52. Which of the following statements about job enlargement is NOT true?a. Job enlargement involving giving employees more tasks requiring them to

be knowledgeable in different areas has fewer positive effects than job enlargement adding tasks that are very simple in nature.

b. When jobs are enlarged, employees view themselves as more capable.c. Job enlargement is beneficial because it is positively related to higher

quality customer service.d. The intent behind job enlargement is to increase the variety of tasks to be

performed so that boredom is reduced and human resources are used effectively.

(a) Medium/Comprehension

53. All of the following statements about job enrichment are true EXCEPTa. job enrichment may lead to dissatisfaction with one’s pay. b. job enrichment is a suitable technique to be used with any employee.c. job enrichment may be associated with increased productivity and

reduced absenteeism.d. the technique allows employees to take on more responsibility.

(b) Medium/Comprehension

54. Olaf works on an assembly line in an Oslo factory. Last month the assembly line processes were overhauled. Olaf is still responsible for soldering electrical contacts on an appliance component, but now he is also expected to review each component for quality defects after he solders. If he notes any defects, he can place the component in the scrap bin. If he scraps five such pieces in succession, he is permitted to press an emergency button above his station, which stops the assembly line completely. This job re-design is a reflection of the _____ ______________ philosophy.a. job rotationb. job enlargementc. job enrichmentd. job specialization

(c) Difficult/Synthesis

Page 131: Fwk Bauer Tif Organizational Behavior All Chapters

55. Maria works at a factory in Lima, Peru where she sews blue jeans for a major American firm. On Mondays, she sews the side seams of the jeans, on Tuesdays she sews on pockets, Wednesdays she presses finished jeans, Thursdays she folds jeans, packages sets of twelve in shrink wrap and places them in boxes, and on Friday she sews decorative stitching on the pockets and side seams. Maria is working under the job design technique ofa. job specialization.b. job enrichment.c. job enlargement.d. job rotation.

(d) Medium/Synthesis

56. Task identity isa. the extent to which the job requires a person to utilize multiple high level

skills.b. the degree to which a person is in charge of completing an identifiable

piece of work from start to finish.c. whether a person’s job substantially affects other people’s health, work or

well-being.d. the degree to which a person has the freedom to decide how to perform

his tasks.(b) Easy/Knowledge

57. Skill variety isa. the extent to which the job requires a person to utilize multiple high level

skills.b. the degree to which a person is in charge of completing an identifiable

piece of work from start to finish.c. whether a person’s job substantially affects other people’s health, work or

well-being.d. the degree to which a person has the freedom to decide how to perform

his tasks.(a) Easy/Knowledge

58. Feedback isa. the extent to which the job requires a person to utilize multiple high level

skills.b. the degree to which a person is in charge of completing an identifiable

piece of work from start to finish.c. the degree to which people learn how effective they are being at work.d. the degree to which a person has the freedom to decide how to perform

his tasks.(c) Easy/Knowledge

59. Task significance isa. the extent to which the job requires a person to utilize multiple high level

skills.b. the degree to which a person is in charge of completing an identifiable

piece of work from start to finish.

Page 132: Fwk Bauer Tif Organizational Behavior All Chapters

c. whether a person’s job substantially affects other people’s health, work or well-being.

d. the degree to which a person has the freedom to decide how to perform his tasks.

(c) Easy/Knowledge

60. Autonomy isa. the extent to which the job requires a person to utilize multiple high level

skills.b. the degree to which a person is in charge of completing an identifiable

piece of work from start to finish.c. whether a person’s job substantially affects other people’s health, work or

well-being.d. the degree to which a person has the freedom to decide how to perform

his tasks.(d) Easy/Knowledge

61. Which of the following is NOT a core job characteristic in the Hackman and Oldham job characteristics model?a. skill varietyb. task significancec. feedbackd. pay

(d) Easy/Knowledge

62. Which of the following is NOT an outcome in the Hackman and Oldham job characteristics model?a. organizational citizenshipb. performancec. satisfactiond. turnover

(a) Easy/Knowledge

63. Which of the following is a psychological state in the Hackman and Oldham job characteristics model?a. motivationb. meaningfulnessc. creativityd. engagement

(b) Medium/Knowledge

64. Which of the following describes the elements of the Hackman and Oldham job characteristics model?a. Three psychological states lead to five core job characteristics which lead

to five outcomes.b. Five core job characteristics lead to five outcomes which produce three

psychological states.c. Five core job characteristics lead to three psychological states which lead

to five outcomes.d. Three psychological states lead to five outcomes which are analyzed for

five core job characteristics.

Page 133: Fwk Bauer Tif Organizational Behavior All Chapters

(c) Medium/Comprehension

65. Which of the following psychological states is the most important for employee attitudes and behaviors?a. responsibilityb. knowledge of resultsc. motivationd. meaningfulness

(d) Medium/Knowledge

66. A fashion designer who creates pieces for a large fashion house for a New York or Paris runway show hasa. high task identity.b. low task identity.c. low skill variety.d. low autonomy.

(a) Medium/Synthesis

67. Employee autonomy has all of the following positive benefits buta. it makes the employees more reactive in doing their jobs.b. it increases motivation at work.c. it increases employee creativity.d. it increases employee effectiveness.

(a) Medium/Comprehension

68. Which of the following statements regarding the relationship between feedback and performance is true?a. The mere presence of feedback is sufficient to motivate employees to

better performance.b. Regardless of whether feedback is positive or negative in nature,

employees will feel motivated by it.c. Feedback is detrimental to performance in about one-third of the cases.d. The manner in which feedback is delivered has no bearing on its

motivation effects.(c) Medium/Analysis

69. According to the formula for the motivating potential score of a job, the most important elements in deciding motivation potential area. skill variety and autonomy.b. task significance and task identity.c. autonomy and feedback.d. skill variety and feedback.

(c) Difficult/Analysis

70. Which of the following is true regarding the motivating potential of a job?a. An employee whose expectation for his job is to pay the bills will have

high growth strength.b. Even though a job is designed with the express purpose of motivating

individuals, some employees may not find the job motivational.c. Career stage does not influence the importance of the five core

characteristics.

Page 134: Fwk Bauer Tif Organizational Behavior All Chapters

d. Employees with high growth need strength will respond less favorably to jobs with high motivating potential.

(b) Difficult/Evaluation

71. In order to increase the feedback you receive, do the following:a. avoid asking your superior for feedback. Managers will look less favorably

upon you if you continually bother them for feedback.b. develop a positive relationship with your manager.c. when you receive negative feedback, defend yourself by explaining your

behavior. d. seek positive feedback rather than negative feedback.

(b) Medium/Analysis

72. Structural empowermenta. refers to making a person powerless.b. is the degree to which a person has higher order needs such as esteem

and self-actualization.c. refers to the aspects of the work environment that give employees

discretion, autonomy and the ability to do jobs effectively.d. is the degree to which a person has the freedom to decide how to perform

his tasks.(c) Medium/Knowledge

73. Which of the following factors do not determine empowerment?a. structureb. access to informationc. leadership styled. pay

(d) Medium/Knowledge

74. ___________ is an example of felt empowerment.a. Meaningful workb. Leadership stylec. Organizational structured. Organizational climate

(a) Easy/Knowledge

75. One of the dimensions of structural empowerment isa. meaningful work.b. confidence that you can perform the job.c. feeling that one has autonomy at work.d. access to information.

(d) Easy/Knowledge

76. Which of the following statements is true about empowerment?a. The notion of empowerment is appropriate for all employees.b. Empowerment is a relatively easy concept to implement in the workplace.c. Employees who are nervous about empowerment may also worry about

the increased responsibility.d. Empowerment is not related to job performance.

(c) Medium/Analysis

Page 135: Fwk Bauer Tif Organizational Behavior All Chapters

77. To empower your employees,a. retain information on the job until it is absolutely needed.b. take away employee power so your manager accountability is not

compromised.c. make sure managers continue to manage in such a manner that they are

stepping in with routine responses to issues.d. change the company structure so employees have more power on the

job.(d) Medium/Analysis

Section II: Motivating Employees through Goal-Setting

78. The “M” in “SMART” goals stands fora. Measurableb. Motivatingc. Minimald. Management

(a) Easy/Knowledge

79. Which of the following statements regarding SMART goals is correct?a. Easy goals are the most effective goals.b. When goals are specific, stress is higherc. Adding a time limit into a goal leads to short term decision making and

leads to ignoring the long term consequences.d. Effective goals tend to be aggressive.

(d) Medium/Analysis

80. If you say to yourself, “As soon as I finish reading this chapter in my textbook, I am going to play Guitar Hero for an hour,” this specific goal has motivated you becausea. it has given you direction.b. it has energized you not to stop until you have accomplished the goal.c. it has provided you with self esteem.d. it has caused you to re-think how you are working.

(b) Medium/Application

81. All of the following conditions contribute to the effectiveness of goal setting EXCEPTa. ability.b. goal commitment.c. feedback.d. group, as opposed to individual goals.

(d) Easy/Knowledge

82. Which of the following would be conditions under which SMART goals would be effective relative to other times?a. when managers have clearly established goals without employee inputb. when employees feel the skill sets needed to perform a goal-oriented task

need to be upgraded

Page 136: Fwk Bauer Tif Organizational Behavior All Chapters

c. when employees have a supportive and trusting relationship with their managers

d. when managers provide feedback to employees on a very irregular basis, if at all

(c) Medium/Synthesis

83. _______________ is (are) a downside to goal setting.a. Learning decreasesb. Adaptability increasesc. Developing broadmindednessd. Ethical problem decreases

(a) Easy/Knowledge

84. Though goal setting is usually viewed as a positive process, there can be downsides ifa. employees can adapt their behaviors in response to unplanned for

threats.b. employees lack skills necessary to reach the goals.c. rewards are established for coming close to, as well as achieving a goal.d. goals account not just for meeting, as an example, production quotas, but

also for meeting quality standards.(b) Difficult/Evaluation

85. A management by objectives program includes all of the following EXCEPTa. detaching individual goals from organizational strategy.b. setting individual goals aligned with corporate strategy.c. developing action plans.d. periodically reviewing performance goals.

(a) Easy/Knowledge

86. Which of the following is accurate for goal setting?a. Management commitment is key to successful implementation of MBO

programs.b. In general, there are no downsides to goal setting.c. Goals should be easily achieved to be motivating.d. Goals are more effective motivators if feedback on achievement is limited

so as not to divert attention from the goal to the feedback.(a) Difficult/Synthesis

Section III: Motivating Employees through Performance Appraisals

87. Research on performance appraisal suggestsa. performance appraisal ratings are usually used only for feedback

purposes.b. employees are always well aware of the basis for their evaluations.c. as organizations become flatter, there is less need for multiple appraisal

perspectives like those of peers, customers and subordinates.d. feedback is most effective if it immediately follows high or low

performance.(d) Difficult/Evaluation

Page 137: Fwk Bauer Tif Organizational Behavior All Chapters

88. The most likely individual to rate an employee isa. the employee’s supervisor.b. a coworker of the employee.c. a customer of the employee.d. a subordinate of the employee.

(a) Easy/Comprehension

89. 360-degree feedbacka. is designed to provide feedback for promotion or pay decisions.b. is designed to deal with all instances where competition between

coworkers is prevalent.c. is designed to provide feedback for developmental purposes.d. is the most frequently used form of performance appraisal in U.S. firms.

(c) Medium/Analysis

90. Fair hearing a. involves letting employees know what criteria are used in appraisal.b. involves documenting performance problems and using factual evidence

in rating performance.c. means ensuring that there is two-way communication during the appraisal

process and the employee’s perspective is heard.d. is the process where feedback is confidentially gathered from peers,

customers and subordinates.(c) Easy/Knowledge

91. Adequate noticea. involves letting employees know what criteria are used in appraisal.b. Involves documenting performance problems and using factual evidence

in rating performance.c. means ensuring that there is two-way communication during the appraisal

process and the employee’s perspective is heard.d. is the process where feedback is confidentially gathered from peers,

customers and subordinates.(a) Easy/Knowledge

92. Judgment based on evidencea. involves letting employees know what criteria are used in appraisal.b. involves documenting performance problems and using factual evidence

in rating performance.c. means ensuring that there is two-way communication during the appraisal

process and the employee’s perspective is heard.d. is the process where feedback is confidentially gathered from peers,

customers and subordinates.(b) Easy/Knowledge

93. A relative ranking system is used whena. students receive an A if they got 93% of the total points in a semester.b. students receive weighted grades during a semester, like: 20 of 25 points

on a quiz worth 5% of the grade, and 10 of 15 on a quiz worth 3% of the grade.

Page 138: Fwk Bauer Tif Organizational Behavior All Chapters

c. a college implements a system where 10% of the class will receive A, 20% will receive B, 70% will receive C, 20% will receive D and 10% will receive F.

d. a student must take all quizzes to receive a C, a student must take all quizzes and write a paper to receive a B, and a student must take all quizzes, write a paper and do a presentation to receive an A.

(c) Difficult/Synthesis

94. Forced ranking systems, like that established by General Electric,a. provide employees with concrete feedback on what their goals are for

next year.b. are beneficial to a firm if they are used in a consistent manner and for a

long period of time.c. can lead to employee stagnation and a performance-aversive culture.d. can carry the danger of lawsuits because of equal employment

opportunity concerns with employees rated.(d) Difficult/Evaluation

95. All of the following are purposes of a performance appraisal EXCEPTa. achieving work/life balance. b. documenting employee performance for pay and reward decisions.c. providing employees with feedback for developmental purposes.d. distributing bonuses, pay increases and promotions.

(a) Medium/Comprehension

96. Conducting an effective performance appraisal meetinga. is always easier if the rater begins the meeting with a criticism of the

employee and moves forward from there.b. is always easier if the rater limits the number of opportunities the

employee has to speak. c. is facilitated if the rater has empathy for the employee.d. is easier if the rater completes the performance appraisal form as he talks

to the employee during the actual performance appraisal meeting.(c) Medium/Synthesis

97. Matthew and Jim are next-door neighbors and their sons are best friends. Matthew and Jim take the boys on many father-son outings. Matthew is Jim’s boss at ABC Corporation. Jim gets high ratings from Matthew. There may be bias in those ratings due toa. leniency.b. stereotypes.c. contrast.d. liking.

(d) Medium/Application

98. Renaldo is preparing to rate Marquez on his annual performance evaluation. As he looks at the first item, Renaldo thinks about the meeting he had with Marquez last week. Marquez shared with Renaldo that his wife was quite ill and that there may be days when he would be called to the hospital unexpectedly. As Renaldo looks back at the form, he thinks about how costly this will be for Marquez. Renaldo may give Marquez ratings that show ______________ bias.

Page 139: Fwk Bauer Tif Organizational Behavior All Chapters

a. likingb. leniencyc. stereotyped. recency

(d) Difficult/Synthesis

99. All of the following statements about rating biases are true EXCEPTa. women in stereotypically male jobs are rated higher than women in

stereotypically female jobs.b. attractive women are rated higher if they hold non-management jobs than

if they hold management positions.c. liking someone may cause the rater to selectively remember positive

things about the person and thus rate him higher.d. leniency bias can be addressed by having the rater use relative rankings.

(a) Difficult/Evaluation

Section IV: Motivating Employees Through Performance Incentives

100. Research on performance incentives suggest all of the following EXCEPTa. employees report lower levels of pay satisfaction under pay-for-

performance systems.b. firms with pay-for-performance systems have higher customer service.c. eighty percent of all American companies use merit pay.d. pay-for-performance systems are more effective at retaining higher

performing employees than praise or recognition systems.(a) Difficult/Evaluation

101. A piece rate system a. is a program providing one-time rewards to employees for specific

accomplishments.b. is a company-wide program where employees are rewarded for

performance gains compared to past performance.c. is a permanent pay raise based on past performance.d. is a program where employees are paid on the basis of individual output

produced.(d) Easy/Knowledge

102. A mid-sized manufacturing company produced one thousand component assemblies in July of this year, far exceeding the previous record of 843 assemblies. One week later, the firm had grills brought on site and employees were treated to a steak luncheon. The steak grill-out is an example ofa. an award.b. a bonus.c. gainsharing.d. merit pay.

(a) Easy/Evaluation

103. Which of the following statements regarding performance incentives is NOT true?a. Employees who are heavily rewarded by commissions may neglect

customers who provide them with a low probability of making a sale.

Page 140: Fwk Bauer Tif Organizational Behavior All Chapters

b. When awarded stock options, employees tend to hold on to them rather than sell them.

c. Profit sharing is not very effective at tying employee pay to individual effort.

d. Gainsharing programs can be successful if the payout formula is generous and employees can participate in the management of the company.

(b) Difficult/Evaluation

104. Which of the following statements regarding performance incentives is true?a. Employees tend to hold onto stock options rather than sell them.b. Merit pay is a successful program because employees recognize they will

be rewarded only if performance increases.c. Profit sharing is very effective in tying individual pay to employee effort.d. Companies are increasingly choosing to implement team- or group-based

pay-for-performance programs over individual-based pay-for-performance programs.

(d) Difficult/Evaluation

Section V: The Role of Ethics and National Culture

105. Which of the following statements regarding goal setting and ethics is NOT true?a. Research shows that teachers who are rewarded for their students’

success may give their students answers during tests to improve the test scores.

b. The tendency to behave unethically occurs when employees know they are going to achieve their goals.

c. Employees have two choices when goal accomplishment is rewarded: work hard to reach the goals, or cheat.

d. When a high percentage of a CEO’s pay package consists of stock options, companies are more likely to misrepresent the financial situation of the company.

(b) Difficult/Evaluation

106. Companies can reduce the temptation to behave unethically bya. rewarding only those who reach high goals.b. rewarding everyone including those who completely missed their goals.c. withholding rewards from those who are demonstrating unethical

behaviors.d. providing small recognitions for all behaviors even if the behavior has

unethical overtones so that no employee feels like he has been singled out.

(c) Difficult/Evaluation

107. Which of the following statements is true regarding cultural aspects of motivation in a work environment?a. Mexican employees work very effectively in self-managed teams.b. Indian employees are very satisfied when they are empowered.c. Chinese employees are very motivated when goals are very difficult.d. Peer appraisals are very effective in collectivism cultures.

(c) Difficult/Evaluation

Page 141: Fwk Bauer Tif Organizational Behavior All Chapters

108. Which of the following statements regarding culture and designing a motivating work environment is true?a. countries high in power distance respond positively to appraisal systems

where lower level employees give performance feedback to their managers.

b. the paternalistic nature of Mexican companies makes methods of increasing employee empowerment easy to implement.

c. workers in India tend to respond positively to empowerment.d. in China, goals with low specificity are more motivational than those with

high specificity.(d) Difficult/Evaluation

Closing Section: Motivation is Key for Success: The Case of Xerox

109. All of the following are reasons why Xerox is so effective at motivating its employees EXCEPT:a. the company’s success.b. the firm has a centralized structure that retains decision making at upper

levels in the organization.c. the belief that employees should feel interested in the work they do.d. the family atmosphere in the large company.

(b) Medium/Comprehension

FILL IN THE BLANK

110. The philosophy of _____________ _____________focuses on the most efficient method to perform a job.

(scientific management)

111. ____ _______________ breaks down jobs into their simplest components and assigns them to employees so that they can perform tasks in a repetitive manner.

(Job specialization)

112. An auto assembly worker who works on the left rear wheel bolts on Monday, the right front headlight screws on Tuesday, the front grille connectors on Wednesday, the right rear wheel bolts on Thursday and the left front screws on Friday is working under a ________ ____________ system.

(job rotation)

113. The job characteristics model features the psychological states of _____________, _______________, and _____________________.

(meaningfulness, responsibility, and knowledge of results)

114. An organizational behavior instructor who develops a unique teamwork project for use in her classroom will have high ______________ with the project.

(task identity)

115. __________ ________ ___________ describes the degree to which a person has higher-order needs such as esteem and self-actualization.

(Growth need strength)

Page 142: Fwk Bauer Tif Organizational Behavior All Chapters

116. Employees who feel motivated and have discretion to make decisions about the content and context of their job are __________________.

(empowered)

117. A Harley-Davidson employee’s authority to stop the assembly line if he sees a blemish on the product has ___________________ empowerment.

(structural)

118. The acronym, SMART, stands for___________, ______________,__________, realistic and time-bound when discussing goal setting.

(specific, measurable, aggressive)

119. The degree to which a person is dedicated to reaching the goal is called _______ _____________________.

(goal commitment)

120. ______________________is the process in which a rater or raters evaluate an employee’s performance.

(Performance appraisal)121. ___-_________ _______________ is the process where supervisors, peers,

subordinates, and sometimes even customers provide feedback in a confidential manner to later share with the employee for developmental purposes.

(360-degree feedback)

122. _______ ________ ensures that there is two-way communication during the appraisal process and the employee’s story is heard.

(Fair hearing)

123. If you answer 90% of the questions right on an exam, you get an A, if you get 80% correct, you get a B, and so on. This approach to grading an exam uses an __________ _________ criteria.

(absolute rating)

124. If a manager does not want to have a confrontation with an employee or wants to avoid hurting that employee’s chances of getting a bonus, he may give the employee a rating higher than warranted. This bias in performance appraisal is called ______________.

(leniency)

125. Beliefs about different groups that may be generalized to an employee who is being evaluated, even though those beliefs may have little basis in reality, leads to a _____________ bias in performance evaluation.

(stereotype)

126. History suggests that one of the first incentive plans in place was when Napoleon promised 12,000 francs to anyone who could find a way to preserve food for the army. This is an example of a ____________.

(bonus)

Page 143: Fwk Bauer Tif Organizational Behavior All Chapters

127. Fruit pickers are paid based on the amount of fruit they pick in a day. This is an example of a ______ _______ _______ system.

(piece rate incentive)

128. A one-time reward that follows a specific accomplishment is called a _________.(bonus)

129. The purpose of ________ __________ is to align company and employee interests by making employees owners.

(stock options)

130. _____________ is a company-wide program where employees are rewarded for performance gains compared to past performance.

(Gainsharing)

131. Chinese employees are much more motivated by ____________ goals than their American counterparts.

(difficult)

SHORT ANSWERS

132. Describe some of Frederick Taylor’s observations that led to his development of the scientific management philosophy.

Taylor saw work being done haphazardly. In addition, workers, not managers, generally were in charge of the work being done. Also, he noticed many inefficiencies in the production methods being used, and little planning was carried out prior to production. Finally, worker equipment was not standardized and little training or instruction was provided.

Based on these observations, Taylor offered a way to minimize waste by identifying the most efficient method to perform the job.

133. How do the job design techniques of job enlargement and job enrichment differ?

Job enlargement is the process of expanding the number and kind of tasks performed by the employee to add more variety in his work.

Job enrichment is the process where workers have more control over how they perform their tasks.

The key difference between the two is the nature of control, authority or responsibility the worker has under job enrichment. For example, in job enlargement, the employee may be responsible on an auto assembly line for the entire left rear wheel component. There are, obviously, a number of specific tasks that would be part of producing that component. The job would be enriched, if in addition to completing tasks to produce the left rear wheel component, the employee was also responsible for checking the quality of the assembly arriving at his work station. If the incoming assembly had quality defects, the enriched worker might be able to stop the entire assembly line.

Page 144: Fwk Bauer Tif Organizational Behavior All Chapters

134. Describe the process of job rotation as it applies to an employee in your college’s photocopying center.

A variety of jobs are performed at most college photocopy centers. Clearly exams and professor handouts are produced there, but there are also a number of binding activities, packing large orders, delivery of orders, and so on that are completed there. One example of a job rotation might be if on Monday, employee A would be responsible for receipt and logging of all job orders either sent through campus mail or delivered in person to the center. On Tuesday, employee A might be stationed at the copy machine doing the actual copying of work. That same employee might move to the spiral binding machine on Wednesday and the flat binding machine on Thursday. Friday would find employee A delivering completed jobs to individual offices throughout the campus. Each individual job is broken down to its simple parts for easy training. As the repetition of the tasks occurs, quality improves, as does efficiency. The rotation between the jobs addresses any monotony that might arise from completing the same tasks on a daily basis.

135. List and define the core job characteristics found in the Hackman and Oldham model.

Skill variety is the extent to which the job requires a person to utilize multiple high level skills.

Task identity is the degree to which a person is in charge of completing an identifiable piece of work from start to finish.

Task significance is whether a person’s job substantially affects other people’s work, health or well-being.

Autonomy is the degree to which a person had the freedom to decide how to perform his tasks.

Feedback is the degree to which people learn how effective they are being at work.

136. What is the formula to calculate the motivating potential score (MPS) of a job, and what is the purpose of the score?

MPS = Skill variety + Task Identity + Task significance x Autonomy x Feedback3

The formula suggests that autonomy and feedback are the most important elements in deciding the motivating potential compared to skill variety, task identity and task significance. The purpose of the formula is to assess the motivating potential of a job.

137. What is the difference between felt empowerment and structural empowerment?

Page 145: Fwk Bauer Tif Organizational Behavior All Chapters

Structural empowerment refers to the aspects of the work environment that give employees discretion, autonomy and the ability to do their jobs effectively.

Felt empowerment is the condition where employees find the work to be meaningful, feeling confident that they can perform the job, feeling that they have discretion and autonomy at work and having the ability to influence how the company operates.

138. What does each of the letters in the acronym, SMART, mean?

S is Specific.M is Measurable.A is Aggressive.R is Realistic.T is Time-bound.

139. Write a SMART goal.

To eliminate 25% of the solid waste from US stores by the year 2015 (Wal-Mart)

140. Why do goals motivate?

Goals give us direction. The goals tell you what to focus on; so they should be set carefully. Employee goals should be aligned with company goals.

Goals energize employees. Even if you are tired of working toward a goal, having a specific goal in mind energizes you.

Goals present challenges. When goals are set and people reach them, they feel a sense of accomplishment.

SMART goals urge you to think outside the box and re-think how you are working. Goals have to be challenging and somewhat difficult to motivate.

141. Explain when goals are more effective.

Goals are more effective when there is feedback. The feedback helps indicate the progression to goal accomplishment.

Employees should have the skills, knowledge and abilities to reach their goals.

Employees should be committed to the goals.

Goals should be challenging, not easy, and not extremely difficult.

142. Are there any downsides to goal setting?

Yes, setting goals for specific outcomes may hamper employee performance if those employees lack skills and abilities needed to reach those goals. Therefore,

Page 146: Fwk Bauer Tif Organizational Behavior All Chapters

goals should be set for learning, not outcomes, so the learning decreases can be addressed.

Goal setting may prevent employees from adapting and changing their behaviors in response to unforeseen threats. In short, adaptability declines.

Goals tend to focus employee behavior on activities that are measured, and thus people sacrifice other important elements of performance as a single-mindedness develops.

Finally, an aggressive pursuit of goals can lead to unethical behaviors. If rewards are given only for achievement of goals and not for coming close, some employees may do whatever it takes to achieve that goal.

143. List the steps of the management by objectives (MBO) approach.

1. Make sure individual goals support team goals and team goals support the company strategy.

2. Specifically, company-wide goals should be derived from corporate strategy.

3.Team and department goals are then determined making sure they derive from the company goals.

4. Individual goals are collaboratively set between the employee and the supervisor that are aligned with the corporate strategy.

5. An action plan is developed.

6. The performance goals are periodically reviewed and revised.

144. How does 360-degree feedback differ from the more traditional form?

360-degree feedback is a system where all sources share, in a confidential manner, feedback on an employee for developmental purposes. These sources include peers, coworkers, supervisors and customers. The traditional feedback system usually involves only supervisors

145. What are the three characteristics of appraisals that increase the perception that they are fair?

Adequate notice involves letting employees know what criteria will be used during the appraisal.

Fair hearing means ensuring that two-way communication occurs during the appraisal process and making certain that the employee’s story is heard.

Judgment based on evidence involves documenting performance problems and using factual evidence as opposed to personal opinions when rating performance.

Page 147: Fwk Bauer Tif Organizational Behavior All Chapters

146. Differentiate between absolute and relative ranking.

Absolute rating is the situation when your performance is rated based upon some objective criteria.

Relative rating is when your rating depends on how your objective performance compares with the rest of the group evaluated.

147. Name three common biases in performance appraisals.

Liking is when the rater and the ratee have an existing relationship. The feelings of like or dislike can bias the ratings.

Leniency is when managers give employees ratings that are higher than what is warranted.

Stereotypes are beliefs about different groups that are generalized to the person in question even if that belief has little basis in reality.

148. Name two types of individual incentives and describe each.

Piece rate incentives are when employees are paid on the basis of individual output produced.

Bonuses are onetime rewards that follow specific accomplishments of employees.

Merit pay involves giving employees permanent pay raises based on past performance.

Sales commissions reward sales employees with a percentage of sales volume or profit generated.

Awards are methods that convey appreciation for worker performance and include such items as plaques, mugs, and gift cards.

149. Name and describe two group- or team-based incentives.

Team bonuses are one-time rewards that follow specific accomplishments of teams of employees.

Gainsharing is a company-wide program where employees are rewarded for performance gains compared to past performance. An example of this is labor cost savings.

Profit sharing involves sharing a percentage of company profits with all employees.

Stock options give an employee the right, but not the obligation, to purchase company stocks at a predetermined price.

Page 148: Fwk Bauer Tif Organizational Behavior All Chapters

ESSAY

150. Using the concepts of job design, goal setting and incentive systems, discuss how Nucor Steel Company successfully motivates its employees.

Nucor employees act almost like owners on their jobs. They are encouraged to fix things when they see they are wrong and have appropriate power to do so. Nucor Steel is very well known for pushing authority and responsibility down to the lowest levels in the hierarchy, which shows how effectively the firm has designed jobs for enrichment.

Incentive systems at Nucor are so effective that Nucor employees are the highest paid steel workers in the world. Much of the earnings comes from ties to performance; in fact everyone, including the CEO, has pay tied to corporate performance. The system is such that low performers are penalized while high performance is rewarded and employees become increasingly committed to the firm.

The formula for Nucor success is simple: Align company goals to employee goals and give employees power to make things happen. The employee turnover rate is one of the lowest in the industry and the firm one of the most successful.

151. Choose a job. Using the Hackman and Oldham job characteristics model, analyze that job for its motivational properties. Comment on ways to enhance the job’s motivational properties and potential.

Example job is college instructor.

Skill variety of the job would be high as instructors teach, develop materials, tutor, counsel, etc.

Task identity is high since the instructor has her own course and teaches it as she sees fit throughout the entire semester or term.

Task significance is also high as the instructor’s job affects students’ well-being among others.

Autonomy is high as instructors have the freedom to choose their own textbooks, make their own handouts, exams, etc.

Feedback is high as instructors are evaluated formally by their students, by their deans and by other groups including informally on websites such as ratemyprofessor.com.

These five factors lead to the instructors perceiving high meaningfulness in their work where they are responsible for outcomes (student grades) and acquire knowledge of results (how the students perform as well as how the instructor performs).

Page 149: Fwk Bauer Tif Organizational Behavior All Chapters

This, in turn, leads to overall job satisfaction, internal motivation, higher performance and lower absenteeism and turnover.

Enhancing the motivational potential could occur through increasing autonomy and feedback to their highest levels possible.

152. Jerome had been working in an accounting department for nine months and has not received much feedback from his supervisor. What steps could he undertake to increase the feedback he is receiving?

Jerome can first seek the feedback, instead of trying to guess how he is doing. Jerome’s manager will become aware that Jerome cares about his performance and wants to be successful.

Jerome needs to be genuine in his desire to learn. He does not want to suggest to the manager that he is only giving an impression of being motivated, but is sincerely interested in improving his performance.

Jerome needs to develop a good relationship with his manager so that he can ask questions about his performance.

Jerome should also find trustworthy peers who can share information regarding his performance.

Finally, Jerome should be gracious in receiving his feedback.

153. Matilda is a new sales manager in a pharmaceutical organization. She decides that she wants to implement an MBO (management by objectives) approach in that department. Describe the steps she will complete to institute such a program.

Matilda will begin the process by setting company-wide goals derived from the pharmaceutical organization’s strategy. From that, team- and department-level goals are set. Next individual level goals will be collaboratively set between Matilda and her employee. Both Matilda and the employee must ensure that the goals are aligned with corporate strategy. Once the goals are set, an action plan is developed and implemented. Periodically, performance is reviewed and goals revised as needed.

154. Brett has been a manager of the accounting department for eight months now. He will be holding his first performance appraisal meetings in the next few days. Describe some ways in which he could enhance the effectiveness of those performance appraisal meetings.

Present feedback in a constructive manner, focusing not on criticizing the person, but on discussing performance problems.

Increase employee participation throughout the meeting. When employees have the opportunity to present their side of the story, they react more positively to the overall appraisal process and feel the system to be fair.

Page 150: Fwk Bauer Tif Organizational Behavior All Chapters

Be knowledgeable about the employee’s performance and the job that is being evaluated.

Prepare before the actual meeting. You might have the employee complete a self-appraisal. The rater definitely should have his form completed before the meeting with specific examples of each kind of behavior evaluated indicated on that form. Also, handle all the logistics of that meeting, including having sufficient time to actually conduct the meeting.

During the actual meeting, show empathy and support for the employee. One way to do so is to avoid opening the meeting with a criticism of the employee. Keep in mind that criticism of performance is acceptable but not of the person.

After the meeting, make a conscious attempt to maintain a steady feedback schedule with the employee and to follow through on all goals set during the appraisal meeting.

155. Harvard uses a forced choice ranking system in evaluating a number of classes in its business curriculum. In this system, for example, 10% of the students might receive As, 20% Bs, 60% Cs and 10% Ds. Discuss the issues surrounding such a grading system.

Students admitted to Harvard are already outstanding students or high performers. To then place them in a situation where they are forced into recognition that some of them are not high performers, but essentially “failures” is problematic. While some students clearly will not be able to make the transition from high school to Harvard, a large number of them will. To assume that only 10% of them will perform well enough to receive As is a bit of a stretch. Those students who have worked very hard and do not receive the grades they believe they deserve due to these artificial standards could experience a real lack of motivation. In general, this system may be counterproductive in the educational setting.

156. Discuss an incentive system that an inner city school system might use to motivate teachers.

Merit pay is an incentive system being considered by a number of school systems. The system involves giving employees a permanent increase in pay based upon past performance. The notion of how to measure such performance in the education setting is, of course, controversial. Some options being instituted include rewarding teachers for increased standardized test scores and higher graduation rates. A formula is established whereby for every point higher a class scores on a standardized test compared to the previous year, a certain bonus is paid.

157. Discuss some of the research findings on goal setting and ethical behavior.

In general, goal setting and rewarding employees for achievement of those goals are successfully implemented by firms. However, in some settings, unintended consequences arise where unethical behavior arises. If goal accomplishment

Page 151: Fwk Bauer Tif Organizational Behavior All Chapters

leads to rewards and the rewards are desirable, employees may feel they have only two choices: work hard to achieve the goals, or cheat.

There is a multitude of examples of unethical behavior undertaken by employees to achieve desirable rewards. Sanitation workers have taken trucks far over their legal limit to landfills, car repair places have found phantom problems with cars brought in for a check-up and CEOs have exercised stock options at questionable times, to provide just a few examples.

To address some of these concerns, it has been proposed that reward achievement not be an all-or-nothing proposition, but that various reward levels be established to address those who fall just short of the established goal.

158. Discuss some of the research on goal setting and performance appraisal in other cultures.

There are differences in the motivational impact of goal setting, depending on the culture in question. American employees need challenging but not difficult goals to be motivated. The Chinese, in contrast, are more motivated when the goals are difficult.

Specific goals motivate salespeople from Western cultures. Again, in China, low-specificity goals are more motivational.

Page 152: Fwk Bauer Tif Organizational Behavior All Chapters

Organizational Behavior, Version 1.1Bauer & Erdogan

FWK Test Item FileChapter 7

TRUE/FALSE

1. There are a variety of ways to lessen stress in the workplace.(True)

2. Stress, or the body’s response to a change that requires a physical, mental or emotional response, is always bad for the individual.

(False)

3. Eighty percent of all American workers report they feel workplace stress at least some of the time.

(True)

4. The amygdala section of the brain is responsible for stimulating fear responses.(True)

5. The human brain responds to outside threats to individual safety with a “fight or flight” message.

(True)

6. The General Adaptation Syndrome looks at how events on the job cause different kinds of people to feel different kinds of emotions.

(False)

7. In the resistance phase of stress, an outside stressor jolts an individual insisting that something must be done.

(False)

8. John’s company downsized, and John is now performing his own job plus most of his former coworker Sidney’s job. John is now experiencing role ambiguity.

(False)

9. Role conflict is a result of contradictory demands at work.(True)

10. Role conflict is the strongest predictor of poor performance on the job.(False)

11. Information overload negatively impacts employee efficiency and creativity.(True)

Page 153: Fwk Bauer Tif Organizational Behavior All Chapters

12. A Health magazine survey indicates that the most stressful job in 2008 was that of police officer.

(False)

13. Overall, work-family conflict is more problematic for men than women.(False)

14. On the Holmes-Rahe scale, the most stressful life event is the death of a spouse.(True)

15. Eustress is “good” stress.(True)

16. Downsizing or the expectation of downsizing is related to greater absence in the workforce.

(True)

17. The outcomes of stress are only psychological in their orientation.(False)

18. Blemishes and other skin problems can be outward manifestations of stress.(True)

19. The link between stress and heart disease has been proven by the American Heart Association to be strong.

(False)

20. Persistent stress has the potential to place vulnerable individuals at an increased risk for depression.

(True)

21. Research shows that excessive stress is related to higher turnover and lower job performance.

(True)

22. Marissa is generally a few minutes late for class. She feels that “class will be there, why rush?” Marissa always thinks through how to address problems that arise for her rather than responding first and thinking later. Marissa has a Type A personality.

(False)

23. Men rely upon stronger social networks to address stressful situations, so men are more likely to be depressed than women.

(False)

24. The old adage “Have a good cry” is great advice to deal with stress that has built up in an individual.

(False)

25. The “Corporate Athlete” approach to dealing with stress is a reactive approach.(False)

Page 154: Fwk Bauer Tif Organizational Behavior All Chapters

26. Flow is the state of consciousness where individuals are totally absorbed in an activity.

(True)

27. Fish contains dopamine, which can increase a person’s brain functions and thus produce feelings of alertness.

(True)

28. Physically active work breaks lead to reduced mental concentration levels and increased mental fatigue.

(False)

29. Most American adults get the appropriate amount of sleep each night.(False)

30. Individuals with a strong social network are more stressed than individuals who do not have such networks because of the increased interpersonal demands of large social networks.

(False)

31. Stress-related issues cost companies billions of dollars annually due to absenteeism, lost productivity, and accidents.

(True)

32. Organizations can help employees reduce stress by offering them greater autonomy in their jobs, making their job duties clear, and providing employee assistance programs.

(True)

33. Firms like Apple are interested in how products are perceived emotionally because strong positive emotions make individuals more likely to use a product and recommend it to others.

(True)

34. Both positive and negative emotions can be contagious, but the spillover from positive emotions lasts longer than negative ones.

(False)

35. According to Affective Events Theory, the six emotions of anger, fear, joy, love, sadness and surprise inspire actions that can benefit or harm others.

(True)

36. The customer service representative at the department store who keeps smiling as she listens to the customer bitterly complain about her newly purchased product is engaging in deep acting.

(False)

37. Americans enjoy much greater leisure time than their European counterparts.(False)

Page 155: Fwk Bauer Tif Organizational Behavior All Chapters

38. Recognizing that customers’ negative emotions were eroding sales representatives’ morale, American Express used emotional resonance to identify employees’ feelings about their work.

(True)

39. American Express’ emotional awareness training program probably raised overall life insurance sales because sales representatives began to view their jobs from the customer’s viewpoint.

(True)

MULTIPLE CHOICE

Opening Section: Facing Foreclosure: The Case of Camden Property Trust

40. What is Camden Property Trust’s motto:a. Have fun!b. Work hard!c. Money is the most important thing!d. You can never work too hard!

(a) Medium/Comprehension

Section I: What is Stress?

41. Stress isa. a fact of nature.b. decreasing in the American workplace.c. a mental barrier, but not a physical one.d. always negative.

(a) Easy/Comprehension

42. Which of the following is not a type of response to stress?a. physicalb. mentalc. emotionald. societal

(d) Easy/Knowledge

43. General Adaptive Syndrome isa. how events on the job cause different kinds of people to feel different

emotions.b. the hypothesis that stress plays a general role in disease.c. the body’s response to good news.d. the mismatch among emotions, attitudes, beliefs and behavior.

(a) Easy/Knowledge

44. Which region of the brain is responsible for stimulating the fear response?a. hippocampus

Page 156: Fwk Bauer Tif Organizational Behavior All Chapters

b. basal gangliac. temporal lobed. amygdala

(d) Easy/Knowledge

45. Hans Selye’s work on stress suggests all the following physical diseases could result from stress EXCEPT:a. high blood pressure.b. infertility.c. depression.d. ulcers.

(b) Easy/Comprehension

46. In what stress phase does the body release cortisol and begin to adjust to the demands of the stress?a. alarm phaseb. exhaustion phasec. adaptation phased. resistance phase

(d) Medium/Knowledge

47. Fight or flight responses include all of the following physical aspects buta. wide-eye focus.b. digestion interruption.c. decreased heart rate.d. shallow breathing.

(c) Medium/Knowledge

It is finals week at State University and Agnes has five finals in three days. What phase of the stress process is she experiencing in each of the scenarios below?

48. Agnes just went through the kitchen cupboards and found a chocolate bar loaded with caramel. She is experiencing the a. exhaustion phase.b. alarm phase.c. resistance phase.d. adaptation phase.

(a) Medium/Application

49. Agnes glances at the clock and notes it is three hours until her first final of the week. She still has two chapters to study. She feels a slight pain in the pit of her stomach. Agnes is experiencing symptoms of what stress phase?a. exhaustion phaseb. alarm phasec. resistance phased. stressor phase

(b) Medium/Application

50. Which of the following statements about the stress process is correct?

Page 157: Fwk Bauer Tif Organizational Behavior All Chapters

a. The amygdala area of the brain is key to managing stress as it helps distinguish the different reactions to deadlines and emergencies.

b. Reactions to stress are mental but not physical in their orientation.c. Jans Kelly’s General Adaptation Syndrome is a well-substantiated

approach to dealing with stress in the workplace.d. Eustress is stress that is healing.

(d) Medium/Comprehension

51. Stressors area. demands to perform interactions and internal calculations that exceed the

supply or capacity of time available for processing.b. states of consciousness in which a person is totally absorbed in an

activity.c. events on the job which cause different people to feel different emotions.d. events or contexts that cause a stress reaction by elevating levels of

adrenaline and forcing a physical or mental response.(d) Easy/Knowledge

52. Which of the following statements about stressors is INCORRECT?a. An American Psychological Association survey suggests that money,

work, and housing are at the top of stressful issues.b. No matter how they are categorized, stressors are always bad for

individuals.c. Stressors cause stress by elevating adrenaline levels and forcing a

response.d. Stress is a cumulative process.

(b) Medium/Comprehension

53. Role ambiguity isa. vagueness in relation to job responsibilities.b. insufficient time and resources to complete a job.c. contradictory demands at work.d. processing demands that exceed the supply of time for such processing.

(a) Easy/Knowledge

54. Facing contradictory demands at work is a form ofa. role ambiguity.b. role conflict.c. role overload.d. information overload.

(b) Easy/Knowledge

55. Role overload isa. inability to reconcile the demands of one’s work and home life. b. facing contradictory demands at work.c. having insufficient time and resources to complete a job.d. vagueness in relation to what job responsibilities are.

(c) Easy/Knowledge

56. Information processing demands that exceed the supply or capacity of time available for such processing is

Page 158: Fwk Bauer Tif Organizational Behavior All Chapters

a. role conflict.b. role ambiguity.c. role overload.d. information overload.

(d) Medium/Application

57. Mary is a new employee in the handbag department of a major department store. She is not entirely sure whether she is to simply stay on the sales floor and sell purses to customers, or if she is to sell the items and then go back into the stockroom to replenish them. Mary is experiencinga. information overload.b. role ambiguity.c. role overload.d. role conflict.

(b) Medium/Application

58. Rodney is experiencing a real dilemma. His boss just called him and told him that he had to attend a dinner meeting Wednesday night with some important out-of-town clients. Rodney’s daughter just called and told him she will be dancing a solo at her dance recital on Wednesday evening. Rodney is experiencinga. role conflict.b. role overload.c. information overload.d. role ambiguity.

(a) Medium/Application

59. Joe was diagnosed with cancer six months ago. He is back at work between chemotherapy treatments but is unable to work more than four hours per day. Diego has been covering all Joe’s assignments that extend beyond Joe’s four-hour day as well as covering the regular tasks of his own eight-hour shift. Diego is likely experiencinga. role ambiguity.b. role overload.c. role conflict.d. information overload.

(b) Medium/Application

60. Omar is sitting at his computer writing his organizational behavior term paper using Internet research while he e-mails his friends and watches the NFL Thursday night football game. His phone just rang and now he is also talking to his girlfriend. Omar is likely experiencinga. role conflict.b. information overload.c. role ambiguity.d. role overload.

(b) Medium/Application

61. Research on workplace stressors indicatesa. role conflict is the strongest predictor of poor performance.b. when role ambiguity is low, new employees have a difficult time fitting into

their new organizations.

Page 159: Fwk Bauer Tif Organizational Behavior All Chapters

c. role conflict hurts job performance, but role overload has little impact on job performance.

d. the fragmented fashion in which we work due to information overload impacts mental acuity and creativity.

(d) Difficult/Evaluation

62. All of the following are key stressors for Health magazine’s “Ten Most Stressful Jobs” EXCEPTa. knowledge demands.b. time demands.c. emotional demands.d. physical demands.

(a) Medium/Comprehension

63. Which is the most stressful job in America as noted in a 2008 Health magazine?a. minerb. inner city high school teacherc. police officerd. judge

(b) Medium/Knowledge

64. Which of the following statements regarding work-family conflict is INCORRECT?a. Work demands and family demands may be incompatible.b. Escalating stressors are found where stress in one area of life spills over

into other parts of life.c. Work-family conflict is greater for men than for women.d. Dual career families have created situations where there is no parent who

has sole responsibility for childcare or household duties.(c) Medium/Analysis

65. Which of the following statements regarding work-family conflict is correct?a. Work-family conflict increases job and life satisfaction.b. Organizations that have programs to assist employees in achieving work-

family balance accept that these programs will reduce productivity while increasing retention.

c. Recent surveys show that Americans rarely work overtime. d. Work-family conflict occurs when the demands of work and family

negatively impact each other.(d) Difficult/Comprehension

66. Which of the following statements regarding life changes is INCORRECT?a. Stress results from negative life changes but not from positive ones.b. The Holmes-Rahe scale ascribes stress values to life events based upon

incidences of illness and death in the 12 months after such events.c. Stressors are cumulative so the greater number of stressors you

experience, the more prone you are to negative consequences of those stressors.

d. Life events can have varying impacts on our health and well-being.(a) Medium/Evaluation

Page 160: Fwk Bauer Tif Organizational Behavior All Chapters

67. Research on downsizing indicates all of the following to be true about downsizing EXCEPTa. since 1980, the service industry has accounted for the largest average

percentage of firms which downsized five or more percent of their workforces.

b. downsizing and job insecurity are related to greater alcohol use.c. downsizing leads to other stressful events such as financial insecurity.d. creativity in the work environment declines significantly during

downsizing.(a) Medium/Evaluation

68. Which of the following statements regarding life stressors is correct?a. Work-family conflict is slightly greater for men than women.b. Expectations of future downsizing leads to greater absenteeism in the

workplace.c. Stress results only from negative life events; positive life events increase

feelings of satisfaction.d. Organizations cannot aid employees in dealing with work-family conflicts

as any interference from the firm would be seen as an invasion of privacy.(b) Difficult/Synthesis

69. Which of the following is not a physiological outcome of stress?a. heart diseaseb. immune system suppressionc. skin problemsd. headaches

(a) Easy/Knowledge

70. Which of the following is not an established psychological outcome of stress?a. emotional problemsb. anxietyc. phobiasd. depression

(c) Easy/Knowledge

71. Research on stress in the workplace indicates all of the following EXCEPTa. stress is related to higher turnover.b. stress is related to decreased job performance.c. stress is related to lower job commitment.d. stress is related to higher organizational citizenship behaviors.

(d) Medium/Analysis

72. Marcus is a student at State University. He seems to run to each of his classes and is very upset with the professor if class does not begin exactly on time. He studies until the late hours of the night and will argue with his professor for every possible point on a multiple choice exam. Marcus would be classified as havinga. Type A personality.b. Type B personality.c. pessimistic personality. d. neurotic personality.

(a) Medium/Application

Page 161: Fwk Bauer Tif Organizational Behavior All Chapters

73. Anita seems to take life as it comes. When the refrigerator and washing machine broke down in the same day flooding the kitchen, she laughed it off saying, “It could always be worse.” She then went and called the appliance repair service. Anita has a _________________ personality.a. Type Ab. Type Bc. pessimistic.d. neurotic

(b) Medium/Application

74. A Type B personality displays which of the following characteristics?a. impatience.b. deep levels of job involvement.c. high level competitivenessd. logical decision making.

(d) Easy/Knowledge

75. Research on individual differences in the type of stress experienced by employees indicates all of the following EXCEPTa. babies may be predisposed to a more stressful life from birth if mothers

experience a great deal of stress during pregnancy.b. due to their stronger social networks, women process stress more

effectively than men.c. men become depressed more often than women.d. individuals with Type A personalities are more likely to experience

negative organizational outcomes.(c) Medium/Evaluation

76. Research on crying as a stress reliever suggestsa. the old adage is true: “Let it all out with a good cry.”b. criers are less depressed, less anxious and less tired than those who

wept less.c. crying is much more successful than humor at combating stress.d. crying may intensify negative feelings because it is a social signal that the

individual weeping is very upset.(d) Medium/Evaluation

Section II: Avoiding and Managing Stress

77. The “corporate athlete approach” to stress isa. a reactive approach to the problemb. to avoid challenges so as not to be overwhelmed by them.c. to train mind and body to perform at peak levels by using better nutrition,

and positive action.d. a technique to change eustress into distress.

(c) Medium/Comprehension

78. Flow isa. applying the principles of athletic performance to workplace performance.b. a state of consciousness where a person is totally absorbed in an activity.

Page 162: Fwk Bauer Tif Organizational Behavior All Chapters

c. the regulation of feelings and expressions for organizational purposes.d. ongoing negative emotional states resulting from dissatisfaction.

(b) Easy/Knowledge

79. Gordon is not focused on his work but has a great deal of energy. He is most likely to a. be disengaged at work.b. be distracted at work.c. be a procrastinator.d. be totally absorbed in work.

(b) Difficult/Synthesis

80. A manager with low focus and low energy woulda. procrastinate at work.b. be distracted at work.c. be disengaged at work.d. be totally absorbed in work.

(a) Difficult/Evaluation

81. Work that flowsa. offers opportunities for a large number of breaks.b. offers the employee little opportunity to comment on the task at hand.c. offers high pay.d. offers opportunities to use skills possessed by the employee.

(d) Medium/Comprehension

82. Work that flows includes all of the following characteristics EXCEPTa. competence.b. challenge.c. conformity.d. choice.

(c) Medium/Comprehension

83. Diet acts as an individual lifestyle choice to impact stress levels. Which of the following statements about diet is INCORRECT?a. Eating light meals in the middle of the day may slow down the body

because there is insufficient food energy available to stimulate the brain to react to stress.

b. Greasy foods make a person feel tired because digestion of the fats in such foods diverts blood from the brain.

c. The dopamine in fish produces a feeling of alertness.d. Eating fish for lunch tends to improves reaction times.

(a) Medium/Evaluation

84. Research indicates that exercise is an effective strategy for managing stress. Which of the following statements best describes some of the findings from studies on exercise and stress?a. Physically active breaks, like walking, actually increase mental fatigue.b. Exercise, like light calisthenics, actually absorbs energy.c. Light exercise tends to erode hand-eye coordination as well as fine motor

control.

Page 163: Fwk Bauer Tif Organizational Behavior All Chapters

d. Regular exercise increases the body’s ability to draw oxygen out of the air, thus combating stress.

(d) Medium/Evaluation

85. Sleep research on stress indicates all of the following EXCEPTa. workers who miss work because of exhaustion become rejuvenated and

face less stress upon their return to work.b. about one third of adults have trouble sleeping.c. work-life conflict makes good sleep difficult to achieve.d. insomnia is a stress-related epidemic.

(a) Medium/Comprehension

86. Which of the following statements about individual approaches to managing stress is INCORRECT?a. Time management practices like prioritizing and keeping a schedule help

reduce stress.b. Social support can buffer the effects of stress.c. When workdays are broken into smaller parts, stress can be shifted to a

state of flow.d. Physical activity increases mental fatigue, which increases stress levels.

(d) Difficult/Synthesis

87. Stress in the form of job challenge leads to: a. Higher turnoverb. Higher performance.c. Higher absenteeism.d. Higher commitment.

(b) Medium/Comprehension

88. All of the following are steps that firms are taking to assist employees in dealing with stress in the workplace EXCEPTa. creating jobs with autonomy and control.b. replacing employees who are stressed with more resilient employees.c. ensuring fairness in the workplace.d. clarifying expectations of employees.

(b) Medium/Comprehension

89. Which of the following is a technique used by firms to reduce the stress experienced by their employees?a. Create job descriptions for employees that are less specific and focused.b. Shift control over critical decisions to higher levels in the organization.c. Base pay decisions on tenure as opposed to performance. d. Train managers to be more interpersonally sensitive.

(d) Difficult/Synthesis

90. Research on telecommuting indicates which of the following?a. Telecommuting decreases stress but increases work family conflict.b. Telecommuting increases job satisfaction but decreases job performance.c. Telecommuting increases job performance and decreases turnover.d. Telecommuting decreases stress but also decreases job performance.

(c) Difficult/Evaluation

Page 164: Fwk Bauer Tif Organizational Behavior All Chapters

91. All of the following statements regarding telecommuting are correct EXCEPTa. telecommuting is a universally effective process; regardless of job or

employee, it is effective.b. increasing numbers of workers, including many government employees

are engaged in telecommuting.c. research on telecommuting indicates that stress is lower and job

satisfaction and job performance higher when the program is instituted.d. telecommuting success is dependent on matching the right employee to

the right job in the right environment.(a) Difficult/Evaluation

92. Which of the following statements regarding organizational steps to managing employee stress is correct?a. Sabbaticals are stress-reduction programs found only in the university

setting.b. Employee assistance programs are found only in large Fortune 500

companies.c. Telecommuting decreases stress, but increases work-family conflict

because the employee is working from home.d. Employees experience less stress when their assignments are clear.

(d) Difficult/Evaluation

93. Which of the following statements regarding organizational steps to managing employee stress is correct?a. Employee assistance programs often address legal or financial problems

of the employee.b. Employee autonomy increases stress and decreases job satisfaction.c. Company sabbaticals are paid vacations for extended periods of time. d. Organizations’ that have a fair environment are less profitable but have

higher employee retention rates. (a) Difficult/Evaluation

Section III: Emotions

94. Which of the following is not a negative emotion?a. angerb. fearc. surprised. sadness

(c) Easy/Knowledge

95. Which of the following statements regarding positive emotions is INCORRECT?a. Positive feelings dispose an individual to optimism but make him less

open to new ideas.b. Individuals feeling positive emotions like contentment experience a

sensation of having something they did not have before.c. Firms are interested in the emotional reaction their products create

because those who experience strong positive emotions for a product tend to recommend it to others.

Page 165: Fwk Bauer Tif Organizational Behavior All Chapters

d. Having positive emotions can lead to upward positive spirals of good emotions.

(a) Difficult/Synthesis

96. Which of the following statements about negative emotions is INCORRECT?a. Negative emotions like anger can result from an event such as not having

your opinions heard by your supervisor.b. Employees who manage their negative emotions are involved in fewer

conflicts than employees who do not.c. Negative emotions that are not managed can become the norm in a

workplace.d. Negative emotions are always destructive in the workplace.

(d) Medium/Evaluation

97. Which of the following statements regarding research on emotional contagion is INCORRECT?a. Both positive and negative emotions can be contagious.b. Some people are more susceptible to emotional contagion than others.c. The spillover from positive emotions lasts longer than that from negative

emotions.d. Emotions are especially salient in teams and thus teams can exhibit

emotional contagion just as individuals can.(c) Medium/Evaluation

Section IV: Emotions at Work

98. Which of the following is NOT an emotion that is part of the Affective Events Theory?a. jealousyb. surprisec. angerd. fear

(a) Easy/Knowledge

99. Jennifer strongly dislikes her next-door neighbors, yet every time she sees them she stops to pleasantly chat with them for a few moments. Jennifer is engaging in a. genuine acting.b. surface acting.c. deep acting.d. cognitive acting.

(b) Medium/Application

100. When an individual is displaying emotions he does not feel, he is engaging ina. deep acting.b. surface acting.c. genuine acting.d. cognitive dissonance.

(a) Easy/Knowledge

101. Genuine acting is a. pretending to experience emotions not really felt.

Page 166: Fwk Bauer Tif Organizational Behavior All Chapters

b. exhibiting physical signs that reflect emotions not felt.c. displaying emotions in alignment with your emotions.d. a mismatch of emotions.

(c) Easy/Knowledge

102. You are a salesperson in a major department store. Though you might not actually believe it, you follow the policy of “the customer is always right” in your daily work at the store. However, since you do not agree with that view, you often experiencea. surface acting.b. genuine acting.c. deep actingd. cognitive dissonance.

(d) Medium/Application

103. Which of the following is not a way to alleviate cognitive dissonance?a. Change your behavior.b. Change the facts of the situation.c. Change your belief.d. Smile more.

(d) Easy/Comprehension

104. Which of the following is NOT a building block of emotional intelligence?a. self-awarenessb. business managementc. relationship managementd. self-management

(b) Easy/Knowledge

105. When you understand how OTHERS feel you are exhibiting which of the building blocks of emotional intelligence?a. self-awarenessb. self-managementc. social awarenessd. relationship management

(c) Easy/Comprehension

106. Marcus knows that he is flexible, enjoys challenging goals, and is very concerned about the security of his job. Marcus understands which of the following building blocks of the emotional intelligence concept?a. self-managementb. relationship managementc. social awarenessd. self- awareness

(d) Medium/Application

107. Research on emotional intelligence suggests that the emotion most helpful to creating a successful team isa. joy.b. empathy.c. flexibility.

Page 167: Fwk Bauer Tif Organizational Behavior All Chapters

d. self-control.(b) Easy/Comprehension

108. Research suggests high levels of emotional intelligence lead to which of the following?a. higher life satisfaction.b. cognitive dissonancec. higher stress levels.d. greater propensity to perceive situations as threats.

(a) Medium/Comprehension

Section IV: The Role of Emotions and Stress for Ethics and Culture

109. Joshua Greene’s study of how our minds and bodies react to difficult situations found all of the following EXCEPTa. Study subjects were able to agree on what was the best of possible evils

in a scenario. b. Human decisions are mostly rational, not impacted by emotion.c. Magnetic imaging suggests emotional scenarios require greater use of

our brains than unemotional scenarios.d. Identification of our emotions can assist us in more appropriately

responding to emotional situations.(b) Difficult/Synthesis

110. Research on stress and leisure time around the world suggests which of the following? a. Compared to Europeans, Americans work longer hours. b. Japanese work 500 hours less than their counterparts in the USA.c. Americans are better in time management compared to their European counterparts. d. Americans are less likely to be employed than Europeans.

(a) Easy/Knowledge

Closing Section: Getting Emotional: The Case of American Express

111. What impact did emotional awareness training have on employees at American Express?a. The training increased sales.b. The training decreased coping skills.c. The training taught aggressive sales tactics to overcome the negative

responses of customers.d. The training created greater barriers between the customers and the

sales representatives.(a) Medium/Comprehension

112. Analysis of emotional training suggestsa. such training can increase a firm’s bottom line.b. such training decreases coping skills in a firm.c. that companies cannot profit if employees recognize clients’ feelings.

Page 168: Fwk Bauer Tif Organizational Behavior All Chapters

d. employees rate caring bosses just below salary increases.(a) Medium/Analysis

113. Emotional resonancea. is a technique that ignores the negative emotions of clients.b. is the gap between individuals’ words and their true feelings.c. is a technique that identifies employees’ feelings about their work.d. Is an emotional barrier between individuals.

(c) Easy/Knowledge

FILL IN THE BLANK

114. American Express used a technique called _________ _______ to identify its employees’ feelings about their work.

(emotional resonance)

115. ________ is the body’s reaction to a change that requires a physical, mental or emotional adjustment or response.

(Stress)

116. The _______________ is the region of the brain responsible for stimulating fear responses.

(amygdala)

117. The _____________ _____________ _____________ is a hypothesis by Hans Selye that suggests that stress plays a general role in disease by exhausting the body’s immune system.

(General Adaptation Syndrome)

118. “Fight or flight” is the reaction to an outside stressor in the first phase, or ____________ phase of stress.

(alarm)

119. During the resistance phase of stress, the body begins to release _________ and draws on fat and sugar reserves to adjust to the demands of stress.

(cortisol)

120. The stressor has significantly weakened the individual during the ___________ phase of stress.

(exhaustion)

121. ________ _______ refers to a vagueness as to what your responsibilities are.(Role ambiguity)

122. Maggie’s boss wants her to complete the department project by next week. Her team leader for the corporate project wants her to travel to New York in the next few days to work on that item. Maggie is experiencing _____ ____________.

(role conflict)

123. Having insufficient time and resources to complete a job is called ______ _________.

Page 169: Fwk Bauer Tif Organizational Behavior All Chapters

(role overload)

124. You are at the computer working on the Internet gathering information for your term paper, your cell phone is ringing, you are receiving emails and the television is on. You are likely experiencing __________ _____________.

(information overload)

125. Flexible scheduling is one approach to addressing _________-______ conflict.(work-family)

126. _________ is “good” stress experienced for such events as an outstanding personal achievement or getting married.

(Eustress)

127. Nancy is always impatient. She seems to run around the office completing this task and that. She is always the first one to work and the last to leave. Nancy has a ________ ___ personality.

(Type A)

128. The state of consciousness when an individual is totally absorbed by an activity is ____________.

(flow)

129. Two individual lifestyle choices that one can make to try to decrease feelings of stress include: ____________ and ______________.

(diet, exercise, sleep, time management, social support network)

130. _____________ is working remotely from home or some other locale during a portion of the workweek.

(Telecommuting)

131. A practice that has been widely used by universities that provides paid time off from the normal routine of work is called a ______________.

(sabbatical)

132. Individuals who are experiencing substance abuse problems may seek help provided by their firms but offered through an outside source in an _____________ _____________ program.

(Employee assistance)

133. Positive emotions include __________, _______ and ___________.(joy, love, surprise)

134. _____, ______, and ______ are examples of negative emotions.(anger, fear, and sadness)

135. The _______ _______ Theory explores how events on the job cause different kinds of people to feel different emotions.

(Affective Events)

Page 170: Fwk Bauer Tif Organizational Behavior All Chapters

136. When a hair cutter smiles even though the young child whose hair she is cutting is crying and squirming, the hair cutting is exhibiting __________.

(surface acting)

137. ________ __________ is the regulation of feelings and expressions for organizational purposes.

(Emotional labor)

138. When a person pretends to experience emotions he does not feel, he is engaging in _________.

(deep acting)

139. The mismatch between emotions, attitudes, beliefs and behavior is ______ ____________.

(cognitive dissonance)

140. The subject of __________ ____________, examined by Daniel Goleman, looks at how people can understand each other more completely by becoming more aware of their own and others’ emotions.

(emotional intelligence)

SHORT ANSWERS

141. Define stress and briefly discuss its prevalence in the workplace.

Stress is the body’s reaction to a change that requires a physical, mental, or emotional adjustment or response. Today, stress is inevitable and everywhere. While we can’t change stress, we can change the manner in which we respond to it.

Polls indicate that 80% of American workers report experiencing stress in the workplace at least some of the time. Stress in the workplace is manifested through such physical effects as neck pain, aching muscles and insomnia.

142. Describe each phase of Hans Selye’s General Adaptation Syndrome.

Phase one is the alarm phase where an outside stressor jolts the individual insisting that something be done. This is the well- known “fight or flight” response.

Resistance is the phase where the body begins to release cortisol and draws on fats and sugars for energy to adjust to the demands of stress. While the body can draw on its reserves to hold off stress for a while, it cannot continue in this state forever.

Phase three is the exhaustion phase where the body’s stores of fats and sugars are depleted and the prolonged release of cortisol has weakened the body considerably. To cope, many individuals reach for something sweet to restore the sugars.

Page 171: Fwk Bauer Tif Organizational Behavior All Chapters

143. Name two workplace stressors and give an example of each.

Workplace stressors include: role demands (role ambiguity, role overload, role conflict), and information overload.

Role ambiguity is the vagueness one feels about what responsibilities he holds. For example, when you begin a new job, you are often unclear what you are supposed to do.

Role conflict is when you have contradictory demands made on you at work. For example, your boss wants you to complete your project for next week while your team supervisor wants his project completed first.

Role overload is when you have insufficient time and resources to complete your job. For example, if you are doing your job and covering for an individual who is out on medical leave, you may feel pulled in many directions.

Information overload is simply the processing demands made on individuals where the time required to complete the tasks far exceeds the amount of time actually available. The number of emails many individuals receive in one day often far exceeds the amount of time available for responding.

144. Name two life stressors and give examples of each.

Life stressors include: work-family conflict, life changes, and downsizing.

Work-family conflict is experienced when the demands from work and family negatively impact one another. An example is when you need to attend a work function and you have sick child at home to care for.

Life changes are events that stress an individual. Life changes can be negative (the death of a spouse) or positive (getting married). Though each produces stress, the negative event produces distress and the positive events, eustress. The Holmes-Rahe Scale provides various life events with values that express the stress experienced.

Downsizing is when firms decide to become smaller often through physical, product and employee means. Currently, many firms are downsizing to stay competitive in the worsening economy.

145. Describe two physiological outcomes of stress.

Stress can be manifested internally as nervousness, tension, anger or irritability. It can also be manifested externally through skin conditions and sweating.

Because stress causes a suppression of the immune system, there are links between stress and high blood pressure, ulcers and illness susceptibility.

146. Name two psychological outcomes of stress.

Page 172: Fwk Bauer Tif Organizational Behavior All Chapters

Anxiety and depression are two psychological outcomes of stress.

147. Research links stress to what work outcomes?

Stress is linked to higher turnover and lower levels of performance. Stressed individuals also tend to be less prone to engaging in organizational citizenship behaviors and to exhibit lower organization commitment.

148. Differentiate between a Type A personality and a Type B personality.

A Type A personality is a person who displays high levels of speed/impatience, job involvement and hard-driving competitiveness.

Type B personalities are calmer than Type A personalities who are more prone to thinking through situations than their Type A counterparts who tend to respond emotionally.

149. What are two individual approaches to managing stress?

The individual approaches to managing stress include: The corporate athlete concept, flow, diet, exercise, sleep, social support networks and time management.

The corporate athlete is training so that the employee is healthy in mind and body and embraces challenges versus avoiding them.

Flow is the concept of totally immersing yourself in the job.

Social support networks include coworkers, friends and family.

150. Describe the concept of a “corporate athlete.”

The corporate athlete is an approach to managing stress developed by Jack Groppel at the University of Illinois. The concept relies on applying principles of athletic performance to workplace performance. The corporate athlete is thus an individual who trains his mind and body to perform at peak levels (and under less stress) due to better eating habits, more focused training and positive action. The resulting strong mind and body embraces challenges, not avoids them.

151. Name two organizational approaches to managing stress.

Organizational approaches to managing stress include: making expectations (job duties, for example) clear, employee autonomy, fair work environment, telecommuting, sabbaticals and employee assistance program.

152. Describe positive emotions.

Positive emotions include joy, love and surprise. These emotions result from events.

153. Name two negative emotions.

Page 173: Fwk Bauer Tif Organizational Behavior All Chapters

Negative emotions include: anger, fear and sadness.

154. Describe Affective Events Theory.

Affective Events Theory looks at how events on the job cause different kinds of people to feel different kinds of emotions.

155. Differentiate between surface, deep and genuine acting.

Surface acting is exhibiting physical signs like smiles to reflect emotions you do not really feel.

Deep acting is an individual pretending to experience emotions he does not feel.

Genuine acting is when individuals are asked to display emotions aligned with their own.

156. What is cognitive dissonance?

Cognitive dissonance is a mismatch between emotions, attitudes, beliefs and behaviors.

Many firms operating in multiple countries find their employees face cognitive dissonance because of the different business practices within those countries. For example, in China, to secure permission to talk about land purchases, a major U.S. company’s executives has to make payments directly to village leaders. Such scenarios created quite a bit of dissonance for those individuals.

157. Name the four building blocks of Emotional Intelligence, and define each.

The four building blocks are: self-awareness, self-management, social awareness and relationship management.

Self-awareness is being able to accurately perceive, evaluate and display your emotions.

Self-management is being able to direct your emotions in a positive way when needed.

Social awareness is when you are able to understand how others feel.

Relationship management is when you are able to help others manage their own emotions and establish supportive relationships with others.

ESSAY

158. You are the new Human Resource manager at Lands Corporation. You have just completed examining a great deal of company data. Turnover is much higher in Lands Corporation than at its competitors or the industry average. The data that you analyzed includes a series of transcripts of exit interviews conducted over

Page 174: Fwk Bauer Tif Organizational Behavior All Chapters

the past eight months. Repeatedly you note that past employees describe being “burned out” and then leaving the firm. What are some areas you can examine to develop organizational approaches to address and manage stress?

There are obviously a host of approaches you can take to help employees handle stress in Lands Corporation. One starting point might be to look at the jobs past “burned out” employees were performing. Are the jobs clearly defined? If the employee does not always know what he is supposed to be doing, he is more likely to feel stressed.

A second step is to examine those same job descriptions for the amount of autonomy given to each employee. Stress results when employees feel a lack of control.

Is the work environment fair, or is your work environment toxic? If the company does not value its employees and continually creates unfair scenarios for its employees, stress will result.

Can you find any jobs for which employees are able to telecommute? Working from home at your own pace obviously reduces stress.

Though universities were the first organizations to widely use employee sabbaticals, more and more companies have taken to using these programs to reduce stress and rejuvenate employees.

Employee assistance programs are not just for those experiencing problems with drugs and alcohol but for those experiencing stress that negatively impacts our lives.

Once you have analyzed Lands Corporation for these issues, you can begin to create action plans to address the burnout issue and ultimately the turnover.

159. Your roommate has major stress issues every finals week. She becomes so upset about her exams and papers that she is almost impossible to live with. You do not want to see her go through these problems this finals week, so you have decided to discuss with her various approaches she can adopt that may help calm her during this time. What are some suggestions you could offer her?

You have just read Jack Groppel’s work on Corporate Athletes. You suggest she concentrate on her mind and body throughout finals week as well as the weeks leading up to her exams. Let her know that lower stress levels tend to result when an individual is concerned about good nutrition, focused training and being positive in his or her actions.

Help her find an activity in which she can immerse herself. Absorption in an activity makes the individual feel in control, plus makes her more alert and strong. Greasy foods make you tired. So, watch the fast foods that students are prone to eating during finals week (and other times).

Page 175: Fwk Bauer Tif Organizational Behavior All Chapters

Talk to her about exercising. Physical activity enhances mental concentration so even a walk would reduce her stress.

Encourage her to sleep. You can’t learn a semester’s worth of material in one night, so absolutely do not pull an all-nighter. But even more important, don’t work until the wee hours of the night either. Get rest, and lots of it.

Rely on her social network. Obviously she has you for support, but let her know that there are others who share her concerns. Use those others for support.

Encourage her to practice some time management according to the following guidelines: Write down what she needs to do. Encourage her to cross off those items she has completed. Have her understand what activities eat up her time. Does she sit down to read emails and notice it is an hour later before she finishes? If so, then set up a specific time during the day when she does nothing but read and respond to emails. But, only complete what she can in a time frame that is established. For example, set aside 20 minutes and only 20 minutes for those emails. When the time expires, she should go on to something else.

These steps should help her have a calmer finals week this time around.

160. Meredith just brought her elderly mother to live at her house because of her mother’s current health problems. What are some options her firm might offer her to help alleviate the work-family conflict she will inevitably experience?

The most obvious and, perhaps, easiest option a firm could offer, would be a flexible schedule. Flextime usually requires the employee to work some core hours and then permits that employee to work the remainder either before the core time or after.

Telecommuting is a second option that the firm could consider. Again most firms require a set number of days or hours per week when the employee is physically working from the office. The remainder of days or hours are worked from home or other remote location.

Other options include work sharing, family leave among others.

161. What research findings support an organization’s attention to stress management programming?

Some of the research findings include:Work-family conflict leads to lower job and life satisfaction.Stressors are cumulative and can occur in trends.Stress can be negative (distress) or positive (eustress).Downsizing and job insecurity lead to greater stress, alcohol use and lower performance and creativity.Stressed individuals have lower organizational commitment.Stress can lead to internal manifestations like anger and irritability or external manifestations like high blood pressure and ulcers.

Page 176: Fwk Bauer Tif Organizational Behavior All Chapters

162. Should we all have a “good cry” sometime to relieve stress?

Though Mom may have told you to “let it all out, have a good cry”, that may not be the right answer. Research indicates that crying may not be as helpful as the old adage suggests. There is no evidence to support the hypothesis that crying improves your mood or health. In fact, one study suggested that venting only increases negative effects of negative emotions.

Some research suggests that laughter may actually be the best choice. Crying may intensify negative feelings and suggest to others that the issue really does bother you. Criers tended to be more depressed, anxious and hostile than those who did not cry.

163. Describe the Affective Events Theory and illustrate how it operates.

Affective Events Theory looks at how events cause different kinds of people to feel different kinds of emotions.

As an example, imagine that a co-worker unexpectedly brings you some of your favorite baked goods. You feel happy and surprised. The positive feelings you are experiencing cause you to want to do something that you had not planned before. So, you help someone else. The small moments of emotion can influence your job satisfaction. The more the employee experiences positive emotions, the more satisfied at work.

164. List the worst job you ever performed. Now describe how you used emotional labor in that position.

Emotional labor is the regulation of feelings and expressions for organizational purposes.

Imagine the worst job listed was customer service representative at a firm like Target. Complaining customers would make life in that job rather difficult.

Imagine one of those customers brought in a damaged good. He is complaining bitterly. You simply smile and tell him that you have a full return policy up to thirty days after the purchase. Though the customer is ranting and raving, you are smiling and practicing surface acting.

Deep acting may be an alternative. In deep acting, you empathize with the customer. Again, the customer is ranting and raving and you put yourself into his shoes and actually note that you experienced the same difficulty when you purchased the same item.

Genuine acting is when individuals are asked to display emotions that are aligned with their own.

Page 177: Fwk Bauer Tif Organizational Behavior All Chapters

165. An organizational behavior researcher suggests that not only should mental intelligence be assessed in each individual, but emotional intelligence should also be determined. Do you agree?

Answers will vary but most individuals will probably disagree.

Either from a support or opposition viewpoint, the student can discuss the various building blocks. Self-awareness, step one is something that all individuals should complete. What are you well versed in, what are your shortcomings? How can you improve on the negatives and cultivate the positives?

Self-management is the phase where you can direct your emotions in a particular manner when needed. Again, it is not a problem knowing how to direct and control your emotions and the relevant aspects of the audience supporting them.

Social awareness is when you are able to understand how others feel. Empathy is a difficult concept, and may not be something many students will feel is a necessity.

Relationship management is when you help others manage their emotions. Since many individuals have difficulty managing their own, this may not be a very well-supported aspect of the emotional intelligence question.

Page 178: Fwk Bauer Tif Organizational Behavior All Chapters

Organizational Behavior, Version 1.1Bauer & Erdogan

FWK Test Item FileChapter 8

TRUE/FALSE

1. Communication is the process by which information is exchanged between individuals through a common system of symbols, signs, or behavior.

(True)

2. The best approach to delivering bad news in the workplace, such as a downsizing decision, is a written one like e-mail.

(False)

3. Research indicates that communication ability is related to a manager’s performance.

(True)

4. Deficient interpersonal communication was a factor in 70-80% of all accidents over the past 20 years.

(True)

5. Though employees seem more satisfied when their company communicates with them about issues that impact them, effective communication does little to improve the firm’s market value.

(False)

6. The communication process is very complex, and thus presents almost insurmountable challenges to understanding and controlling it.

(False)

7. The sender in the communication process is totally responsible for successful, effective communication between the sender and receiver.

(False)

8. In the communication process, noise is found only in the external environment of the communication situation.

(False)

9. Filtering, emotional disconnects, and lack of credibility are all barriers to effective communication.

(True)

10. Selective perception is the personal filtering of what we see and hear so as to suit our own needs.

(True)

Page 179: Fwk Bauer Tif Organizational Behavior All Chapters

11. Nancy goes to work at her job at the university after the December holiday break. She opens her email and finds 312 emails after her vacation. Nancy is experiencing information overload.

(True)

12. The EEOC is an acronym for the Equal Employment Opportunity Commission in the Human Resource area.

(True)

13. Men like to ask questions before starting a project; women just “jump right in.”(False)

14. Men tend to focus more on data in their communications, while women focus on intuition.

(True)

15. The word or term “secretary” is an example of biased language.(True)

16. Listening is an art, not a skill, and thus individuals cannot be trained to be better listeners.

(False)

17. The phrase, “You had better…” is a communication freezer because it is somewhat threatening.

(True)

18. There are two types of communication in organizations: verbal and written.(False)

19. Storytelling is largely an ineffective means of communication as receivers perceive the senders to be largely unable to control the process.

(False)

20. Crucial conversations are discussions where the stakes are high, so opinions need to be in line with the majority in the group.

(False)

21. Verbal communication is largely asynchronous.(False)

22. Written communication is the most often used form of communication in business.

(False)

23. People speak much faster than they hear words.(False)

24. Nonverbal communication is at least as important as verbal communication.(True)

Page 180: Fwk Bauer Tif Organizational Behavior All Chapters

25. If I emphasize the word “Max” in the following sentence, “I did not tell Max you were late,” it could suggest that I might have implied it to anyone listening.

(False)

26. There are cultural differences in the use of nonverbal communication elements like body language and facial expression.

(True)

27. Proxemics suggests that standing at an intimate distance to a colleague increases communication effectiveness.

(False)

28. The most information-rich communication channel is a formal written document like a report.

(False)

29. A face-to-face information channel is high in information richness.(True)

30. Written communication is best utilized when the ideas are simple.(False)

31. In a business e-mail, using a subject line is often unnecessary as it may create unrealistic expectations about the message content.

(False)

32. Communication flows upward, downward, and laterally in an organization, but it is not possible for communication to flow diagonally.

(False)

33. Written and verbal communication tends to seek, give, or exchange information.(True)

34. A press release generally appears to be independent of its sender. (True)

35. People are not happy when something unexpected happens, but they are less upset when things are somewhat hidden.

(False)

36. In crisis situations, it is extremely important that an executive be the spokesperson for the firm.

(True)

37. Communication methods and forms differ little across cultures.(False)

38. A nonverbal gesture, like the “OK symbol” made in the United States, could be offensive in another culture.

(True)

Page 181: Fwk Bauer Tif Organizational Behavior All Chapters

39. In China, as in the United States, direct eye contact is important in a communication situation.

(False)

40. The sentence construction of the “Isn’t it?” question we ask in the United States is confusing for non-native speakers.

(True)

MULTIPLE CHOICE

Opening Section: You’ve Got Mail…and You’re Fired! The Case of RadioShack

41. Which of the following statements about communicating downsizing decisions is INCORRECT?a. Employee termination should be conducted in a standardized and

consistent manner, so using e-mails is a highly acceptable technique.b. The communication of employee termination decisions requires careful

crafting of the message and its delivery.c. Many firms have outsourced the employee termination process to third

parties.d. A number of new terms have been developed which attempt to convey

the employee termination decision in a softer, more impersonal manner.(a) Difficult/Synthesis

Section I: Understanding Communication

42. Which of the following statements about communication is correct?a. Communication is the process by which information is exchanged

between individuals, through a common system of symbols, signs or behavior.

b. Miscommunication is an annoyance, but seldom has serious consequences.

c. Between 10 and 20 percent of a manager’s time is spent communicating.d. Communication ability is particularly important in lower levels of

organizations.(a) Medium/Analysis

43. Research on communication indicates all of the following EXCEPTa. Poor communication can lead to lawsuits.b. Effective communication skills are the top skills sought by recruiters

today.c. Poor communication is seen as a causal factor in 70-80% of all accidents.d. Although good communication helps retain effective performers, it does

not increase a firm’s market share.(d) Difficult/Evaluation

44. Communication fulfills all of the following organizational functions EXCEPTa. transmitting information.b. sharing emotions.

Page 182: Fwk Bauer Tif Organizational Behavior All Chapters

c. control.d. coordination.

(c) Easy/Comprehension

45. In the communication process model, the task of translating an idea into words is calleda. decoding.b. encoding.c. medium.d. noise.

(b) Easy/Knowledge

46. The individual originating a message is called the a. sender.b. encoder.c. receiver.d. decoder.

(a) Easy/Knowledge

47. Jason sent an email to his mother, “Ur never going 2 believe this. That prof gave me no credit for my paper. I h8 him.” (with emoticons included). When his mother got the e-mail, it took her quite some time to decipher the message. Jason should have considered his audience when he _________ the message.a. decodedb. validatedc. receivedd. encoded

(d) Easy/Comprehension

48. Ariel was writing a paper at her laptop when the phone rang. She answered it but kept writing as she talked. It was a friend who needed an assignment that had been given last week. Ariel e-mailed her the assignment. A second e-mail just arrived from the friend, which said: “I said the assignment for Monday, this is Wednesday’s.” Ariel likely made the mistake because of the _________ in her communication situation.a. message.b. noise.c. sender.d. feedback.

(b) Easy/Analysis

A professor decides to save paper by posting her syllabus on the university’s web-based course application program instead of printing it and handing it out during the first class. She sends an e-mail to all her students prior to the actual start of the course that reads, “Your syllabus is available on Courseweb. I will go over that syllabus during the first class but, since I am attempting to conserve paper, will not make a copy available to you. Students however, are welcome to print out their own copy of the syllabus.”

49. In this scenario, the professor has what role in the communication process?a. receiver

Page 183: Fwk Bauer Tif Organizational Behavior All Chapters

b. mediumc. decoderd. sender

(d) Easy/Application

50. Given the above scenario, the students have what role in the communication process?a. receiverb. mediumc. decoderd. sender

(a) Easy/Application

51. The email sent by the professor represents what aspect of the communication model?a. feedbackb. noisec. mediumd. decoding

(c) Easy/Application

52. Students come to class and ask why they are not receiving syllabi. This question represents what aspect of the communication model?a. noiseb. feedbackc. mediumd. message

(b) Medium/Application

53. If students try to go online to view the syllabus and the university’s computer system keeps shutting down on them as they read the syllabus, the students are dealing with what aspect of the communication model?a. messageb. mediumc. feedbackd. noise

(d) Difficult/Application

Section II: Communication Barriers

54. Which of the following is not a barrier to communication?a. filteringb. intention of communicationc. selective perceptiond. jargon

(b) Easy/Comprehension

55. Jason received the sales report and noted how badly the southern sector performed. As he prepared the a presentation for the weekly executive meeting, Jason decided to focus his remarks on the northern and western sectors, which

Page 184: Fwk Bauer Tif Organizational Behavior All Chapters

had record sales levels, and refrain from mentioning the southern numbers. This scenario focuses on the communication barrier of a. filtering.b. selective perception.c. emotional disconnects.d. semantics.

(a) Medium/Analysis

56. Which of the following statements about filtering information is INCORRECT?a. Filtering is withholding information to manage a person’s reactions.b. Filtering prevents people from getting a complete picture of a situation.c. People tend to filter downward communication more than upward

communication.d. To maximize the chances of sending effective communication, it is

advisable to use multiple channels to send it.(c) Medium/Analysis

57. Which of the following is not a criterion that individuals may use in deciding whether or not to filter a message?a. past experienceb. month of the yearc. emotional state, involvement with the topic and level of attentiond. knowledge and perception of speaker

(b) Medium/Comprehension

58. Martin is the financial director of the small business. He recently read a memo from one department presenting a new idea for a product. During the discussion on the item afterward, Martin kept focusing on the cost of producing and marketing that product. The barrier facing Martin in this scenario isa. filtering.b. selective perception.c. emotional disconnects.d. semantics.

(b) Medium/Comprehension

59. The personal filtering of what we see and hear so as to suit our own needs isa. filtering.b. semantics.c. selective perception.d. information overload.

(c) Easy/Knowledge

60. Martha’s mom asks her to go to the grocery store for her. Martha agrees and heads upstairs to get ready. Before she leaves her house, she sits down at her desk to check her e-mail and receives a text message from her friend Nancy to turn on the television and see the story about their friend, Jill. As the story about Jill comes on, Martha’s mom comes in and asks Martha to get her an eight-ounce can of diced tomatoes. Martha smiles at Mom and nods and listens to the story on television as she e-mails Sandra and picks up her phone to text Nancy. When Martha gets to the store, she heads to the tomato aisle and stands in front

Page 185: Fwk Bauer Tif Organizational Behavior All Chapters

of the pureed tomatoes trying to remember if mom wanted an 8- or 16-ounce can. This scenario depicts which barrier to communication?a. filteringb. selective perceptionc. semanticsd. information overload

(d) Medium/Application

61. Bill is speaking to a group of fellow marketing specialists at a national convention. He opens his presentation with a dumb blonde joke. Unfortunately, there is dead silence in the room. Bill did not fully address which barrier to communication in preparing his speech?a. filteringb. selective perceptionc. lack of source familiarity or credibilityd. semantics

(c) Medium/Application

62. Which of the following statements about workplace gossip is not correct?a. Research finds that only a minimal amount of workplace communication

occurs at the grapevine level.b. Politically motivated employees use the grapevine to disseminate self-

promoting information in the organization.c. Employees trust their peers as a source of information.d. Managers can control and manipulate the grapevine if they understand it.

(a) Medium/Analysis

63. The acronym SOX refers to what? If you are not clear, then you have been impacted by a communication barrier. Which one?a. selective perceptionb. lack of source familiarity or credibilityc. filteringd. semantics

(d) Medium/Application

64. Men and women tend to differ in their communication styles. Which of the following statements regarding those differences is correct?a. Women focus more on competition in their communication; men on

cooperation.b. Women use metaphors from the home; men from work.c. Women focus on intuition in communicating; men on data.d. Men focus on requests in their communication; women on orders.

(c) Medium/Analysis

65. Which of the following terms is an example to biased language?a. salesmanb. cleanerc. nursed. engineer

(a) Medium/Application

Page 186: Fwk Bauer Tif Organizational Behavior All Chapters

66. Which of the following statements about listening is correct?a. Listening takes practice and concentration.b. Managers listen up to 70% of their day.c. Listening skills are related to promotions.d. All of the above

(d) Medium/Analysis

67. A communication receiver who is preparing his comment or answer and not really listening to the sender isa. using jargon.b. using semantics.c. biased.d. rehearsing.

(d) Easy/Comprehension

68. All of the following are rules for active listening EXCEPTa. find areas of agreement with the speaker.b. listen for feelings.c. note all cues.d. listen for message content.

(a) Medium/Analysis

69. Active listening involvesa. interrupting the message sender to clarify what she is saying.b. giving partial attention to the sender so you can be thinking about your

response.c. refraining from asking senders any questions about their messages.d. taking time to understand the points being made.

(d) Medium/Analysis

70. Listeninga. is an art, not a skill, and cannot be learned.b. does not require close attention to the sender so multitasking is possible.c. can be demonstrated to the sender by nonverbal signals like the nod of a

head.d. creates a bond between communicators, but does not increase

communication flow or accuracy.(c) Difficult/Analysis

71. Which of the following pieces of advice is not a tip to improve your listening habits?a. Start by stopping.b. Think about what you will say when the time comes.c. Don’t multitask during listening.d. Empathize with the sender’s viewpoint.

(b) Medium/Evaluation

72. Maria is conducting a seminar on active listening. She is preparing a list of ways to improve one’s active listening. Which of the following statements is not on her list?a. To not be rude, do not ask questions of the sender.

Page 187: Fwk Bauer Tif Organizational Behavior All Chapters

b. Establish eye contact.c. Pay attention to what is not said.d. Be surprised.

(a) Medium/Evaluation

73. Which of the following would freeze a conversation?a. giving praiseb. asking questionsc. empathizing with the other persond. telling the other person what to do

(d) Medium/Evaluation

74. All of the following would freeze a conversation EXCEPTa. You think you have problems!b. You are very skilled at that.c. You’re jealous.d. You had better listen.

(b) Difficult/Synthesis

75. Which of the following statements about communication barriers is correct?a. Employees are often eager to share bad news with those above them in

an organization to get credit for discovering potential problems.b. Jargon spoken between people from different areas and backgrounds helps them forge a professional bond. c. Managers can use grapevine to send and receive messages. d. Men often jump right into a project while women ask a lot of questions

before beginning.(d) Difficult/Synthesis

76. Which of the following statements about communication barriers is correct?a. men focus on intuition to make decisions, women on data.b. a receiver who is emotionally upset still clearly receives communication

from the sender.c. there are very few words that inflame or insult people.d. selective perception is a time saver, but it can lead to mistakes.

(d) Difficult/Synthesis

Section III: Different Types of Communication and Channels

77. Which of the following is not a type of communication?a. verbalb. directc. nonverbald. written

(b) Easy/Knowledge

78. Anna is placing her workgroup’s lunch order over the phone with the Pizza Shoppe. She is using what type of communication?a. verbalb. directc. nonverbal

Page 188: Fwk Bauer Tif Organizational Behavior All Chapters

d. written(a) Easy/Comprehension

79. Storytelling is an effective form of communication for which of the following situations?a. assigning tasks to an employeeb. explaining the benefit package changes to employeesc. orienting new employees to the company’s cultured. correcting an employee’s approach to a project

(c) Difficult/Synthesis

80. Which of the following statements about storytelling is INCORRECT?a. Storytelling helps construct common meanings for individuals in an

organization.b. The more frequently stories are shared in an organization, the lower the

organizational commitment.c. The greater the strength of the stories shared in an organization, the

greater the organization commitment.d. The higher quality story relayed by an entrepreneur, the greater the ability

to secure capital for his firm.(b) Medium/Analysis

81. An example of a crucial conversation isa. asking for a raiseb. taking part in a weekly meetingc. assigning weekly work to an employeed. chatting about the company’s latest email on the firm’s holiday party plans

(a) Easy/Comprehension

82. All of the following are aspects of crucial conversations EXCEPTa. crucial conversations are ones where opinions vary and emotions run

strong.b. high stakes conversations require more planning, reflection and skill than

many other types of verbal communication.c. communications experts suggest using “but” not “and” in crucial

conversations.d. crucial conversations require flexible communication styles to be most

effective.(c) Medium/Evaluation

83. Nina is a career counselor in a university’s career development office. She is making recommendations to a student in a career planning session related to improving the quality of the student’s conversation. Which piece of advice should be deleted from Nina’s list?a. Speak more than the receiver.b. Ask questions.c. Use eye contact and smile.d. Be receptive to new ideas.

(a) Medium/Application

84. A memo is an example of

Page 189: Fwk Bauer Tif Organizational Behavior All Chapters

a. verbal communication.b. direct communication.c. nonverbal communication.d. written communication.

(d) Easy/Knowledge

85. Communication that is asynchronousa. is written for one receiver, but can be read by many.b. is written for one receiver and read by one receiver.c. is written by the sender and can be read any time by the receiver.d. is written for many but often read by only a few.

(c) Easy/Knowledge

86. Which of the following statements about written and verbal communication is correct?a. Verbal communication is constructed over a longer period of time than

written.b. Written communication is better at conveying feelings than verbal

communication.c. Humans listen faster than they speak.d. Written communication is used more often than oral communication.

(c) Medium/Analysis

87. Which of the following statements regarding verbal and written communication is true?a. Written communication is an inherently flawed medium for conveying

specific facts.b. Individuals hear about 150 words per minute, but speak around 400–500

per minute.c. It is very difficult to teach individuals to write clearly.d. Over ninety percent of employers consider writing a key skill for hiring

new employees.(d) Medium/Analysis

88. What has research shown to be true about nonverbal communication?a. Nonverbal cues are only minimally important. b. Our gestures and other nonverbal movements are largely habitual and

really do not convey much to your audience.c. When individuals are lying, they blink more frequently, shift their feet, and

shrug more often.d. Only 7% of our message is conveyed through nonverbal communication

while 55% is conveyed through the actual words of the message.(c) Medium/Analysis

89. In the sentence, “I did not tell the teacher you were cheating,” if the word “I” was emphasized by the tone of your voice, what are you implying?a. Someone else told the teacher you were cheating.b. I was talking about someone else.c. I told the teacher you were cheating.d. I may have implied it.

(a) Difficult/Synthesis

Page 190: Fwk Bauer Tif Organizational Behavior All Chapters

90. Which of the following statements about nonverbal cues is INCORRECT?a. If your posture mirrors that of the person to whom you are speaking, it

shows you are not listening and generally, not paying attention to that speaker.

b. Happiness is conveyed through facial expressions of an upturned mouth and slightly closed eyes.

c. In some countries, like the United States, direct eye contact conveys trustworthiness.

d. In many parts of the world, simple touch is a necessary, positive part of doing business.

(a) Medium/Evaluation

91. You are part of a marketing team that has just secured a contract for a national ad campaign. The client firm is holding a luncheon to celebrate the contract signing and you are attending. According to Edward Hall’s work, what distance will you maintain from members of the client firm as you network with them?a. personalb. socialc. intimated. public

(a) Medium/Application

92. Information-rich channels convey more a. verbal information.b. nonverbal information.c. effective information.d. ineffective information.

(b) Difficult/Comprehension

93. Effective managers tend to use morea. information-rich channels.b. verbal channels.c. written channels.d. nonverbal channels.

(a) Medium/Application

94. Which of the following communication channels would be the most information-rich?a. Written letters.b. Handheld devices.c. E-mails.d. Videoconferencing.

(d) Medium/Knowledge

95. Which of the following communication channels has the lowest information richness?a. emailsb. formal written documentsc. blogsd. telephone conversation

Page 191: Fwk Bauer Tif Organizational Behavior All Chapters

(b) Medium/Knowledge

96. Marcus wants a record of his communication and is not physically near the person to whom he is communicating. He knows it will take the recipient of his communication some time to understand the message conveyed. What communication channel should Marcus utilize?a. written reportb. meetingc. telephone conversationd. blog

(a) Difficult/Synthesis

97. Nancy is the human resource manager at Alba Company. She needs to lay off twenty employees who are all located at her facility. Which communication channel should she use to deliver the layoff information?a. telephone conversationb. e-mailc. face-to-face conversationd. videoconferencing

(c) Difficult/Synthesis

98. Verbal communication should be used when a. immediate feedback is needed.b. conveying facts.c. ideas are complicated.d. no feedback is needed.

(a) Medium/Comprehension

99. Written communication should be used whena. there is time urgency.b. ideas are simple.c. emotions or feelings need to be conveyed.d. facts need to be conveyed.

(d) Medium/Comprehension

100. Which of the following statements about e-mail is INCORRECT?a. Over 100 million American adults use e-mail at least once a day.b. Some companies are so concerned about the amount of e-mail their

employees receive that they are instituting “no e-mail Fridays.”c. E-mail is the most popular medium of communication today.d. A large portion of e-mails received are either scam or spam attempts.

(c) Medium/Analysis

101. Which of the following is not a piece of good advice regarding business e-mails?a. Do not sign off your e-mails with a closing like “Thank you”; it just wastes

space. Simply send your contact information.b. Do not place anything in an e-mail you would not want the world to see.c. Do not use capital letters for your message.d. Do not hit the send button until you check your e-mail.

(a) Medium/Analysis

Page 192: Fwk Bauer Tif Organizational Behavior All Chapters

102. Communication usually travels diagonally to a. supervisors.b. coworkers.c. subordinates.d. other departments.

(d) Easy/Knowledge

103. Which of the following is not a function of business communication?a. creating informationb. exchanging informationc. seeking informationd. giving information

(a) Easy/Knowledge

104. Recent surveys indicate that managers spend a majority of their time communicating with what group?a. those external to the organizationb. superiorsc. subordinatesd. others that are internal to the firm (e.g., those in other departments).

(c) Medium/Analysis

105. Which of the following is not an example of external communications?a. adsb. press releasec. attitude surveysd. web page

(c) Easy/Knowledge

106. External communication about a client’s product or service created for a specific receiver isa. a company reportb. a web pagec. an add. a press release

(d) Easy/Knowledge

107. An external communication that combines the look of an article with the focused message of an ad isa. a web page.b. an advertorial.c. a press release.d. a sweepstakes form.

(b) Easy/Knowledge

108. Which of the following statements regarding external communications is INCORRECT?a. Press releases are sometimes constructed without acknowledging the

sender’s identity.b. Ad fees are based on the perceived value of the audience who watches,

reads or listens to the channel where the ad appears.

Page 193: Fwk Bauer Tif Organizational Behavior All Chapters

c. The key to a successful external communication to customers is to keep the business message formal and professional.

d. Blogs are growing in popularity, so much so that some firms have created official positions for those bloggers.

(c) Medium/Evaluation

Section IV: The Role of Ethics and Culture during Communication

109. Which of the following statements regarding communication and ethics is INCORRECT?a. Ethical, forthright communication by an organization should occur

internally as well as externally.b. A high profile employee, such as an executive, should be the chief

external communicator during a crisis situation for a firm.c. People are unhappy with the occurrence of the unexpected, but are even

more unhappy when they find out someone tried to hide it.d. Exxon Oil is widely recognized as having been successful communicators

during the Valdez incident.(d) Difficult/Evaluation

110. Which of the following statements regarding cross cultural communication is INCORRECT?a. Nonverbal communication is consistent across cultures.b. The question “Isn’t it?” should be avoided by English speakers with non-

English speaking colleagues.c. Low situational context cultures, like the United States, “get down to

business” in negotiating deals.d. Within the same culture, words can be misunderstood because of a lack

of common experiences.(a) Medium/Evaluation

111. Which of the following statements regarding nonverbal body signals is true?a. Hand waving indicates greetings in Europe just as it does in the United

States.b. A “thumbs up” means five in Germany and one in Japan.c. The American “ok” gesture would be inappropriate or offensive in Brazil or

Russia.d. The “hook ’em horns” gesture from the University of Texas means you

are warding off evil in Italy.(c) Medium/Analysis

FILL IN THE BLANK

112. ___________ is the process by which information is exchanged between individuals through a common system of symbols, signs or behavior.

(Communication)

113. Communication fulfills three main functions within an organization including _____________, _________________, and ______________.

(coordination, transmission of information and sharing emotions and feelings)

Page 194: Fwk Bauer Tif Organizational Behavior All Chapters

114. A ______________ originates the communication process.(sender)

115. The process of translating ideas into words is ______________.(encoding)

116. Communication media include spoken words, written words, or signs. (media)

117. Anything that interferes with or distorts the message being transformed is __________________.

(noise)

118. _____________ is a communication barrier where information is withheld or distorted to manage a person’s reactions.

(Filtering)

119. Personal filtering of what we see and hear so as to suit our own needs is _____________ _______________.

(selective perception)

120. When we receive more information than we can take in, the imbalance is known as __________ ______________.

(information overload)

121. An informal gossip network that can be a barrier to effective communication is called a ____________.

(grapevine)

122. Words mean different things to different people; this is the concept of __________.

(semantics)

123. In accounting, students talk about LIFO and FIFO; these are two examples of shortcuts called _________ that many in the field take to talk in code.

(jargon)

124. Men and women differ in their communication styles. When using analogies in their discussion, men reference ____________ and women reference ____ ____________.

(sports, the home)

125. To resolve the issue of biased language, instead of calling an individual a businessman, you should call that individual a _______________.

(businessperson)

126. The behavior where we are not really listening but preparing our response is called ________________.

Page 195: Fwk Bauer Tif Organizational Behavior All Chapters

(rehearsing)

127. Giving full attention to what other people say, taking time to understand the points made, asking questions as needed, and not interrupting at inappropriate times is called ____________ ______________.

(active listening)

128. _____________ _______________ put an end to effective communication by making the receiver feel judged or defensive and include criticizing, blaming, ordering or judging.

(Communication freezers)

129. The three types of communication are: _____________, _______________, and _______________.

(verbal, written, and nonverbal)

130. Discussions that are high stakes and where opinions vary and emotions run strong are ____________ ______________.

(crucial conversations)

131. A message that can be read at any time is ____________ unlike a conversation, which is carried on in real time.

(asynchronous)

132. ______________ communication does a better job of conveying feelings while _______________ communication does a better job of conveying facts.

(Verbal, written)

133. Nonverbal cues including the tone, pace, and volume of speech is called ______________.

(paralanguage)

134. ___________ is a term coined by Edward T. Hall to denote the different kinds of distance that occur between people.

(Proxemics)

135. The distance between two people, like a mother and child, usually in the zero to eighteen inch range is called the ___________ distance.

(intimate)

136. _________________ __________ communication channels convey more nonverbal information and tend to be used by effective managers.

(Information-rich)

137. __________ communication is best utilized when you need feedback and there is time urgency.

(Oral)

138. ____________ communication is best utilized when ideas are complicated or you are conveying facts.

(Written)

Page 196: Fwk Bauer Tif Organizational Behavior All Chapters

139. _____________ are symbols that offer clues to the emotional side of words used in e-mail messages.

(Emoticons)

140. _________ ________ professionals create external communication about a client’s products, services, or practices for specific receivers.

(Public relations)

141. High __________ cultures place a high value on establishing relationships prior to working with others while low ____________ cultures “get down to business.”

(context, context)

SHORT ANSWERS

142. Briefly discuss the elements in the communication process.

A sender encodes a message that translates an idea into words. The message is sent through a medium that may include spoken or written words or signs. The receiver receives the message and decodes it by assigning meaning to the words. The receiver may then decide to respond to the original message by also encoding or translating his idea into words. This process of responding is called feedback. The entire process from sender to receiver may be altered or distorted due to noise in the system. The noise can be external to the environment or internal. If too much noise impacts the encoding or decoding, or any other aspect of the communication process, miscommunication can occur.

143. Name the three functions which communication fulfills and provide an example of each.

The three main functions of communication are: coordination, transmission of information, and sharing emotions and feelings. Coordination would ensure that everyone is aware of the strategy and goals of the organization and all are working together toward them. Transmission of information would be demonstrated by, for example, a written copy of the firm’s objectives such that all employees would know the goals and their general direction. Finally, the sharing of emotions and feelings could be exemplified by praising a department for exceeding annual goals set at a luncheon in their honor.

144. Name, describe and provide an example of two barriers to effective communication.

Barriers to communication include:

Filtering: a distortion or withholding of information to manage a person’s reaction. An example might be a manager who keeps negative comments about his department from his superior.

Page 197: Fwk Bauer Tif Organizational Behavior All Chapters

Selective perception is personal filtering of what we see and hear so as to suit our own needs. An example is when a sports fan reads the newspaper, he often does not “see” news stories but only those items related to sports.

Information overload is when the information processing demands on an individual’s time to perform interactions and internal calculations exceed the supply or capacity of time available for such processing. When an individual returns from vacation, he often has hundreds of email messages, phone messages and memos on his desk.

Emotional disconnects are when there are opinion or personality differences which impact communication. I have had a terrible day already and then you say something to me that is actually pretty innocuous, but I take offense.

Lack of source familiarity or credibility. There is no common context for the sender and receiver in the communication situation. This is a prime example of why some jokes just fail to make anyone laugh.

Workplace gossip is an informal gossip network. Sometimes people hear about events that will occur in a firm over lunch and then get the official e-mail about it later.

Semantics are the meanings we give to words. SOX (Sarbanes-Oxley Act) is an acronym widely understood by accounting and finance people but is something lost on others not in the field.

Gender differences in communication. Example: Men tend to focus on orders in their communication, women on requests.

Differences in meaning between the sender and receiver: The perspectives of the two parties differ and miscommunication occurs. For example, I use generational slang like the word “cool.” I think it means, “that’s interesting,” you are from a previous generation, so you think it has to do with temperature.

Biased language: words or actions that offend or stereotype others. For example, gender bias is shown when the word “salesman” is used. To avoid gender bias, use the term “salesperson.”

145. Give two examples of biased language and indicate why they would be considered biased.

Biased language is words or actions that offend or stereotype others. Examples: cleaning lady instead of cleaner. Also, chairman instead of chairperson.

146. Give an example of a communication freezer and indicate why it ends effective communication.

Communication freezers are words or phrases that put an end to effective communication by making the receiver feel judged or defensive. Examples include:

Page 198: Fwk Bauer Tif Organizational Behavior All Chapters

Telling the other person what to do: “You must…”Threatening: “You had better…”Judging the other person negatively: “You’re not thinking straight.”

147. Why is storytelling an important verbal communication skill?

Storytelling is important to the organization because it helps to establish common meanings for organization members. Stories clarify key values and indicate how tasks are to be performed. Story frequency, strength and tone are related to higher organizational commitment. Story telling on an individual level enhances interviewing skills.

148. Define a crucial conversation and give an example of it.

A crucial conversation is a discussion where the stakes are high and where opinions vary and emotions run strong. An example is during a performance evaluation where you might be asking for a raise.

149. Compare and contrast the uses for written and oral communication.

Written communication is printed messages. These include memos, proposals, e-mails, letters, training manuals, and operating policies. They may be printed on paper or appear on the screen. Written is asynchronous, which means that the message can be read at any time. Further, written communication can be read by many people, and can be constructed by many people in a collaborative fashion. Written communication is better for conveying facts.

Oral communication takes place in person or over the phone. The verbal communication takes place in real time and is generally one on one. Verbal communication is a better way of conveying feelings. Oral communication is the most frequently used method of communication.

150. Define and provide an example of paralanguage.

Paralanguage is the tone, pace and volume of speech. The tone can change even with the same sentence thereby changing the meaning of the sentence. For example, if I say, “I did not tell the teacher you copied the problem,” you are implying that someone else might have told. Changing the emphasis to another word in the sentence, “I did NOT tell the teacher you copied the problem,” implies that this event did not occur.

151. Define and provide an example of body language.

Body language is a series of nonverbal cues that arise from use of various parts of the body. For example, eye contact when direct in the United States shows that you are interested and listening; when indirect that you are not really paying attention to the speaker.

152. Define proxemics and provide an example of it.

Page 199: Fwk Bauer Tif Organizational Behavior All Chapters

Proxemics is the study of the different kinds of space or distance between people and their meaning. For example, a space of 0 to 18 inches is called intimate space and is reserved for those who are very close to an individual. 1.5 feet to 5 feet is the personal distance, and 5 to 7 feet is the social distance.

153. Define an “information-rich” communication channel and provide an example of such a channel that is high in information richness.

Information-rich communication channels convey more nonverbal information in addition to the verbal or written information. A channel high in information richness is the face-to-face conversation.

154. Provide two instances when written communication should be utilized and two when the choice should be oral communication.

Written communication is used when: conveying facts, the message needs to be part of a permanent file, there is little time urgency, you do not need immediate feedback, and ideas are complicated.

Oral communication is used when: conveying feelings and emotions, the message does not need to be permanent, there is time urgency, you need feedback, and the ideas are simple or can be simple with explanations.

155. Briefly discuss the different directions information can flow in an organization.

Information flows downward to subordinates, upward to supervisors, laterally to coworkers, and diagonally to different departments.

156. Provide two examples of external communications.

External communications include: press/public relations releases, ads, web pages, and customer communications.

Press releases are external communication about a client’s products, services or practices for specific receivers. Ads are external business messages for target receivers reached through media buys. Web pages combine elements of public relations, advertising, and editorial content and include banner ads, blogs or advertiser-driven click-throughs. Customer communications include letters, catalogs, direct mail, e-mails, text messages, or telemarketing messages.

ESSAY

157. Using the process model of communications as a focal point, discuss how a manager might prepare for his first meeting as the new head of a marketing department.

The manager will initiate the meeting, so he will serve as the original sender. The new manager must recognize that first impressions are lasting impressions, so the first message he sends is a critical communication to his future success in his position. Announcing the meeting would be the first communication and he wants

Page 200: Fwk Bauer Tif Organizational Behavior All Chapters

to ensure he encodes the message carefully to convey the importance of that meeting. It is at the initial meeting that the strategy for subsequent meetings will be set, so that importance must be obvious in the words chosen. The medium through which the meeting announcement will be sent must also be carefully considered. Multiple channels are always best since individuals’ schedules differ and the medium choice is most dependent on which channel works best for which individual at any given moment in those busy schedules. For all intents and purposes, e-mail is effective but bulletin board notices would also be workable as might face-to-face reminders of this meeting.

Clearly words will be used in the various channels of communication but some attention-getting symbols or signs might also be used. The receivers are all those in the marketing department. The words chosen for the communication might contain some jargon from the marketing area so that some credibility is initially established as well as some camaraderie, since those in other departments who may view, for example, the bulletin board reminder, may not understand that jargon. The marketing department members will decode the message and, hopefully, provide some feedback. The feedback could itself be a written response email or simply showing up at the meeting at the designated time and place.

This entire communication scenario will take place within the context of noise in the environment. Some examples of the distortions or interferences that may occur include the email for some recipients “bouncing back,” the recipient being distracted when he/she reads the e-mail and focuses on the wrong date or time for the meeting, and/or some offense taken at words contained in the message because of their gender-basis or other aspects.

158. Maynard is the production department supervisor in a manufacturing firm. Recently rumors have been spreading through the department grapevine regarding a possible company-wide downsizing that will lay off approximately 20% of the employees. If you were Maynard, how would you handle this situation?

The grapevine is an informal network and can be the key source of company information for many employees. It is a two-edged sword for managers and supervisors. The grapevine must be recognized as an organizational fact of life, but must also be managed, or the gossip within the grapevine will manage the department and, ultimately, the firm.

Since the grapevine is a grassroots structure, it is viewed as having a great deal of credibility. In fact, the grapevine is often viewed as having greater credibility than managerial information because it is located at the grassroots level. Managers like Maynard must be wary of politically motivated employees who will use the grapevine for misinformation or information supportive of their views.

In Maynard’s position, one must understand who the key information disseminators in the grapevine are and the manner in which they choose the type of information to disseminate and how they so disseminate it. Once these two aspects are thoroughly understood, then Maynard can begin to place the nature of information he needs to have placed at the times he needs to do so. An

Page 201: Fwk Bauer Tif Organizational Behavior All Chapters

alternative approach is to understand the dynamics of the grapevine and provide official communication that beats the grapevine to its dissemination.

159. Maria is the first female manager of the Finance Department at Xylox Corporation. She is contemplating her opportunities and threats as they are presented her over the next few months and realizes that communication between herself and her subordinates will be vital to success. Maria has six subordinates, all of them male. What are some of the key gender-based communication differences she should be attuned to as she begins her interactions with her employees?

Maria should first recognize that there are gender-based differences in communication. Women, for example, like to ask questions before beginning a project, while men tend to “jump right in.” Maria should be aware then that her male subordinates are not “knee jerk reactors,” but utilize that approach regularly.

Men speak quite often in sports metaphors while women tend to use home-based metaphors. Again, Maria must recognize her male subordinates are not just wasting time “chewing the fat about the weekend’s game,” but actually may be making some tactical or strategic decisions.

Maria must also be aware that she may need to change the manner in which she talks to better understand her subordinates or to ensure they better understand her. So, for example, Maria must recognize that while she focuses on cooperation, intuition and requests in her communication, her male subordinates will be focusing on competitiveness, data and order in their style. Again, the first step in dealing with these gender differences in is recognizing their existence and then their pervasiveness and moving forward from there.

160. Sergei has a career coach. The coach has just finished two extensive assessment sessions with Sergei and is now reporting to him on his results. The career coach tells Sergei that he has poor listening skills and must address them immediately before focusing on any other weaknesses. If you were the career coach, what tips might you give to him to begin an improvement plan focused on enhancing listening skills?

The first step in any listening improvement plan is to stop talking. If you are talking, you cannot be listening so the best first step is to just stop the talking.

It is also important to recognize if Sergei physically can hear. Many people do suffer from slight to moderate or worse hearing loss and may need some assistance.

Active listening is giving full attention to what other people are saying, taking the time to understand the points being made, asking questions as needed and not interrupting at inappropriate times. Active listening creates a relationship between the sender and the receiver. The process focuses on how to eliminate rehearsing, which is when the recipient of a verbal message is not really listening because he is preparing to respond to your item or question. Active listening

Page 202: Fwk Bauer Tif Organizational Behavior All Chapters

involves not only listening to the content of the message, but also the feelings associated with that message.

The “plan” to improve listening begins with deciding that you want to listen. Then, you need to show the sender that you are listening. Nonverbal cues can help you convey the notion that you are listening. For example, one could shake his head as the sender speaks to acknowledge receipt of the message. In addition, what the sender has spoken can be repeated, either in full or part to convey receipt.

Some quick pieces of advice for the active listening training include:

Stop talking.Don’t worry about what will be said when the time comes.Join the sender’s team by indicating in some way your receipt of the information sent.Don’t multitask while listening.Try to empathize with the speaker’s point of view.Ask questions if confused.Establish and retain eye contact.Listen with an open mind.Pay attention to what is not said; body language is very important.

161. Networking is a very vital part of a student’s job search process. One key element in networking is the ability to carry on conversations. Like listening, conversation making is a skill that can be learned. What are some recommendations for being a better conversationalist?

Be the first to say, “Hello.” Think before you speak. Don’t be afraid to pause before making a statement so that you are making an intelligent comment and not making a preventable mistake.Be receptive to new ideas. If you disagree with someone, at least get more information on his position without dismissing him verbally at the initial mention of the item.Repeat someone’s name to yourself and then aloud when being introduced. Ask questions. This helps establish and demonstrate interest.Listen as much if not more than you speak.Use eye contact, smiles, and positive body language. Be aware of your posture.Mirror the other person to some degree by repeating what they said.Have an exit strategy ready.Be prepared.

162. You have been asked to teach a business communication lesson on business e-mails. What are some important topics and tips you might want to include in that lesson?

Knowing the volume of e-mails sent each day and the number each individual employee is faced with each day, companies should provide their employees with some assistance in using this form of written communication, which studies indicate is the second most popular form of communication worldwide. Some tips include:

Page 203: Fwk Bauer Tif Organizational Behavior All Chapters

Do not send or forward chain e-mails.Do not send cc e-mails to everyone.Do not send, “reply to all” responses to every e-mail received.Do not put anything into an e-mail that you would not want the world to see.Do not write your message in all capital letters.Do not hit the send button until you are absolutely sure you wish to send the e-mail.Use a subject line with the e-mail.Make your request early in the e-mail, preferably in the first line if possible.End your e-mail with a brief sign-off like “thank you.”Make sure to provide contact information on each e-mail.Keep it brief.Make sure the e-mail contains complete thoughts. Keep in mind that the recipient may not want to take the time to pull up and closely look at the preceding e-mail and thus you want to make sure that recipient understands what you are conveying.

163. Your company has just been received a phone call from the local news station requesting a response to a claim made on the air by a viewer. The viewer accuses your store of selling a toy she purchased that has made her child sick. The customer alleges that the paint used on the toy contains lead. Further, she has stated, on the air, that your firm was aware of the problem but because the toy is so popular and made exclusively for your store, you were unwilling to pull the item from the store shelves, choosing instead to make the profit on the toy. What will you do?

This scenario is obviously a crisis situation and the communication used in this instance is critical. Clear messages must be sent that are credible and that get to the parties necessary to address the issue and prevent further damage to your firm and the buying public.

A lesson learned from the Exxon Valdez incident as well as the Tylenol situation is to make certain that a high ranking official be the official spokesperson in any crisis situation. Also, keep in mind that while no one is happy when the unexpected occurs, they are even more unhappy when they find out you tried to hide something about the matter. Also, make certain that all communication is ethical and forthright. If the truth is missing people are demoralized. Employees want to be “in the know” and trusted with the truth about the business. If such a trust is established, employees are more loyal.

164. You are part of a group in a business communication class. You have a project due next week where you are to discuss nonverbal communication cues that differ depending on the country or culture. What will your segment of the project look like?

One aspect of nonverbal communication that can be examined is gestures around the globe. Some examples follow:

“V” for victory. In the U.S., this is a victory symbol, but in England, it is something like, “Take this.”

Page 204: Fwk Bauer Tif Organizational Behavior All Chapters

The “OK” gesture means things are going well in North America, but in France, it means that a person is worthless. In Brazil, Russia or Germany it means something not appropriate for the workplace.

The ‘thumbs up” means one in Germany, but five in Japan.

“Hook ’em horns” is the symbol for the Texas Longhorns at the University of Texas. In Italy, however, it means that you are being tricked.

Waving your hand in the United States means greetings; in Europe however, it signals disagreement.

Page 205: Fwk Bauer Tif Organizational Behavior All Chapters

Organizational Behavior, Version 1.1Bauer & Erdogan

FWK Test Item FileChapter 9

TRUE/FALSE

1. In the 1980s, General Electric showed that empowering workers to work in teams can be a profitable venture.

(True)

2. A group is a cohesive coalition of people working together to achieve mutual goals while a team is a collection of individuals who interact with each other such that one person’s actions have an impact on another’s.

(False)

3. Tuckman’s Group Development Stage Model sequences group evolution in the following order: forming to norming to storming to performing.

(False)

4. The forming stage of group development involves member examination of such questions as “What will my role be?” and “Will I be accepted?

(True)

5. The appointed leader is often tested during the performing phase of group development.

(False)

6. Because the storming stage is a very chaotic one, many groups get stuck in that phase of group development.

(True)

7. The norming stage of group development involves the establishment of ground rules for the group and an increased commitment to other members and the group goal.

(True)

8. The performing stage of group development involves attention to work content and process.

(True)

9. The adjourning stage of groups is seldom met with any emotional feelings or concern.

(False)

10. The punctuated equilibrium model proposed by Connie Gersick is a linear model.(False)

Page 206: Fwk Bauer Tif Organizational Behavior All Chapters

11. The punctuated equilibrium model suggests that groups remain static for long periods of time experiencing only incremental change and radical, revolutionary change occurs in brief, punctuated bursts that provide opportunity for innovation.

(True)

12. Generally speaking, the more cohesive a group is, the less productive it is because the preservation of individual feelings supersedes any task oriented role.

(False)

13. The more similar group members are with regard to age, gender, education, and skill set, the more cohesive the group will be.

(True)

14. Groupthink is the tendency for individuals to put forth less effort when working in a group context.

(False)

15. Groups with high group cohesion and high task commitment generally have high performance.

(True)

16. Collective efficacy is generally influenced by verbal persuasion, group observation, and pay level.

(False)

17. Process loss is any aspect of group interaction that inhibits group functioning.(True)

18. The key properties of a true team are collaborative action, sacrifice for the common good, and compensation based upon individual outcomes.

(False)

19. Research evidence exists to demonstrate that teams can reduce labor costs, increase product quality, and raise revenues.

(True)

20. If a variety of knowledge, skills, and abilities is needed, if feedback from different organizational groups is needed, if tasks are largely independent, and if wide cooperation is needed to achieve mutual goals, then a team is needed.

(False)

21. Idea generation tasks include coming up with plans for actions and making decisions.

(False)

22. Pooled interdependence is when team members work independently and combine their efforts to create the team’s output.

(True)

Page 207: Fwk Bauer Tif Organizational Behavior All Chapters

23. Reciprocal interdependence exists when team members work on tasks simultaneously.

(True)

24. Task roles in a group include consul, cooperator, and completer.(False)

25. Social roles in a group or team include cooperator, communicator, and calibrator.(True)

26. A task force is a temporary team charged with addressing a specific problem or issue until it is resolved.

(True)

27. Virtual teams represent special management challenges because of issues related to trust and communication.

(True)

28. A firm’s top management team often represents a variety of functional areas.(True)

29. Succession planning is the identification of future members of a top management team.

(True)

30. Empowered teams have the responsibility and authority to achieve their goals.(True)

31. The difference between a self-managed and self-directed team is that the leader in the self-managed team has less decision-making responsibility.

(True)

32. As a rule of thumb, a good size for a team is between two and twenty members.(True)

33. The more diverse a team is with regard to expertise, gender, age, and background, the more likely the team is to avoid groupthink.

(True)

34. Norms are shared expectations about how things operate within a group or team.(True)

35. An effective way of dealing with overbearing behavior is to evaluate the balance of participation by team members.

(True)

36. When team members are found to be poor performers due to a lack of ability, the team is likely to try and train the individual.

(True)

Page 208: Fwk Bauer Tif Organizational Behavior All Chapters

37. Research suggests that sometimes replacing a hierarchy with self-managing teams decreases control over individual workers.

(False)

38. Collectivist societies value individual achievements over interpersonal relationships.

(False)

39. People in high power distance countries expect unequal power distribution and greater stratification in economic, social, and political settings.

(True)

40. The meaning of teamwork differs across cultures.(True)

MULTIPLE CHOICE

OPENING SECTION: Teamwork Takes to the Sky: The Case of General Electric

41. General Electric generated more wealth than any other company in the history of the world from the early 1980s until 2000. All of the following are contributing factors to this success EXCEPTa. providing employees with the highest pay in the industry.b. providing ample support to team members so that all efforts were focused

on the work of the team.c. providing high level training to teams.d. providing teams the opportunity to “self-manage.”

(a) Easy/Knowledge

Section I: Group Dynamics

42. An informal work group isa. a collection of individuals who interact with each other such that one

person’s actions have an impact on the others.b. made up of managers, subordinates, or both, with close associations

among group members that influence the behavior of individuals in the group.

c. made up of two or more individuals who are associated with one another in ways not prescribed by the formal organization.

d. a cohesive coalition of people working together to achieve mutual goals.(c) Easy/Knowledge

43. A group isa. a collection of individuals who interact with each other such that one

person’s actions have an impact on the others.b. made up of managers, subordinates, or both with close associations

among group members that influence the behavior of individuals in the group.

Page 209: Fwk Bauer Tif Organizational Behavior All Chapters

c. made up of two or more individuals who are associated with one another in ways not prescribed by the formal organization.

d. a cohesive coalition of people working together to achieve happiness.(a) Easy/Knowledge

44. To successfully facilitate a group, a leader needs to move through a series of leadership styles over time. Which of the following styles is the FIRST a successful group leader should exhibit?a. delegatorb. directorc. coordinatord. coach

(b) Difficult/Synthesis

45. Which of the following leadership styles best serves a successful group as it has evolved to its final stages?a. directorb. coordinatorc. coachd. delegator

(b) Medium/Knowledge

46. Which of the following is the order Tuckman proposes for group development stages?a. forming, norming, storming, performing, and adjourningb. forming, storming, performing, norming, and adjourningc. forming, performing, storming, norming, and adjourningd. forming, storming, norming, performing, and adjourning

(d) Easy/Comprehension

47. What is the second stage in Tuckman’s four-stage map of group evolution?a. normingb. stormingc. formingd. performing

(b) Easy/Knowledge

48. What was the final stage that Tuckman added to his enhanced group development model?a. formingb. normingc. adjourningd. storming

(c) Easy/Knowledge

49. Which of stage in Tuckman’s forming-storming-norming-performing model is characterized by the establishment of ground rules and operating procedures and goals by group participants?a. formingb. stormingc. norming

Page 210: Fwk Bauer Tif Organizational Behavior All Chapters

d. performing(c) Easy/Comprehension

50. Which stage of Tuckman’s group development model is characterized by general information sharing?a. stormingb. normingc. performingd. forming

(d) Difficult/Comprehension

51. Tuckman’s forming stage is characterized by a. uncertainty, with observant, conflict avoidant group members who are

trying to achieve several goals at once.b. competent group members who work interdependently as part of a

greater entity.c. chaos with group members exploring power and influence roles.d. high energy with group members committed to each other and the overall

goal.(a) Medium/Evaluation

52. The norming stage in Tuckman’s model of group evolution featuresa. uncertainty, with observant, conflict avoidant group members who are

trying to achieve several goals at once.b. competent group members who work interdependently as part of a

greater entity.c. chaos with group members exploring power and influence roles.d. high energy with group members committed to each other and the overall

goal.(d) Medium/Evaluation

53. Tuckman’s five stage group evolution model asks which of the following questions in the performing stage?a. Are we using appropriate information on this task? b. Will I be accepted? c. Why do I have to listen to you?d. Can you help me plan this project?

(a) Medium/Evaluation

54. Which of the following questions might be asked during the norming stage of Tuckman’s five-stage group evolution model?a. Are we using appropriate means to address differences so that conflicts

can be avoided?b. Will I be successful at completing this task? c. I wonder what the other people in this group are like?d. Can you help me with this section of the project?

(a) Easy/Evaluation

55. Group members in the norming stagea. not only get the work done, but they also pay greater attention to how

they are doing it.

Page 211: Fwk Bauer Tif Organizational Behavior All Chapters

b. begin to explore their power and influence.c. find it easy to establish their own ground rules.d. try to figure out who likes whom.

(c) Medium/Analysis

56. Which of the following statements is correct with regard to the forming stage of group development?a. Leadership does not matter at this stage.b. Group members are trying to learn about one another.c. Group members are cohesive and cooperative.d. Group members are trying to clarify what needs to be done and who

needs to do it.(b) Easy/Analysis

57. All of the following statements characterize a group at the storming stage EXCEPTa. the group is competent, autonomous, and insightful.b. the group’s members are forming cliquesc. the group members are argumentative.d. group discussions may be heated.

(a) Easy/Analysis

58. Michael is part of an organizational behavior project group. The group decided on the overall theme of its project and individual members are now completing specific sections of the project. Michael has been e-mailing Rachel almost every day asking her opinion on various parts of his section. The group tends to sit together in class and they have even decided to go out for coffee after class next Tuesday. What phase of group development is Michael’s group in?a. stormingb. adjourningc. formingd. performing

(d) Medium/Synthesis

59. Jennifer has been very emphatic in her comments supporting the new addition to the product line. Christopher has been equally spirited in his opposition to the new item. Gordon and Antonio seem to be leaning to Jennifer’s perspective while Marques supports Christopher. The discussion about the product has consumed the last hour of the meeting of the marketing group. At what stage of Tuckman’s model does this group seem to be functioning?a. forming.b. storming.c. norming.d. performing.

(b) Medium/Comprehension

60. Which of the following is NOT a recommended step to take to avoid getting stuck in the storming phase of Tuckman’s group development?a. Make sure to limit discussions and discussants. It is not productive to re-

hash all the same points over and over.b. Do not rush the group’s development. It takes time to get things right.

Page 212: Fwk Bauer Tif Organizational Behavior All Chapters

c. Normalize conflict. Conflict is normal and can be cathartic.d. Support all group members. Do not allow anyone to feel insecure as he or

she operates in the group.(a) Easy/Analysis

61. Which of the following statements regarding Gersick’s punctuated equilibrium model is INCORRECT?a. The model views group development as a linear process.b. Generally groups are static in their orientation.c. For most periods of time in groups, change is incremental.d. While disruption, chaos and conflict are seen as inevitable in a group,

those events can represent opportunities for innovation.(a) Difficult/Comprehension

62. According to Gersick’s punctuated equilibrium model, groups repeatedly cycle through which two stages?a. storming and norming.b. forming and performing.c. forming and norming.d. storming and performing.

(d) Easy/Comprehension

63. Which of the following statements regarding Gersick’s study of punctuated equilibrium is correct?a. Most change in groups is characterized by periods of revolutionary chaos.b. All groups are able to readily adapt to new environments that arise due to

change.c. The punctuated equilibrium model is cyclical in nature.d. Groups generally cycle through the Storming and Norming stages of

group development.(c) Medium/Comprehension

64. A group where there is a great deal of camaraderie is a group experiencinga. group action.b. cohesion.c. social loafing.d. collective efficacy.

(b) Easy/Knowledge

65. Characteristics of cohesive groups include all of the following EXCEPTa. having a collective identity.b. working together on a meaningful task.c. having a random pattern of communication.d. having a desire to remain as a member of the group.

(c) Easy/Comprehension

66. Which of the following is NOT a fundamental factor impacting a group’s cohesion?a. sizeb. heightc. support

Page 213: Fwk Bauer Tif Organizational Behavior All Chapters

d. stability(b) Easy/Knowledge

67. Which of the following statements related to fundamental factors affecting a group’s cohesion is correct?a. The larger a group, the more cohesive it is likely to be.b. The more similar group members are, the more cohesive the group is

likely to be.c. The more coaching group members receive, the less cohesive the group

is likely to be.d. The less time a group has stayed together, the more cohesive a group is

likely to be.(b) Medium/Analysis

68. Marina is encouraging her sorority sisters to function as a cohesive group. She lists all of the following as benefits EXCEPTa. you will have greater self-confidence and self-esteem if you are a regular

part of the group.b. if you are experiencing stress, as you might during finals week, you can

depend on your sisters to help get you through.c. if you experience a personal tragedy or challenge, your sisters will help

you cope.d. If you are part of the sorority, you do not have to attend meetings and

events often because you can depend on your sisters to cover for you.(d) Easy/Evaluation

69. Eduardo is the new manager of Great Corporation. He is sitting at his desk and writing down a list of steps that he can follow to create and maintain a cohesive work group. Which of the following does NOT appear on that list?a. Celebrate differences so each individual’s contribution is noticed.b. Give praise sparingly; if it is given too often, it loses its luster.c. Create some regular, common rituals like Friday morning coffee.d. Let group members have choices in setting their goals.

(b) Easy/Analysis

70. NASA endured the Challenger disaster when group members conformed with top managers’ decisions that launching the space shuttle in freezing temperatures was better than missing the launch window. This is an example of what group process?a. social loafingb. cohesionc. collective efficacyd. groupthink

(d) Difficult/Synthesis

71. Symptoms of too much cohesion include a. failures are blamed on external factors.b. internal pressures to question the status quo.c. greater questioning of external constraints.d. tolerance for deviants.

(a) Difficult/Evaluation

Page 214: Fwk Bauer Tif Organizational Behavior All Chapters

72. Which group would be the most effective from a performance standpoint?a. A group with low task commitment and low group cohesion.b. A group with high group cohesion and low task commitment.c. A group with high task commitment and low group cohesion.d. A group with high group cohesion and high task commitment.

(d) Easy/Analysis

73. Michael’s group is very cohesive and very committed to the task at hand. What is their likely performance level?a. low performanceb. high performance if they do not value performance c. average performanced. high performance if they value performance

(d) Medium/Application

74. The Ringelmann effect is also known as a. group cohesion.b. collective efficacy.c. social loafing.d. groupthink.

(c) Medium/Comprehension

75. Which of the following statements is INCORRECT with regard to the concept of social loafing?a. The incidence of social loafing declines as group size increases.b. A tendency for social loafing develops when a perception of inequity in

regard to rewards and/or blame arises in a group.c. Social loafing is a consistent effect across groups.d. Social loafing is a consistent effect across cultures.

(a) Medium/Analysis

76. Maria strongly disliked working in groups in college because she always felt there was someone who did not “pull his weight.” However, now that she is a manager, she must organize teams in her firm. Which of the following is NOT a tip to prevent social loafing she has received from other managers in her company?a. Design and communicate to all group members how they will be

evaluated.b. Assign engaging, challenging and rewarding tasks to group members.c. Choose a larger number of employees for each team.d. Define each individual’s tasks in front of the group.

(c) Medium/Analysis

77. Collective efficacy isa. a tendency of individuals to put in less effort when working in a group

context.b. a group’s perception of its ability to perform well.c. a group pressure phenomenon that increases the risk of the group

making flawed decisions.d. the degree of camaraderie within the group.

(b) Easy/Knowledge

Page 215: Fwk Bauer Tif Organizational Behavior All Chapters

78. Collective efficacy isa. the tendency of individuals to put in less effort when working in a group

context.b. the degree of camaraderie within the group.c. any aspect of group interaction that inhibits group functioning.d. a group’s perception of its ability to successfully perform well.

(d) Easy/Knowledge

79. Collective efficacy is influenced by all of the following factors EXCEPTa. how a person feels about the team.b. ties to the organizationc. watching others.d. verbal persuasion.

(b) Medium/Comprehension

Understanding Team Design Characteristics

80. A group isa. a temporary team that addresses a specific problem or issue until it is

resolved.b. a cohesive coalition of people working together to achieve mutual goals.c. a collection of individuals who interact with each other such that one

person’s actions have an impact on the others.d. teams where members are not located in the same physical space.

(c) Easy/Knowledge

81. How does a group differ from a team?a. A group is a cohesive coalition of people; a team is a collection of people.b. In a group, individuals have personal agendas. In a team, personal

agendas are totally suppressed.c. Groups are smaller than teams.d. A group is often a collection of people working independently on their own

goals; a team is assembled to accomplish complex goals.(d) Difficult/Synthesis

82. How does a team differ from a group?a. A team is focused on joint goals; a group often has a collection of

individual goals.b. A team involves the simple sum of independent actions of individuals; a

group involves the integrated, collaborative actions of the collective of people.

c. A team has personal agendas; a group has a shared vision.d. A team has compensation based upon individual outcomes; a group has

compensation based upon shared outcomes.(a) Difficult/Synthesis

83. Process lossa. is a group pressure phenomenon that increases the risk of the group

making a flawed decision.b. is a group’s perception of its ability to successfully perform well.

Page 216: Fwk Bauer Tif Organizational Behavior All Chapters

c. refers to any aspect of group interaction that inhibits group functioning.d. is the tendency for individuals to put in less effort when working in a group

context.(c) Easy/Knowledge

84. All of the following are key properties of a team EXCEPTa. compensation based on shared outcomes.b. collaborative action.c. clear simple goals.d. sacrifice for the common good.

(c) Medium/Analysis

85. Analyses of companies that utilize teams indicate some impressive results. Team-based organizations tend to show all the following results buta. an increase in productivity.b. a reduction in costs.c. an increase in revenue.d. an increase in marketing.

(d) Difficult/Synthesis

86. Which of the following is a situation where a team will NOT be needed in the organization?a. Tasks to be performed are independent of one another. b. The organization benefits from shared goals. c. A variety of skills and abilities are needed. d. Wide cooperation is needed.

(a) Difficult/Evaluation

87. Teams differ by all of the following task classes EXCEPTa. production tasks.b. idea generation tasks.c. development tasks.d. problem-solving tasks.

(c) Easy/Comprehension

88. Arielle is part of a team charged with developing a new name for a product. This is a(n) ________________ task.a. developmentb. productionc. idea generationd. problem-solving

(c) Medium/Application

89. Hank is part of a team doing engineering design on a new hybrid automobile. Hank’s team has completed designing the vehicle. His team is now ready to hand off to another team who will be dealing with _____________ tasks.a. developmentb. productionc. idea generationd. problem

(b) Medium/Application

Page 217: Fwk Bauer Tif Organizational Behavior All Chapters

90. Problem solving tasksa. include actually making something such as a building, product or

marketing plan.b. deal with creative tasks such as brainstorming a new direction or creating

a new process.c. refer to coming up with plans for actions and making decisions.d. involve team members working on each task simultaneously.

(c) Easy/Knowledge

91. Idea generation tasksa. include actually making something such as a building, product or

marketing plan.b. deal with creative tasks such as brainstorming a new direction or creating

a new process.c. refer to coming up with plans for actions and making decisions.d. involve team members working on each task simultaneously.

(b) Easy/Knowledge

92. Task interdependencea. exists when team members work independently and simply combine their

efforts to create a team’s outputs.b. refers to one individual’s output becoming another’s input.c. involves team members working on each task simultaneously.d. refers to the degree that team members are dependent upon one another

to get information, support or materials from other team members to be effective.

(d) Easy/Knowledge

93. An organizational behavior class has students working in teams. The teams have a final paper due. Maria’s group has divided the paper up into sections making each group member responsible for one section. Maria is responsible for integrating all those sections. The group is organizing these tasks by usinga. sequential interdependence.b. pooled interdependence.c. reciprocal interdependence.d. outcome interdependence.

(b) Medium/Application

94. Nathan’s group is responsible for a presentation on their Human Resource project. The group is meeting this evening to prepare the presentation. Nathan has never prepared a presentation as part of a group before, and he is hoping that using such ________________ interdependence will not take a great deal of time because of side conversations and general camaraderie among group members.a. reciprocalb. outcomec. sequentiald. pooled

(a) Medium/Application

Page 218: Fwk Bauer Tif Organizational Behavior All Chapters

95. Outcome interdependencea. exists when team members may work independently and simply combine

their efforts to create the team’s output.b. involves team members working on each task simultaneously.c. refers to the degree that team members are dependent upon one another

to get information, support or materials from other team members to be effective.

d. is when the rewards that an individual receives depend on the performance of others.

(d) Easy/Knowledge

96. Task roles include contractor, creator, anda. cooperatorb. consul.c. coordinator.d. completer.

(d) Easy/Knowledge

97. Boundary spanning roles include coordinator anda. consul.b. cooperator.c. contributor.d. critic.

(a) Easy/Knowledge

98. Which of the following is a social role?a. consul.b. coordinator.c. calibrator.d. contractor.

(c) Easy/Knowledge

99. Eduardo always challenges team members’ positions on various issues discussed in the weekly meeting. Nannette told Eduardo yesterday, “I always have to be prepared when I present ideas to our team because I know you will question everything and make us see the other side of every issue.” Eduardo performs the task role of ____________________ on the team.a. creatorb. contributorc. criticd. completer

(c) Medium/Application

100. The superintendent of the school district is writing a new mission statement for the district. He is engaged in the _____________ role.a. creatorb. contractorc. communicatord. calibrator

(a) Medium/Application

Page 219: Fwk Bauer Tif Organizational Behavior All Chapters

101. The _____________ role in a team creates deadlines for the team, schedules production, and sequences tasks.a. contractorb. contributorc. completerd. creator

(a) Easy/Knowledge

102. Which of the following is NOT correct about the nature of social roles in a team?a. If social roles are filled in a firm, groups are more prone to suffer process

losses.b. If social roles are filled in a firm, groups are more cohesive.c. If social roles are filled in a firm, group members are less likely to engage

in social loafing.d. If social roles are filled in a firm, group members are less likely to have

biases.(a) Medium/Analysis

103. The calibrator rolea. keeps the team on track by suggesting any needed changes to the team’s

process.b. supports those with expertise toward the team’s goals.c. uses good listening skills and humor to defuse tense situations.d. helps transform ideas into action.

(a) Easy/Knowledge

104. Mike listens carefully to the exchanges between team members during meetings and is particularly adept at using humor to lighten any tense situations that arise between team members because of those exchanges. Mike plays the a. completer role.b. communicator role.c. cooperator role.d. calibrator role.

(b) Medium/Analysis

105. Because the ________ role involves gathering information from the larger organization and informing those within the organization about team goals, activities and success, it is often filled by the team manager.a. coordinatorb. completerc. consuld. contractor

(c)

106. All of the following are examples of temporary teams EXCEPTa. cross-functional teamb. product development teamc. task forced. self-managed team

(d) Easy/Comprehension

Page 220: Fwk Bauer Tif Organizational Behavior All Chapters

107. The special challenges that virtual teams pose for a manager include all of the following EXCEPTa. building trust with virtual team.b. not being physically seen by managers.c. nature of decision making by virtual team.d. nature of communication used with virtual team.

(c) Medium/Analysis

108. All of the following statements are correct about top management teams EXCEPTa. Top teams are most effective when there is limited diversity in their

composition.b. Top management teams generally function without formal rules for team

design or structure.c. Top management teams are important for their symbolic roles.d. Top management teams make decisions on new products, markets and

expansions and divestitures.(a) Difficult/Analysis

109. Self-managed teamsa. increase the hierarchy in an organization.b. operate under goals provided by top level management and passed down

through the chain of command.c. have an appointed leader, but can select individual team members.d. can be found at all levels in an organization.

(d) Difficult/Evaluation

110. Self managed teams typically have all the following goals EXCEPTa. increased profitability.b. improved quality.c. cost reduction.d. meeting deadlines.

(a) Medium/Comprehension

111. Empowerment meansa. having the responsibility to reach goals.b. having the authority to reach goals.c. having the ability to make decisions.d. having the responsibility and authority to reach goals.

(d) Medium/Knowledge

112. A self-directed teama. manages itself but still has a team leader and has the potential for low,

medium or high autonomy.b. has a leader who resides outside the team and has the potential for low,

medium or high autonomy.c. has a leader that resides outside the team and has the potential for low

autonomy.d. makes all decisions internally about leadership and how the work is done

and has the potential for high autonomy.(d) Difficult/Evaluation

Page 221: Fwk Bauer Tif Organizational Behavior All Chapters

113. An effective team can be ensured when decisions are made on all of the following items buta. team composition.b. team diversityc. team size.d. All of the above.

(b) Medium/Knowledge

114. Research on team size suggestsa. groups larger than 20 members are highly effective.b. teams with fewer members are less effective at working through

differences.c. larger groups are generally required for very complex tasks.d. there is little relationship between team size and performance.

(c) Medium/Evaluation

115. Which of the following statements regarding team size and diversity is correct?a. The more diverse the team, the more likely the team is to engage in

groupthink.b. The rule of thumb for team size is between 10 and 20 members.c. Teams that believe in the value of diversity perform better than those

which do not.d. Teams whose members have complementary skills are less successful

than those with specialized skills.(c) Difficult/Evaluation

Section III: Organizing Your Team

116. Norms area. agreements on established ground rules, goals and roles.b. shared expectations about how things operate within a group or team.c. formal approaches to decision making in the group or team.d. measures of camaraderie in the group or team.

(b) Easy/Knowledge

117. To make a team contract meaningful, include all the following sections EXCEPTa. team compensation.b. team decision making.c. team performance.d. team values.

(a) Easy/Knowledge

118. Which of the following items is not a key aspect to ensuring the success of a meeting during the course of that meeting?a. Start the meeting on time.b. Don’t worry about group dynamics.c. Manage the group dynamic for full participation.d. Summarize the meeting with action items.

(b) Easy/Comprehension

Page 222: Fwk Bauer Tif Organizational Behavior All Chapters

Section IV: Barriers to Effective Teams

119. All of the following are barriers to effective teams EXCEPTa. overly helpful team members.b. challenges of knowing where to begin.c. poorly managed team conflict.d. poor performance of team members.

(a) Medium/Comprehension

120. Rodney is a new project team leader. Jerome is chatting with him about some of the problems that could arise and prevent his team from being successful. Jerome offers all the following pieces of advice to overcome barriers to team success EXCEPTa. floundering often results from a lack of clear goals; clarify goals from the

outset of the project.b. disagreements among team members may be the result of long

simmering personality conflicts; be aware of them and be prepared to facilitate interaction between them.

c. dominant personalities can only help a group; let those that are the most vocal have their say.

d. poor performance by a group member could be due to a lack of ability; be prepared to train any group members needing it.

(c) Difficult/Evaluation

Section V: The Role of Ethics and Culture in Teams

121. Which of the following statements regarding ethics and teams is INCORRECT?a. Research demonstrates that hierarchical systems always constrain team

members more than a self- managed team does.b. Self- managed teams are viewed as a technique to overcome the

negative aspects of bureaucracy.c. Team pressure can be more powerful than managerial control.d. Team pressure can result in unethical behavior by team members.

(a) Difficult/Synthesis

122. Collectivist societies a. value individual achievement over interpersonal relationships.b. have employees who adapt readily to working in different teams.c. have employees who evaluate their group members higher than their

counterparts in individualistic societies do.d. value loyalty much less than individualistic societies do.

(c) Difficult/Evaluation

123. Which of the following statements regarding power distance and teams is correct?a. People in high power distance countries expect unequal power

distribution in the social, political and economic areas.b. Countries with high power distance tend to use participative decision

making.c. Countries high in power distance tend to use sports metaphors more

often in training than family metaphors.

Page 223: Fwk Bauer Tif Organizational Behavior All Chapters

d. Countries with high power distance define the roles of team members very clearly.

(a) Difficult/Evaluation

CLOSING SECTION: Green Teams at Work: The Case of New Seasons

124. New Seasons is an example of a company with _________.a. providing teams the opportunity to “self-manage.”b. extensive rules driving change.c. pay for performance.d. profits driving decisions.

(a) Easy/Knowledge

FILL IN THE BLANK

125. A ___________ is a collection of individuals who interact with each other such that one person’s actions have an impact on the others.

(group)

126. Two or more individuals who are associated with one another in ways not prescribed by the formal organization are a(n) _____________ _____________.

(informal group)

127. Tuckman’s four-stage model of group development features the forming- ___________-_____________ and performing stages.

(storming, norming)

128. In the _________ stage, group members can be defensive, competitive or jealous.

(storming)

129. Many groups get stuck in the _____________ phase.(storming)

130. Group members establish ground rules and define operating procedures during the ___________ phase of Tuckman’s model.

(norming)

131. The _________ stage was added to Tuckman’s original model and can feature participant emotions ranging from victory to grief to insecurity.

(adjourning)

132. The _______ ____________ model is a cyclical theory that suggests that change in groups occurs in radical spurts rather than incrementally over time.

(punctuated equilibrium)

133. ____________ groups have a collective identity, share a sense of purpose, work together on meaningful tasks, and establish a structured pattern of communication.

Page 224: Fwk Bauer Tif Organizational Behavior All Chapters

(Cohesive)

134. __________ is a group pressure phenomenon that increases the risk of the group’s making flawed decisions by allowing reductions in mental efficiency, reality testing and moral judgment.

(Groupthink)

135. The Ringelmann effect, or ________ ________ is the tendency of individuals to put in less effort when working in a group context.

(social loafing)

136. A group’s perception of its ability to successfully perform well is _________ _______.

(collective efficacy)

137. A ________ is a cohesive coalition of people working together to achieve mutual goals.

(team)

138. Team tasks include __________ tasks, ________ __________ tasks and _____________ tasks.

(production, idea generation, problem-solving)

139. __________ _____________ is where team members work independently on items and then combine their efforts for a final product while _________ ___________ is where team members work on each task simultaneously.

(Pooled interdependence, reciprocal interdependence)

140. The Team Role Typology suggests that ___________ and ____________ are boundary spanning roles.

(consul, coordinator)

141. ___________, ______________, and _______________ are three task roles in the Team Role Typology.

(Contractor, creator, contributor, completer, critic)142. The task role of ____________ is includes “devil’s advocate” behavior.(critic)

143. The social role of ________________ serves to keep the team on track with regard to needed changes in team process.

(calibrator)

144. A _________ ____________ is a temporary team that is created to address a specific problem or issue until it is resolved.

(task force)

145. __________ teams are those where members are not located in the same physical place.

(Virtual)

Page 225: Fwk Bauer Tif Organizational Behavior All Chapters

146. Empowered teams have the ___________ as well as the __________ to achieve their goals.

(responsibility, authority)

147. _______________________ teams make all decisions internally about leadership and how work is done and have the potential for high autonomy.

(Self-directed)

148. Shared expectations of how things operate with a group or team are _________.(norms.)

149. _______ ___________ include agreements on established ground rules, goals and roles.

(Team contracts)

SHORT ANSWERS

150. List the stages in Tuckman’s model of group development including the phase added later in his work. Briefly describe the nature of each stage.

The stages are forming, storming, norming and performing with adjourning added later.

Forming is the initial stage where the group comes together for the first time.

Storming is the second stage where group members keep up their guard but become more argumentative. Also, during this phase, group members explore their power and influence and stake out their territory by differentiating themselves from others.

Norming is the phase where group members establish their own ground rules and define operating procedures and goals.

Performing is the phase where participants are getting the work done and paying attention to how they are doing it.

Adjourning is the phase added later which is focused on how a group ends. Endings are often accompanied by emotions that can range from relief to grief.

151. Briefly discuss the punctuated equilibrium model.

The punctuated equilibrium model was developed by Connie Gersick. The model suggests that groups remain fairly static for long periods of time where change is largely incremental. At some point, however, revolutionary change occurs in a brief, punctuated burst that shakes the very essence of the organization and poses a real opportunity for innovation and creativity to take root.

152. List and describe three factors which affect group cohesion.

Any of these factors impact group cohesion:

Page 226: Fwk Bauer Tif Organizational Behavior All Chapters

Similarity: The more similar group members are with regard to gender, age, education, skills, attitudes and values, the more likely the group will bond.

Stability: The longer a group stays together, the more cohesive it becomes.

Size: The smaller the group, the more cohesive the group.

Support: When group members support their fellow team members, their groupidentity strengthens.

Satisfaction: Cohesion is correlated with how pleased group members are witheach other’s performance, behavior and conformity to group norms.

153. What is the difference between a group and a team?

A group is a collection of individuals who interact with each other such that one person’s actions have an impact on the others. A team is a cohesive coalition of people working together to achieve mutual goals. The difference is that teams are focused on a joint goal that gives them a common purpose, performance goal and approach for which they are mutually accountable.

154. Hackman identified three major classes of tasks. List those classes and describe them.

Production tasks are those involved with making something.

Idea generation tasks deal with creative tasks like brainstorming.

Problem-solving tasks refer to coming up with plans for action and making decisions.

155. Describe task interdependence levels as they relate to teams.

Task interdependence is the degree to which team members are dependent upon one another to get information, support, or materials from other team members to be effective. The three types of task interdependence are: pooled interdependence, sequential interdependence and reciprocal interdependence.

Pooled interdependence exists when team members work independently and combine their efforts to create a team’s output. Sequential interdependence exists when one person’s output is another’s input. Reciprocal interdependence exists when team members work on a task simultaneously.

Outcome interdependence is not specific to a task but exists when rewards received by an individual are dependent on the performance of others.

156. Describe the three general categories of work team roles and then give an example of each. Describe the nature of each example role given.

The three categories of team roles are: task roles, social roles and boundary spanning roles.

Page 227: Fwk Bauer Tif Organizational Behavior All Chapters

Task roles include: critic, completer, contributor, creator and contractor.

A critic is a “devil’s advocate”; a completer is one who transforms ideas into action; a contributor brings information and expertise to the team; a creator deals with changes in the team’s process and a contractor organizes the team’s work.

Social roles include: calibrator, communicator, and cooperator.

A cooperator supports those with expertise to aid in the team’s goals; a communicator includes behaviors targeted at collaboration like humor and good listening skills; and a calibrator keeps the team on track in terms of any changes needed to the team’s process.

Boundary spanning roles include: consul and coordinator.

A consul gathers information from the larger organization and informs those within the organization about the team’s activities, goals and successes; and a coordinator interfaces with others so that the team’s efforts are in line with other individuals and teams within the organization.

157. There are several types of temporary teams. Describe one such temporary team.

A task force is one designed to address a specific issue or problem until it is resolved.

A product development team is one that develops a product.

A cross functional team involves individuals from different parts of the organization staff.

158. Teams vary in the manner in which they are led. Choose one type of team leadership and briefly describe it.

Traditional manager led teams have a manager who is outside the team. The manager assigns work to team members and has the power to hire and fire team members. Manager led teams have little potential for autonomy.

Self-managed teams are those that manage themselves and do not report directly to a supervisor. Team members select their own leader and can even take turns in the leadership role. A self-managed team has the potential for low, medium or high autonomy.

Self-directed teams are a special form of self managed teams where members determine who will lead them with no external oversight. Thus, the team makes all decisions internally about how work is done. There is the potential for high autonomy in this team.

ESSAY

Page 228: Fwk Bauer Tif Organizational Behavior All Chapters

159. Think about a group that you belong or belonged to. Describe the phases of the Tuckman model that the group progressed through or the current phase at which it is now functioning.

Answers will vary. Students might discuss a group they are part of from one of their classes. Students are often quite willing to discuss the formation problems many groups have. The initial, and indeed early meetings are characterized by members being very polite and very general and noncommittal in most of their comments as they observe what is going on to figure out the direction the group might be headed. If a leader has been appointed for the group, that leader might be tested.

The group might have progressed to the storming stage and here the stories will really flow. As the name suggests there are battles fought during the storming stage as group members explore power and influence. There may be cliques, they may take sides, but whatever occurs, there will be chaos.

Though many groups in the workplace may not get out of the storming stage, class groups must progress because terms end and final projects or papers are ultimately due. So, while a group may get its project out while functioning in the storming phase, class groups move to adjournment. Some of the leave-taking in the groups is bittersweet, some joyful.

If a class group progressed out of the storming stage to norming, the analysis will discuss the ground rules and procedures agreed upon by the group. In the performing stage there will be discussion of how the work is getting done and the manner in which process issues or items that may impact the process (like conflicts) are addressed so the work of the group moves forward.

160. Marta has just been appointed a project team leader for the first time. She is interested in understanding key characteristics that could impact the success of her team. You are her supervisor and she is coming to see you tomorrow for advice. What are some of the key elements you will mention to her and what impact do they have upon a team?

One item you could mention is the notion of building cohesion in the team. Cohesive groups have a common identity, and members want to be part of the group. Help Marta understand how to build that notion in the team members.

Share with Marta that similarity, stability, size, support and satisfaction are all issues connected to cohesion. Discuss with her how groups that are similar in terms of gender, age, background, skills, attitudes and beliefs are more cohesive, but warn her of going too far that the group becomes involved in a groupthink situation. Stable groups are more cohesive. Small groups are easier for building cohesion.

161. One of the biggest complaints about college courses is often the requirement to work in groups. Many students are uncomfortable with group work because some of their colleagues do not fully participate in the assignments leaving individual

Page 229: Fwk Bauer Tif Organizational Behavior All Chapters

members with extra work to do. What are some actions you can take the next time you are assigned to a group that will help prevent such social loafing in your group?

Carefully choose the number of people you work with if that number is not assigned by the instructor of the course. Obviously, the larger the group, the easier it is for a student to “hide.”

During the initial meetings, make certain that all members are aware of their roles and publicly acknowledge those roles. Be specific about the tasks assigned and make sure to assign them to individuals. Specificity provides for greater accountability.

Have a way to evaluate each person’s contribution to the group. Many courses create group feedback forms so individuals are aware of the expectations they must fulfill as well as the evaluation provided for completion of such.

Build a cohesive group. Recognize that mutual goals bond people. Work on building that shared experience.

Assign tasks that are engaging and rewarding.

Make sure everyone feels needed.

With these tips in mind, social loafing should be minimized in the group.

162. As part of a Fundamentals of Business Communication course, students are placed in four-person teams to engage in various team activities throughout the course. The first exercise the groups are required to complete is the development of a team contract. Discuss what some key elements of that team contract should be, given that work produced by the team will be evaluated and will impact the individual grade achieved by a student.

Some categories to include are:Team goal: A mutually shared goal is paramount to building cohesion.

Team roles: Who is responsible for what in the team? When assigning roles, first ensure that the team covers all three key categories of roles (boundary spanning, task and social).

Team decisions: How are decisions made in the team?

Team communication: Who will be the individual to inform all group members of meetings or cancellations? What medium will be used to communicate?

Team performance: What criteria will you use to evaluate your team members? What is good performance of a team?

163. A second assignment for the Fundamentals of Business Communication is to be the leader of a team meeting held for a project. Offer some advice to a student

Page 230: Fwk Bauer Tif Organizational Behavior All Chapters

taking the Fundamentals class on how to make his meeting the most effective possible.

The very first step is to ascertain whether a meeting actually needs to be held. While in the scenario above it will take place because a grade is awarded for the meeting project, in the workplace, sometimes meetings simply do not need to be held. If one is needed, before the meeting, make sure everyone has an agenda. And don’t forget to remind people of the meeting.

During the meeting make sure the meeting opens on time and that the meeting agenda is followed. If there is a time limit indicated for each agenda item, honor it. Make sure you get equitable participation from all group members. Summarize the meeting with specific action items. End the meeting on time.

After the meeting, follow up on action items. Send minutes of the meeting in a timely manner.

Page 231: Fwk Bauer Tif Organizational Behavior All Chapters

Organizational Behavior, Version 1.1Bauer & Erdogan

FWK Test Item FileChapter 10

TRUE/FALSE

1. Social issues of ego and corporate pride played a key role in the falling apart of PointCast negotiations.

(True)

2. Managing conflict and engaging in effective negotiations are key factors for a successful organization.

(True)

3. Conflict management and negotiation tactics are art, not skills, and are very difficult to learn.

(False)

4. Intrapersonal conflict can arise due to role ambiguity and role conflict.(True)

5. Interpersonal conflicts should be centered around individual differences, not ideas, to avoid conflict escalation.

(False)

6. Conflict in an organization is always bad.(False)

7. If conflict in an organization is too low, performance is also likely low.(True)

8. Personal conflicts can be good in certain circumstances, but task conflicts are never good.

(False)

9. Research on effective teams indicates that they are characterized by low but increasing levels of process conflict.

(True)

10. Research on effective teams indicates that they are characterized by low but increasing levels of task conflict.

(False)

11. Matrix-structured organizations often experience decisional conflict because each manager reports to two bosses.

(True)

Page 232: Fwk Bauer Tif Organizational Behavior All Chapters

12. Task independence is a root cause of much conflict in the organization.(False)

13. Conflict can result in both positive and negative outcomes.(True)

14. Conflict can be dysfunctional if it involves personal attacks.(True)

15. A high-risk situation where workplace violence has an increased risk is being a bank teller.

(True)

16. If your job involves caring for others in a nursing home, you have an increased risk of experiencing workplace violence.

(True)

17. If the conflict in your workplace is between two individuals on a team, a good way to manage the conflict is to take a majority vote among other team members to determine who is right.

(False)

18. The avoidance conflict handling style is characterized by high levels of cooperativeness and assertiveness.

(False)

19. An individual who deals with conflict by saying, “Maybe we can both agree to give in a little,” has an accommodating conflict handling style.

(False)

20. Competition is an effective conflict handling style if the alternatives proposed are unethical.

(True)

21. The strategy to utilize to achieve the best outcome from conflict is the collaborating style.

(True)

22. Research suggests that when it comes to dealing with conflict, managers are more likely to engage in avoiding, accommodating or compromising styles than their subordinates.

(False)

23. The key to keeping conflict or disagreement healthy in an organization is to focus the discussion on the task, not the personalities.

(True)

24. Healthy conflict can be stimulated in a firm by having appointed individuals play “devil’s advocate.”

(True)

Page 233: Fwk Bauer Tif Organizational Behavior All Chapters

25. The BATNA phase of the negotiation process should be ignored.(False)

26. The first place to begin the investigation phase of negotiation is to look at yourself.

(True)

27. The party with the best alternative to a negotiated agreement is in the best negotiating position.

(True)

28. Making a concession during negotiation is a sign of weakness.(False)

29. The closure phase of the negotiation process may include finding a final offer that is unacceptable and from which the party has walked away.

(True)

30. The integrative approach is a new creative approach to negotiation that can lead to a “win-win” situation.

(True)

31. It is a perfectly acceptable business practice to attempt to negotiate a higher salary with your boss.

(True)

32. A common mistake women make is failing to negotiate their salary offers.(True)

33. Those with unreasonable expectations in negotiations are no more likely to fail at negotiations than those with reasonable expectations.

(False)

34. Mediation is the process of bringing in a third party who has the authority to act as a judge and make a binding decision in the conflict situation.

(False)

35. The arbitration-mediation approach had led to voluntary agreements in a greater percentage of situations than the more common mediation-arbitration approach.

(True)

36. If your negotiating counterpart feels you are unfair or dishonest, he or she is less likely to make concessions.

(True)

37. The platinum rule of negotiations is to care for others enough to treat them the way they want to be treated and long-term relationships will be established.

(True)

38. Americans have a higher tolerance for conflict as a way of working through issues than their Japanese counterparts.

Page 234: Fwk Bauer Tif Organizational Behavior All Chapters

(True)

39. Japanese negotiators learn information from what is not said as well as what is said that is then used in negotiations.

(True)

40. Western cultures view negotiations as a social activity while their Eastern culture counterparts view it as a business activity.

(False)

MULTIPLE CHOICE

Opening Section: Negotiation Failure: The Case of PointCast

41. The lesson learned from PointCast isa. social issues in negotiation can be deal-breakers.b. take the first offer you are presented.c. financial issues always make or break a deal.d. unless you have to disclose something, don’t.

(a) Medium/Evaluation

Section I: Understanding Conflict

42. Conflict isa. a process whereby two or more parties work toward an agreement.b. a process where an outside third party enters the situation with the goal of

assisting the parties to reach an agreement.c. a process that involves people disagreeing.d. a process that involves bringing in a third party who has the authority to

act as a judge and will make a decision to which parties must adhere.(c) Easy/Knowledge

43. Jared is a member of the engineering department in the manufacturing firm and also serves on the Strategy 2015 team, which is crafting an engineering strategy for the firm’s future. He has a “Do not miss” meeting in the engineering department scheduled for Wednesday at 8 a.m. and just received an urgent email about a “Can’t miss” meeting for the strategy group at 8:15 a.m. Wednesday morning. Jared is experiencinga. interpersonal conflict.b. role ambiguity.c. role conflict.d. group conflict.

(c) Medium/Application

44. Intrapersonal conflict isa. a type of conflict between two people.b. conflict that takes place between different groups such as between

different departments.c. a process that involves people disagreeing.d. a type of conflict that arises when a person is uncertain about what is

expected or wanted or has a sense of being inadequate to the task.

Page 235: Fwk Bauer Tif Organizational Behavior All Chapters

(d) Easy/Knowledge

45. All of the following are examples of intrapersonal conflict EXCEPTa. role conflict.b. role differences.c. role ambiguity.d. role reversal.

(d) Easy/Comprehension

46. Conflict between two individuals such as coworkers isa. intrapersonal conflictb. interpersonal conflict.c. intergroup conflict.d. interdepartment conflict.

(b) Easy/Knowledge

47. Nancy and Mary are in the same sorority. Nancy is running for the university’s student government treasurer position. Mary says, “I’ll never vote for her, whether she is my sister or not.” Mary is experiencing what kind of conflict?a. interpersonalb. intergroupc. intrapersonald. interdependent

(a) Medium/Application

48. Intergroup conflicta. occurs between two individuals.b. occurs within the individual.c. occurs between two departments.d. occurs between team members.

(c) Easy/Comprehension

49. Which of the following statements about conflict is INCORRECT?a. Conflict can paralyze an organization.b. Conflict can lead to less than optimal performance.c. Conflict generally makes people uncomfortable.d. Conflict is always problematic in an organization.

(d) Medium/Comprehension

50. Which of the following statements regarding the relationship between conflict and performance is INCORRECT?a. Low conflict leads to low performance.b. Medium conflict leads to high performance.c. High conflict leads to low performance.d. Low conflict leads to high performance.

(d) Difficult/Evaluation

51. In which of the following scenarios would you expect high performance?a. The five team members never have a negative word to say to each other;

there never seems to be any conflict on the team.

Page 236: Fwk Bauer Tif Organizational Behavior All Chapters

b. The two members of the team are constantly bickering. One never says anything without the other disagreeing.

c. The team members seem to get along well enough, but on occasion they will argue about a number of issues.

d. The three partners have an interesting relationship. Partner A is almost always in agreement with Partner B, but fights constantly with Partner C. Partners B and C get along at times and fight at times.

(c) Difficult/Synthesis

52. Research on effective teams indicates that they experiencea. high levels of relationship conflict.b. moderate levels of process conflict at the beginning of the project

timeline. c. moderate levels of task conflict in the middle of the project timeline.d. moderate levels of relationship conflict and high levels of process conflict.

(c) Difficult/Evaluation

Section II: Causes and Cures for Conflict

53. Which of the following is not a root cause of conflict in a workplace?a. organizational structureb. task interdependencec. compatible goalsd. personality differences

(c) Easy/Comprehension

54. In which of the following scenarios are you most likely to experience conflict?a. Resources within the firm are relatively plentiful and most departments in

the organization are seeing a modest increase in their annual budget.b. Interdepartment meetings have just been concluded that created a list of

goals for each department that are compatible with one another.c. The firm just instituted a matrix structure.d. Self-awareness exercises have been completed and most of the team

members were categorized as Type B.(c) Difficult/Evaluation

55. Which of the following is not a root cause of conflict in the workplace?a. task satisfactionb. communication issuesc. limited resourcesd. personality differences

(a) Easy/Comprehension

56. Which of the following is not a positive outcome of conflict in the organization?a. increased creativityb. accurate assumptionsc. enhanced decision makingd. individual viewpoint clarification

(b) Medium/Comprehension

57. Which of the following is NOT a negative outcome of conflict in the organization?

Page 237: Fwk Bauer Tif Organizational Behavior All Chapters

a. excessive personal attacksb. climate of distrustc. employee feelings of being demeaned with resulting lower moraled. low individual stress

(d) Medium/Comprehension

58. Which of the following factors does NOT increase your risk for violence in the workplace?a. caring for others emotionallyb. dealing with valuablesc. being well paidd. supervising others

(c) Medium/Comprehension

59. Which of the following is a high-risk situation where your job might be at risk for workplace violence?a. supervising othersb. denying requests made by othersc. caring for othersd. all of the above

(d) Medium/Comprehension

60. Which of the following occupations is least likely to offer an employee a high-risk situation for workplace violence?a. law enforcementb. educationc. bartendingd. overnight gas station attendant

(b) Medium/Comprehension

61. Which of the following statements regarding conflict and its cures is INCORRECT?a. A bureaucratic organizational structure can lead to conflict between high

and low power people.b. To avoid conflict in communication, focus on the person, not behavior or

its effects.c. Conflict can be dysfunctional in an organization if it is excessive.d. Consideration of a broader range of ideas is a positive outcome of conflict

in the organization.(b) Difficult/Synthesis

Section III: Conflict Management

62. Which of the following is an effective way to manage organizational conflict?a. consensus decision makingb. change the team composition to have more diversity c. create a common opposing forced. eliminate hierarchy

(c) Easy/Comprehension

Page 238: Fwk Bauer Tif Organizational Behavior All Chapters

63. If there is conflict between two members of a team and the underlying reason for the conflict is personality differences, what is a viable way of managing the organizational conflict?a. Change the structure of the organization.b. Focus attention on a common enemy of the two clashing team members.c. Take a majority vote on which individual is causing more issues on the

team.d. Change the composition of the team by separating the individuals at

odds.(d) Difficult/Synthesis

64. If two marketing groups within a firm’s marketing department are vying against each other for a larger percentage of the budget, which option that the marketing manager could consider to manage the organizational conflict would be MOST effective?a. Take a majority vote on what the two departments feel is the best budget

allocation.b. Focus the attention of the two groups on a common “enemy” such as

another firm that that they compete with for business.c. Change the composition of the two departments.d. Restructure the entire department so that teams are no longer used.

(b) Difficult/Synthesis

65. The conflict handling style that is uncooperative and unassertive isa. compromise.b. avoidance.c. accommodation.d. competition.

(b) Easy/Knowledge

66. The conflict handling style that is middle ground where an individual’s desire to get his or her way but respect others’ goals isa. competition.b. avoidance.c. accommodation.d. compromise.

(d) Easy/Knowledge

67. The competition conflict handling stylea. is a middle ground style.b. is uncooperative and unassertive.c. is cooperative and unassertive.d. is highly assertive but low on cooperation.

(d) Easy/Knowledge

68. The collaborative conflict handling style isa. low in cooperativeness and low in competitiveness.b. low in competitiveness and high in cooperativeness.c. high in cooperativeness and high in competitiveness.d. high in competitiveness and low in cooperativeness.

(c) Medium/Comprehension

Page 239: Fwk Bauer Tif Organizational Behavior All Chapters

69. The accommodation conflict handling style is a. cooperative and unassertive.b. uncooperative and unassertive.c. cooperative and assertive.d. uncooperative and assertive.

(a) Medium/Comprehension

70. Nicholas is the production manager for a manufacturing firm. He has two supervisors who are experiencing conflict with each other based upon personality differences. Nicholas should have held a meeting last week to discuss next year’s budget, but cancelled it because the two supervisors had a verbal confrontation on the shop floor the previous day. What conflict handling style is Nicholas demonstrating?a. compromiseb. collaborationc. avoidanced. accommodation

(c) Medium/Application

71. The United Auto Workers (UAW) and Ford Motor Company were negotiating a new agreement to address the auto industry woes during a severe economic downturn. Ford offered to consider delaying layoffs if the UAW agreed to changes in contractual work rules related to the jobs bank. This negotiation suggests the use of what conflict handling style?a. avoidanceb. collaborationc. accommodationd. compromise

(d) Medium/Application

72. Ned and Michael are managers of two different departments at Beckett Corporation. The two have not gotten along well over the years. Yesterday the two attended a budget meeting for the new fiscal year. Both wanted more funding for their departments. When the finance vice president suggested an increase for Ned’s department, Michael became quite agitated and made a number of accusations about the nature of the numbers that Ned had used to support his position. After the meeting, Michael cornered the finance vice president in his office and continued his spiel on why his department deserved the extra funding but Ned’s did not. What style of conflict handling does Michael exhibit?a. compromiseb. accommodationc. competitiond. avoidance

(c) Medium/Application

73. Olivia and her teenage daughter, Natalie, are arguing again about whether Natalie should go to a family gathering this Sunday or attend an outing with her friends. Olivia and her daughter argue about this topic every week. This time, Olivia turns to her daughter and says, “OK, if it is that important that you go out

Page 240: Fwk Bauer Tif Organizational Behavior All Chapters

with Jenna Sunday, then I can agree this time around.” Olivia is using what conflict handling style?a. accommodationb. controllingc. collaborationd. avoidance

(a) Medium/Application

74. Maris Manufacturing is a unionized company producing customized metal units for the appliance and medical equipment industries. Each contract negotiation is a contentious affair for the union and management. However, over the past few months both union and management officials have become very concerned about the number of Maris union employees who are experiencing alcohol-related issues. Last month alone, six employees sought medical assistance under the employee assistance program. Management and union officials will be meeting next week to address the issue. This is an example of what conflict handling style?a. accommodationb. collaborationc. compromised. competition

(b) Medium/Application

75. Which conflict handling style is indicated by an individual who says, “I don’t really care if we work this out.”?a. accommodationb. collaborationc. avoidanced. compromise

(c) Medium/Application

76. Which conflict handling style is indicated by an individual who says, “Maybe we can both agree to give in a little.”?a. compromiseb. collaborationc. accommodationd. avoidance

(a) Medium/Application

77. Though no conflict handling style is perfect, which style seems to be the most effective in many different situations?a. compromiseb. competitionc. collaborationd. accommodation

(c) Medium/Comprehension

78. Research shows that when managers are dealing with conflict they prefer which conflict handling style?a. compromisingb. accommodating

Page 241: Fwk Bauer Tif Organizational Behavior All Chapters

c. avoidingd. forcing

(d) Medium/Comprehension

79. Research shows that when confronted with a conflict situation, subordinates are most likely to use all of the following conflict handling styles EXCEPTa. compromisingb. forcingc. avoidingd. accommodating

(b) Medium/Analysis

80. With regard to conflict, which of the following statements is INCORRECT?a. The best solution often requires a discussion of varying opinions.b. A lack of disagreement may indicate employees are withholding opinions.c. Conflict is inherently bad.d. A key to a healthy discussion is a focus on tasks, not personalities.

(c) Difficult/Analysis

81. All of the following would be effective means of stimulating conflict EXCEPTa. assigning someone to play devil’s advocate.b. encouraging disagreement with no fear of reprisal.c. creating competition between teams or individuals with a bonus for those

with the best solution.d. building clarity into the situation.

(d) Medium/Analysis

Section IV: Negotiations

82. Negotiation isa. the process that involves bringing in a third party who has the authority to

act as a judge and will make a binding decision.b. the process that involves two people disagreeing.c. the process where a third party enters a situation with the goal of

assisting the parties to reach an agreement.d. the process whereby two or more parties work toward an agreement.

(d) Easy/Knowledge

83. Which of the following is the first phase of the negotiation process?a. investigationb. presentationc. determining BATNAd. bargaining

(a) Easy/Knowledge

84. During which phase of the negotiation process does an individual assemble the information gathered in such a manner that his position is supported?a. investigationb. determining the BATNAc. bargainingd. presentation

Page 242: Fwk Bauer Tif Organizational Behavior All Chapters

(d) Easy/Comprehension

85. The investigation phase of the negotiation process involvesa. understanding what your alternatives are.b. gathering information.c. making concessions.d. agreeing on terms.

(b) Easy/Comprehension

86. Which of the following statements regarding to the first phase of the negotiation process, investigation, is INCORRECT?a. Investigation is the phase of the negotiation process that is often ignored.b. Focus on the party with whom you are negotiating first and then analyze

yourself.c. The investigation phase of negotiation is designed to gather information.d. The investigation phase necessitates a clear understanding of priorities.

(b) Difficult/Evaluation

87. The BATNA isa. the first phase of the negotiation process.b. a delaying tactic in negotiation.c. important to assisting you in deciding whether to accept an offer or not.d. the process of making concessions in negotiation.

(c) Medium/Analysis

88. All of the following statements are correct with regard to determining your BATNA EXCEPTa. The party with the best BATNA is in the best negotiating position.b. You must determine your own BATNA but there is little you can do with

regard to the BATNA of the party with whom you are negotiating.c. Assessing the best alternative to a negotiated agreement assists you in

deciding whether to accept an offer.d. The reason you negotiate is to obtain something better than what you

would obtain without negotiating, so know your alternatives.(b) Difficult/Evaluation

89. Which of the following statements regarding the bargaining phase of negotiation is correct?a. The bargaining phase gets the least amount of attention during the

negotiation process.b. Making concessions during the negotiation process is a sign of

weakness.c. A key to the bargaining phase is asking questions.d. This is the phase of negotiation where you determine your BATNA.

(c) Difficult/Evaluation

90. Which of the following statements regarding the closure phase of negotiations is INCORRECT?a. Closure is the phase of the negotiation process where you either have

come to an agreement or walk away from an unacceptable offer.b. Rejection of an offer during closure can be an opportunity to learn.

Page 243: Fwk Bauer Tif Organizational Behavior All Chapters

c. It is not always clear why an agreement has not been reached during negotiation.

d. If the best offer in a negotiation has been rejected, there is nothing left to do.

(d) Medium/Analysis

91. The department managers for Zylog Corporation are meeting for a budget meeting. If the marketing manager gets the 6% increase he is requesting, the human resource manager will not get any increase in his department budget. This is an example of what kind of negotiation strategy?a. integrative approachb. accommodating approachc. distributive approachd. concession approach

(c) Medium/Application

92. If both parties to a budget negotiation seek to expand the overall budget so that each party gets more, this is an example of which kind of negotiation strategy?a. integrative approachb. accommodating approachc. distributive approachd. concession approach

(a) Medium/Application

93. The first step to an integrative approach to negotiation isa. listening.b. exploring options to achieving goals.c. adopting a cooperative stance.d. adopting an adversarial stance.

(c) Medium/Analysis

94. Which of the following statements regarding negotiation strategies is correct?a. An adversarial stance is needed to begin integrative negotiation.b. Focusing on a “fixed pie” in negotiations is a good approach because it

controls the number of options to be considered.c. Reviews of negotiation experiments indicate that in almost all those

situations where an integrative approach to negotiations could be adopted, such an approach was utilized.

d. Listening is a key element in the integrative approach to negotiation.(d) Difficult/Synthesis

95. Which of the following steps is the FIRST you should undertake in negotiating a higher salary?a. Overcome your fear.b. Get the facts.c. Know what you want.d. Be assertive.

(a) Easy/Comprehension

96. Which of the following is NOT a recommended step in negotiating a higher salary for yourself?

Page 244: Fwk Bauer Tif Organizational Behavior All Chapters

a. Overcome your fear.b. Begin aggressively.c. Get the facts.d. Listen more than you talk.

(b) Medium/Analysis

97. Which of the following is NOT a common mistake in negotiations?a. Letting your ego get in the way.b. Deciding to negotiate.c. Having unrealistic expectations.d. Getting overly emotional.

(b) Medium/Analysis

98. Mark has been the teachers’ union representative for contract negotiations for the past five contracts. Today the school district presented a counterproposal in which they requested that teachers make a minimal contribution for their healthcare premiums. The expiring contract makes the school district entirely responsible for the premiums. Mark is outraged. He throws the proposal given him on the floor and yells at the school district representative as he exits the negotiation room, “This is a pure and simple insult to my members and I will not sit at a table with anyone who treats them so poorly.” Mark is committing what negotiation mistake?a. having unrealistic expectationsb. letting past negative outcomes impact the present onesc. getting overly emotionald. letting his ego get in the way

(c) Difficult/Evaluation

99. Which of the following statements regarding common mistakes in negotiations is correct?a. Research shows that negotiation avoidance is particularly pronounced

among men.b. Setting high goals decreases some of the tension of negotiations.c. Negotiators that have engaged in ineffective negotiations in the past are

more likely to learn lessons which make them successful negotiators in the future.

d. Those with greater power are more likely to be effective when using anger in negotiations.

(d) Difficult/Analysis

100. Research indicates which of the following findings regarding negotiations is correct?a. Those who encounter anger during negotiation are likely to counter with

an accommodation approach to negotiations.b. Though men have a greater tendency to negotiate in a salary situation,

they are no more successful than women in securing an increase.c. Setting integrative goals is no more likely to improve the chances of

arriving at an agreement than going into negotiations with adversarial goals.

Page 245: Fwk Bauer Tif Organizational Behavior All Chapters

d. Individuals who were unable to negotiate a deal in previous negotiation situations tended to have lower outcomes in subsequent negotiations than those who had initially negotiated deals in the past.

(d) Difficult/Evaluation

101. Which of the following is NOT a good piece of advice to offer individuals who want to engage in successful negotiations?a. Don’t establish deadlines; when the negotiations are over, they are over.b. Be patient; don’t rush things.c. Don’t worry about periods of silence; just wait.d. Focus on agreement first and then go back to those areas that are

unresolved.(a) Medium/Analysis

102. Mediation isa. the process of bringing in a third party who has the authority to act as a

judge and will make a binding decision.b. the process where a third party enters the situation with the goal of

assisting the parties to reach an agreement.c. the process whereby two or more parties work toward an agreement.d. the process that involves people disagreeing.

(b) Easy/Knowledge

103. Allan is a member of a union. The supervisor at the firm where he works gave Allan a three-day suspension for not wearing his protective shoes on the manufacturing floor of his firm. Allan appealed the suspension and the company’s plant manager refused to rescind the suspension. The union wants to appeal this decision to a neutral third party for a ruling. What procedure does the union want to utilize?a. mediationb. negotiationc. arbitrationd. conflict management

(c) Easy/Application

104. Which of the following statements regarding the process of mediation is INCORRECT?a. A mediator determines the underlying interests.b. A mediator clears up misunderstandings.c. A mediator resolves the charge made in a negotiation.d. A mediator incorporates areas of agreement into resolutions.

(c) Medium/Comprehension

105. Which of the following statements regarding third party negotiations is correct?a. Individuals who utilized mediation indicated in a survey that they would

not be supportive of using the process again.b. The mediator assists the parties in agreeing on mutually exclusive

resolutions.c. Arbitrators help prepare for a formal hearing.d. Arbitration is an acceptable practice for many companies to resolve

challenging problems.

Page 246: Fwk Bauer Tif Organizational Behavior All Chapters

(d) Medium/Evaluation

106. A mediator should enter a negotiation whena. personal differences are detected.b. the parties are only talking for short periods of time and tension is rising.c. the parties are unable to find a solution.d. a quick resolution is desired but not imperative.

(c) Medium/Evaluation

107. Which of the following statements regarding the arbitration-mediation approach is INCORRECT?a. Agreements between the two parties are less likely to occur with an

arbitration-mediation approach than a mediation-arbitration approach.b. In the arbitration-mediation approach, the arbitrator makes a decision and

then places it in a sealed envelope while the mediation progresses.c. The arbitration-mediation approach begins with both sides formally

presenting their cases to the arbitrator.d. The mediation-arbitration approach is a more common approach than the

arbitration-mediation one.(a) Medium/Evaluation

Section V: The Role of Ethics and Culture in Negotiations

108. Which of the following statements regarding ethics and negotiations is INCORRECT?a. Hardball negotiation tactics are fair game; the goal after all is to “get” all

you can.b. If you try to destroy one party in negotiating, that party may return the

favor at a future date.c. A party who feels you are being unfair will be less likely to make

concessions.d. A party who feels you are dishonest may refuse to negotiate with you at

all.(a) Medium/Evaluation

109. Which of the following statements regarding negotiations around the world is correct?a. American negotiators like to establish a strong relationship first while

Chinese negotiators like to “get down to business”.b. Japanese negotiators have a higher tolerance for conflict as a way of

working through issues than their American counterparts.c. Japanese negotiators reveal more information during negotiations than

their American counterparts.d. Japanese negotiators are likely to interpret negotiation offer patterns over

time and include them in their negotiations.(d) Difficult/Evaluation

110. All of the following statements regarding negotiations around the world are correct EXCEPTa. Given their culture, Chinese negotiators are uncomfortable saying “no.”

Page 247: Fwk Bauer Tif Organizational Behavior All Chapters

b. Some cultures find it disrespectful to negotiate prior to establishing a trusting relationship.

c. Since Japan is a high context culture, information for negotiation is learned from what is not said as well as what is said.

d. Eastern cultures tend view negotiations as a business activity while Western cultures see it as a social activity.

(d) Difficult/Evaluation

FILL IN THE BLANK

111. The social issues of _________ and ____ derailed the PointCast deal and are among the most difficult aspects of negotiating mergers today.

(ego, corporate pride)

112. __________ is the process that involves people disagreeing.(Conflict)

113. When you are uncertain about what is expected or wanted, or have the sense of being inadequate to perform a task, you are experiencing _____________ _____________.

(intrapersonal conflict)

114. Union and management conflicts are examples of ____________ conflict.(intergroup)

115. A __________ amount of conflict is related to high performance.(moderate)

116. Two root causes of work conflict are task _______________ and ____________ goals.

(interdependence, incompatible)

117. The _________ conflict handling style is uncooperative and unassertive.(avoidance)

118. A person with a(n) _________ conflict handling style might say, “If it’s important to you, I can go along with it.”

(accommodation)

119. The conflict handling style in which an individual has some desire to express his own concerns and get his way but still respects the other person’s goals is the _______________ style.

(compromising)

120. The conflict handling style in which people are highly assertive but low on cooperation is _______________.

(competition)

121. When two people who are in conflict emphasize problem solving and integration of each other’s goals, they are utilizing a ____________ conflict handling style.

Page 248: Fwk Bauer Tif Organizational Behavior All Chapters

(collaboration)

122. The ____________ conflict handling style has the most potential to be highly effective in many different situations.

(collaboration)

123. The process where two parties work toward an agreement is ____________.(negotiation)

124. BATNA stands for _____ ___________ ___________ ___________ ____________.

(best alternative to negotiated agreement)

125. A key stage that is often ignored in the negotiating process is the ____________ phase where information is gathered.

(investigation)

126. ___________ is the third phase of the negotiation process where you assemble the gathered information in a way to support your position.

(Presentation)

127. The ______________ phase of the negotiation process is where each party discusses his goals and seeks to get an agreement.

(bargaining)

128. A _____________ is giving up something to get something in return.(concession)

129. The final phase of the negotiation process, ______________, is where the two parties either come to an agreement on terms or one party has found the final offer to be unacceptable and walks away from it.

(closure)

130. The traditional “fixed-pie” approach where negotiators see the situation as a pie that has to be divided between them is the ______________ negotiation strategy.

(distributive)

131. The ____________ negotiation strategy looks for ways for the two parties to integrate goals under a larger umbrella.

(integrative)

132. Mediation, arbitration and other ways of resolving conflicts with the help of a specially trained third party without need for a formal trial or hearing is called _____________ _________ _____________.

(Alternative Dispute Resolution)

133. ___________ is a process where an outside third party enters the situation with the goal of assisting the parties to reach an agreement.

(Mediation)

Page 249: Fwk Bauer Tif Organizational Behavior All Chapters

134. The conflict resolution process where a neutral third party listens to each side’s case and makes a binding decision is called ___________.

(arbitration)

135. Commonly, the conflict resolution process of _________ follows that of ___________ but research indicates that the process of _____________ followed by ____________ actually leads to more voluntary agreements between the parties.

(arbitration, mediation, arbitration, mediation)

136. If your counterpart feels you are being __________ in your negotiations, he is less likely to make any concessions.

(unethical or dishonest or unfair)

SHORT ANSWERS

137. Briefly describe the “social issues” that are some of the most difficult aspects of negotiating that keep firms such as PointCast from reaching agreement.

The chapter opens with a description of PointCast, one of the Silicon Valley’s hottest start-ups on the 1990s, in merger negotiations with Rupert Murdoch’s News Corporation. Though media speculation valued PointCast at $750 million, Murdoch offered a package of $450 million that PointCast rejected. Subsequent decisions to go public also failed and ultimately, PointCast was sold for $7 million. The missed opportunity is often cited as one of the biggest mistakes in Internet history and is largely credited to the social issues of ego and corporate pride which created conflict in the initial merger negotiations. Apparently PointCast CEO Christopher Hassett believed the hype about his firm and was unwilling to alter his perception during negotiations. Social issues can get in the way of fruitful negotiations and produce painful lessons.

138. Name the three types of conflict and briefly describe each.

Conflict is the process that involves people disagreeing. The three types of conflict include intrapersonal conflict (which arises in a person due to uncertainty about what is wanted or expected or when you feel inadequate to perform a task), interpersonal conflict (which is conflict between individuals sometimes due to personality or competition differences), and intergroup conflict (which occurs between groups like union and management or different departments within the same firm).

139. Describe the inverted-U relationship that exists between conflict and performance.

Conflict in an organization is not always bad. Research tends to suggest that both low and high levels of conflict lead to lower performance. Moderate conflict, on the other hand, leads to higher levels of performance. The moderate conflict is viewed as optimal because it creates a situation where there is healthy debate on ideas, stimulating, for example, creativity. In general, personal conflict is never healthy, but others, like task conflict can be beneficial to the firm.

Page 250: Fwk Bauer Tif Organizational Behavior All Chapters

140. Describe two root causes of conflict at work.

Six root causes of workplace conflict are: organizational structure, limited resources, task interdependence, incompatible goals, personality differences and communication problems.

An organizational structure like a matrix can present workplace conflict because it has each manager reporting to two bosses while a bureaucratic structure has different levels of people with different power creating high-power and low-power clashes.Limited resources like money create conflict around budget time when an increase in one department’s budget will necessitate a decrease in a second department’s budget.

Task interdependence creates conflict when the output of one individual is the input of another. If the first individual delays his output, he has negatively impacted another.

Incompatible goals create conflict when two parties believe their goals to be mutually exclusive and then end up being connected in some manner. For example, compensation systems often reward salespeople for the amount of sales they make regardless of the production department’s ability to fill those orders.

Personality differences are the most common basis of conflict in the organization. Type A and Type B personalities often do not mix when work needs to be completed on a deadline.

Communication problems also create conflict. Dealing with people who do not return phone calls can be very trying for many individuals.

141. What are two positive and two negative outcomes of conflict?

Positive conflict outcomes include: considering a broader range of ideas resulting in a stronger idea; bringing forward assumptions that may be inaccurate or invalid; increasing participation or creativity; and clarifying individual views.

Negative outcomes of conflict include: increased stress and anxiety which decreases productivity and satisfaction; the feeling of defeat or detachment that some people experience which lowers their morale; and the development of a climate of distrust which hinders teamwork and cooperation.

142. What are some of the factors that increase the risk for you to experience workplace violence on the job?

Two general categories of situations increase your risk of experiencing workplace violence: dealing with people and being in high-risk situations.

In dealing with people, circumstances like caring for others either emotionally or physically, interacting with frustrated customers, supervising other and denying

Page 251: Fwk Bauer Tif Organizational Behavior All Chapters

requests others make of you increases your chances of experiencing workplace violence.

Being in high-risk situations like dealing with valuables and money; handling weapons; working with drugs, alcohol or those under the influences of either of them; and working nights and weekends increases your chances of experiencing workplace violence.

143. Describe two ways in which organizational conflict can be managed.

Change the structure: When a structure in an organization is dysfunctional, as when a firm uses a matrix and there is confusion as to which of two bosses a manager should attend to first, then the structure may need to be altered.

Change the team composition: When team members clash repeatedly, it may be a good idea to change one or two team members to ensure greater harmony.

Create a common opposing force: When two work groups within a department are clashing over, for example, funding, a manager could divert their anger toward each other by focusing their attention on another department entirely which received substantial funding increases in the last budget cycle.

Consider majority rule: When group members are disagreeing over ideas, take a vote and the idea with the most support wins.

Problem solve: A common approach, the problem solving approach gets group members to focus on the task and not on the people involved so that the root cause of the problem is found.

144. List and briefly describe each of the five conflict handling styles.

Avoidance is where the individual is uncooperative and unassertive. The person practicing this conflict resolution style denies conflict is there.

Accommodation is where the individual is cooperative and unassertive. The person gives in to what the other side wants even if it means giving up personal goals.

Compromise is a middle ground style where individuals express their own concerns and want to get their own way, but respect the other person’s goals.

Competition is where the person wants to reach his goal regardless of what others say or feel.

Collaboration is the style high in both assertiveness and cooperation. This is the strategy that tends to be most effective in the most different conflict situations.

145. List the steps in the negotiation process.

Investigate, determine your best alternative to a negotiated agreement, present, bargain, and close.

Page 252: Fwk Bauer Tif Organizational Behavior All Chapters

146. Briefly describe the two negotiation strategies.

The distributive approach to negotiation is the fixed pie approach where if one department increases its budget, another department’s budget is cut.

The integrative approach to negotiation is a more creative approach where both parties look for ways to integrate their goals under a larger umbrella.

147. List and describe two common mistakes made in negotiation.

Failing to negotiate or taking the first offer: Research indicates that women are less likely to negotiate their starting salary than men and that this may be a contributing factor to the lower salaries women receive than their male counterparts.

Letting your ego get in the way: PointCast is a classic example of a firm that could have been part of a successful merger until its CEO let his ego do the talking and the deal was squelched.

Having unrealistic expectations: In labor contract negotiations, the unrealistic expectations often set forth in the initial contract proposal by the union are called “pie in the sky” demands.

Getting overly emotional: Anger never serves anyone when negotiating. Bargaining should involve the task or issue at hand and not the personalities involved.

Letting past negative outcomes affect the present ones: Research indicates that those who have been less successful in past negotiations tend to have lower outcomes than those who had successfully negotiated deals in the past.

148. Describe two alternative dispute resolution techniques.

Mediation is the process where a neutral third party enters a stalled negotiation to assist the parties involved in getting back on track. The mediator works with both parties to reach a solution without representing either side.

Arbitration is the process where a neutral third party listens to each party’s side in a dispute and then renders a decision. In most cases the decision is said to be binding, which means both parties must adhere to it.

ESSAY

149. Janice is a new manager for XYZ Corporation. She is about to attend two weeks of management training. The first topic she will examine is that of conflict and its management. You are the instructor for the conflict seminar. Describe the types of conflict Janice and other managers may face in the workplace, their underlying causes and some strategies for managing those conflicts in the workplace.

Page 253: Fwk Bauer Tif Organizational Behavior All Chapters

There are three types of conflict that should be noted at the outset of the seminar session. The three are: intrapersonal, interpersonal and intergroup conflict.

Intrapersonal conflict arises when an individual does not know what is expected or wanted of him in the workplace or where he feels inadequate to perform a task. Perceived lack of trust of the employee, role ambiguity and role conflict are causes of intrapersonal conflict. Clarifying the employee’s role and empowering him with various responsibilities are two viable options for managing this type of conflict in the workplace.

Interpersonal conflict is likely the most common form in the workplace. Personality differences are a key cause of such conflict. Problem solving approaches that focus attention on the task or issue at hand and not on the personalities are two options to consider in dealing with this type of conflict. If neither of those works, there is always the possibility that a manager should consider moving or removing one of the personalities from his department.

Intergroup conflict is often found between groups in a single department or between departments in a workplace. In unionized settings in the United States, the union and management tend to have longstanding adversarial relations. One effective way of dealing with intergroup conflict is to create a common “enemy” for the clashing parties. Diverting their attention from each other and refocusing it on an outside group can actually result in more cooperation among group members and ultimately, a more cohesive group.

150. Angelo is the new superintendent of the school district. In his first two weeks on the job he makes it a point to talk to every staff and faculty member as well as key stakeholders in the community and those who interact with the district on a regular basis. His assessment after those interviews is that the district is conflict-ridden. Seemingly everywhere he turns intrapersonal, interpersonal and intergroup conflicts are found. Discuss how he can get his arms around this situation and begin to manage some of the conflicts he is encountering.

Angelo needs to determine the root causes of the conflicts. Such root causes can cover a wide spectrum but some common ones are: organizational structure, limited resources, task interdependence, incompatible goals, personality differences and communication problems.

Each of the root causes identified could take substantial time and effort to address, but identification is always the first step. If organizational structure seems to be an issue, Angelo might consider changing it. He needs to understand how authority really flows in the district as well as how communication and decision making flows. Once that is assessed, the nature of changes needed may be obvious.

Limited resources are always an issue regardless of the nature of the organization, but Angelo may want to consider getting a clear handle on what each department’s budget is and how such budgets are determined yearly. He may want to institute some changes in identification of new spending items

Page 254: Fwk Bauer Tif Organizational Behavior All Chapters

separating out, for example, those items basic to the educational process and required by law and those discretionary that enhance the curriculum.

Task interdependence may be an issue in the staff areas. Again, a thorough assessment of what jobs’ outputs become the inputs for other jobs and the timing of each could lead to the redesign of some jobs for enhanced efficiency and effectiveness.

Incompatible goals and communication problems could be intertwined. Understanding how information is disseminated to employees in the district may indicate a very haphazard pattern. Some of the information partially or incompletely disseminated may be the district and individual goals. When that information is clarified and input appropriately received and processed, both problems may be addressed.

Personality differences are the common denominator in the conflicts of many organizations. Focusing employee attention on the task and not the personalities involved is always the first step but sometimes that proves to provide only temporary relief and the ultimate step of moving or removing individuals must be assessed.

151. Nejah and Omar are two supervisors in the production department of a manufacturing firm. The two have known each other since high school intensely dislike each other and their animosity repeatedly spills over into the workplace. Kareem is the production department manager. After yesterday’s five-minute shouting match between Nejah and Omar on the factory floor with the production crew looking on, Kareem is at wit’s end. Using the conflict handling style grid as a focal point, discuss what style Kareem might use to handle this situation.

Kareem might have used the avoidance style of conflict handling to have allowed this situation to escalate to this point. In short, he may have denied that the two individuals had a personality conflict or simply delayed handling it because it was so longstanding he felt it would resolve itself as it had in the past.

Kareem might want to consider collaboration or compromise as his conflict handling style with Omar and Nejah but it will take time and patience to use either because of how much ill-will has built up between the two. In compromise, Kareem will need to find a middle ground for the two where he accounts for and assists the pair in recognizing the desires of each and how those wants can be expressed and then both rescinded a bit. Kareem must be cautious in making sure each party gives in the same amount as the other and that each party accepts and perceives the fallback in position to be equivalent. An alternative approach to this conflict scenario might be collaboration. In this scenario both parties will argue for their respective positions, personalities taken out of the mix, supporting each position with facts and rationale. Then, with Kareem’s oversight, both sides will attempt to secure a win-win situation. Again the time and effort Kareem might invest in trying such a conflict resolution needs to be considered.

The competition style of conflict handling would not be effective in this scenario since the two parties involved are already competitive. Adding Kareem’s competitive style would only make it seem like he is playing favorites further

Page 255: Fwk Bauer Tif Organizational Behavior All Chapters

escalating the situation. The accommodation style may also prove to be less effective in this scenario since it involves one side giving up a personal goal to facilitate the other. Since the feud between these two is so long-standing, this style might take so much time to initiate and carry through to fruition that it is a pointless exercise.

152. The Challenger incident is widely recognized as a situation of groupthink at NASA. While some testing had indicated potential concerns with cold weather and the sustainability and performance of “O-rings” in the shuttle design, the desire of NASA executives to meet a timetable for budgetary and public relations reasons caused that information to not be properly presented in the debate over launch timing. What are some ways that NASA executives could stimulate conflict in such meetings to avoid future tragedies?

Conflict can be stimulated by encouraging people to raise issues and disagree with the status quo without fear of any retribution or reprisal. The freedom to dissent without retribution must be a part of the organization culture. Written procedures ensuring such positive conflict outcomes are a good beginning but a conscious management of vital discussions by a manager in which such dissent is featured is also critical.

Assign an employee a devil’s advocate role in all meetings. If you know that someone will always question the issue or the solution to it, you are more accepting and prepared for such dissent and it brings out the key inaccuracies in assumptions much faster.

Create competition among teams or individuals with “prizes” or bonuses for those who come up with the best solution to a problem or issue. Rewards that are understood, attainable and appreciated are motivating.

Build ambiguity into the process. When clarity is not found, questions are asked and those questions can stimulate greater creativity or pinpoint problematic areas.

153. Mario is preparing to negotiate a marketing agreement for his product with an advertising firm. What are the key steps he should prepare for in that negotiation?

At the outset, Mario needs to do some investigation and gather information. The better informed you are, the better bargaining position you place yourself in. In Mario’s case, he should begin by looking at his own goals. What does he want to get out of the negotiation? What can he achieve? What can he concede?

His second step will be to develop his best alternative to a negotiated agreement or BATNA. The reason he is negotiating is to produce results that are better than what he could have obtained without negotiating, so what are those results? Thinking through his BATNA will assist him in ultimately deciding to go with the advertising firm or walk away for another option.

Mario must prepare for his presentation where he will bring together the information he has gathered in a manner that will support his position.

Page 256: Fwk Bauer Tif Organizational Behavior All Chapters

The actual bargaining will be when the two parties meet to discuss their goals and attempt to reach an agreement. Mario must keep in mind that a regular part of such bargaining is making some concessions. His BATNA will guide him in assessing such concessions. Further, Mario must understand that he can ask questions during this phase to ensure he is getting the best deal for himself and to understand, possibly, why a deal is not made.

Finally, the bargaining needs to come to a close either with an agreement or with one or the other party walking away.

154. Clarissa is preparing for a meeting with a manager at a firm that has just offered her a job. Clarissa was approached by this firm while she was working for one of its competitors and the new firm has convinced her that she has greater opportunities there so she has tentatively accepted its offer contingent on salary negotiations. What advice would you offer to Clarissa to assist her in obtaining a higher salary from those negotiations?

The first piece of advice to be offered to Clarissa is to negotiate! Research indicates that women are less likely to engage in negotiations over salary and less likely attain higher levels of salary than their male counterparts. To get to a higher level, Clarissa must negotiate.

In order to negotiate, Clarissa must overcome any fears she might have of the process. Especially since she wants to go to this firm, she may be afraid of angering her manager before she even gets going in the new firm. That said, it doesn’t hurt to ask, particularly if you are prepared to defend your request.

Get the facts. Make sure you have checked with various websites to understand what the market rate is. Clarissa does not want to over or undersell herself and her skill set in the market.

Build your case. Clarissa must be very clear as to why she believes she is worth a specific dollar amount. She must clearly indicate what she will contribute to this firm and why she believes such a contribution is equate with a specific dollar figure.

Know what you want: Clarissa must be clear on her salary goal and how set she is in that goal. If the new firm is unwilling to meet her salary demands, will she walk away?

Don’t make the first offer. Clarissa should allow her new manager to initially name the starting figure. Negotiations generally work with one party starting high and the other countering.

Listen more than talk. Clarissa may learn a lot by listening so temper your comments with periods when you listen to the new manager for best results.

155. Antonio is preparing to negotiate his individual contract with a financial services firm. He is very nervous and does not want to shortchange himself. Provide him some advice for being successful at negotiations.

Page 257: Fwk Bauer Tif Organizational Behavior All Chapters

Focus on securing an agreement. If the goal is an agreement, when an issue arises that might lead to an impasse, simply set it aside as something you agree to disagree on and address it later. Dealing with contentious issues at a later time or in a different environment often leads to their resolution.

Be patient. If there is no deadline in place to reach an agreement by, be flexible.

Recognize whose reality you are dealing with. Each side perceives their version of reality. Be clear on yours.

Deadlines: If a deadline is established, negotiators tend to move toward agreement as that deadline approaches.

Be comfortable with silence: After an offer or counteroffer is made, allow the other party to contemplate it. Do not expect an immediate response.

156. Alemeda has just been hired by an aluminum maker to negotiate supply agreements between the aluminum firm and various international companies. What are some cultural issues that Alemeda should keep in mind as she begins negotiations?

Alemeda should recognize that some cultures have a lower threshold for conflict than others. Countries like Japan and Korea, for example, prefer harmony to conflict. Americans and Germans, in contrast, have a higher tolerance for conflict as a way to work through issues.

The nature of the relationship between negotiating parties also differs based upon the culture. Americans are prone to “getting right down to business” while negotiators in Japan and China must establish a friendly, trust-based relationship before bargaining. Consequently, Japanese negotiators, because of their high context culture are likely to look for patterns in what is not said as well as what is said during negotiations and factor those patterns into their offers.

Similarly, the relationship between the negotiators impacts the view of the overall activity. Americans view the negotiation as a business activity while Brazilians see it as a social activity.

In all cases, Alemeda must assess the cultural implications of negotiations before initiating the activity.

Page 258: Fwk Bauer Tif Organizational Behavior All Chapters

Organizational Behavior, Version 1.1Bauer & Erdogan

FWK Test Item FileChapter 11

TRUE/FALSE

1. All decisions have major consequences and require much thought.(False)

2. Decision making requires action as a solution.(False)

3. Programmed decisions are unique, creative decisions.(False)

4. A decision rule is an automated response to a problem or issue that occurs frequently.

(True)

5. An example of a nonprogrammed decision is deciding whether to merge with another firm.

(True)

6. Strategic decisions are usually made by middle level managers.(False)

7. Tactical decisions are those concerned with how things get done.(True)

8. The rational decision-making model limits the number of alternatives considered.(False)

9. The first step of both the rational decision-making model and the creative decision-making process is to identify the problem.

(True)

10. The most difficult step of the rational decision-making process is to establish the decision criteria.

(False)

11. Analysis paralysis is when more and more time is spent on gathering information and thinking about it, but no decisions are made.

(True)

12. People are always interested in making an optimal decision.(False)

Page 259: Fwk Bauer Tif Organizational Behavior All Chapters

13. To satisfice is to accept the first alternative that meets minimum criteria.(True)

14. In the intuitive decision-making process, only one choice is considered at a time.(True)

15. Innovation and creativity are the same thing.(False)

16. Immersion is to conscious thought as incubation is to unconscious thought.(True)

17. The three factors that evaluate the level of creativity in the decision-making process are fluency, flexibility, and originality.

(True)

18. Creativity is the interaction between personality traits, attributes, and serendipity.(False)

19. While setting high idea quotas appears to logically maximize the effectiveness of brainstorming, in reality it has just the opposite effect.

(False)

20. Wildstorming is a process where the group focuses on ideas that are impossible and then tries to imagine what would need to happen to make them possible.

(True)

21. The intuitive decision-making model is best used when the decision maker has experience with the problem and there is time pressure.

(True)

22. The rational decision-making process is best used when the decision is important and you are trying to maximize outcomes.

(True)

23. Hindsight bias is the opposite of framing bias.(False)

24. Given research on anchoring bias, individuals are more likely to focus on “60% of all people taking the test pass”, rather than “40% of all people taking the test fail.”

(False)

25. Escalation of commitment is also known as “sunken cost fallacy.”(True)

26. One way to avoid escalation of commitment is to have identifiable turning back points.

(True)

27. Group decisions tend to be more creative than individual ones, but they are often not more effective than those made by individuals.

Page 260: Fwk Bauer Tif Organizational Behavior All Chapters

(False)

28. Groupthink is one of the reasons cited for the tragedy of the Challenger space shuttle.

(True)

29. Group decisions regularly outperform the decision of the group’s best member.(False)

30. Individual decision making produces a greater commitment to the ultimate decision than does group decision making.

(False)

31. Groupthink is characterized by symptoms like questioning the morality of the group.

(False)

32. Having a devil’s advocate in meetings is a technique that can help avoid groupthink.

(True)

33. The nominal group technique involves using written responses to a series of questionnaires instead of physically bringing individuals together to make a decision.

(False)

34. Research shows that consensus decision making is less accurate and can even make group members feel less satisfied with a decision.

(False)

35. Group decision support systems could make employees more reluctant to share information due to lack of control.

(True)

36. Decision trees are helpful in avoiding errors such as overconfidence bias.(False)

37. Unethical decisions in bank lending is likely one of the causes of the current economic crisis in the United States.

(True)

38. One basic question to ask to ascertain the ethics of a decision is: “How would I feel if this decision was broadcast on the news?”

(True)

39. American managers tend to value quick decision making while Chinese managers favor more reflective decision making.

(True)

40. Ingar Skaug, CEO of Wilhelmsen Lines, empowered his employees to make their own decisions when he assumed his position at the global maritime company.

Page 261: Fwk Bauer Tif Organizational Behavior All Chapters

(True)

MULTIPLE CHOICE

Opening Section: Decision Making Culture: The Case of Google

41. Google is known for having _________.a. conformity.b. encourages risk taking and innovation.c. emotional decision making.d. making decisions by consensus.

(b) Easy/Comprehension

Section I: Understanding Decision Making

42. Making choices among alternative courses of action, including inaction, isa. decision making.b. programmed decisions.c. satisficing.d. consensus

(a) Easy/Knowledge

43. Programmed decisions area. unique, nonroutine, and important, requiring conscious thinking,

information gathering and careful consideration of alternatives.b. decisions that are made to set the course of an organization.c. a set of parameters against which all of the potential options in decision

making will be evaluated.d. ones which occur frequently enough that an automated response is

developed for them.(d) Easy/Knowledge

44. Mark works as the frozen food manager in a major grocery store chain. When his stock of two-pound bags of frozen shrimp gets down to two cases, he e-mails his warehouse to send ten cases to restock.

The type of decision Mark is making about restocking shrimp is aa. strategic decision.b. nonprogrammed decision.c. programmed decision.d. tactical decision.

(c) Medium/Application

45. The “automated” ordering of ten cases when supply gets to two cases in the store is called a(n) a. consensus decision.b. decision rule.c. alternative.d. anchor.

(b) Medium/Application

Page 262: Fwk Bauer Tif Organizational Behavior All Chapters

46. A unique, nonroutine, important decision requiring conscious thinking, information gathering, and careful consideration of alternatives is a(n)a. programmed decision.b. operational decision.c. nonprogrammed decision.d. decision rule.

(c) Easy/Knowledge

47. In 2003, six people died from Hepatitis A and 660 were sickened after eating at Chi Chi’s, a popular Mexican restaurant in suburban Pittsburgh, Pennsylvania. In the days following the first death, the CEO of the Chi Chi’s chain made a decision to have all restaurant workers and the restaurant itself tested to determine what established the dangerous conditions. (Eventually the hepatitis outbreak was traced to the green onions in the firm’s salsa.) The decision to order such testing in the crisis situation is an example ofa. a programmed decision.b. a nonprogrammed decision.c. a decision rule.d. a tactical decision.

(b) Medium/Application

48. Strategic decisions a. refer to those that make the organization run on a daily basis.b. refer to how things get done.c. refer to those setting the course of the organization.d. refer to those that occur frequently enough to have an automated

response to them.(c) Easy/Knowledge

49. Who is most responsible for making strategic decisions?a. managersb. engineersc. low-level employeesd. CEOs.

(d) Medium/Knowledge

50. An example of a strategic decision isa. How often should I communicate with my new coworkers?b. How should we market the new product line?c. Should we downsize our organization?d. What should I say to customers about our new product?

(c) Medium/Comprehension

51. A decision which centers on how things get done is a(n) a. tactical decision.b. strategic decision.c. operational decision.d. programmed decision.

(a) Easy/Comprehension

Page 263: Fwk Bauer Tif Organizational Behavior All Chapters

52. Tactical decisions are generally made bya. CEOs.b. engineers.c. managers.d. Boards of Directors.

(c) Medium/Knowledge

53. Which of the following questions is an example of a tactical decision?a. Should we takeover our competitor?b. How should we market the new product line?c. What should I say to the customers about our return policy?d. How will I balance my master’s degree workload with my work

assignments?(b) Medium/Comprehension

54. Operational decisions a. refer to those things that employees do each day to make the

organization run.b. refer to things that might happen in the future.c. refer to those that set the course of the organization.d. refer to those that occur frequently enough to develop an automated

response to them.(a) Medium/Knowledge

55. Operational decisions are made bya. managers.b. top management teams.c. employees throughout the organization.d. CEOs.

(c) Medium/Comprehension

56. An example of an operational decision isa. Should we consider a merger with our biggest competitor?b. How often do I go back to the stockroom to get additional products for

display?c. Should we develop a new corporate structure?d. Which advertising firm should we choose for our fall advertising

campaign?(b) Medium/Analysis

57. Antonio, Alpha Company’s marketing manager, will be listening to the advertising pitches from the final four firms today as he chooses the television ad campaign for his product for the fall season. Antonio is making a(n)a. programmed decision.b. strategic decision.c. tactical decision.d. operational decision.

(c) Difficult/Application

58. Doris and Lydia are members of the Board of Directors at Beta Corporation. They are giving very serious consideration to voting to merge their company with

Page 264: Fwk Bauer Tif Organizational Behavior All Chapters

Zeta Company for enhanced efficiency, effectiveness and competitive advantage. Doris and Lydia are making a(n) a. strategic decision.b. tactical decision.c. operational decision.d. programmed decision.

(a) Medium/Application

59. Claudia is a salesperson with a major department store chain that is currently running a “secret coupon sale.” The program allows the salesperson to randomly award a savings coupon to any shopper of the salesperson’s choice. Who the salesperson chooses to award the coupon to is a(n)a. strategic decision.b. tactical decision.c. operational decision.d. programmed decision.

(c) Medium/Application

60. A series of steps that decision makers should consider if their goal is to maximize their outcome and make the best choice describes a. the bounded rationality model.b. the rational decision-making model.c. the intuitive decision-making model.d. the creative decision-making model.

(b) Easy/Comprehension

61. If the goal of your decision making is to make the optimum decision or make the best choice, you should utilizea. the bounded rationality model of decision making.b. the intuitive decision-making model.c. the creative decision-making model.d. the rational decision making model.

(d) Medium/Comprehension

62. When the goal of the decision making exercise is to make a satisfactory decision because you are limited in some way such as time, you should utilizea. the rational decision-making model.b. the intuitive decision-making model.c. the bounded rationality decision-making model.d. the creative decision-making model.

(c) Medium/Comprehension

63. The bounded rationality decision-making modela. describes a series of steps that decision makers should consider if their

goal is to maximize their outcome and make the best choice.b. recognizes the limitations of decision making processes by having

individuals knowingly limit their options to a manageable set and choose the best alternative without conducting an exhaustive search of alternatives.

Page 265: Fwk Bauer Tif Organizational Behavior All Chapters

c. refers to arriving at decisions without conscious reasoning, arguing that experts make decisions by scanning the environment for cues to recognize patterns.

d. refers to arriving at decisions after first gathering information about the problem and then setting the problem consciously aside until an insightful solution to the problem arises.

(b) Medium/Knowledge

64. The creative decision-making processa. describes a series of steps that decision makers should consider if their

goal is to maximize their outcome and make the best choice.b. recognizes the limitations of decision making processes by having

individuals knowingly limit their options to a manageable set and choose the best alternative without conducting an exhaustive search of alternatives.

c. refers to arriving at decisions without conscious reasoning, arguing that experts make decisions by scanning the environment for cues to recognize patterns.

d. refers to arriving at decisions after first gathering information about the problem and then setting the problem consciously aside until an insightful solution to the problem arises.

(d) Medium/Knowledge

65. The decision-making process where decisions are arrived at without conscious reasoning, arguing that experts make decisions by scanning the environment for cues to recognize patterns isa. the creative decision-making model.b. the bounded rationality model.c. the intuitive decision-making model.d. the rational decision-making model.

(c) Medium/Comprehension

66. Jennifer has to decide which of two job offers she is going to choose. She begins her process by listing the key criteria she is looking for in a job including salary level, location, promotional opportunities, and so on. She then takes each job offer letter and carefully goes through each line assessing the offer in relationship to the criteria she has established. Jennifer is using which of decision-making model to choose her job?a. the rational decision-making modelb. the creative decision-making modelc. the intuitive decision-making modeld. the programmed decision-making model

(a) Medium/Application

67. Which is the first step in the rational decision-making model?a. Establish decision criteria.b. Identify the problem.c. Weigh decision criteria.d. Generate alternatives.

(b) Easy/Comprehension

Page 266: Fwk Bauer Tif Organizational Behavior All Chapters

68. What is the most challenging or difficult step in the rational decision-making process?a. Identify the problem.b. Establish decision criteria.c. Generate alternatives.d. Evaluate alternatives.

(c) Medium/Comprehension

69. Which of the following statements regarding the rational decision-making model is INCORRECT?a. The decision maker should generate alternatives before establishing

criteria.b. The decision maker should make certain to clearly identify the problem

before undertaking any other step.c. Successful managers tend to be clear on what they want at the outset of

the decision making process.d. One research study indicated that no alternative generation occurred in

85% of the decisions examined.(a) Medium/Evaluation

70. Which of the following is NOT an unrealistic assumption made in the rational decision making model?a. People completely understand the decision to be made.b. People know all their available choices.c. People want to make optimal decisions.d. People have perceptual biases.

(d) Medium/Analysis

71. The availability of too much information leading to more and more time being spent on gathering information and thinking about it, resulting in no decisions being made isa. satisficing.b. analysis paralysis.c. wildstorming.d. anchoring.

(b) Easy/Knowledge

72. To satisfice is to a. generate new ideas that are original, fluent and flexible.b. set parameters against which all of the potential options can be

evaluated.c. accept the first alternative that meets your general criteria.d. be influenced by the way in which problems are framed.

(c) Easy/Knowledge

73. Anita finishes her college semester on April 15. She will be home from April 15 until May 20 when she is scheduled to have her wisdom teeth removed. She will be going on a family vacation during the third week in July and will be returning to school on August 10 for majorette camp. Anita is looking for a job for the summer. She figures she will take the first job that pays minimum wage and will

Page 267: Fwk Bauer Tif Organizational Behavior All Chapters

allow her flexibility for her dental appointment and vacation. Anita is making a decision using the a. rational decision-making model.b. bounded rationality decision-making model.c. intuitive decision-making model.d. creative decision-making model.

(b) Medium/Application

74. Susan is a paramedic for the county ambulance service. Yesterday there was a terrible accident on the interstate when a bus carrying senior citizens to an Atlantic City casino was struck by a small dump truck as the truck tried to change lanes. Fifteen of the bus riders were killed and 25 injured. As Susan arrived on the scene to perform triage on the accident victims, she quickly made decisions as to which victims needed immediate care and who could wait. Susan was using what decision-making model in making these stressful decisions?a. bounded rationality model of decision makingb. creative decision-making modelc. intuitive decision-making modeld. rational decision-making model

(c) Difficult/Evaluation

75. Experts make decisions in the intuitive decision-making models based on all the following buta. climate.b. training.c. experience.d. knowledge.

(a) Medium/Comprehension

76. The incubation step in the creative decision-making process isa. the step where the decision maker consciously thinks about the problem.b. the step where the decision maker sets the problem aside and does not

consciously think of it.c. the step where the decision maker gains insight into the problem and has

a “eureka” moment.d. the step where the decision maker verifies the feasibility of the solution

and implements the decision.(b) Medium/Knowledge

77. Which of the following statements about the creative decision-making model is INCORRECT?a. Creativity is the generation of new, imaginative ideas.b. The dynamic nature of today’s organizations, including structural changes

and cost cutting, have driven creativity in the business.c. Problem identification is the first step in the creative decision making

model.d. Innovation and creativity are the same process.

(d) Medium/Analysis

78. Which of the following is the correct order of the phases in the creative decision-making model?

Page 268: Fwk Bauer Tif Organizational Behavior All Chapters

a. Problem identification, incubation, immersion, illumination, verification and application.

b. Problem identification, immersion, incubation, illumination, verification and application.

c. Problem identification, illumination, immersion, incubation, verification and application.

d. Problem identification, illumination, incubation, immersion, verification and application.

(b) Medium/Comprehension

79. All of the following are factors that researchers focus on to evaluate the level of creativity in the decision making process EXCEPTa. fluency.b. flexibility.c. rationality.d. originality.

(c) Easy/Knowledge

80. The creativity level evaluation factor of fluency is a. how different the ideas are from one another.b. the number of ideas a person is able to generate.c. how unique a person’s ideas are.d. the set number of ideas a group must reach.

(b) Easy/Knowledge

81. Experts propose that creativity occurs as a result of the interaction between all of the following factors EXCEPT a. situational context (like physical structure).b. personality traits (like risk-taking).c. serendipity or luck.d. attributes (like expertise).

(c) Medium/Comprehension

82. To enhance organizational creativity, a manager should consider focusing on all the following areas EXCEPTa. team composition.b. employee pay.c. culture.d. leadership.

(b) Medium/Comprehension

83. Which of the following is NOT a way to enhance organizational creativity?a. Promote brainstorming as a way to generate ideas.b. Make teams more homogeneous so as to reduce the possibility of conflictc. Build a physical space conducive to creativity.d. Role model creative behavior.

(b) Medium/Analysis

84. Which of the following would NOT be good piece of advice to offer a company that is trying to enhance organizational creativity?

Page 269: Fwk Bauer Tif Organizational Behavior All Chapters

a. Ensure team stability by keeping team membership intact for extended periods of time.b. Use the nominal group technique to avoid the pitfalls of the common

group process.c. Diversify your team.d. Incorporate creative behavior into the performance appraisal process.

(a) Medium/Analysis

85. Which decision-making model would you use when your goals are unclear, there is time pressure, and you have experience with the problem?a. the rational decision making modelb. bounded rational modelc. intuitive decision making modeld. creative decision making model

(c) Medium/Evaluation

86. The bounded rationality model should be used to make decisions whena. the minimum criteria are clear.b. the decision is important.c. there is time pressure.d. new solutions need to be generated.

(a) Medium/Evaluation

87. If you do not have relevant expertise in the issues to make a decision, you should NOT use which decision making model?a. bounded rationality decision-makingb. rational decision-making modelc. creative decision-makingd. intuitive decision-making

(d) Medium/Evaluation

Section II: Faulty Decision Making

88. Individuals are influenced in their decision making by all of the following EXCEPTa. escalation of commitment.b. foresight bias.c. anchoring.d. framing bias.

(b) Easy/Knowledge

89. Overconfidence bias a. is the tendency of decision makers to be influenced by the way that

problems are framed.b. occurs when looking backward in time where mistakes seem obvious

after they have already occurred.c. refers to the tendency for individuals to rely too heavily on a single piece

of information.d. occurs when individuals overestimate their ability to predict future events.

(d) Easy/Knowledge

Page 270: Fwk Bauer Tif Organizational Behavior All Chapters

90. Sara’s laptop started causing her problems. It was taking a long time to boot up, and froze unexpectedly a number of times. Now the laptop has shut down completely and she cannot get it started up again. One of the group members on her project said to Sara, “You should have gone for help when it started acting up, now you’ve affected all of us.” This scenario is an example of what type of decision-making trap?a. framing biasb. anchoringc. overconfidence biasd. hindsight bias

(d) Difficult/Application

91. A movie called the “Money Pit” starring Tom Hanks and Shelley Long focused on the couple buying a house and continually having to spend money repairing one part of it after another. A number of times they should have sold the house, but they kept thinking that if they did just one more thing, the house would be great. This is situation is an example of what decision-making trap?a. anchoringb. escalation of commitmentc. framing biasd. overconfidence bias

(b) Medium/Application

92. The tendency for decision makers to be influenced by the way that a situation or problem is presented isa. escalation of commitment.b. anchoring.c. overconfidence bias.d. framing bias.

(d) Easy/Knowledge

93. Escalation of commitment occurs becausea. decision makers do not want to admit they were wrong.b. strict “turn back” points have been established.c. persistence pays off.d. decision makers lack personal pride.

(a) Medium/Evaluation

Section III: Decision Making in Groups

94. The advantages of group decision making over individual decision making include all of the following EXCEPTa. the decision is more creative.b. the decision-making process is more enjoyable.c. the decision-making process is more efficient.d. the implementation of the decision is easier.

(c) Medium/Analysis

95. The tendency to avoid a critical evaluation of ideas that the group favors is calleda. anchoring.b. groupthink.

Page 271: Fwk Bauer Tif Organizational Behavior All Chapters

c. analysis paralysis.d. wildstorming.

(b) Easy/Knowledge

96. Which of the following is true for group decision making?a. Group decision making is faster than individual decision making.b. Groups often perform lower than the best individual in the group.c. Groups make it easier to achieve accountability for decisions.d. Groups generate fewer ideas than individuals.

(b) Medium/Evaluation

97. Which of the following is NOT a symptom of groupthink?a. collective rationalizationsb. illusion of vulnerabilityc. direct pressured. illusions of unanimity

(b) Difficult/Analysis

98. The decision-making technique designed to help with group decision making by ensuring that all members participate fully isa. majority rule.b. consensus.c. the Delphi technique.d. the nominal group technique.

(d) Easy/Knowledge

99. Which of the following statements regarding techniques for making better decisions is INCORRECT?a. The nominal group technique is a technique used routinely at most

meetings.b. Consensus requires more time to carry out, but it works well when

support is needed for a plan.c. The Delphi technique is a group process using written responses to a

series of questionnaires so individuals are not physically brought together to make a decision.

d. Majority rule is simple, speedy and easy to use.(a) Difficult/Synthesis

100. Which of the following statements regarding group decision support systems (GDSS) is INCORRECT?a. GDSS could make employees reluctant to share information.b. GDSS could become too complex.c. GDSS improves the output of group collaborative work through higher

information sharing.d. GDSS avoids all possibilities of information overload.

(d) Difficult/Analysis

Section IV: The Role of Ethics and Culture in Decision Making

101. Which of the following is a guideline in determining whether a decision is ethical?

Page 272: Fwk Bauer Tif Organizational Behavior All Chapters

a. Will I feel better or worse about myself after I make this decision?b. Does the decision break any organizational rules?c. Is the decision fair?d. All of the above

(d) Medium/Evaluation

102. Which of the following statements regarding decision-making styles in other cultures is correct?a. Chinese managers value quicker decision making more than their

American counterparts.b. Though they use consensus group decision making, the Japanese make

much faster decisions than Dutch decision makers.c. Dutch managers tend to complete consensus decision making much

more than their Japanese counterparts.d. Japanese managers using consensus decision making implement those

decisions much faster than other cultures.(d) Medium/Evaluation

Closing Section: Empowered Decision Making: The Case of Ingar Skaug

103. Ingar Skaug, CEO of Wilhelmsen Lines, changed the decision-making style of his firm to one thata. gave him all the decision-making authority in his firm.b. empowered employees to make their own decisions.c. led his employees to think that he wanted their input but made the

decisions himself.d. made decisions by consensus.

(b) Medium/Analysis

FILL IN THE BLANK

104. _________ _________ is making choices among alternative courses of action including inaction.

(Decision making)

105. __________ decisions occur frequently and have automated responses developed for them, while ___________ decisions are unique and require conscious thinking, information gathering and careful alternative generation.

(Programmed, nonprogrammed)

106. Automated responses that we use to make decisions are called _______ _________.

(decision rules)

107. ___________ decisions set the course for an organization and are made by CEOs, while ______________ decisions are those that make the organization run and are made daily by employees.

(Strategic, operational)

Page 273: Fwk Bauer Tif Organizational Behavior All Chapters

108. “How should we market the new product line?” is an example of a ____________ decision made by managers.

(tactical)

109. When a decision is important and outcomes need to be maximized, use the __________ ___________ __________ __________ to make your decision.

(rational decision-making model)

110. The __________ _____________ __________ recognizes the limitations of the decision-making process and thus the tendency of individuals to satisfice in their decisions.

(bounded rationality model)

111. When time pressures arise, as when a life or death situation occurs, and an individual has expertise in the area, the _________ ________ ____________ __________ is often used to make decisions.

(intuitive decision-making model)

112. The generation of new, imaginative ideas is ___________.(creativity)

113. The phase of the creative decision-making process where the individual sets the problem aside and does not consciously think about it for a while is the ________________ phase.

(incubation)

114. The insightful or “eureka” moment in the creative decision-making process is the ___________ phase.

(illumination)

115. In assessing the level of creativity in the decision-making process, ______ is the number of ideas a person is able to generate.

(fluency)

116. _________________ is how different ideas generate in the creative decision-making process are from one another.

(Flexibility)

117. The uniqueness of ideas generated during the creative decision-making process is _______________.

(originality)

118. Some experts suggest that creativity is the interaction among the three factors of _______________, ________________, and ________________.

(personality traits, attributes, situational context)

119. Ideas for enhancing organizational creativity focus on team composition, team process, ________ and ____________.

(leadership, culture)

Page 274: Fwk Bauer Tif Organizational Behavior All Chapters

120. The group process of generating ideas that follow a set of guidelines including no criticism of ideas during the process, the notion that no idea is too crazy, and that builds on other ideas is _______________.

(brainstorming)

121. Since research suggests that the quantity of ideas leads to better quality ideas in the end, setting ________ ___________ ________ where the group must reach a set number of ideas before they are done brainstorming is a recommended practice.

(high idea quotas)

122. _______________ is a variation on brainstorming where the group focuses on ideas that are impossible and then imagines what would need to happen to make them possible.

(Wildstorming)

123. A decision-making trap where people overestimate their ability to predict future events is called _______________.__________.

(overconfidence bias)

124. When you say, “I know I should have taken this car in for service when I first heard the noises, and now it’s just quit on me,” you are exhibiting the decision-making trap, _________ ________.

(hindsight bias)

125. Maria relied on knowing what an outstanding worker Gordon was to hire Gordon’s sister for the entry-level position. Gordon’s sister is very unreliable. Maria fell into the decision-making trap of _______________ in making her hiring decision.

(anchoring)

126. The tendency for people to focus on television ads that say, “Medicine X is 60% effective in reducing symptoms” does not look at the fact that that statement means that “Medicine X fails to reduce those symptoms 40% of the time.” This tendency is called ____________ ____________.

(framing bias)

127. The old adage, “Don’t throw good money after bad” is another way to describe the decision-making trap of ____________ _______ ____________ also known as the “sunken costs fallacy.”

(escalation of commitment)

128. Research suggests that _____________ decision making offers fewer ideas than ________ decision making.

(individual, group)

129. ___________ is the group pressure phenomenon that increases the risk of the group making flawed decisions by leading to a reduction in mental efficiency, reality testing, and moral judgment.

(Groupthink)

Page 275: Fwk Bauer Tif Organizational Behavior All Chapters

130. One of the symptoms of ________ is the illusion of invulnerability that is shared by all group members and creates excessive optimism and encourages them to take extreme risks.

(groupthink)

131. The ________ _________ ____________ is a tool designed to help group decision making by ensuring that all members participate fully.

(nominal group technique)

132. The __________ ________ is a group process using written responses to a series of questionnaires instead of physically bringing individuals together to make a decision.

(Delphi technique)

133. When the goal is to gain support for a particular idea or plan of action, the _______ form of decision should be used as it is inclusive, participatory, cooperative and democratic in nature.

(consensus)

134. An interactive computer-based system that combines communication and decision technologies to help groups make better decisions is a _________ _____________ __________ _____________.

(group decision support system)

135. A ___________ ____________ is a diagram where answers to yes or no questions lead decision makers to address additional questions until they reach the end of the tree.

(decision tree)

136. In Japan, nemawashi refers to building _______________ within a group before a decision is made.

(consensus)

SHORT ANSWERS

137. What is the difference between a programmed and an unprogrammed decision? Given an example of each.

A programmed decision is a decision that occurs frequently enough that an automated response is developed for it. An unprogrammed decision is one that is unique and requires conscious thinking, information gathering and careful consideration of alternatives.

An example of a programmed decision is when a restaurant automatically re-orders napkins and placemats for use in the restaurant when the stock of those items gets to, for example, two cases in the stockroom. A nonprogrammed decision might be when a firm faces a crisis situation as when Procter and Gamble was faced with allegations that its corporate logo was supportive of the devil and had to address such charges.

Page 276: Fwk Bauer Tif Organizational Behavior All Chapters

138. Give an example of a strategic decision and indicate who usually makes it.

A strategic decision sets the course for the organization. An example of a strategic decision is to decide whether you should acquire another company or not. Strategic decisions are made by CEOs, boards of directors, or other top level teams.

139. Give an example of a tactical decision and indicate who makes it.

A tactical decision is focused on how things get done. An example is how to market a new product. Tactical decisions are made by managers.

140. Give an example of an operational decision and indicate who makes it.

An operational decision is made by employees throughout the organization and refers to decisions that employees make each day to make the organization run. An example is when an employee is expected to visit the stockroom in a retail establishment.

141. Compare the rational decision-making and the bounded rationality model.

The rational decision-making model is a series of steps that decision makers consider if their goal is to maximize the quality of their outcomes. The optimal outcome desired creates a need to thoroughly examine alternatives. The bounded rationality model recognizes the limitations of searching for alternatives and suggests that decision makers often satisfice, accepting the first alternative that meets minimum criteria because an exhaustive search is not possible.

142. Differentiate between the intuitive and creative decision-making processes.

The intuitive decision-making process is one where experts scan the environment for cues and once a pattern is discerned, choose a course of action based upon their experience. The creative decision-making process is one where problem identification is followed by immersion, incubation and then illumination and finally verified and applied. The creative process is characterized by the incubation period where the problem is set aside for a time period and illumination is the “eureka” moment when the decision is made.

143. What are the three factors that are used to evaluate the level of creativity in the decision-making process?

The three factors are: fluency or the number of ideas a person is able to generate; flexibility or how different the ideas are from one another; and originality or how unique a person’s ideas are.

144. Describe two decision-making traps and how to avoid them.

There are five decision-making traps:

Overconfidence bias is when individuals overestimate their ability to predict future events. To avoid this trap, stop and be realistic in your judgments.

Page 277: Fwk Bauer Tif Organizational Behavior All Chapters

Hindsight bias is the opposite of overconfidence bias as it looks backward in time where mistakes seem obvious after they have occurred. It is important that decision makers remember this bias when judging other people’s actions.

Anchoring is the tendency for individuals to rely too heavily on a single piece of information. To avoid this bias, the individual needs to gather sufficient data for assessment.

Framing bias is the tendency for decision makers to be influenced by the way a situation or problem is presented. Avoidance of this bias occurs through simple awareness of the problem and a close monitoring of actions to discern when it occurs.

Escalation of commitment is when individuals continue on a failing course of action after information reveals it may be a poor path to follow. The approach to dealing with this is to assess the scenario and make a clear decision recognizing that sometimes the best choice is to admit you failed.

145. Do groups make better decisions than individuals?

It depends on the specifics of the situation. What research does tell us however, is that groups, particularly those that are diverse, often make better decisions than individuals because more options are considered. When groups make decisions, they are often implemented much more easily because those involved in the process have already “bought into” the decision. What must be guarded against is the group becoming too cohesive and not evaluating alternatives critically but simply accepting them because the group supports them.

146. What are two symptoms of groupthink?

There are eight symptoms of groupthink: illusion of invulnerability; collective rationalization; unquestioned belief in the group’s inherent morality; stereotyped views of outgroups; direct pressure; self-censorship; illusions of unanimity and the emergence of self appointed mindguards.

147. Describe the steps in the nominal group technique.

The nominal group technique was developed to ensure all members in a group participate fully. The first step is when each member of the group silently and independently writes down ideas. Step two is going around the room in order gathering all the ideas that were generated. The third step is a discussion that focuses on each of the ideas generated and finally, the group votes on their favorite ideas.

148. What is the difference between majority rule and consensus?

Majority rule is when each individual within a group gets a vote and whatever alternative received the most votes, wins. It is considered effective and is simple, speedy, easy to use and fair.

Page 278: Fwk Bauer Tif Organizational Behavior All Chapters

Consensus is a decision-making rule that groups use when the goal is to gain support for an idea or plan of action. This process requires more time and focuses on the aspect of plan support.

149. Explain how group decision support systems can become counterproductive.

Individuals can only process so many ideas and so much information at one time. As virtual meetings grow larger, it is reasonable to figure that information overload can occur and good ideas will fall through the cracks, recreating the problem that the group decision support system was designed to prevent. Also, the system could also just get too complicated.

150. What are two basic questions that can be asked to assess the ethics of a decision?

Is this decision fair?Will I feel better or worse about myself after I make the decision?Does this decision break any organizational rules?Does this decision break any laws?How would I feel if this decision was broadcast on the news?

151. Briefly describe how decision making differs around the globe.

Decision-making styles and approaches differ depending on the culture. Research, for example, shows that Japanese and Dutch decision makers are consensus-oriented while American decision makers value quick decisions.

ESSAY

152. Using the rational decision-making model as a template, discuss a recent decision you made.

Step one: Identify the problem.Choosing a college to attend.

Step two: Establish decision criteria.Offers specific major, cost parameters, distance from home.

Step three: Weigh decision criteria.Prioritize each of the criteria, say the cost is the most important

factor.

Step four: Generate alternatives.List the schools you are considering

Step five: Evaluate alternatives.What is the positive and negative of each school?

Step six: Choose the best alternative.Pick a school.

Page 279: Fwk Bauer Tif Organizational Behavior All Chapters

Step seven: Implement the decision.Apply to the school and gain admission.

Step eight: Evaluate the decision.After a semester, do you still like it?

153. Darlene is a new manager at XYZ Corporation. She is most interested in enhancing the creativity of her department. What recommendations would you suggest to her?

Four areas can be the focus of creativity enhancement efforts. They are team composition, team process, leadership and culture.

Team composition might be enhanced if you diversify the team, change group membership or use leaderless teams to allow teams freedom to create without trying to please anyone up front.

Team process can be enhanced by engaging in brainstorming, using the nominal group technique, or using analogies to envision problems and solutions.

Leadership: Challenge teams so they are engaged, not overwhelmed; let people decide how to achieve goals, support and celebrate creativity, and role model creative behavior.

Culture: institute organizational memory; build a physical space conducive to creativity; and incorporate creative behavior into the performance appraisal process.

154. You work for a company that was a sub-contractor for the Challenger space shuttle. Cognizant of the groupthink that made the fateful launch decision, you want to ensure that such a condition does not exist in your firm. The company executives have decided that a seminar is necessary to define groupthink, its causes, and provide recommendations do avoid it in the future. You are in charge of that seminar.

Groupthink is a group pressure phenomenon that increases the risk of the group’s making flawed decisions leading to a reduction in mental efficiency, reality testing, and moral judgment. The eight symptoms of groupthink are:

illusion of invulnerability; collective rationalization; unquestioned belief in the group’s inherent morality; stereotyped views of outgroups; direct pressure; self-censorship; illusions of unanimity and the emergence of self-appointed mindguards.

To avoid groupthink, groups should: discuss the symptoms of groupthink, assign a rotating devil’s advocate, invite experts who are not part of core decision making; encourage a culture of difference; and debate the ethical implications of the decisions.

Page 280: Fwk Bauer Tif Organizational Behavior All Chapters

Individuals should: monitor personal behavior for signs of groupthink, check for self-censorship; carefully avoid mindguard behavior; avoid putting pressure on other group members to conform; and remind members of the ground rules for avoiding groupthink.

Finally, group leaders should break the group into subgroups from time to time; have more than one group work on the same problem; remain impartial and refrain from stating preferences at the outset of decisions; set a tone of encouraging critical evaluations throughout deliberations; and create an anonymous feedback channel through which all group members can contribute if desired.

155. Ron is the owner of a small business. He is deciding whether to expand his operation to a second location or remain only at his original site. He decides to perform a “premortem” on the project and has asked your assistance in making sure he covers all appropriate steps in the process. What recommendations would you provide him?

A premortem is designed to imagine what could go wrong and avoid it before spending any money or having to change course.

The six steps to be followed are:

A planning team creates a plan outline for the issue.Either the existing group or a unique group imagines the issue at its worst. The group writes down all the reasons they can imagine that led to this failure.The list of ideas is reviewed for additional ideas.The issues are sorted into categories in a search for themes.The plan should be revised to account for the flaws detected.

Page 281: Fwk Bauer Tif Organizational Behavior All Chapters

Organizational Behavior, Version 1.1Bauer & Erdogan

FWK Test Item FileChapter 12

TRUE/FALSE

1. Leaders do not need to rely on the use of force to influence people.(True)

2. A formal leader is one who holds a position of authority and may utilize the power that comes from that position as well as personal power to influence others.

(True)

3. When intelligence is measured with a paper and pencil test, its relationship to leadership is a bit weaker than when intelligence is viewed as the perceived intelligence of the leader.

(True)

4. Emotional intelligence helps an individual attain the management ranks, but once there, mental intelligence becomes important.

(False)

5. All effective leaders are extraverts.(False)

6. The only trait of the Big 5 personality traits that is related to leadership emergence or leader effectiveness is conscientiousness.

(False)

7. There is no relationship between height and being viewed as a leader.(False)

8. Three key traits associated with leadership are: integrity, conscientiousness and introversion.

(False)

9. Agreeable people who are modest, good-natured and avoid conflict are less likely to be perceived as leaders.

(True)

10. A key finding from research completed on traits is that all traits are not equally effective in predicting leadership potential.

(True)

11. Research supports the argument that demonstrating both initiating structure and consideration makes leaders more effective.

(False)

Page 282: Fwk Bauer Tif Organizational Behavior All Chapters

12. Initiating structure involves structuring the roles of subordinates, providing them instructions, and behaving in ways to increase the group’s performance.

(True)

13. Authoritarian decision makers make decisions alone without necessarily involving employees in the decision making process.

(True)

14. Research suggests that the democratic decision-making style helps satisfy employees and improves decision quality.

(False)

15. Laissez-faire leaders create high levels of ambiguity about job expectations for their employees and thus lower employee satisfaction.

(True)

16. If you have a low least preferred coworker rating, it means that you have a people-oriented personality and are able to separate your liking of a person from your ability to work with that person.

(False)

17. Fiedler’s contingency theory suggests different people can be effective in different situations.

(True)

18. Research supports Fiedler’s predictions about when low LPC leadership should be used.

(True)

19. House’s path-goal theory is based on equity theory.(False)

20. Path-goal theory predicts that the type of leader behavior effective under different circumstances will depend on the characteristics of the employee and the work environment.

(True)

21. A supportive leadership style works best when there is high role ambiguity for employees, they have low abilities, and they have an external locus of control.

(False)

22. When employees have high abilities and high achievement motivation, the directive leadership style is best.

(False)

23. In House’s path-goal theory, the leader’s style is viewed as fixed and the environment changes; Fiedler’s theory assumes just the opposite.

(False)

Page 283: Fwk Bauer Tif Organizational Behavior All Chapters

24. Research suggests that on average, the leadership style recommended by Vroom and Yetton’s decision tree model tends to make more effective decisions compared to leaders using a style not recommended.

(True)

25. Transformational leaders use charisma, inspirational motivation, intellectual stimulation, and individualized consideration to influence individuals and create commitment to company goals.

(True)

26. Passive management by exception involves leaving employees alone and waiting until something goes wrong before entering the picture.

(True)

27. Transactional leadership is a more effective form than transformational leadership.

(False)

28. The key factor in making transformational leadership more effective is extraversion.

(False)

29. Charisma is an inherent characteristic, so those firms attempting to teach charisma, are not making good use of their time or money.

(False)

30. Individuals with high LMX receive higher levels of resources than those with a low LMX relationship.

(True)

31. All employees want a high-quality relationship with their manager.(False)

32. Servant leaders put their employees first.(True)

33. Authentic leaders are self-aware, introspective, and have thorough understanding of their own values.

(True)

34. People in positions of authority are influential in driving people to ethical behaviors but not unethical behaviors.

(False)

35. The global leadership and organizational behavior effectiveness project found, thus far, that there are no universal traits that are desirable or undesirable in a leader.

(False)

Page 284: Fwk Bauer Tif Organizational Behavior All Chapters

MULTIPLE CHOICE

Opening Section: Taking the Pepsi Challenge: The Case of Indra Nooyi

36. Indra Nooyi has proven to be an effective leader for PepsiCo since she took over the firm in 2006. She exhibits all of the following leadership qualities EXCEPTa. vision.b. charisma.c. proven track record in the industry.d. laid-back performance standards and style.

(d) Medium/Analysis

37. The act of influencing others toward a goal isa. leadership.b. charisma.c. power.d. motivation.

(a) Easy/Knowledge

38. Leaders who hold a position of authority and utilize the power that comes from their position as well as personal power to influence others are calleda. informal leaders.b. transformational leaders.c. formal leaders.d. charismatic leaders.

(c) Easy/Knowledge

Section I: Who is a Leader? Trait Approaches to Leadership

39. The earliest approach to the study of leadership wasa. the trait approach.b. the behavioral approach.c. the transactional approach.d. the transformational approach.

(a) Easy/Comprehension

40. The trait approach to leadership examines all of the following factors EXCEPTa. psychological attributes.b. behaviors.c. personality characteristics.d. physical characteristics.

(b) Medium/Comprehension

41. All of the following are examples of the Big 5 Personality traits EXCEPTa. openness.b. extraversion.c. intelligence.d. agreeableness.

(c) Easy/Comprehension

Page 285: Fwk Bauer Tif Organizational Behavior All Chapters

42. Which of the following statements regarding intelligence is INCORRECT?a. One’s intelligence is the strongest predictor of leadership performance.b. People with high mental abilities are more likely to be viewed as leaders

in their environment.c. Effective leaders tend to have high emotional intelligence as well as high

mental intelligence.d. IQ matters for entry into the high level managerial ranks, but once there,

IQ is less effective since all others at that level also possess high IQ.(a) Difficult/Evaluation

43. Jien has the ability to control his emotions in multiple situations and has empathy for others; he is internally motivated and has many admirable social skills. Jien hasa. high mental intelligence.b. high emotional intelligence.c. low mental intelligence.d. low emotional intelligence.

(b) Medium/Application

44. Being organized, systematic, punctual, achievement-oriented and dependable are characteristics of the personality traita. openness.b. agreeableness.c. extraversion.d. conscientiousness.

(d) Easy/Knowledge

45. Openness isa. being outgoing, talkative, sociable, and enjoying social situations.b. being affable, tolerant, sensitive, trusting, kind and warm.c. being curious, original, intellectual, creative and receptive to new ideas.d. being organized, systematic, punctual, achievement-oriented and

dependable.(c) Easy/Knowledge

46. Being anxious, irritable, temperamental and moody isa. anxiety.b. neuroticism.c. anger.d. aggressiveness.

(b) Easy/Knowledge

47. Being affable, tolerant, sensitive, trusting, kind and warm isa. agreeableness.b. extraversion.c. conscientiousness.d. openness.

(a)

48. Extraversion is

Page 286: Fwk Bauer Tif Organizational Behavior All Chapters

a. being curious, original, intellectual, creative and open to new ideas.b. being organized, systematic, punctual, achievement oriented and

dependable.c. being affable, tolerant, sensitive, trusting, kind and warm.d. being outgoing, talkative, sociable, and enjoying social situations.

(d) Easy/Knowledge

49. Maria is part of an organizational behavior project group. Her group members chose her as the leader of the group because she is always on task, on time for meetings, sets achievable goals and is willing to assist anyone who needs help. Maria is a. agreeable.b. extraverted.c. conscientious.d. open.

(c) Medium/Application

50. Nick’s roommate is attending a networking event for his business fraternity this weekend. He says, “Nick, you are so good at going up to someone and starting a conversation. You always seem to know what to do and what to say. You never seem awkward and you can talk to anyone about anything. I want to handle this networking event well—give me some tips.” Nick seems to be very a. agreeable.b. conscientious.c. narcissistic.d. extraverted.

(d) Medium/Application

51. The Big 5 personality trait ___________ has the strongest relationship to both leader emergence and leader effectiveness.a. conscientiousnessb. agreeablenessc. extraversiond. openness

(c) Medium/Comprehension

52. Research has found all of the Big 5 personality traits to be related to leader emergence and leader effectiveness EXCEPTa. conscientiousness.b. agreeableness.c. extraversion.d. openness.

(b) Difficult/Synthesis

53. The degree to which a person is at peace with himself and has an overall positive assessment of one’s self worth and capabilities isa. agreeableness.b. self-efficacy.c. self-esteem.d. integrity.

(c) Medium/Comprehension

Page 287: Fwk Bauer Tif Organizational Behavior All Chapters

54. Martin sent an anonymous memo to various members of his company charging his manager with cheating the employees out of pay bonuses. Martin’s manager seems to lacka. integrity.b. agreeableness.c. extraversion.d. neuroticism.

(a) Medium/Application

55. Which of the following statements regarding self-esteem and leadership is INCORRECT?a. Leaders with high self esteem support their subordinates more than

leaders with lower self esteem.b. Research shows a strong relationship between height and being viewed

as a leader which may be accounted for by self-esteem.c. Those with high self esteem have high self confidence which may affect

the image perceived by followers.d. Leaders with high self esteem punish their subordinates less effectively,

when such punishment is due, than do leaders with lower self esteem.(d) Medium/Analysis

56. Which of the following is not a key trait associated with leadership?a. High conscientiousnessb. High introversionc. High intelligenced. High integrity

(b) Medium/Synthesis

57. Which of the following is not a key trait associated with leadership?a. High integrityb. High conscientiousnessc. High agreeablenessd. High openness to experience

(c) Medium/Synthesis

58. Which of the following statements regarding traits and leadership is INCORRECT?a. All traits are equally effective in predicting leadership potential across

organizations.b. In large, bureaucratic organizations, leader traits have less impact on

behavior than in a small, entrepreneurial organization.c. Some traits have a stronger impact on leader emergence than on leader

effectiveness.d. Recognition of leader traits can help select appropriate managers for

organizations.(a) Difficult/Synthesis

Section II: What Do Leaders Do?: Behavioral Approaches to Leadership

59. The two broad categories of leader behaviors are

Page 288: Fwk Bauer Tif Organizational Behavior All Chapters

a. task-oriented and authority-oriented structures.b. task-oriented and initiating structures.c. people-oriented and authority-oriented structures.d. task-oriented and people-oriented structures.

(d) Easy/Knowledge

60. People-oriented behaviors are also calleda. initiating structure.b. consideration.c. laissez-faire.d. authoritarian.

(b) Easy/Knowledge

61. Task-oriented behaviors are also calleda. authoritarianism.b. consideration.c. initiating structured. laissez-faire.

(c) Easy/Knowledge

62. People-oriented behaviors includea. providing employees instructions.b. treating employees with respect.c. structuring employee roles.d. planning for increased employee performance.

(b) Medium/Comprehension

63. Research findings of leader behaviors suggesta. when leaders are task oriented, employees are more satisfied.b. task oriented behaviors tend to be more effective in large companies.c. extremely high levels of leader task-oriented behaviors may lead to

burnout in employees.d. employees who work under people oriented leaders are more productive.

(c) Difficult/Evaluation

64. When leaders make decisions alone without involving employees in the decision making process, they are usinga. laissez-faire decision making.b. participative decision making c. authoritarian decision making.d. democratic decision making.

(c) Easy/Knowledge

65. In democratic decision makinga. leaders leave employees alone to make decisions.b. the leader provides minimum guidance and involvement in the decision.c. leaders make the decision alone without necessarily involving employees

in the decision making process.d. employees participate in the making of the decision.

(d) Easy/Knowledge

Page 289: Fwk Bauer Tif Organizational Behavior All Chapters

66. Nelson is the manager of a marketing department in a medium sized company. Nelson has six employees reporting to him. In deciding the goals for the department for next quarter, Nelson is holding a meeting for employee input and direction. Nelson uses a(n) _________ style of decision making in the marketing department.a. authoritarianb. democraticc. laissez-faired. supportive

(b) Medium/Application

67. Research on decision-making styles findsa. Laissez-faire decision making is positively related to employee

satisfaction.b. Democratic decision making leads to stronger employee productivity.c. Employees care more about the overall participativeness of the

organizational climate than involvement in every single decision.d. Scientists favor authoritarian decision making over democratic decision

making.(c) Difficult/Synthesis

Section III: What is the Role of the Context? Contingency Approaches to Leadership

68. Nancy argues with Janet over every work project and generally does not want to be assigned to work with her. Yet, Nancy thinks Janet is a very nice person and enjoys her company at social outings.a. Nancy would have a low least preferred coworker score.b. Nancy would have a high least preferred coworker score.c. Nancy would have a midrange least preferred coworker score.d. Nancy would not have a least preferred coworker score.

(b) Difficult/Application

69. If you dislike working with a person and do not care for them on a personal level either,a. you would have a low least preferred coworker score.b. you would have a high least preferred coworker score.c. you would have a midrange least preferred coworker score.d. you would not have a least preferred coworker score.

(a) Medium/Comprehension

70. In situations of medium favorableness, a. low LPC leaders are effective.b. high LPC leaders are effective.c. medium LPC leaders are effective.d. either low or high LPC leaders will be effective.

(b) Difficult/Evaluation

71. Situational favorableness includes all of the following conditions EXCEPTa. leader-subordinate relations.b. position power.

Page 290: Fwk Bauer Tif Organizational Behavior All Chapters

c. motivation level.d. task structure.

(c) Easy/Comprehension

72. Paolo is a manager of a department in a small, family-run retail department store. He and his subordinates clash over scheduling and other procedural aspects of the job almost daily. While having the title of manager, Paolo makes few decisions for the department, instead implementing those relayed by his manager. The sales associate position as well as his managerial position do not have specific duties tied to them; instead everyone is supposed to do “whatever it takes.” The situation would best be described asa. favorable.b. medium favorablec. unfavorable.d. controlled.

(c) Medium/Application

73. If leader-subordinate relations are good, position power is low and task structure is high, the situation would be categorized asa. favorable.b. medium favorable.c. unfavorable.d. controlled.

(a) Medium/Analysis

74. If leader-subordinate relations are poor, position power is high and task structure is high, the situation would be categorized asa. favorable.b. medium favorable.c. unfavorable.d. controlled.

(b) Medium/Analysis

75. Karli is the designated leader of a class project group. She gets along well with the group members but often is not sure exactly what she should do to assist her group in producing a high-quality project. Which LPC style is best for this scenario?a. Low LPC leaders are effective.b. High LPC leaders are effective.c. Medium LPC leaders are effective.d. Either low or high LPC leaders will be effective.

(b) Difficult/Synthesis

76. Michael is the United Way pledge leader for his department. He gets along quite well with his coworkers. His pledge leader duties, with regard to specific steps to follow in asking coworkers for pledges and how to collect and report them, are clearly laid out for him in a United Way brochure. Which LPC style is best for this scenario?a. Low LPC leaders are effective.b. High LPC leaders are effective.c. Medium LPC leaders are effective.

Page 291: Fwk Bauer Tif Organizational Behavior All Chapters

d. Either low or high LPC leaders will be effective.(a) Difficult/Synthesis

77. Research on Fiedler’s Contingency Theory indicates which of the following?a. Predictions about where high LPC leadership will be effective are

supported more than those where low LPC leadership is effective.b. There is full support for Fiedler’s Contingency Theory.c. Fiedler’s theory is important because it recognizes the importance of the

leadership context.d. Fiedler’s theory is a useful framework for thinking about which personality

traits are useful in predicting leadership.(c) Medium/Analysis

78. Expectancy theory suggests individuals are motivated when all of the following conditions are met EXCEPTa. effort leads to high performance.b. rewards lead to high performance.c. rewards are valuable to the performer.d. high performance will be rewarded.

(b) Medium/Analysis

79. Which of the following statements regarding House’s path-goal theory of leadership is INCORRECT?a. The role of the leader in the path-goal theory is to remove roadblocks and

create a motivational environment for workers.b. Each leadership style identified in the path-goal theory is effective

depending on the employee characteristics and those of the work environment.

c. House’s path-goal theory of motivation is based upon the equity theory of motivation.

d. Leaders can create high performing employees by making sure employee effort leads to performance and that performance is rewarded with desired rewards.

(c) Medium/Synthesis

80. Maurice is a manager in the production area of a large manufacturing firm. He sets a tight production schedule every day and makes certain each employee knows what job he is performing that day and how to perform it. Maurice has a ____________ leadership style.a. directiveb. supportivec. participatived. achievement-oriented

(a) Difficult/Application

81. Nina makes it a point to talk to her subordinates every day about what is going on in their personal lives. She always congratulates them when they share successes and has a kind word and “a shoulder to cry on” if necessary when things are not going well. Nina has a _____________ leadership style.a. directiveb. supportive

Page 292: Fwk Bauer Tif Organizational Behavior All Chapters

c. participatived. achievement-oriented

(b) Difficult/Application

82. Leaders who ensure their employees take part in decision making have a(n) _________________ leadership style.a. achievement-orientedb. supportivec. directived. participative

(d) Medium/Comprehension

83. An achievement-oriented leadera. makes sure employees are involved in decision making.b. provides directions for his employees.c. provides emotional support to employees.d. sets goals for employees and encourages them to reach their goals.

(d) Medium/Knowledge

84. In which scenario would directive leadership be most effective?a. when employees have low role ambiguityb. when employees have high abilitiesc. when employees have an internal locus of controld. when employees have an external locus of control

(d) Difficult/Evaluation

85. If jobs are boring and repetitive or tasks are stressful, the ________ style of leadership is most effective.a. supportiveb. achievement orientedc. directived. participative

(a) Medium/Knowledge

86. The information system department’s employees are highly skilled individuals who possess a very internal locus of control. The firm is looking to hire a manager for the department to replace the retiring manager. What type of leadership style would likely be most effective with this group?a. supportiveb. achievement-orientedc. directived. participative

(d) Medium/Analysis

87. Which of the statements regarding research on House’s path-goal theory is correct?a. House’s theory suggests that the leader’s style is fixed and the

environment changes.b. The theory has been fully tested and verified.c. The theory’s greatest contribution is the importance of the leader’s ability

to change styles depending on the circumstances.

Page 293: Fwk Bauer Tif Organizational Behavior All Chapters

d. House’s theory is based upon goal-setting theory.(c) Difficult/Evaluation

88. The leadership decision treea. focuses on a static leadership style.b. focuses on leader emergence.c. focuses on an adaptive leadership style.d. leads to the determination of how much involvement leaders should seek

in making decisions.(d) Medium/Comprehension

89. According to the leadership decision tree, a leader who makes a decision alone using available information has a(n)a. autocratic decision-making style.b. consultative decision-making style.c. group decision-making style.d. laissez-faire decision-making style.

(a) Medium/Analysis

90. Bob meets with his subordinates on a regular basis to share concerns and information with the group. He takes their feedback but makes decisions on his own. According to the leadership decision tree, Bob has a(n) _____________ decision-making style.a. laissez-faireb. consultativec. groupd. autocratic

(b) Medium/Analysis

Section IV: What’s New? Contemporary Approaches to Leadership

91. ___________ leaders lead employees by aligning employee goals with the leader’s goals.a. Directiveb. Transactionalc. Transformationald. Achievement-oriented

(c) Easy/Knowledge

92. Which one of the following traits is NOT a tool that transformational leaders use to influence employees and create commitment to the company goals?a. charismab. management by exceptionc. intellectual stimulationd. inspirational motivation

(b) Medium/Comprehension

93. Most research on leadership has been conducted on which of the following contingency theories of leadership?a. Fieldler’s contingency theoryb. House’s path-goal theory

Page 294: Fwk Bauer Tif Organizational Behavior All Chapters

c. transformational leadership theoryd. authentic leadership theory

(c) Difficult/Synthesis

94. Malcolm is the manager of his department. After performance evaluations, he gives those who have met their goals for the year a pay raise. He gives his employees their assignments and leaves them to do their work but does monitor them so that if he notices a potential problem, it gets resolved before causing difficulties. What kind of contemporary leadership style is Malcolm demonstrating?a. directive leadershipb. transformational leadershipc. transactional leadershipd. laissez-faire leadership

(c) Difficult/Application

95. Research on transformational and transactional leadership shows all of the following EXCEPTa. transformational leaders increase the performance of their followers.b. transactional leaders have positive influences over employee attitudes.c. transformational leaders create higher levels of commitment to

organizational change efforts.d. transactional leaders increase the intrinsic motivation of their followers.

(d) Difficult/Evaluation

96. The key factor in the effectiveness of transformational leadership isa. trust.b. creativeness.c. extraversion.d. charisma.

(a) Medium/Comprehension

97. Which of the following is an INCORRECT statement about leader-member exchange (LMX) theory?a. High LMX relationships involve an active dislike between the leader and

member.b. In high LMX relationships, the leader has a trust-based relationship with

the member.c. In high LMX relationships, both leader and member go above and beyond

their job descriptions to assist the other party in succeeding.d. In low LMX relationships, the leader does his job and the member does

his.(a) Difficult/Evaluation

98. All of the following are factors contributing to building high-quality leader-member exchange EXCEPTa. leader fairness.b. leader delegation.c. employee job satisfaction.d. personality similarity.

(c) Medium/Analysis

Page 295: Fwk Bauer Tif Organizational Behavior All Chapters

99. All of the following are consequences of a high leader-member exchange relationship EXCEPTa. job satisfaction.b. lower turnover.c. organizational commitment.d. effort in building good relationships.

(d) Medium/Analysis

100. Which of the following statements regarding leader-member exchange relationships is INCORRECT?a. A problem with a low-quality exchange relationship is that employees may

not have access to a positive work environment.b. All employees would like to have a positive relationship with their leader.c. If a leader is unethical, employees may not want to have a relationship so

as not to suffer from guilt by association.d. When high LMX employees succeed, leaders credit the efforts they put

forward and their high level abilities.(b) Difficult/Evaluation

101. Servant leadership a. is a leadership approach that defines the leader’s role as serving the

needs of others.b. refers to effective leaders who stay true to themselves.c. is leading employees by aligning employee goals with the leader’s goals.d. involves structuring the roles of subordinates, providing them with

instructions, and behaving in ways that will increase the performance of the group.

(a) Easy/Knowledge

102. Servant leadership has an explicit focus on all of the following factors EXCEPTa. ethics.b. community development.c. paternalism.d. self-sacrifice.

(c) Medium/Analysis

103. Servant leadersa. put themselves first.b. make their overriding goal getting their employees to contribute to the

community.c. make their primary mission high-level personal achievement.d. feel an obligation to their employees, customers and the larger

community.(d) Medium/Analysis

104. Authentic leadershipa. is a leadership approach that defines the leader’s role as serving the

needs of others.b. refers to effective leaders who stay true to themselves.c. makes sure employees are involved in decision making.

Page 296: Fwk Bauer Tif Organizational Behavior All Chapters

d. sets goals for employees and encourages them to reach those goals.(b) Easy/Knowledge

105. Which of the following is NOT a key characteristic of authentic leaders?a. They are not afraid to act they way they are.b. They understand where they are coming from.c. They have no set priorities.d. They have high levels of personal integrity.

(c) Medium/Comprehension

Section V: The Role of Ethics and National Culture

106. Which of the following statements regarding leadership and ethics is INCORRECT?a. People in positions of authority are influential in driving others to ethical or

unethical behaviors.b. Leaders with high levels of moral development create more ethical

organizational climates.c. The contingency theories of leadership are explicit in noting the important

role that ethics play in effective leadership.d. The level of ethical leadership shown is related to a willingness to report

job related problems to the leader.(c) Medium/Evaluation

107. Which of the following statement regarding leadership and other cultures is INCORRECT?a. Ninety-eight percent of all leadership studies have been completed in the

United States.b. The applicability of leadership theories to other cultures is uncertain.c. Only decisiveness is a universal leadership trait.d. Given the differences between cultures, a sensitivity to those differences

and the ability to adapt style to circumstances are needed in global leaders.

(c) Medium/Analysis

108. Which statement regarding global leader characteristics is correct?a. In Western and Latin cultures, leaders who speak in a monotonous voice

convey the ability to control emotions.b. Universally, people feel that honesty, trustworthiness and fairness are

related to leadership effectiveness.c. Transactional leaders are found to be the most influential leaders around

the world.d. There is no universal agreement on characteristics viewed as undesirable

in leaders.(b) Difficult/Evaluation

FILL IN THE BLANK

109. ___________ is the act of influencing others to work to a goal.(Leadership)

Page 297: Fwk Bauer Tif Organizational Behavior All Chapters

110. Leaders who demonstrate leadership by influencing others, but are without a formal position are ____________ leaders.

(informal)

111. “IQ” refers to _____________ ___________ while “EQ” refers to _____________ _______________.

(mental intelligence, emotional intelligence)

112. A person who is talkative, outgoing, sociable and enjoys social situations is ___________.

(extraverted)

113. ___________ is being organized, systematic, punctual, achievement-oriented and dependable.

(Conscientiousness)

114. The two personality traits of the Big 5 that are not related to leader emergence or leader effectiveness are ____________ and ___________.

(agreeableness, neuroticism)

115. ____________ may explain the relationship between being tall and being viewed as a leader.

(Self-esteem)

116. Researchers at the Ohio State University and the University of Michigan found that there are two broad categories of behaviors related to leadership; they are __________ and ____________.

(task-oriented, people-oriented)

117. A __________ leader decision-making style leaves employees alone to make a decision.

(laissez-faire)

118. A ______ ______ score from Fiedler’s contingency theory means that an individual has a people-oriented personality and can separate his liking of a person from his ability to work with that person.

(high LPC)

119. The three conditions creating Fiedler’s situational favorableness are ______ ________, ___________ ___________and _______ _______________.

(leader-subordinate relations, position power, task structure)

120. The __________ leadership style is appropriate for situations when tasks are boring and repetitive and they are stressful.

(supportive)

121. Leaders who provide specific directions for employees, treat them well, care about them on a personal level and are encouraging have the ____________ leadership style.

(directive)

Page 298: Fwk Bauer Tif Organizational Behavior All Chapters

122. A ____________ __________ is a decision-making tool to help leaders determine how much involvement they should seek in making decisions.

(decision tree)

123. Those leaders who lead employees by aligning employee goals with the leader’s goals are _______________ leaders.

(transformational)

124. ______________ leaders ensure that employees demonstrate the right behaviors because they provide resources in exchange.

(Transactional)

125. A magnetic personality is _____________.(charisma)

126. ____________ _______________ _ _____________ involves leaving employees to do their jobs without interference but at the same time proactively predicting problems.

(Active management by exception)

127. Leaving employees alone, and, as a manager, waiting until something happens before interacting is _____________ ____________ ____ _____________.

(passive management by exception)

128. _______________is the key factor in making transformational leadership more effective.

(Trust)

129. In a high _______________, the leader forms a trust-based relationship with the members.

(leader-member exchange or LMX)

130. When a leader puts his employees first, he is exhibiting ___________ _____(servant leadership)

131. Key characteristics of _________leadership are self-awareness, a thorough understanding of values and priorities, and personal integrity.

(authentic)

132. The ________________ experiments, where those participating provided electric shocks to others answering questions incorrectly, are an example of unethical behaviors by people in positions of authority.

(Milgram)

133. __________leaders are found to be one of the most influential leaders around the world.

(Charismatic)

SHORT ANSWERS

Page 299: Fwk Bauer Tif Organizational Behavior All Chapters

134. List and describe two traits that are associated with leadership. Indicate what impact they have on leadership.

Traits that can be presented include: general mental intelligence and emotional intelligence, the Big 5 traits, self-esteem and integrity.

General intelligence is often viewed as getting individuals to the leader level, but once there emotional intelligence may take over, as everyone at the leader level has mental intelligence.

The Big 5 traits of openness, conscientiousness and extraversion are related both to leader emergence and leader effectiveness. Extraversion has the strongest relationship to both leader emergence and effectiveness.

Individual leaders with high self-esteem tend to have greater levels of self-confidence, and studies show that they support their subordinates more and when they punish, punish more effectively. Also, self-esteem may be the underlying reason why height and leadership are connected, because people who are taller have greater self-esteem and therefore project more charisma and get more followers.

Those individuals who have greater integrity are viewed as more trustworthy and thus gain more followers.

135. Differentiate between mental intelligence and emotional intelligence. What impact does either have on leadership in an organization?

Mental intelligence is represented by IQ and obviously reflects abilities such as mathematical and verbal comprehension. Emotional intelligence is a concept popularized by Daniel Goleman and refers to individuals’ ability to control their own emotions, understand other people’s emotions, internal motivations and their social skills.

Goleman suggests that mental ability gets the individual to the leader level but once there, emotional intelligence takes over, as everyone at the leader level has general intelligence.

136. Describe the ties between each of the Big 5 personality traits and leadership.

Extraversion is the Big 5 trait with the strongest relationship between both leader emergence and leader effectiveness.

Conscientious individuals are organized, take initiative and demonstrate persistence, so this factor also is tied to leader emergence, and then effectiveness in that position.

Openness is the trait representing originality and creativity and tends to lead to leader emergence, then effectiveness.

Agreeable people are modest, good-natured and avoid conflict but are less likely to be perceived as leaders.

Page 300: Fwk Bauer Tif Organizational Behavior All Chapters

Clearly, neuroticism, which is being anxious, irritable, temperamental and moody, is not a characteristic tied to leader emergence or effectiveness.

137. What is the difference between task-oriented and people-oriented behaviors?

Task-oriented or initiating structure, delineate the roles of subordinates, provide them instructions and behave in ways that generally increase the performance of the group. Task-oriented people get things done and meet organizational goals.

People-oriented behaviors, or consideration, include showing concern for employee feelings and treating employees with respect. People-oriented managers care about their employees and demonstrate that concern through action and decision.

Both types of behaviors are beneficial to an organization but for different purposes. For example, when people-oriented behaviors are demonstrated, employees are more satisfied. When task-oriented behaviors are demonstrated, productivity tends to be higher.

138. Describe each of the types of leader decision making.

There are three types of leader decision-making styles. Authoritarian decision making is when leaders make the decisions alone without involving employees in the decision-making process. Laissez-faire decision making is when leaders leave employees alone to make the decision providing minimal guidance in the decision. Democratic decision-making leaders have employees participate in the decision making process.

139. How does Fiedler’s contingency theory compare to that of House’s path-goal theory of leadership?

Fiedler’s contingency theory looks at individual scores on a least preferred coworker scale. This score indicates whether you can dislike an individual personally and still work effectively with him. This theory assumes the leader’s style is fixed and the environment changes. Thus you assess the leader style in relationship to situational favorableness (leader-subordinate relations, position power, task structure) to determine the most effective style.

House’s path-goal theory is based upon expectancy theory and looks at how a leader’s style can change depending on the circumstances. Underlying path-goal theory is the expectation that effort leads to high performance, high performance will be rewarded. and the rewards will be valuable. The leader’s job is to remove roadblocks from this process and create environments that subordinates see as motivational.

140. Describe each of the four leadership styles and under what circumstances the style is most effective.

Directive leaders are those who provide specific directions to employees. They lead by clarifying role expectations, setting schedules, and making sure

Page 301: Fwk Bauer Tif Organizational Behavior All Chapters

employees how what to do. This style works best when there is high role ambiguity for employees, where employees have low abilities and external loci of control.

Supportive leaders provide emotional support to employees. They treat employees well, treat people on a personal level and are encouraging. This style is most effective when the tasks are boring and repetitive and stressful.

The participative leadership style makes certain employees are involved in making important decisions. The style is most effective when employees have high-level abilities, when the decision is relevant to them and when they have a high locus of control.

The achievement-oriented leader sets goals for employees and encourages them to reach those goals. The circumstances under which this style works most effectively are when employees have high abilities and a high achievement motivation.

141. Define transformational leadership.

Transformational leadership is when leaders align employee goals with their own. To do so, the leader uses his charisma, inspirational motivation, intellectual stimulation and individualized consideration to influence subordinates.

142. Describe transactional leadership.

Transactional leadership ensures that employees demonstrate the right behaviors because the leader provides resources in exchange. The leaders provide contingent rewards and management by exception.

143. Why would some employees not want to have a high quality relationship with their leader?

All employees may not want to have a high quality relationship with their manager. Some employees do not like their managers, at times because they see them as unethical or unkind and do not want to be labeled as such simply because they associate with him. Some employees do not have any interest in advancing their career in a particular company or industry so they are simply in the firm to get their paycheck and there is no need for additional time spent in a relationship. Finally, some leaders simply cannot influence some people.

144. What is servant leadership?

Servant leadership is where the leader serves the needs of others including employees, customers, and the external community. Servant leaders feel a need to understand their employees’ personal needs, to empower them and to help them develop their careers.

145. What is authentic leadership?

Page 302: Fwk Bauer Tif Organizational Behavior All Chapters

The key to authentic leadership is to be true to yourself. Authentic leaders are self-aware, they are introspective and understand where they are coming from and have a thorough understanding of their own values and priorities. They are not afraid to act the way they are and they have high personal integrity.

146. What implications do the Milgram experiments have for the concept of leadership?

The Milgram experiments, where subjects were directed by an individual in a lab coat to administer electric shocks to other people when those individuals gave a wrong answer to questions, demonstrates that people in positions of authority are influential in driving others to ethical or unethical behaviors. In the Milgram situation, the individuals giving the INCORRECT answers were actually actors, and the electric shocks subjects thought they were delivering were not real. Yet, the actors pretended the shocks were occurring. About two-thirds of the subjects continued to administer the shocks to INCORRECT answers even when those shocks reached “dangerous” levels.

ESSAY

147. Choose any two of the leadership theories presented in this chapter, define them and then compare and contrast the theories.

There are four sets of theories in this chapter: Trait theories, behavioral theories, contingency and contemporary theories. Students can choose in the trait theory category to discuss specific traits like the Big 5 personality traits, mental and emotional intelligence, self-esteem and integrity. Their discussions should center on the relationship between the individual trait and either leadership emergence or leadership effectiveness. The intent behind the trait theories was to define a limited number of attributes that could define a leader or the success of a leader. The major limitation of the trait theories is that they do not take into account the situation in which the leadership takes place.

A second group of theories that can serve as a comparison point are the behavioral theories. These theories look at what leaders actually do. The Ohio State and University of Michigan studies pointed out that leaders tend to demonstrate task- or people-oriented behaviors, and students can discuss the relationship between those behaviors and leadership effectiveness. Further, students can discuss the style of leader decision making and its relationship to leader effectiveness. The value of the behavioral theories is that they indicate the need to understand the context of leadership, and so the third category, contingency theory, might be a point of comparison for the writer.

Contingency theories presented include Fiedler’s and House’s theories. Fiedler’s work assumed the leader style to be fixed and only the environment can be changed. House took a different track underscoring the importance of varying one’s style depending on the situation.

The final group of theories are contemporary theories including those of transformational, transactional, leader-member exchange, servant and authentic

Page 303: Fwk Bauer Tif Organizational Behavior All Chapters

leadership. Each of these new theories focuses on a different aspect of leadership, but all suggest that effective leaders need to change their style based upon the demands of the situation as well as using their own values and needs.

148. What contribution to building effective leaders does the trait theory offer?

The trait theories attempt to create a limited set of attributes that can define a leader or the success of the leader. While the number of traits is large and the theory does not account for the situation a leader faces, the notion of understanding leader attributes is a good way to begin building such strengths in your organization or your personal repertoire of skills. The more likely individuals are to demonstrate these attributes, the more likely they are to be successful and to make their organizations successful. So while the topics that could be discussed in a leadership development seminar from a trait perspective are large, the value of those traits to individual development is unquestionable.

149. Under what conditions would a task-oriented manager be most successful?

Task-oriented managers work best in scenarios where structured roles for subordinates, clear instructions and behaviors to enhance performance are warranted. Research suggests small businesses are appropriate places for such attributes.

150. Compare and contrast transformational and transactional leadership.

Transformational leadership is where managers lead by aligning employee goals with the leader’s own. To do so, the leader uses charisma, individualized consideration, intellectual stimulation and inspirational motivation. In contrast, transactional leaders ensure that employees demonstrate the right behaviors because the leader provides resources in exchange. These resources include contingent rewards. Transactional managers often use active management by exception as a tool to success. Both of these leadership approaches are part of the contemporary body of work on leadership. Both have shown to lead to effectiveness, but the circumstances where that occurs obviously differs. Transformational leadership is viewed as more effective because of the trust established in the relationship.

151. How would you train an individual so as to increase his charisma?

There is some argument as to whether charisma is an inherent characteristic and untrainable. If you do believe you can train charisma, you might want to consider:

Teaching individuals how to create a vision that you understand and are immersed in and then demonstrate how to share that with others so they too pursue it with you.

This vision must be tied to history in the department or organization, so the notion of teaching the history or culture of the firm needs to be addressed in training.

Page 304: Fwk Bauer Tif Organizational Behavior All Chapters

Body language recognition is very important, so a session of training could be devoted to the study of body language and its implications.

Charisma involves confidence. Do self-esteem assessments during a training session and develop techniques to enhance self-esteem.

Charisma also involves having novel approaches to problems, so training could focus on developing out-of-the-box thinking by using different brainstorming techniques including wildstorming.

152. Critique the leader-member exchange theory (LMX) with regard to the implications it has for effective leadership in the organization.

There are many different relationships in a work organization between the leader and subordinates. Clearly there will be in-group and out-group situations. There are advantages to high LMX relationships particularly from the perspective of the subordinate and the manager often gets a committed, productive worker because of the resources available from that relationship. That said, not everyone can have a high LMX relationship, so, some of the best advice for a leader is that he be fair in all his dealings with employees and that he treat all his employees with respect. The key issue to remember as a leader is relationships have an important influence over employee attitudes and behaviors, so be aware how you build relationships and how you could build relationships—put conscious effort into it.

153. Denise wants to improve her relationship with her manager. Right now, she feels like she really does not know him well. She recognizes that part of the issue is that she works in her office with the door closed and opens it to meet with people only on an “as needed” basis. What are some recommendations for Denise?

Create interaction opportunities with your manager. Denise should not stay in her office. Get out, ask questions. Recognize that people are more attracted to others like them. Denise needs to establish and convey the similarities she and her manager share so they have a common starting point.Tactfully utilize impression management techniques. For example, compliment your manager on work-related issues.Be a reliable employee. Trust is key to building and sustaining relationships.Be aware that relationships develop early. Denise created some issues for herself by staying in her office. She is working from behind but using the right techniques, can catch up.

154. In what context might a servant leader be successful?

Servant leadership can actually work in any firm. In today’s business climate where daily news bulletins present yet another leader who has created a Ponzi scheme or otherwise defrauded firms and individuals, the move to putting employees, customers and the larger external community first is radical but needed. Those who practice servant leadership will get attention and will be successful just for the simple reason of trust. People will begin to pull out of the

Page 305: Fwk Bauer Tif Organizational Behavior All Chapters

economic environment when that trust is gone, but will enter full scale when they see integrity demonstrated.

155. What leadership theory or characteristics are most applicable to other countries?

The answer for all situations is, it depends. Students can cite specific examples provided in this chapter with regard to the Global Leadership and Organizational Behavior Effectiveness project, but it comes down to understanding that there is variation and similarity across cultures. Being aware, being open and constantly monitoring other cultures are the best pieces of advice for understanding what the key characteristics are.

In this situation, the characteristics of vision, team orientation and participative leadership seem to be preferred styles around the world. Traits like how confident leaders should be and whether they should sacrifice themselves for the good of their employees differ. Universally charismatic and support leadership is common but how those traits are achieved varied.

Page 306: Fwk Bauer Tif Organizational Behavior All Chapters

Organizational Behavior, Version 1.1Bauer & Erdogan

FWK Test Item FileChapter 13

TRUE/FALSE

1. Power is the ability to get things done the way you want them to be done.(True)

2. Power distribution is usually not visible in organizations.(False)

3. Power has negative consequences but never has positive consequences.(False)

4. One reason why power can be so easily abused is because individuals are quick to conform.

(True)

5. Conformity is people’s tendencies to behave consistently with social norms.(True)

6. The Milgram, Asch, and Hawthorne studies illustrate how important it is to create checks and balances to help individuals resist the tendency to conform or abuse authority.

(False)

7. In the Milgram studies, participants were willing to administer harmful levels of voltage to learners when those learners provided incorrect answers.

(True)

8. The Asch experiments showed that a dissenting minority does not have much power.

(False)

9. Meta-analysis suggests that the level of conformity in the United States has been increasing since the 1950s.

(False)

10. The Zimbardo experiment was stopped early because all parties involved became too entrenched in their experimental roles.

(True)

11. The more that a party is dependent upon you, the less power you have.(False)

12. The scarcer a resource is that you control, the more power you possess.(True)

Page 307: Fwk Bauer Tif Organizational Behavior All Chapters

13. The more vital the resource you control, the greater the level of power you possess.

(True)

14. The more substitutes available for a resource, the higher the power level of the person possessing that resource.

(False)

15. The CEO of an organization has legitimate power because of his or her role.(True)

16. Coercive power tends to accompany legitimate power in an organization.(False)

17. Coercive power works through fear and forces people to do things they otherwise would not consider.

(True)

18. Michael is the individual in the firm who knows everything about computer issues. When anyone has a problem, they go to Michael. Michael has referent power.

(False)

19. Charisma is a type of referent power.(True)

20. Influence is getting others to do what we want them to do.(True)

21. The most commonly used influence technique is personal appeal.(False)

22. Compliance is when the target of influence not only agrees to your request, but actively supports it.

(False)

23. Rational persuasion uses facts, data and logical arguments to influence a target person.

(True)

24. When college students are told by a speaker seeking their support, “You can make a difference. Do this for future generations,” that speaker is using inspirational appeals to convince students to join the cause.

(True)

25. Using any form of flattery in an appeal is an example of the exchange influence tactic.

(False)

26. Research shows that individuals who lived near stairwells in dorms were the most well liked people in those dorms because they were seen most often by

Page 308: Fwk Bauer Tif Organizational Behavior All Chapters

everyone who was coming and going. This is an example of the personal appeal influence tactic.

(True)

27. Research shows that managers with high referent power tend to use pressure tactics much more frequently than those with low referent power.

(False)

28. Legitimating tactics are those whose appeal is based upon position power.(True)

29. Referent power is more effective than formal power bases and is positively related to employees’ satisfaction with supervision.

(True)

30. The most effective approach to impression management on the job is to build credibility and maintain authenticity.

(True)

31. Impression management has been shown to be related to higher performance ratings by increasing liking and perceived similarity.

(True)

32. The tone of your voice, rate of speech, and how you deliver your message are relatively unimportant in the impression management area.

(False)

33. Impression management is particularly salient in job interviews and promotional contexts.

(True)

34. Upward influence includes appealing to a higher authority or citing a firm’s goals as a reason for others to follow your suggestion.

(True)

35. Managers report using personal appeals most frequently with other managers and rational persuasion most frequently with subordinates.

(False)

36. Effective politics is about winning at all costs.(False)

37. Almost all managers surveyed suggest workplace politics exist in their organizations and that to be successful individuals must engage in politics.

(True)

38. If employees feel their organizations are too driven by politics, they are less committed to the organization and perform worse on the job.

(True)

39. Individuals high in internal locus of control engage in more political behavior.

Page 309: Fwk Bauer Tif Organizational Behavior All Chapters

(True)40. When resources such as raises or promotions are limited, employees see the

organization as more political.(True)

41. When individuals feel role ambiguity in their jobs, they feel the organization is more political.

(True)

42. The more democratic the decision making is in a firm, the more the firm will be perceived as political.

(True)

43. Social network analysis can reveal who can be trusted in an organization, who is important to decision making and even who is innovative.

(True)

44. Boundary spanners are people who are linked to the greatest number of people in the organization.

(False)

45. Social network analysis indicating strong ties is demonstrating informational, not emotional support.

(False)

46. Peripheral specialists in a social network are those with special expertise who can be called upon for advice even though they work independently from the group.

(True)

47. Strong ties in a social network are especially useful for innovation.(False)

48. Company presidents usually possess at least legitimate, coercive, and referent power.

(False)

49. High power distance countries are those where power is centralized in the hands of a few.

(True)

50. In low power distance countries, decisions are often made based upon loyalty rather than some formal review process.

(False)

MULTIPLE CHOICE

Opening Section: Focus on Power: The Case of Steve Jobs

Page 310: Fwk Bauer Tif Organizational Behavior All Chapters

51. Steve Jobs is one of the best-known CEOs in business. Which of the following statements is INCORRECT in describing the way that Steve Jobs runs Apple Inc.?a. Jobs has an uncanny ability to persuade and influence people.b. Jobs instills enthusiasm in his employees.c. Jobs has a vision that he shares with his employees.d. Jobs is always respectful of his employees.

(d) Difficult/Synthesis

52. Steve Jobs is best known for using what kind of power in running Apple Inc.?a. coerciveb. referentc. legitimated. all of the above

(d) Medium/Analysis

Section I: The Basics of Power

53. The ability to influence the behavior of others to get what you want isa. charisma.b. power.c. conformity.d. pressure.

(b) Easy/Knowledge

54. Which of the following statements regarding power distribution research is INCORRECT?a. Power distribution is visible in organizations.b. Individuals ranked themselves higher than their coworkers did.c. In comparing rankings given to managers with regard to their influence,

there was general agreement on the top five but no agreement on the bottom five.

d. Research indicates the ability to rank managers by influence is a consistent practice in various industries and firms.

(c) Difficult/Analysis

55. Which of the following statements regarding the positive and negative consequences of power is INCORRECT?a. Individuals are often quick to question the actions of those in power.b. Powerful CEOs can align an organization to achieve goals.c. The adage, “Power corrupts and absolute power corrupts absolutely”

seems to have some merit.d. Power can corrupt and lead to the destruction of companies.

(a) Difficult/Evaluation

56. People’s tendencies to behave consistently with social norms isa. culture.b. conformity.c. power.d. dependency.

(b) Easy/Knowledge

Page 311: Fwk Bauer Tif Organizational Behavior All Chapters

57. Which of the following statements regarding the Milgram experiment on conformity to authority is correct?a. Participants believed they were engaged in an experiment on job design.b. Over half of the participants refused to shock the learner at very high

voltage levels regardless of the encouragement to do so provided by the experimenter.

c. Several learners were treated for depression at the conclusion of the study.

d. The researcher/experimenter’s role was very important in obtaining group compliance with the experiment procedures/guidelines.

(d) Difficult/Evaluation

58. Which of the following statements regarding the Asch studies is INCORRECT?a. Most of the participants in the Asch groups were confederates.b. A dissenting minority influenced the nature of response given by

participants. c. Conformity with an incorrect answer was attributed to the perception that

some members of the group had more information regarding the correct choice.

d. A dissenting minority response caused the participants to give the correct answer only if that minority also gave the right answer.

(d) Difficult/Evaluation

59. Which of the following statements regarding the Zimbardo study is correct?a. The experimental guards received training before they were told to keep order.b. The guards became abusive and aggressive, but the prisoners exhibited

little reaction.c. Study participants refused to adapt to their roles. d. The Zimbardo study was ended early due to the participant’s deep

entrenchment of the roles.(d) Difficult/Evaluation

60. The strategic contingencies model suggestsa. the more difficult it is find a substitute, the less power held by the person

with the substitute.b. the more unique your resource, the more power you will have.c. the more vital the resources you control, the less power you hold.d. the more dependent an individual is on another, the less power the one

being depended on has in the situation.(b) Difficult/Analysis

61. Mark is the only individual in his organization who speaks Spanish fluently. His firm just purchased a firm located in Mexico. Mark’s boss just told him how valuable he is to the firm. This scenario is an example of what factor in the strategic contingency model?a. scarcityb. dependentc. independenced. substitutability

Page 312: Fwk Bauer Tif Organizational Behavior All Chapters

(a) Medium/Application

62. The value of the resource isa. its scarcity.b. its substitutability.c. its importance.d. its dependency.

(c) Easy/Knowledge

63. The organizational behavior course has a project as a key requirement in the course. As part of the course’s evaluation, students must indicate which group member is the most valuable to the group. The designated individual will receive bonus points. Nick is the only one in his course project group who can create effective PowerPoint presentations. As the group’s presentation must feature a visual aid, Nick is very confident he will be chosen as the most valuable group member. Elizabeth just arrived at the group meeting with ideas for depicting colorful poster boards with the same information. The group was very enthusiastic about her novel ideas. This scenario demonstrates the impact ___________ has on the level of power one has.a. importanceb. dependencyc. scarcityd. substitutability

(d) Medium/Application

64. Which of the following is correct regarding aspects of the strategic contingency model?a. If the resources you control are vital to the organization, you will be

viewed as a threat to the others and will have less job security.b. If there is a clear substitute that exists, the level of power held is greater.c. The more difficult something is to obtain, the less value it tends to have.d. The more dependent you are on another, the less power you have.

(d) Medium/Analysis.

Section II: The Power to Influence

65. A professor assigns grades due to _________ power.a. coerciveb. referentc. rewardd. legitimate

(d) Medium/Application

66. Managers at a pharmaceutical research and development firm have checkbooks that they carry around with them as they monitor their subordinates. If a manager notes outstanding behavior by a subordinate, they can write the high-level performer a check on the spot. This is an example of the manager’s ___________ power.a. referentb. reward

Page 313: Fwk Bauer Tif Organizational Behavior All Chapters

c. expert powerd. information

(b) Medium/Application

67. Whitt Company made a substantial monetary offer to Bob to leave his design job at Lincoln Industries and come to work at Whitt. The Whitt CEO, on hearing some complaints from members of the Whitt Board of Directors about the size of Bob’s pay package, responded, “Bob is one of the few people in the industry who really understands what the customer wants and can design it into the product.” Bob has ____________ power.a. expertb. legitimatec. referentd. legitimate

(a) Medium/Application

68. Professor Lamb is a fabulous speaker—funny, topical, and very knowledgeable about his subject. His classes are always filled as soon as they open for scheduling and the waiting list to get into them is long. Students never miss his class and they highly recommend him to their friends. Graduate students want to study with him to learn his techniques. Professor Lamb has ___________ power.a. coerciveb. legitimatec. referentd. reward

(c) Medium/Application

69. Al Dunlap was recognized as a turnaround artist for firms experiencing financial and performance difficulties. When Dunlap arrived at a firm, downsizing usually followed. Dunlap received the nickname, “Chainsaw Al” due to his cost-cutting abilities. When Dunlap arrived at a firm he would recommend major restructuring and though they were not supportive of his tactics, his managers implemented his plan. Dunlap had ___________ power.a. legitimateb. likabilityc. referentd. informational

(a) Medium/Application

70. When negotiating a deal on the purchase of a new automobile, Bradley had manufacturing cost data from his neighbor who worked for the car manufacturer. Bradley got himself a good deal on his purchase due to his ___________ power in the negotiation.a. referentb. legitimatec. informationd. coercive

(c) Medium/Application

71. Access to specific knowledge or skills is _______________ power.a. expert

Page 314: Fwk Bauer Tif Organizational Behavior All Chapters

b. informationc. referentd. reward

(b) Easy/Comprehension

72. The ability to grant an increase in pay, a perk, or an attractive job assignment is ___________ power.a. referentb. legitimatec. expertd. reward

(d) Easy/Comprehension

73. Coercive power a. is the ability to take something away or punish someone for

noncompliance.b. is the ability to influence due to the possession of knowledge or skill.c. stems from the personal characteristics of the person such as the degree

to which we respect and want to be like them.d. stems from one’s organizational role or position.

(a) Easy/Knowledge

74. Referent power a. is the ability to take something away or punish someone for

noncompliance.b. is the ability to influence due to the possession of knowledge or skill.c. stems from the personal characteristics of the person such as the degree

to which we respect and want to be like them.d. stems from one’s organizational role or position.

(c) Easy/Knowledge

75. Power that comes from one’s organizational role or position isa. expert power.b. referent power.c. legitimate power.d. information power.

(c) Easy/Comprehension

76. Power that comes from knowledge and skill isa. expert power.b. referent power.c. legitimate power.d. reward power.

(a) Easy/Comprehension

77. Attempting to get others to do what we want isa. commitment.b. power.c. charisma.d. influence.

(d) Easy/Knowledge

Page 315: Fwk Bauer Tif Organizational Behavior All Chapters

78. The ability to attract others, win their admiration, and hold them spellbound isa. power.b. influence.c. charisma.d. commitment.

(c) Easy/Knowledge

79. According to surveys, the most frequently used influence tactic isa. personal appeals.b. rational persuasion.c. legitimating.d. pressure.

(b) Easy/Comprehension

80. Mike was playing video games. His mother came downstairs and said, “Mike, your room is a mess. I want you to go upstairs right now and clean it up or you will not be able to go out on Saturday.” Mike wanted to finish his game but saved it instead and went to clean his room. Mike has responded to Mom’s influence attempt witha. compliance.b. resistance.c. commitment.d. exchange.

(a) Medium/Application

81. When a target agrees to a request and actively supports it, the target’s response is a. resistance.b. compliance.c. commitment.d. conformity.

(c) Easy/Comprehension

82. The college organization president spoke to her members noting that the organization needed monetary donations to reach their goals. She cited numerous reasons why the money was needed and encouraged all members to call alumni members of the group for such donations. She suggested that the group members remind the alumni of the organization’s importance and their ties to that group. John dislikes asking anyone for money and decided not to make the calls. John is responding to the president’s influence tactics witha. resistance.b. conformity.c. compliance.d. commitment.

(a) Medium/Application

83. When the influence target does not wish to comply with the request and either passively or actively repels the influence attempt, they are exhibitinga. conformity.b. compliance.

Page 316: Fwk Bauer Tif Organizational Behavior All Chapters

c. commitment.d. resistance.

(d) Easy/Knowledge

84. The school district negotiator notes during contract negotiations with the teachers’ union that the district cannot afford a 3% wage increase. The district negotiator presents the current expenditure budget, the projected tax revenue and state contributions, and the currently increasing unemployment rate in the community’s tax base as evidence of the lack of extra money for the increases. The district negotiator is attempting to convince the teachers’ negotiator to pull back his demands usinga. inspirational appeals.b. pressure.c. rational persuasion.d. personal appeals.

(c) Medium/Application

85. The Teach for America representative faced the group of college students and asked, “Do you want to make a difference in this world? Do you want to give back for all the good things you have had happen to you over your lifetime? Do you want to help a child have a chance, a chance like yours? Then, won’t you consider giving just nine months of your life to make a difference? Consider joining me and thousands of your peers in leading this country in a new direction by taking part in Teach for America.” The representative is using the _______________ influence tactic.a. rational persuasionb. personal appealc. inspirational appeald. pressure

(c) Difficult/Application

86. Giving students a free mug, t-shirt, or blanket just for getting their names on a mailing list at an organization is an example of the ____________ influence tactic. Research suggests individuals try to repay what has been given them.a. ingratiationb. exchangec. pressured. personal appeal

(b) Medium/Analysis

87. Using different forms of making others feel good about themselves is the influence tactic of a. exchange.b. ingratiation.c. pressure.d. personal appeal.

(b) Easy/Comprehension

88. The use of client lists by advertisers and businesses to promote goods and service is an example of the _____________ influence tactic. And, it works!a. pressure

Page 317: Fwk Bauer Tif Organizational Behavior All Chapters

b. personal appealc. exchanged. coalition

(d) Medium/Comprehension

89. The influence tactic that works most effectively in organizations that value democratic decision making isa. pressure.b. coalition.c. consultation.d. ingratiation.

(c) Medium/Comprehension

90. A number of weight loss products advertised on television feature doctors who note the safety and effectiveness of the product. This approach to convincing viewers to purchase the product is an example of the _______________ influence tactic.a. likingb. personal appealc. pressured. legitimating

(d) Medium/Analysis

91. Getting a person to enter an employee assistance program for a substance abuse problem as a condition of keeping their job is an example of using the ____________ influence tactic.a. legitimatingb. pressurec. personal appeald. consultation

(b) Medium/Comprehension

92. Your roommate says to you, “I really need to make sure that I do not have any typos or misspelled words on my cover letter for my business communications class. Will you please read it for me?” This is an example of the a. exchange influence tactic.b. ingratiation influence tactic.c. personal appeal influence tactic.d. rational persuasion influence tactic.

(c) Medium/Application

93. _______________ power is more effective than formal power bases and is positively related to employee performance and organizational commitment.a. Legitimateb. Referentc. Rewardd. Information

(b) Medium/Comprehension

94. Which of the following statements regarding impression management is INCORRECT?

Page 318: Fwk Bauer Tif Organizational Behavior All Chapters

a. Whether you are actively managing your image or not, people are forming impressions of you.

b. You can cause yourself psychological distress by contradicting your personal values in trying to be someone you are not.

c. At work, the most effective approach to impression management is to build credibility and be the person you want to be.

d. You can actively shape the way people perceive you.(c) Medium/Synthesis

95. Which of the following is NOT an impression management category that an individual can use to accomplish the outcomes desired?a. personalityb. verbalc. nonverbald. behavior

(a) Easy/Comprehension

96. Your posture, hand gestures, and eye contact are all examples of what aspect of impression management?a. behaviorb. verbalc. nonverbald. personality

(c) Medium/Analysis

97. Which of the following is not an aspect of the verbal strategy of impression management?a. tone of voiceb. rate of your speechc. intent of your speechd. what you choose to say

(c) Difficult/Comprehension

98. Research suggests all of the following are outcomes of impression management EXCEPT impression managementa. is related to higher performance ratings for the subordinate due to

increased liking on the part of the manager.b. occurs throughout the workplace.c. influence is especially salient in interview and promotional contexts.d. is less influential in unstructured interviews than structured interviews.

(d) Medium/Comprehension

99. The ability to influence those in positions lower than yours isa. upward influence.b. downward influence.c. peer influence.d. diagonal influence.

(b) Easy/Knowledge

100. Which of the following statements regarding upward influence is INCORRECT?

Page 319: Fwk Bauer Tif Organizational Behavior All Chapters

a. While helping higher-ups be more effective can help employees gain more power for themselves, there is no gain for those same employees from allowing those below to influence them.

b. Asian American and Caucasian American managers use different tactics in influencing their superiors than they utilize in influencing their subordinates.

c. As organization complexity grows, the need for upward influence increases.

d. Upward influence examples include forming an alliance or a perceived alliance with a higher up and appealing to a higher authority.

(a) Difficult/Synthesis

101. Downward influence is best achieved through a. citing the firm’s goals.b. forming alliances.c. an inspiring vision.d. forming support networks.

(c) Medium/Analysis

102. Research on the direction of influence and the effectiveness of various influence techniques indicatesa. rational persuasion is the most frequently used influence tactic across all

functional groups.b. subordinate use of assertiveness and rationality as influence tactics lead

to negative outcomes.c. extraverts are more likely to use inspirational appeal as an influence

tactic.d. the better the quality of relationship between the subordinate and

supervisor, the more positively resistance to influence techniques are seen.

(b) Difficult/Synthesis

Section III: Organizational Politics

103. Political behaviors in organizations include all of the following EXCEPTa. negotiating.b. rational decision making.c. alliance building.d. resolving conflicting interests.

(b) Medium/Comprehension

104. All of the following statements regarding organizational politics are correct EXCEPTa. organizational politics are informal, unofficial, and even behind-the-scene

efforts to influence, increase power or achieve objectives.b. politics arise in an organization as an attempt to influence the allocation of

scarce resources.c. effective politics is all about winning at all costs.d. organizations are made up of different interests that must be aligned.

(c) Medium/Synthesis

Page 320: Fwk Bauer Tif Organizational Behavior All Chapters

105. Research on employee perception of organizational politics finds all of the following EXCEPT if employees think that their organization is overly driven by politics,a. employees have lower job satisfaction, but are no less committed to the

organization.b. employees perform worse on their jobs.c. employees have higher levels of job anxiety.d. that perception can be reduced by an infusion of a high level of feedback

by the leader.(a) Difficult/Synthesis

106. All of the following are individual antecedents of political behavior EXCEPTa. political skill.b. role ambiguity.c. expectations of success.d. internal locus of control.

(b) Easy/Comprehension

107. All of the following are organizational antecedents of political behavior EXCEPTa. expectations of success.b. role ambiguity.c. resource scarcity.d. democratic decision making.

(a) Easy/Comprehension

108. Which of the following statements regarding antecedents of political behavior is INCORRECT?a. Role ambiguity presents opportunities for employees to negotiate their

duties.b. Authoritarian decision making leads to more political behavior in the

organization.c. Individuals high in political skill are more effective at their jobs.d. Individuals high in internal locus of control engage in more political

behavior.(b) Medium/Synthesis

Section IV: Understanding Social Networks

109. Informal social networks serve all of the following important functions EXCEPTa. delivering private information.b. providing support.c. allowing individuals access to diverse skill sets.d. helping create power.

(b) Medium/Comprehension

110. Social network analysis reveals all of the following EXCEPT whoa. is very knowledgeable in the group.b. in the group can be trusted.c. in the group is innovative.d. in the group is important in decision making.

(a) Medium/Comprehension

Page 321: Fwk Bauer Tif Organizational Behavior All Chapters

111. Which of the following is NOT a key role in a social network?a. central connectorsb. boundary spannersc. external specialistsd. peripheral specialists

(c) Easy/Knowledge

112. People who connect one network to another within the company or even across organizations area. central connectors.b. boundary spanners.c. external specialists.d. peripheral specialists.

(b) Easy/Comprehension

113. Allan does not work for Core Company, but the human resource director calls him whenever he wants some advice on labor unions. Allan is classified as a _________ __________ in a social network analysis.a. central connectorb. boundary spannerc. external specialistd. peripheral specialist

(d) Medium/Analysis

114. ___________ __________ indicate emotional as well as informational support in a social network.a. Strong tiesb. Weak tiesc. Social tiesd. Casual ties

(a) Easy/Comprehension

115. Brokering power in a social network means you a. have a high level role in a social network.b. can introduce people to each other.c. have expert power in a social network.d. have resources that others in a social network desire.

(b) Easy/Knowledge

116. Which of the following statements regarding the analysis of social networks is correct?a. Weak ties are more difficult to maintain than strong ties.b. Strong ties indicate emotional support but no informational support.c. Strong ties are particularly useful for innovation.d. Ties that are reciprocated are strong ties.

(d) Medium/Synthesis

Section V: The Role of Ethics and National Culture

Page 322: Fwk Bauer Tif Organizational Behavior All Chapters

117. In countries with high power distancea. decisions are sometimes based on loyalty, not performance.b. it is unclear who is in power.c. people expect small differences in pay.d. people perceive inequity to be inherently wrong.

(a) Medium/Analysis

118. Which of the following statements is correct about power distance?a. Leaders in high power distance countries emphasize equality and

opportunity for everyone.b. In low power distance countries leaders are not to “stand out too much”

from the rest of their firms.c. Organizations in low power distance countries are more hierarchical in

nature than in high power distance countries.d. In high power distance countries formal review mechanisms are in place

to give everyone a fair chance at pay raises.(b) Medium/Analysis

119. Which of the following statements regarding the perceptions of power in countries is INCORRECT?a. A country’s culture determines how people will attempt to influence each

other.b. In Mexico, which has a higher power distance dimension than in the

United States, lower level managers were unable to make decisions on behalf of their bosses.

c. Rational persuasion is seen as more effective in the United States than in China.

d. Chinese managers rated influence tactics like coalitions lower than their American counterparts.

(d) Medium/Analysis

FILL IN THE BLANK

120. _______________ is the ability to influence the behavior of others to get what you want.

(Power)

121. People’s tendencies to behave consistently with social norms is __________.(conformity)

122. The __________studies looked at conformity and authority by having participants shock learners when those learners provided an INCORRECT response.

(Milgram)

123. Individuals could be influenced to say that two lines were the same length when one was clearly shorter than the other due to conformity in the _________ studies.

(Asch)

Page 323: Fwk Bauer Tif Organizational Behavior All Chapters

124. The ___________ study was stopped early because participants and the experimenter became too entrenched in their roles of aggressive jail guard and helpless prisoner.

(Zimbardo)

125. When discussing dependency, the concept of ___________ means the uniqueness of a resource.

(scarcity)

126. To determine how dependent you are on someone you need to assess three key factors— ___________, ____________, and ____________.

(scarcity, importance, substitutability)

127. Power derived from one’s role in the organization is ______________ power.(legitimate)

128. A manager possesses ___________ power because he can provide you an increase in pay or a promotion.

(reward)

129. Punishing someone for noncompliance, as a parent does to a child when the child does not clean her room, is an example of __________ power.

(coercive)

130. __________ power comes from knowledge and skills such as the long-time employee who knows everything about company history and procedures.

(Expert)

131. When negotiations are going on, the person computing the numbers to determine if a requested pay increase can be given possesses _________ power.

(information)

132. Oprah Winfrey has tremendous charisma and is adored by many fans. Oprah has ____________ power.

(referent)

133. When we try to get others to do what we want, we are using _________.(influence)

134. The influence tactic used most frequently is __________ __________ in the United States.

(rational persuasion)

135. When an influence target does not wish to comply with the request and either passively or actively repels the influence attempt, the influence attempt response of ____________ is occurring.

(resistance)

136. The influence attempt response______________ occurs when the target not only agrees to the request but also actively supports it as well.

(commitment)

Page 324: Fwk Bauer Tif Organizational Behavior All Chapters

137. The influence tactic that uses facts, data and logical arguments is ___________ __________.

(rational persuasion)

138. The influence tactic that seeks to tap into your values, emotions and beliefs is ____________ _________.

(inspirational appeals)

139. The influence tactic that causes you to help another person because you like him or her and he or she asked for your help is _________ ___________.

(personal appeal)

140. _____________ is the influence tactic that uses flattery or other forms of making others feel good about themselves.

(Ingratiation)

141. The influence tactic where there is give and take such that if someone does something for you, you do something for them is called ___________.

(exchange)

142. Exerting undue influence on someone to do what you want or else something undesirable will occur is the __________ influence tactic.

(pressure)

143. When someone tries to influence you by saying, “by the power vested in me…,” he or she is using __________.

(legitimation)

144. __________ ____________ is actively shaping the way others perceive you such as being concerned about your nonverbal and verbal behaviors.

(Impression management)

145. The ability to influence your boss and others in positions above you is ________ ________.

(upward influence)

146. People’s ability to relate well to others, self-monitor, inspire confidence and trust, and alter their reactions depending upon the situation they are in is ______________ ___________.

(political skill)

147. The visual map of relationships between individuals is a ________ __________.(social network)

148. One of the key roles in a social network is a _________ ________, or people who are linked to the greatest number of people.

(central connector)

149. A ______________ ___________ is a person who connects one network to another within the company or even across organizations.

Page 325: Fwk Bauer Tif Organizational Behavior All Chapters

(boundary spanner)

150. In a social network analysis, ________ ______ are characterized by less frequent interaction with less emotional attachment, which are easier to maintain and thus people have more of them.

(weak ties)

SHORT ANSWERS

151. What is conformity? Give an example of an instance where people conformed.

Conformity is people’s tendencies to behave consistently with social norms. An example is the Abu Ghraib prison scenario where guards tortured prisoners when they were ordered to do so.

152. What is the relationship between dependency and power?

Dependency is directly related to power. To ascertain how dependent you are on someone you need to assess:

Scarcity or the uniqueness of the resource. The more unique your resource and the more difficult to obtain, the higher the level of power you possess.

Importance or the value of the resource. The more important the resource, the more power you have.

Substitutability or the ability to find another option that works as well. The fewer options available, the more power possessed.

The interaction of the three factors determines the level of dependence and power.

153. Identify two bases of power.

There are six bases of power:

Legitimate power coming from the organizational role or position.Reward power coming from the ability to grant a reward like an increase in pay.Coercive power which is the ability to take something away or punish someone for noncompliance.Expert power comes from knowledge and skill.Information power is similar to expert power but information power centers on the access to specific information.Referent power comes from the personal characteristics of the person. Referent power is also called charisma.

154. Name two of the most commonly used influence tactics.

The influence tactics are:

Page 326: Fwk Bauer Tif Organizational Behavior All Chapters

Rational persuasion using data, facts and logical arguments.Inspirational appeals tapping into values, emotions and beliefs.Consultation using an influence agent to ask others for help in directly influencing or planning to influence another person or group.Ingratiation refers to different forms of making others feel good about themselves.Personal appeal is helping another person because you like him or her and he or she asked for your help.Exchange is give and take where if someone does something for you, you do something for him or her in return.Coalition tactics refer to a group of individuals working together toward a common goal to influence others.Pressure is exerting undue influence on someone to do what you want or else something undesirable will occur.Legitimating tactics occur when the appeal is based on legitimate or position power such as “By the power vested in me….”.

155. What is impression management?

Impression management means actively shaping the way you are perceived by others. This is particularly important for those who are seeking their first jobs. In interviews you must be aware of the cues about yourself you are sending through verbal and nonverbal behaviors.

156. What are the directions that influence attempts can take?

Upward influence is the ability to influence those in positions higher than yours.Downward influence is the ability to influence those in positions lower than yours.Peer influence must be carried out in such a manner that it does not become destructively competitive.

157. What are some individual antecedents of political behavior?

Individual antecedents include: political skill, internal locus of control, investment in the organization, and expectations of success.

158. What are some organizational antecedents of political behavior?

Organizational antecedents include: scarcity of resources, role ambiguity, performance evaluations, promotions, and democratic decision making.

159. What is a key role in a social network?

There are three key roles in a social network. Central connectors are those linked to the greatest number of people. Boundary spanners are people who connect one network to another. Peripheral specialists have special expertise that can be drawn upon even if they work independently of the group.

ESSAY

Page 327: Fwk Bauer Tif Organizational Behavior All Chapters

160. Choose a leader from entertainment, business or government. Describe the kinds of power that leader possesses.

Answers will vary. Barack Obama has legitimate power as the President of the United States. He has coercive power as the Commander-in-Chief of the Military. He can appoint individuals to Cabinet positions, which gives him reward power. He has expert power given his years in government, and referent power based upon the size of his victory in the November, 2008 election. His regular briefings on national security and domestic economic issues give him information power.

161. Discuss the three sets of classic studies on conformity and what the implications of their findings are.

The Milgram studies involved an experimenter, study participants and learners who were actually confederates chosen by the experimenter. The experimenter directed the participants to ask questions of the learners and when the learners were INCORRECT in their answers, to deliver a shock to those learners. The participants delivered shocks for INCORRECT answers long after the “fake” shocks would have been in a damaging range and long after they heard “screams” of pain from the learners. The study showed that people conform to the direction given to them by someone in authority.

In the Asch studies, individual participants were paired with confederates and asked about the length of lines. The confederates were to influence the individuals to say that two lines were the same length when one was clearly shorter than others. Research participants went along with wrong answers about 37% of the time. When a confederate acted as a dissenting minority, and gave the correct answer, the INCORRECT response decreased by 75%. In short, the power of a small dissenting minority was demonstrated.

In the Zimbardo study, volunteers were placed in a prison simulation. The prisoners were picked up by real police officers and placed in the basement of a Stanford building. Guards were chosen from among the volunteers and were told to keep order without any training. It did not take long for the guards to become aggressive and abusive and the prisoners to become depressed and helpless. Because the participants were becoming so entrenched in their roles, the experiment was terminated early.

All three studies demonstrate how authority can produce conformity. Also, conformity is a tendency people gravitate toward, so checks and balances must be put into place to ensure abuse does not occur.

162. Present a scenario when someone attempted to influence you to do something. Indicate what influence tactics or tactics they used to get you to do what they wanted.

Answers will vary with the student. Among the influence tactics that can be presented are: rational persuasion, inspirational appeals, consultation, ingratiation, personal appeal, exchange, coalition tactics, pressure and legitimating tactics.

Page 328: Fwk Bauer Tif Organizational Behavior All Chapters

One scenario that occurs on college campuses is where students come to speak to juniors and seniors to encourage them to join “Teach for America.” The speakers are usually recent graduates of the particular college who are part of the program and are attempting to recruit their peers to join the program. They use influence tactics centered on inspirational appeals particularly with regard to how the soon-to-be graduates can make a difference in the world before they begin their corporate lives. The inspirational appeals are accompanied by visuals showing how one can aid America’s troubled youth.

163. Discuss Dale Carnegie’s recommendations for get others to like you. Do you find them viable?

Carnegie’s recommendations are:

Be genuinely interested in other people.Smile.Remember that a person’s name is the most important sound in any language.Be a good listener.Talk in terms of the other person’s interests.Make the other person feel important.

The viability is an issue that is student specific. Generally, Carnegie’s work relies on referent power and the recognition that referent power grows if others like, respect and admire you.

164. How can you use impression management in an interviewing situation?

Some key aspects of impression management for the interviewing situation are the three main categories of:

Nonverbal: When someone looks at you what does your clothing have to say about you? Do you have body piercings? What will the interviewer, who likely will be of a different generation than you, think?

Verbal: Your tone, rate of speech and what you say and how you say it are important. About 35% of our comprehension of the verbal comes from these elements.

Behavior: Do you shake hands at the outset of the interview? Are you still when you sit in the interview or do you squirm? Do you make eye contact? Be aware of the impression you are making.

165. Choose a social network of which you are a part. How would you go about doing a social network analysis? What would you be looking for in the results of that analysis?

A social network analysis looks at the structure of social relationships in the group. You might look at who emails whom in a class group you have. Who calls whom? What do they talk about? How frequently do these actions occur? After the data is collected, you use software to determine the contacts and membership in the central connectors, boundary spanners and peripheral

Page 329: Fwk Bauer Tif Organizational Behavior All Chapters

specialist categories. Further, you assess strong ties and weak ties. Both ties are important to understand where one might begin to lay the groundwork for a change in the group, for example.

Page 330: Fwk Bauer Tif Organizational Behavior All Chapters

Organizational Behavior, Version 1.1Bauer & Erdogan

FWK Test Item FileChapter 14

TRUE/FALSE

1. Toyota employees feel a sense of ownership in their company, which is supported by the firm’s matrix structure.

(True)

2. Toyota employees are expected to be experts in what they do.(True)

3. Toyota uses a system of continuous improvement and learning.(True)

4. Just-in-time production leads to antagonistic relations between Toyota and its suppliers.

(False)

5. Decentralization is the concentration of decision making at higher levels in the organization.

(False)

6. Employees feel that decentralized organizations provide greater levels of procedural fairness.

(True)

7. If a firm is operating in a stable environment, the centralized organizational structure may lead to the most efficient operations.

(True)

8. A unionized firm has very formalized structures.(True)

9. A formalized structure leads to faster decision making.(False)

10. Tall structures have wider span of control than flat structures.(False)

11. Flat structures are better at satisfying employee self-actualization needs while tall structures are better at satisfying employee security needs.

(True)

12. In divisional structures, employees tend to be more specialists than generalists.(False)

Page 331: Fwk Bauer Tif Organizational Behavior All Chapters

13. Functional structures are most effective for companies with large product lines.(False)

14. Employees working in dynamic environments depend on their general mental abilities for good performance.

(True)

15. Organic structures are highly formalized and centralized.(False)

16. The biggest advantage of a mechanistic structure, especially in a stable environment, is its efficiency.

(True)

17. Organic structures are related to higher levels of job satisfaction.(True)

18. Matrix organizations violate the unity of command principle.(True)

19. Conflict at various levels is a potential issue for matrix organizations.(True)

20. A strategic alliance is a type of boundaryless organization where all nonessential functions are outsourced.

(False)

21. A learning organization is one where acquiring knowledge and changing behavior as a result of the newly acquired knowledge is part of the organization’s design.

(True)

22. Organizational change can be either radical or incremental.(True)

23. Moore’s law says the overall complexity of computer circuits will double every 18 months with no increase in costs, so firms will rapidly change their technology.

(True)

24. While manufacturing jobs have been outsourced, knowledge-based jobs are safe from outsourcing.

(False)

25. Change is more likely to occur in firms that are performing poorly because successful firms keep doing what they have done to make them successful.

(True)

26. Surveys indicate that resistance to change is one of the top two reasons why change efforts within organizations fail.

(True)

Page 332: Fwk Bauer Tif Organizational Behavior All Chapters

27. Passive resistance refers to sabotaging the change effort and being outspoken about the negative effects of change.

(False)

28. When people react to organizational change negatively, they are absent more often and are more likely to voluntarily leave the company.

(True)

29. Employees with a positive self-concept are better at coping with change.(True)

30. Those employees who feel they can perform well under a new system will be more committed to change.

(True)

31. People do not welcome change whether it is favorable or not to them.(False)

32. Resistance to change is never a positive force for an organization.(False)

33. The first stage in Lewin’s Model of planned change is unfreezing.(True)

34. Employees with more complete information are no more committed to the change effort than those who have less than complete information.

(False)

35. Convincing the opinion leaders in your organization of the need for organizational change is a more successful technique to ensure change than trying to get everyone on board at the same time.

(True)

36. During the change efforts, the organization should focus on big wins to demonstrate the success of the effort.

(False)

37. Cultures low in uncertainty avoidance are comfortable with change.(True)

MULTIPLE CHOICE

Opening Section: Organizational Structure: The Case of Toyota

38. The lean manufacturing system employs all of the following human resource strategies EXCEPTa. high bonuses.b. high investment in training.

Page 333: Fwk Bauer Tif Organizational Behavior All Chapters

c. internal promotions.d. employment stability.

(a) Medium/Comprehension

39. Which of the following statements regarding Toyota’s structure is INCORRECT?a. Toyota’s structure facilitates communication among functions.b. Toyota uses cross-functional module development teams.c. Toyota’s production system has led it to be structured as a matrix

organization.d. Toyota requires all its employees to be generalists.

(d) Difficult/Evaluation

40. Which of the following statements regarding Toyota’s just-in-time production and suppliers is INCORRECT?a. Around 75% of the final Toyota vehicle is produced by its suppliers.b. Long-term relationships are established with Toyota suppliers.c. Toyota work goes to the lowest bidding supplier.d. Toyota sends engineers to supplier firms to assess and improve their

production processes.(c) Difficult/Evaluation

Section I: Organizational Structure

41. Organization structure isa. the degree to which decision-making authority is concentrated at higher

levels in the organization.b. how the individual and team work within an organization is coordinated.c. the degree to which policies, procedures, job descriptions and rules are

written and explicitly articulated.d. the number of employees reporting to a single manager.

(b) Easy/Knowledge

42. The degree to which decision-making authority is concentrated at higher levels in an organization isa. formalizationb. centralizationc. hierarchical levelsd. departmentalization

(b) Easy/Knowledge

43. In an organization, the union contract is very explicit as to employee work rules, the tasks each individual employee will perform, the pay each individual will receive, how each individual can move from one position to another and so on. This organization’s structure is an example of a ____________ structure in the organization.a. centralizedb. departmentalizedc. hierarchicald. formalized

(d) Medium/Application

Page 334: Fwk Bauer Tif Organizational Behavior All Chapters

44. _________ is the number of levels an organization has.a. Hierarchyb. Departmentsc. Structured. Centralization

(a) Easy/Knowledge

45. The company is organized into accounting, marketing, human resource management, and production departments. This firm has aa. divisional structure.b. functional structure.c. product structure.d. matrix structure.

(b) Easy/Application

46. The company was organized into the detergent area, the dishwashing soap area, and the health and beauty aid area. This firm has a a. divisional structure.b. functional structure.c. mechanistic structure.d. matrix structure.

(a) Easy/Application

47. Centralized organizationsa. tend to have faster, more efficient decision making.b. attract more job candidates.c. put less pressure on higher level managers.d. can lead to large cost savings in purchasing operations.

(d) Medium/Analysis

48. Formalizationa. can lead to greater innovativeness.b. enhances employee motivation.c. provides for quicker decision making.d. makes employee behavior predictable.

(d) Medium/Analysis

49. In which of the following firms is formalization likely to occur?a. small, local pizza shopb. advertising agencyc. unionized manufacturing firmd. nonunion heavy construction firm

(c) Difficult/Synthesis

50. Tall structures providea. employees greater levels of role ambiguity.b. employees with greater self-actualization need satisfaction.c. employees with greater security need satisfaction.d. managers with a greater span of control.

(c) Difficult/Evaluation

Page 335: Fwk Bauer Tif Organizational Behavior All Chapters

51. Flat structuresa. have narrow spans of control.b. provide lower levels of self-actualization than tall structures.c. provide opportunities for greater role clarity for employees.d. provide limited advancement opportunities.

(d) Medium/Analysis

52. Functional structures a. are most effective when an organization has a large number of products.b. are most effective in dynamic environments.c. tend to facilitate the effective performance of employees with general

mental abilities.d. are cost-effective structures for the human resource and information

technology departments of most companies.(d) Medium/Analysis

53. Highly formalized and centralized structures similar to bureaucracies area. mechanistic structures.b. organic structures.c. matrix structures.d. product structures.

(a) Easy/Knowledge

54. Organic structuresa. are those that resemble a bureaucracy and are highly formalized and

centralized.b. cross a functional structure with a product structure.c. are flexible and decentralized structures with low levels of formalization

where communication lines are more fluid.d. are how individual and team work within an organization are coordinated.

(c) Easy/Knowledge

55. Mechanistic structuresa. are conducive to entrepreneurial activity and general innovativeness.b. maximize efficiency and minimize costs.c. enhance intrinsic motivation on the job.d. are flexible and react well to change.

(b) Medium/Analysis

56. Organic structuresa. are related to lower job satisfaction.b. have formal communication channels.c. maximize efficiency and minimize costs.d. are conducive to entrepreneurial activity and general innovativeness.

(d) Medium/Analysis

57. All of the following statements regarding the matrix structure are correct EXCEPTa. The matrix structure increases the frequency of informal communication

but decreases the frequency of formal communication.b. The matrix structure is a response to uncertainty and dynamism in the

environment.

Page 336: Fwk Bauer Tif Organizational Behavior All Chapters

c. Matrix structures violate the unity of command principle.d. Matrix structures require managers to be more interdependent than in

traditional structures and thus more of their time must be spent coordinating their work.

(a) Difficult/Synthesis

58. Unity of command is NOT found in what type of organizational structure?a. functional structureb. divisional structurec. boundaryless structured. matrix structure

(d) Easy/Comprehension

59. Which of the following is NOT a good piece of advice to give to individuals who have multiple bosses in a matrix structured organization?a. Make an effort to establish a good relationship with each manager.b. Keep conflicts you are experiencing to yourself.c. Make sure all managers are familiar with your overall workload.d. Do not assume that having multiple bosses is a bad thing.

(b) Medium/Evaluation

60. An organization that eliminates traditional barriers between departments as well as barriers between the organization and the external environment is aa. matrix organization.b. functional organization.c. boundaryless organization.d. divisional organization.

(c) Easy/Knowledge

61. Learning organizations use all of the following approaches to acquire knowledge that they can use to change behavior EXCEPTa. conducting formal retrospective meetings.b. studying customer habits.c. benchmarking against best industry practices.d. acquiring a firm to gain its research and development area.

(d) Medium/Evaluation

62. Two or more companies that find an area of collaboration and combine their efforts to create a partnership beneficial to both parties is aa. strategic alliance.b. modular organization.c. matrix organization.d. learning organization.

(a) Easy/Comprehension

63. An organization where all the nonessential functions are outsourced is aa. strategic alliance.b. modular organization.c. matrix organization.d. learning organization.

(b) Easy/Comprehension

Page 337: Fwk Bauer Tif Organizational Behavior All Chapters

Section II: Organizational Change

64. Which of the following statements regarding change is INCORRECT?a. Organizational change is the movement of an organization from one state

of affairs to another.b. Organizational change takes on limited forms.c. Organizational change, at its most basic, involves effective people

management.d. Organizational change can be forced or planned.

(b) Medium/Comprehension

65. All of the following are external forces that are creating change for organizations EXCEPTa. technology.b. workforce demographics.c. market conditions.d. growth of unions.

(d) Easy/Knowledge

66. All of the statements regarding workplace demographics and their impact on change are correct EXCEPT an older workforcea. that chooses retirement may cause a loss of valuable knowledge for

firms.b. will create a need for new types of benefits.c. will likely cause flexible work hours and job sharing to diminish as older

people like the structure of set hours of work.d. will create a need to train managers so that they are not using age-related

stereotypes in managing those employees.(c) Medium/Evaluation

67. The practice of outsourcing as a response to the globalization of business has created all of the following changes for organizations EXCEPT a need toa. retrain the workforce.b. deal with employee stress.c. understand how to compete in a global marketplace.d. restructure employee communication channels.

(d) Medium/Analysis

68. Which of the following statements regarding factors of change in the organization is INCORRECT?a. Rapid developments in technology force a firm to change its technology

to keep up.b. Manufacturing jobs have been outsourced, but knowledge-based jobs are

largely safe from such movement.c. Though 50% of all small businesses fail, those that succeed often evolve

into large, complex organizations over time.d. Changes in market conditions or the economy may cause companies to

change the manner in which they do business.(b) Difficult/Synthesis

Page 338: Fwk Bauer Tif Organizational Behavior All Chapters

69. Which of the following statements regarding performance and organizational change is INCORRECT?a. More successful firms find it easier to change than those performing

poorly.b. Nothing fails like success is a fitting saying to describe the relationship

between organizational change and performance.c. Change in upper level management is a motivator for organizational level

change.d. Successful companies often have special practices in place to keep the

organization open to change.(a) Medium/Synthesis

70. The most negative reaction to a proposed change attempt isa. enthusiastic support.b. active resistance.c. passive resistance.d. compliance.

(b) Easy/Knowledge

71. Andy is vehemently opposed to the restructuring of the jobs in his department in the manufacturing firm. He decides that since no one is listening to what he has to say he will get their attention by damaging each of the parts he is making for tomorrow’s shipment. This is an example of what kind of change reaction?a. enthusiastic supportb. passive resistancec. active resistanced. compliance

(c) Medium/Application

72. Marta has been a part-time faculty member at the university for nine years. She has been teaching two to three writing courses a semester. Last month, the dean held a meeting where he announced that next academic year, part-time instructors would be limited to teaching only two classes during the entire year. Marta is very upset and tells her husband, “I may look into teaching at the community college next year but don’t let anyone at the university know.” This is an example of what kind of change reaction?a. passive resistanceb. enthusiastic supportc. complianced. active resistance

(a) Medium/Application

73. The new CEO of XYZ Corporation has developed a plan that will call for the restructuring of the organization from a functional structure to a matrix one. In his meeting with his subordinates, manager Ervin says, “This will be great. You will still work for this area and me, but you will have variety in that you will also all be reporting to a second manager for various projects to which you will be assigned. This new structure will only help grow this company.” This is what kind of a reaction to change?a. complianceb. enthusiastic support

Page 339: Fwk Bauer Tif Organizational Behavior All Chapters

c. active resistanced. passive resistance

(b) Medium/Application

74. Georgio just returned from a meeting where his boss informed everyone of the new procedures to use in requesting reimbursement for travel. Georgio says, “I just figured out how to fill the other one out and now they are changing things. Oh, well, what are you going to do?” This is what kind of a reaction to change?a. enthusiastic supportb. active resistancec. complianced. passive resistance

(c) Medium/Application

75. Being disturbed by changes without voicing your opinion isa. active resistance.b. passive resistance.c. enthusiastic support.d. compliance.

(b) Easy/Comprehension

76. Compliance isa. going along with proposed changes with little enthusiasm.b. the most negative reaction to a proposed change.c. defending the change and encouraging others to give their support to the

change as well.d. being disturbed by changes without voicing those opinions.

(a) Easy/Knowledge

77. All of the following statements regarding resistance to change are correct EXCEPTa. all change attempts have to overcome the resistance of people.b. if resistance to change is not addressed by the firm, time and energy will

be lost.c. when people react negatively to change, they are absent more often.d. when people react negatively to change, they experience negative

emotions, but are no more likely to leave the firm than they would be in a firm that was not undergoing organizational change.

(d) Difficult/Synthesis

78. People resist change for all of the following reasons EXCEPTa. disrupted habits.b. personality.c. feelings of certainty.d. fear of failure.

(c) Easy/Comprehension

79. All of the following statements regarding reasons employees resist change are correct EXCEPT

Page 340: Fwk Bauer Tif Organizational Behavior All Chapters

a. those who feel they can perform well under a new system will be more committed than those who have lower confidence in their ability to perform.

b. we are creatures of habit.c. feelings of uncertainty create stress.d. people with a negative self-concept view change as an opportunity.

(d) Medium/Synthesis

80. Which of the following statements regarding reasons employees resist change is correct?a. people may not welcome the change currently proposed, but they do not

usually place those changes into any past context, so former unsuccessful changes have little impact on the situation.

b. people resist change even if it is personally beneficial to them.c. people are less likely to resist a change that is big in scope and has far

reaching consequences.d. people resist change that negatively impacts their influence and power in

the firm.(d) Difficult/Synthesis

81. Abdul has been downsized. Which of the following is NOT a piece of advice to offer him?a. Polish your resume and job hunting skills.b. Know our rights.c. Get help.d. Get angry! You just lost your job, you deserve to vent.

(d) Easy/Comprehension

82. Which of the following is NOT a stage of Lewin’s model of planned change?a. refreezeb. unfreezec. assessd. change

(c) Easy/Comprehension

83. Making sure that organizational members are ready for and receptive to change is what step of the Lewin model?a. refreezeb. unfreezec. assessd. change

(b) Easy/Comprehension

84. Ensuring that change becomes permanent and the new procedures become the norm is what stage of Lewin’s model of planned change?a. refreezeb. unfreezec. assessd. change

(a) Easy/Comprehension

Page 341: Fwk Bauer Tif Organizational Behavior All Chapters

85. What is the first step in Lewin’s model of planned change?a. refreezeb. unfreezec. assessd. change

(b) Easy/Comprehension

86. All of the following factors are important when unfreezing for planned change EXCEPTa. taking things slowly; do not suggest a crisis is imminentb. create a vision for changec. communicate the plan for changed. provide support

(a) Medium/Analysis

87. Which of the following statements related to the unfreezing stage of planned change is INCORRECT?a. Allow employee participation in planning for change efforts.b. Get everyone on board as soon as the change is initiated.c. Make sure management provides emotional and instrumental support.d. Make sure employees hear about planned changes via official channels,

not the grapevine.(b) Medium/Evaluation

88. Which of the following statements regarding the second stage of planned change is INCORRECT?a. Early success gives change planners credibility.b. Employees will be stressed; support them.c. Create big wins because people need to see that things are happening.d. Ensure that management keeps alert to and deals with obstacles that

arise.(c) Medium/Evaluation

89. Which of the following statements regarding the final stage of planned change is INCORRECT?a. Employees should be informed regularly of the results of the change.b. Declare victory early in making the change and get into a routine.c. Reward those who embraced the change effort.d. Change the mindset of people so change becomes a regular part of the

corporate culture.(b) Medium/Evaluation

90. Which of the following statements regarding overcoming resistance to your proposals is INCORRECT?a. Do not listen to naysayers.b. Involve those around you in the change.c. Understand the reasons for resistance.d. Present data to your audience.

(a) Medium/Analysis

Page 342: Fwk Bauer Tif Organizational Behavior All Chapters

Section III: The Role of Ethics and National Culture

91. All of the following alternatives to downsizing are more ethical actions EXCEPTa. early retirement programs.b. outsourcing.c. job sharing.d. hiring freezes.

(b) Medium/Analysis

92. The system where proposals at lower levels are signed and passed along to higher level management in an effort to build consensus is thea. boundaryless organization.b. modular organization.c. ringi system.d. strategic alliance.

(c) Easy/Knowledge

93. Which of the following statements regarding structure and change around the world is correct?a. France is relatively comfortable with change because it is low in

uncertainty avoidance.b. The United States is more likely to use rational persuasion as an

influence tactic than China.c. The United States is characterized by higher levels of centralization than

firms in Singapore and Hong Kong.d. Japanese organizations are largely decentralized as evidenced by their

ringi system.(b) Medium/Evaluation

FILL IN THE BLANK

94. Toyota’s production system is built on principles of _____ ___ __________ production

(just-in-time)

95. A culture of “never being satisfied” and rethinking how things are done facilitates __________ _____________

(continuous improvement or learning)

96. ____________ production requires harmonious relationships between Toyota and its suppliers.

(Just-in-time)

97. ___________ ___________ refers to how individual work and team work are coordinated within an organization.

(Organization structure)

98. The degree to which decision-making authority is concentrated at higher levels in an organization is _____________.

(centralization)

Page 343: Fwk Bauer Tif Organizational Behavior All Chapters

99. An organization where policies, procedures, job descriptions and rules are written and explicitly articulated is ____________.

(formalized)

100. ____________structures are associated with reduced motivation and job satisfaction and a slower pace of decision making.

(Formalized)

101. A firm with several layers of management between frontline employees and the top level has a __________ structure.

(tall)

102. The number of employees reporting to a single manager is the ________ ____ ________.

(span of control)

103. A firm that has departments for finance, manufacturing, human resources, accounting, and marketing is an organization in a ___________ structure.

(functional)

104. A pharmaceutical company with medical devices, consumer products, baby care, and health supplements as its departments is organized as a _____________ structure.

(divisional or product)

105. _________ structures are most effective for organizations that do not have large numbers of products.

(Functional)

106. ___________ structures are highly formalized and centralized.(Mechanistic)

107. A flexible, decentralized structure with low levels of formalization where communication lines are more fluid is a(n) __________ structure.

(organic)

108. __________ organizations, which combine functional structures with product structures, violate the unity of command principle.

(Matrix)

109. A ___________organization is a type of boundaryless organization where all nonessential functions are outsourced. Toyota is one such organization.

(modular)

110. ____________ organizations are organizations that eliminate traditional barriers between departments as well as barriers between the organization and external environment.

(Boundaryless)

Page 344: Fwk Bauer Tif Organizational Behavior All Chapters

111. A form of boundaryless organization where two or more companies find an area of collaboration and combine their efforts to create a partnership that is beneficial to both parties is a ________ _________.

(strategic alliance)

112. In ________ ________ experimenting, learning new things and reflecting on new knowledge are the norms.

(learning organizations)

113. The most negative reaction to a proposed change attempt is ___________ ___________.

(active resistance)

114. Going along with proposed changes with little enthusiasm is ___________.(compliance)

115. Michael defends the new change in company healthcare registration as helping the firm be more efficient. He says, “After all, we have healthcare. Let’s learn how to do this since it will help us.” Michael is reacting to the change using the _______ __________ form.

(enthusiastic support)

116. The first stage of Lewin’s process of change model is ___________, which ensures that employees are ready for change.

(unfreezing)

117. _______ is the final stage of Lewin’s process of change model which ensures that the change becomes permanent.

(Refreezing)

118. The ________ ____ is used in Japan to build consensus by having proposals signed at lower levels and passed along to higher level management.

(ringi system)

SHORT ANSWERS

119. What are the four building blocks of a company’s structure?

Centralization, departmentalization, formalization and hierarchical levels.

120. In what type of firms does centralization seem to work best?

Centralization is the concentration of decision making at higher levels in the organization. Centralization works well with organizations that need efficiency in their operations and works especially well in stable environments.

121. Formalization works well in what type of organization?

Page 345: Fwk Bauer Tif Organizational Behavior All Chapters

Formalization works best where explicitly articulated, clearly written rules, procedures, plans and programs are needed. Unionized settings are one such organization where such specificity is needed.

122. Provide an example of an organization that has a functional structure.

A company organized into human resource, finance, manufacturing, marketing and information technology departments would be a functional organization. Manufacturing firms are often organized in this manner.

123. Give an example of a company that is organized along divisional lines.

A firm organized along divisional lines may be organized based upon the products it produces. For example, a consumer snack food company like Frito-Lay might have its beverage division, its snack foods division, its restaurant division, etc.

124. Compare and contrast mechanistic and organic structures.

Mechanistic structures resemble a bureaucracy and are highly formalized and centralized. Organic structures are flexible and decentralized with low formalization.

125. What is a matrix organization?

A matrix organization is a cross between a functional structure and a product structure. Thus, employees report to department managers and project or team managers. Such an arrangement violates the unity of command principle but is effective in uncertain and dynamic environments.

126. What is a boundaryless organization?

A boundaryless organization is a term coined by GE’s Jack Welch to refer to an organization that has eliminated traditional barriers between departments as well as barriers between the organization and external environment. There are two forms of the boundaryless organization: the modular organization and the strategic alliance.

127. What is a learning organization?

A learning organization is one where acquiring knowledge and changing behavior as a result of newly acquired knowledge is part of an organization’s design.

128. Describe some forms that organizational change takes.

Organizational change may impact a company’s strategy, structure, policies, procedures, technology or culture. The change can be radical or incremental.

129. List some issues in the external environment that might create a need for change in an organization.

Page 346: Fwk Bauer Tif Organizational Behavior All Chapters

Workplace demographics like the aging of that workforce, technological changes, globalization of business and market conditions are all issues that might create a need for change.

130. What impact might workforce demographics have on an organization?

A workforce demographic that might impact a firm is the aging workforce. Older people might require new or different benefits not currently offered to the existing workforce. They also might need different work arrangements and schedules.

131. What impact might technology have on an organization?

As technology rapidly changes, organizations must keep up. Firms struggle with the technology itself, funding the technology changes and training employees to keep up with those changes. The text mentioned the technological changes occurring in the music industry.

132. What impact does globalization have on an organization?

Globalization can be a threat or an opportunity for a firm. Many firms have begun to outsource much of their production to be competitive through low-cost structures. Sometimes the outsourcing is a success, sometimes the whole adaptation must be reviewed. The toy companies in the United States that outsourced to China and now deal with concerns about lead paint on those toys are doing some rethinking of the outsourcing process.

133. What impact do market conditions have on an organization?

The upheaval in the economy currently has created a dramatic need for change. Consolidations in multiple industries, ongoing corporate scandals and general uncertainty have necessitated a need to examine the way each firm does business and to look at alternatives that might help it remain viable.

134. Name two ways in which people react to change.

People react in four ways: active resistance, passive resistance, compliance and enthusiastic support. Any change attempt must overcome resistance on the part of people and recognizing these reactions makes a manager better able to address that reaction and provide for a successful change implementation.

135. Name two reasons why people resist change.

People resist change for a variety of reasons including: it disrupts their habits; because of their personality, feelings of uncertainty, fear of failure, the personal impact of the change, the prevalence of change and the perceived loss of power.

136. Is resistance to change always bad?

No, resistance to change can be a positive force. When resistance provides feedback to assess the changes and the methods to achieve such change it can be serving a constructive purpose. When people provide constructive criticism,

Page 347: Fwk Bauer Tif Organizational Behavior All Chapters

those suggestions can be integrated into the change process for better implementation.

137. Briefly describe the Lewin process of change model.

Lewin’s model is comprised of three stages: unfreezing, change and refreezing. The first stage ensures that employees are ready for change. The second stage executes the intended change and the third stage ensures the change is permanent.

138. What are two key steps to take in ensuring success in unfreezing for change?

The steps include: creating a vision for change, communicating a plan for change, developing a sense of urgency for change, building a coalition, providing support and allowing employees to participate.

139. What are two key steps to take to ensure change is appropriately executed?

The steps include: continue to provide support, create small wins, and eliminate obstacles.

140. What are two key steps in refreezing for change?

The steps include: publicizing success, building on prior change, rewarding change adoption, and making change a part of the organizational culture.

141. What impact does organizational structure have on ethics?

If the organizational structure is too rigid or hierarchical, employees are not provided opportunities to develop moral intelligence. This moral intelligence is key in assessing what is ethical or not in the business setting. Similarly, when the structure prohibits an employee from making decisions with some degree of autonomy, ethical behaviors fail to develop.

142. Do organizational structures differ around the world?

The answer is very simply, yes. The student may choose to cite specific countries and their structures but generally they will be noting the nature of centralization or decentralization of organizations in each country. In addition, the student may note the culture’s level of uncertainty avoidance in noting structure.

ESSAY

143. What impact does structure have on Toyota Company’s success in the automobile industry?

Toyota is successful because its structure supports its strategy. Toyota functions under just-in-time production. Such a production focus requires employees who are expert at their positions, and who are continuously learning and improving. Toyota, which is functionally structured, has aspects of a matrix organization

Page 348: Fwk Bauer Tif Organizational Behavior All Chapters

whereby employees are placed on teams to engage in that continuous learning process. Further, Toyota has strategic alliances with a number of firms that, again, support the just in time production process. In fact, Toyota’s supplier firms are responsible for roughly 75% of the car’s production. Toyota shows the success associated with an organizational structure aligned with a firm strategy.

144. Choose a firm and discuss the nature of its structure using the four building blocks of structure as your focus.

Student responses will vary. But they should include a discussion of centralization, formalization, departmentalization and hierarchical levels.

The university is generally centralized with key decisions made in the chancellor/president’s office and implemented at the college level.

The university is highly formalized with explicit policies, rules, and procedures in place to guide all activities from teaching to tuition payment by students. Everyone is covered by a general employee handbook and each school has specific additional rules.

The university is departmentalized into schools based on functional area.

The hierarchical levels include the chancellor/president, vice presidents, deans, full professors, assistant professors, associate professors, etc.

145. Max has just been assigned to a project team after his organization restructured to a matrix organization. What kinds of advice could be offered him to help him deal with multiple bosses?

Having multiple bosses is a difficult adjustment for many individuals because we are accustomed to the unity of command structure. The potential for conflict with the multiple boss, multiple duty/obligation structure is thus obvious. Some tips to meet the challenges include:

Do not assume that having multiple bosses is necessarily a bad thing.Make sure all your managers are familiar with your overall workload.Make conflicts known to your bosses.Do not be afraid to request a meeting with all your managers.Make an effort to establish effective relations with each manager.Be cognizant of the relationships among managers as well.

With these tips in mind, Max should be able to make the adjustment.

146. Describe a time when a change was made in some organization of which you were a part. Were you resistant to the change? Why or why not?

Answers will vary with the student but the response should clearly delineate what the change was that was instituted. Also, the reaction to the change should be noted with the commentary on whether the student was resistant or not.

Page 349: Fwk Bauer Tif Organizational Behavior All Chapters

Sample: A manufacturing firm reclassified secretarial jobs. The company suggested the reclassification was necessary because the nature of jobs had changed. In this particular organization, computers had been provided to all members of the firm. Thus individuals throughout the organization were now able to type their own communication; they did not have to send work to the secretary for typing. Further, the organization had purchased a new telephone system so all organization members were able to put calls through to voice mail if they were busy. Again, secretaries had no need to answer phones. The overall result of the reclassification was that many secretarial jobs were placed at a lower level in the pay scale to reflect few duties performed. There was very vocal resistance to the plan, but that active resistance never rose to the level of sabotage. Clearly, the fact that employees’ livelihoods were impacted made the change a difficult one for the firm to implement. A number of secretaries ultimately left the firm, which likely was the intent of the company in reclassifying the jobs to begin with, as those positions were never filled when vacated.

147. Describe an organization that you believe needs to undergo a change. Describe the change that is needed and, using Lewin’s model of planned change, discuss how you would implement that change.

Answers will vary depending on the organization chosen. A family business owner of a mid-size manufacturing firm is aging and wants to leave the organization for retirement. He has two sons who are both part of the business and both desire to lead the organization. Before choosing a successor, he needs to work on stage one of change, unfreezing. He must prepare all employees of the firm and his external stakeholders for his departure. As the founder of the firm, he is the face of the firm so he must inform everyone of his desires but also of the smooth transition that will not interrupt their service. Once he informs the stakeholders of his plans, he must choose a leader. That leader must create a vision for the future. This vision must be communicated to the employees. A sense of urgency must be imparted as transitions cannot drag on but must take place. Change leaders need to be found who will serve as key supporters for the new leader. All management members must support each other and the employees as this change takes place. Clearly, stage two is the actual change. As the transition occurs, small wins must be publicized so the employees believe that the firm will continue to prosper. The final stage of the change, refreezing, will be a publicizing of the success of the transition and how the firm continues to serve its customers.

148. You have just been downsized from the firm where you have worked for nine years. What do you do to recover?

Though a painful experience, given the economic circumstances and the fact that firms now compete in a global setting, downsizing is a possibility at any time for any level employee. To recover:

Be calm. Easier said than done, but rational approaches to the incident as well as your next job search are needed at this critical time.Do not get angry. This is not your fault, it is circumstances. Do not burn bridges because of those circumstances.

Page 350: Fwk Bauer Tif Organizational Behavior All Chapters

Know your rights. Make sure you understand severance packages and continued health insurance coverage.Think about your ideal job situation. Are you in the right field? Are you in the right size firm? Now is the time to make any changes that help you realize your ideal job.Get help. If the firm is not providing some job search assistance, find organizations that offer the resources you need including assistance on resume writing, job interview skills, etc.Polish your resume and job-hunting skills.Do not give up. You will find a job.

149. Grace has been with her firm for a year and sees a number of procedures that she feels can be changed to enhance efficiency and effectiveness. She has offered those suggestions during group meetings and even through written communication to her boss. No one seems to pay any attention to the suggestions she has offered. How can she get support for her suggested changes?

Grace might consider the following:

Listen to the naysayers. Grace may believe she has a great idea, but maybe it is not as good as she thinks. Those who dissent may offer suggestions for change that make the original proposal, now a viable one.Assess whether your change is revolutionary. If Grace’s suggestions are dramatic changes, resistance is to be expected.Involve those around you in planning the change. Grace should not provide everyone her suggestions, but make them a part of building those suggestions into a useful plan of action.Do you have credibility? Grace has ideas for changes but how is she perceived in the organization? Grace must be honest with herself and assess whether others trust her and thus will be willing to follow through on her suggestions.Present data to the audience. Grace needs to not just say “this needs to be done”; she needs to provide rationale for that change and why it would be successful.Appeal to the audience’s ideals. What is the purpose behind the suggestion? Will the suggestion in some manner make the company employees happier? Will it improve the company’s prospects?Understand the reasons for resistance to change. Grace must understand why employees are ignoring her proposals. Do her suggestions threaten the personal influence of some members? Are employees afraid to fail if this suggestion is implemented?

As Grace reviews these items, she can reshape and reword her suggestions and resubmit them for consideration.

Page 351: Fwk Bauer Tif Organizational Behavior All Chapters

Organizational Behavior, Version 1.1Bauer & Erdogan

FWK Test Item FileChapter 15

TRUE/FALSE

1. Nordstrom’s has a lengthy employee handbook full of rules and regulations designed to emphasize quality in customer service.

(False)

2. Organizational culture is a system of shared assumptions, values and beliefs that show employees what are appropriate and inappropriate behaviors.

(True)

3. Individuals are more aware of their firm’s culture when they have an opportunity to compare it with that of another firm.

(True)

4. Research suggests that corporate strategy is the most important factor for business success, but organization culture is a close second.

(False)

5. Researchers found a relationship between organizational success, measured by indicators like revenues, sales and market share, and organizational culture.

(True)

6. The three levels of organizational culture are beliefs, assumptions and artifacts.(False)

7. The deepest or innermost level of organizational culture is values.(False)

8. Firms with an aggressive culture can face a number of lawsuits because of the focus on outperforming the competitor at all costs.

(True)

9. Outcome-oriented cultures hold managers accountable for performance but not employees.

(False)

10. If performance pressures increase too much, individuals begin to see their peers as competitors and short-term results become important, which can create unethical behaviors.

(True)

11. Creating a safety culture can reduce accidents, improve employee retention and increase profitability due to reduced workers’ compensation claims.

Page 352: Fwk Bauer Tif Organizational Behavior All Chapters

(True)

12. A strong culture always outperforms a weak culture because of the consistency of expectations.

(False)

13. Subcultures can arise from the personal characteristics of employees and managers or the conditions of the workplace.

(True)

14. Employee perceptions of subcultures have little impact on employee performance or commitment to the organization.

(False)

15. Countercultures are never tolerated in an organization once they are identified.(False)

16. Organizational structure is the most important factor in the creation of an organization’s culture.

(False)

17. A founder’s values could be one explanation for the difficulty encountered when trying to change a culture.

(True)

18. Founder values are a part of the company culture regardless of the success of the firm.

(False)

19. Companies within the same industry generally have similar organizational cultures, but on occasion, they can be wildly different.

(True)

20. The industry influence over culture suggests that it may not be possible to duplicate the culture of a firm in another industry.

(True)

21. Organizational cultures help determine who is hired by a firm.(True)

22. Organization choice of individuals for employment is impacted by culture, but that culture also impacts the self-selection out of that process by some individuals.

(True)

23. Individuals high in agreeableness would likely seek out firms with aggressive cultures.

(False)

24. Person-organization misfit is one of the important reasons for employee turnover.(True)

Page 353: Fwk Bauer Tif Organizational Behavior All Chapters

25. The attraction-selection-attrition process is an example of a method that maintains a certain level of homogeneity in personalities and values within an organization.

(True)

26. Onboarding activities in a firm produce higher levels of job satisfaction in new employees but have little impact on their organizational commitment long term.

(False)

27. The inability to network is a major reason why new employees either leave their jobs voluntarily or are terminated within the first two years of employment.

(True)

28. Computer-oriented orientation programs are more cost effective and more effective in conveying the firm’s culture than classroom-oriented programs.

(False)

29. While managers and supervisors are very important in helping new employees adjust to a firm’s culture, coworkers actually hamper the process.

(False)

30. Mentors and protégés who had input into the matching process were no more satisfied with the outcome of the program then were those who were placed into a mentor-protégé pair.

(False)

31. It is less important to find an individual to serve as a mentor who personifies company values than to find an individual who is enthusiastic about participating in the program.

(False)

32. Leader role modeling is an important influence in the creation or change in an organization’s culture.

(True)

33. A firm that rewards the achievement of goals only, and not the process to achieve those goals, is likely to have an outcome-based culture.

(True)

34. A firm that uses a ranking system where employees are pitted against each other for the top rewards is more likely to have an aggressive culture.

(True)

35. The behaviors that are punished, ignored or rewarded are likely to help determine how a culture evolves in a firm.

(True)

36. All mission statements are effective because they describe who the companies are and what they do.

(False)

Page 354: Fwk Bauer Tif Organizational Behavior All Chapters

37. Extending benefits to full- and part-time employees as well as spouses and domestic partners can convey to those employees the people orientation of the firm.

(True)

38. While a firm’s physical layout impacts motivation and job satisfaction, it does little to convey company culture.

(False)

39. Culture is generally resistant to change efforts.(True)

40. Two conditions that help effect a cultural change include experiencing failure in a firm and changes in the external environment impacting the firm.

(True)

41. The first step in the cultural change process is changing leaders and the leadership team.

(False)

42. Renovating the firm’s facilities and updating a firm’s logo can help enable a cultural change.

(True)

43. Making ethics assessment a regular part of performance evaluation can ensure ethical behavior becomes part of a firm’s core values.

(True)

44. The culture of a company is heavily impacted by the culture of its nation of origin.(True)

45. The Japanese culture emphasizes harmony, so it is highly unlikely a Japanese company would have an aggressive culture.

(True)

MULTIPLE CHOICE

Opening Section: Customer Service Culture: The Case of Nordstrom

46. Nordstrom is most recognized for and has been ranked in Fortune Magazine’s “100 Best Companies to Work For” because of itsa. upscale environment.b. quality apparel.c. service culture.d. employee rewards.

(c) Medium/Analysis

Section I: Understanding Organizational Culture

Page 355: Fwk Bauer Tif Organizational Behavior All Chapters

47. A system of shared assumptions, values, and beliefs that shows employees what is appropriate and inappropriate behavior isa. culture.b. mission.c. goals.d. strategy.

(a) Easy/Knowledge

48. Which of the following statements regarding organizational culture and its impact on the firm is INCORRECT?a. Employees become more aware of their organization’s culture when they

have an opportunity to compare it to that of another firm.b. Having a culture that fits with the company leads to good company

performance, but having a culture that does not fit has no impact.c. Organizational cultures that are hard to imitate can create a competitive

advantage for a firm.d. Organizational culture is an effective control mechanism for dictating

employee behavior.(b) Medium/Evaluation

49. Organizational culture consists of three levels:a. assumptions, orientations, and beliefs.b. beliefs, values, and artifacts.c. artifacts, assumptions, and beliefs.d. assumptions, artifacts, and values.

(d) Easy/Comprehension

50. Cultural assumptionsa. lie below the awareness level and reflect beliefs about human nature and

reality.b. are shared principles, standards and goals.c. are a set of values unique to a limited cross-section of the organization.d. are the visible and tangible elements of culture.

(a) Easy/Comprehension

51. The visible and tangible elements of culture area. assumptions.b. values.c. artifacts.d. beliefs.

(c) Easy/Knowledge

52. Shared principles, standards and goals area. artifacts.b. values.c. beliefs.d. assumptions.

(b) Easy/Comprehension

53. The corner office, mahogany desks and credenzas, gold name plates on office doors and reserved parking places are examples of

Page 356: Fwk Bauer Tif Organizational Behavior All Chapters

a. cultural beliefs.b. cultural assumptions.c. cultural artifacts.d. cultural values.

(c) Medium/Application

54. “People are generally dishonest” is an example of aa. cultural orientation.b. cultural artifact.c. cultural value.d. cultural assumption.

(d) Difficult/Application

55. Security, safety and social equality are all examples of a. cultural assumptions.b. cultural artifacts.c. cultural values.d. cultural orientations.

(c) Difficult/Application

Section II: Characteristics of Organizational Culture

56. 3M Corporation provides its employees time each week to generate ideas for new products. This practice is responsible for the development of such products as Post-it notes. 3M has a(n) _____________ culture.a. innovativeb. people-centeredc. detail orientedd. aggressive

(a) Medium/Application

57. The Internal Revenue Service has many rules and regulations with regard to tax code and the filing of returns. The organization is large and very bureaucratic. If an individual has issues with the agency, getting it resolved is a laborious process filled with “red tape” and takes an extensive amount of time and patience to resolve. The Internal Revenue Service has a(n) _________ culture.a. team-orientedb. stablec. people-orientedd. outcome-oriented

(b) Medium/Application

58. The school district has established achievement-oriented goals. Teachers are told that their instruction must thoroughly prepare students for high scores on the state achievement tests. Each nine-week grading period, teachers meet to discuss overall progress toward the goals set for increasing state achievement test scores. Recently the superintendent of the school district proposed establishing a bonus fund to reward teachers whose students’ scores increased the most from year to year on those tests. This school has what kind of culture?a. people-oriented

Page 357: Fwk Bauer Tif Organizational Behavior All Chapters

b. detail-orientedc. aggressived. outcome-oriented

(d) Medium/Application

59. Lincoln Manufacturing has a large sign posted at the entrance to the firm’s parking area that keeps count of the number of days since the last accident in the firm. Currently, the number stands at 150. The count is changed every day in somewhat of a “ceremony,” and every time another 50 days go by without an accident, the firm provides a free lunch to employees. Posters throughout the plant remind workers to wear protective clothing including earplugs, safety hats and glasses and safety shoes. Lincoln has what kind of culture?a. person-orientedb. aggressivec. safetyd. stable

(c) Medium/Application

60. The mother of the young man always has such difficulty getting him jeans to wear to elementary school. He needs the husky size and does not want any color EXCEPT the dark blue denim. The mother needs four pair of jeans. She usually purchases them at a major department store located in the mall near her home. She is at the mall today and is disappointed that only one pair in the size she needs is available. The salesclerk says that more should be available soon, but she is not sure exactly when. The day after the purchase, the phone rings in the mother’s home. It is the salesclerk from the major department store reporting that three pairs of the husky jeans in the child’s size have arrived and she is holding them for the mother. The mother thanks the clerk and asks how she got the mother’s phone number. “It took me some time, says the clerk, “but I found your receipt from yesterday and looked up your phone number through our computer system.” This store has a _________ culture.a. serviceb. detail-orientedc. stabled. people-oriented

(a) Medium/Application

61. At Walt Disney World, guests staying at The Grand Floridian Resort and Spa find thick white terrycloth robes hanging in the cupboard when they arrive in the room. Maids do not just place washcloths in the bath area; they form the washcloths into animals and arrange them on the vanity ready for a photograph. At night mints appear on the pillows and when the beds are made in the morning, any plush animals children have purchased or brought from home will be arranged in an entertaining formation on the finished bed. These examples suggest that Walt Disney Company is _______ __________.a. people-oriented.b. detail-oriented.c. team-oriented.d. outcome oriented.

(b) Medium/Application

Page 358: Fwk Bauer Tif Organizational Behavior All Chapters

62. The firm pays above average wages to its employees. Each employee is given at least two weeks of vacation and any time family matters arise and time is needed, flexible arrangements are made to facilitate the employee. Each Thanksgiving, a turkey is provided to each employee, $15 gift certificates are given for the Christmas holiday, and during the summer, the firm pays for each employee and one other individual to attend an expense-paid day at the local amusement park. This firm seems to be a(n) _________ company.a. outcome-orientedb. service-orientedc. people-orientedd. team-oriented

(c) Medium/Application

63. ABC Corporation has its sights clearly set on XYZ Company. The firm’s employees often talk about “cutting XYZ off at the knees.” Recently a lawsuit was filed against ABC by XYZ because ABC “raided” the middle-level management level of XYZ and hired away 5 of its 7 managers. ABC has a(n) ____________________ culture.a. serviceb. outcomec. stabled. aggressive

(d) Medium/Application

64. The firm places its employees into five-person groups and gives them projects that are to be completed by each group with minimum direction from a project leader. Compensation in the firm is divided into two parts—that earned for your primary job, and that earned for work in your group. Once each year, training seminars are conducted for all company employees to learn the latest techniques in collaboration. This firm has a(n) ____________ culture.a. people-orientedb. detail-orientedc. team-orientedd. outcome-oriented

(c) Medium/Application

65. Firms that are flexible, adaptable and experiment with new ideas are _________ firms.a. aggressiveb. innovativec. stabled. outcome-oriented

(b) Easy/Knowledge

66. People-oriented cultures a. value fairness, supportiveness and respecting individual rights.b. value competitiveness and outperforming competitors.c. are flexible, adaptable, and experiment with new ideas.d. emphasize precision and paying attention to details.

(a) Easy/Comprehension

Page 359: Fwk Bauer Tif Organizational Behavior All Chapters

67. Companies that are collaborative and emphasize cooperation among employees have a(n) __________ _________ culture.a. people-orientedb. outcome-orientedc. detail-orientedd. team-oriented

(d) Easy/Comprehension

68. A company with a stable culture isa. flexible, adaptable, and experiments with new ideas.b. predictable, rule-oriented, and bureaucratic.c. achievement-oriented, results-oriented, and action-oriented.d. fair, supportive, and respects individual rights.

(b) Easy/Comprehension

69. A culture that emphasizes precision and paying attention to details is a. a service culture.b. an aggressive culture.c. a detail-oriented culture.d. an outcome-oriented culture.

(c) Easy/Comprehension

70. Those who emphasize achievement, results and action create a(n) _________ __________ culture in their organizations.a. outcome-orientedb. team-orientedc. detail-orientedd. people-oriented

(a) Easy/Comprehension

71. Which of the following statements regarding culture dimensions is INCORRECT?a. Employees tend to stay longer in people-oriented cultures.b. Outcome-oriented cultures hold managers accountable for success, but

not other employees.c. In team-oriented cultures, managers have more positive relationships with

their subordinates than in other cultures.d. Proactive behavior tends to emerge often in service companies.

(b) Medium/Evaluation

72. To ensure all cast members share the history of Walt Disney World, new hires at the resort must take the courses on traditions at Disney. With these classes, the Walt Disney Company is creating aa. weak culture.b. counterculture.c. strong culture.d. subculture.

(c) Medium/Analysis

73. Strong culturesa. facilitate the changes that must occur in firms during mergers and

acquisitions.

Page 360: Fwk Bauer Tif Organizational Behavior All Chapters

b. outperform weak cultures regardless of the volatility of the environment.c. are no more difficult to change than weak cultures.d. are evidenced by consensus among employees on the values of the

company.(d) Medium/Evaluation

74. A defined set of values unique to a limited cross-section of the organization isa. a subculture.b. a counterculture.c. a strong culture.d. a service culture.

(a) Easy/Comprehension

75. The marketing department at Langston Industries meets once a week for a pizza luncheon after which the marketing manager holds his weekly “venting session” where department members can express their concerns about what is going right and wrong in the department and firm. The finance department, on the other hand, never meets. The finance manager simply sends emails to his subordinates each Monday with a list of the items to be accomplished each week and tries to get around to seeing each individual sometime during the week for any questions the employee might have. Langston appears to have _________ in its organization.a. counterculturesb. subculturesc. onboardingd. strong cultures

(b) Medium/Application

76. The culture of Altmira Corporation is characterized by participative decision making, innovation and openness. The accounting department, however, is run using authoritarian decision making to ensure efficiency and effectiveness. Accounting manager Gordon Paul believes in tight control to prevent errors. The accounting department’s culture could be classified as aa. strong culture.b. weak culture.c. subculture.d. counterculture.

(d) Medium/Application

77. Shared values and beliefs that are in direct opposition to the values of the broader organizational culture represent aa. strong culture.b. weak culture.c. subculture.d. counterculture.

(d) Easy/Comprehension

78. Which of the following statements regarding culture in organizations is INCORRECT?a. Multiple subcultures can exist in a single organization.

Page 361: Fwk Bauer Tif Organizational Behavior All Chapters

b. Subcultures arise in a firm due to personal characteristics of managers and employees and the different conditions under which work is performed.

c. Countercultures are never tolerated by an organization once they are identified.

d. Employee perceptions on subcultures are related to employee commitment to the organization.

(c) Medium/Evaluation

Section III: Creating and Maintaining Organizational Cultures

79. Which of the following statements regarding culture creation is correct?a. An organization’s culture is shaped only by internal environmental factors.b. New organization members are rarely taught the “way of business” in the

firm; they simply acquire that knowledge through daily activities.c. When the organization’s way of doing business provides a successful

adaptation to environmental challenges, those beliefs and values are retained.

d. Studies show the only factor consistently found to determine culture creation in a firm is the founder’s values.

(c) Difficult/Evaluation

80. Which of the following statements regarding founder’s values is INCORRECT?a. Founder values continue to be reflected in a firm only to the extent they

continue to aid the firm’s success.b. The difficulty in changing firm culture can be, at least, partially attributed

to the shaping provided by founder values.c. Founder values can provide a competitive advantage for a firm.d. Founder values lay the cornerstone of the firm’s culture, but values have

a rapidly decreasing impact as the firm grows and ages.(d) Difficult/Evaluation

81. If an industry is highly regulated, firms competing within it likely havea. bureaucratic structures.b. dynamic, innovative cultures.c. low concern for rules and authority.d. businesses processes characterized by agility and quick action.

(a) Difficult/Synthesis

82. _____________ is (are) part of the culture maintenance phase.a. Founder valuesb. Industry demandsc. Leadershipd. Preferences

(c) Easy/Knowledge

83. In completing a Big 5 personality inventory, Manuel was found to have high openness to experience. He would be attracted to an organization likea. Target where jobs tend to be structured in their duties and responsibilities.

Page 362: Fwk Bauer Tif Organizational Behavior All Chapters

b. the IRS, which has a bureaucratic structure and authoritarian decision making processes.

c. Google with a laid-back, innovative culture.d. General Motors with a tradition-bound culture experiencing serious

viability issues.(c) Medium/Application

84. Which of the following statements regarding the attraction-selection-attrition (ASA) process is INCORRECT?a. Individuals self-select the companies for which they choose to work.b. Due to economic circumstances, companies today hire people for fit with

the job only and are much less concerned about fit with the culture.c. The ASA process aids organizations in maintaining the relative

homogeneity of employee personalities and values.d. Person-organization misfit is an important reason for employee turnover.

(b) Difficult/Evaluation

85. The process through which new employees learn the attitudes, knowledge, skills, and behaviors required to function effectively within an organization isa. attraction-selection-attrition.b. organizational socialization.c. mentoring.d. assessment.

(b) Easy/Comprehension

86. If onboarding activities are successfula. employees experience greater job satisfaction but are no more likely to

remain with the firm than those who do not take part in onboarding.b. employees feel comfortable in the organization but have no greater belief

that their abilities are sufficient for work in the firm than those employees who did not take part in the onboarding activities.

c. employees feel accepted by their peers but do not necessarily share the values and norms of the larger organization.

d. employees are more confident about their ability to perform in the organization and are likely to remain longer with the firm.

(d) Difficult/Analysis

87. During onboarding activities, employees must do all of the following to be successful EXCEPTa. be reactive.b. seek feedback.c. network.d. be proactive.

(a) Easy/Comprehension

88. Research on the employee role in onboarding activities indicates all of the following to be true EXCEPTa. feedback-seeking enhances employee adjustment.b. reactive individuals are more successful than proactive individuals.c. active relationship building is particularly important to avoid turnover in

those firms lacking systematic onboarding activities.

Page 363: Fwk Bauer Tif Organizational Behavior All Chapters

d. failure to network often causes employees to turnover in a job.(b) Medium/Evaluation

89. Angelina has just accepted a new job. Her career counselor would not offer her which of the following pieces of advice if she wanted to ensure that Angelina gets “on board” in the new firm quickly?a. Build relationships with your manager and coworkers. Do not leave every

day exactly at quitting time; get to know people in your firm.b. Practice impression management, particularly that of your first

impression.c. Do not ask too many questions, as this would reveal your weaknesses.d. Try to build success early on in your tenure at the company. Volunteer for

projects and complete them effectively. Show your value to the firm.(c) Difficult/Evaluation

90. A trusted individual who provides the employee with advice and support regarding career-related matters is aa. coworker.b. manager.c. mentor.d. counselor.

(c) Easy/Comprehension

91. Which of the following statements regarding the organization’s role in onboarding and orientation activities is INCORRECT?a. The mere existence of mentoring programs does not guarantee their

success.b. The speed with which new employees learn the company’s culture is

strongly influenced by the support of supervisors and managers.c. Adjustment to a new company is facilitated by information provided by

peers and leaders.d. How companies do onboarding doesn’t really matter much.

(d) Easy/Comprehension

92. Research on mentoring suggestsa. the mere existence of the program ensures its success.b. mentor and protégé input to the program has little impact on satisfaction

with the program.c. mentor training enhances program outcomes.d. mentor characteristics are less important in their selection to the program

than willingness to participate.(c) Difficult/Evaluation

93. Leaders impact organization culture in all of the following ways or practices EXCEPTa. role modeling.b. leader style.c. leader intelligence.d. reaction to employee actions.

(c) Easy/Comprehension

Page 364: Fwk Bauer Tif Organizational Behavior All Chapters

94. Which if the following statements regarding the impact of reward systems on culture shaping is correct?a. A firm that uses a forced choice method of evaluation where a percentage

of employees are ranked as excellent, average and failures, and the failures are threatened with turnover, is likely to have a team-oriented culture.

b. Firms that reward based on how an employee meets a goal and not just whether the goal is met are likely to have highly aggressive cultures.

c. A firm that rewards purely on the basis of goal achievement is likely to have an outcome-oriented culture.

d. In outcome oriented cultures, supportive, cooperative behaviors are rewarded.

(c) Difficult/Evaluation

95. Which of the following statements regarding the creation and maintenance of culture is INCORRECT?a. One reason for the difficulty encountered in trying to change a firm’s

culture is the early shaping of culture based on the founder’s vision.b. If a leader motivates by rewarding for performance, an outcome-oriented

culture can develop.c. The attraction-selection-attrition process is a self-protective mechanism

by which a firm maintains a level of homogeneity regarding the values of organization members.

d. Computer-based orientation programs are more cost efficient and are more effective at conveying corporate culture than classroom-based programs.

(d) Difficult/Synthesis

96. Best practices for onboarding programs in firms suggest all of the following EXCEPTa. Make interaction key to the program. Get participation from all

organization members to the greatest extent possible.b. Keep the program as an informal plan. Formalization of the plan will

prevent its adaptation to new types of employees.c. Be clear on goals of the program and those who are actively participating.d. Establish milestones for the program and monitor them. Try to build a

rhythm for the program.(b) Medium/Analysis

97. A statement of purpose describing who a company is and what it does isa. a policy.b. a ritual.c. a mission statement.d. a rule.

(c) Easy/Knowledge

98. Which of the following statements about the effectiveness of a mission statement is INCORRECT?a. The mission statement must be provided to the employees when they first

enter the firm to be effective.

Page 365: Fwk Bauer Tif Organizational Behavior All Chapters

b. The mere act of publishing the mission statement makes it effective as it clarifies to all employees the purpose of the organization.

c. Effective mission statements are well known by employees.d. Effective mission statements influence employee behaviors.

(b) Medium/Analysis

99. A well known Fortune 500 CEO dresses up in a different outlandish costume each year for his presentation during the annual stockholder meeting. The presentation is simulcasted to all divisions and offices of the firm and employees enthusiastically view the proceedings. This scenario is an example of what visual element of culture?a. ritualsb. storiesc. policiesd. artifacts

(a) Medium/Comprehension

100. If the firm has a strict dress code policy in its employee handbook calling for male employees to wear “ties and white or light colored shirts, no pastels” and female employees to wear “dark skirts of knee length or below or dress slacks,” the culture of the firm is likelya. innovative.b. stable.c. aggressive.d. team-based.

(b) Medium/Application

101. Alcoa Corporation’s headquarters facility has a minimal number of traditional offices with four walls and a door. The elevators on each floor are located at the ends of the long corridors in out-of-the-way alcoves. An open-air escalator runs in the center of the building. Workers largely are seated in low-walled cubicles. Copy machines and other office equipment are centrally located on each floor. This physical layout encourages what kind of culture?a. aggressiveb. team-basedc. detail-orientedd. outcome-oriented

(b) Medium/Application

102. Which of the following statements regarding visual elements of culture is INCORRECT?a. Stories highlight critical events faced or heroic efforts put forth by

individual employees to instill cultural values in employees.b. Employee handbooks are developed to convey rules which determine

acceptable and unacceptable behavior in an organization.c. Repetitive activities in an organization, like weekly meetings opened with

calisthenics, have symbolic meaning to employees.d. While physical layouts impact motivational levels in employees, they do

little to convey the firm’s culture to those employees.(d) Medium/Synthesis

Page 366: Fwk Bauer Tif Organizational Behavior All Chapters

Section IV: Creating Culture Change

103. What is the first step in increasing the chances of success in effecting a cultural change?a. creating a sense of urgencyb. role modelingc. changing the reward systemd. changing key players

(a) Easy/Comprehension

104. What is the final step in a successful cultural change process?a. trainingb. role modelingc. changing the reward systemd. creating new stories and symbols

(d) Medium/Comprehension

105. Which of the following statements with regard to steps to effecting successful cultural change is INCORRECT?a. Renaming a firm and creating a new logo for it enables cultural change

and ensures the communication of the new value system.b. Replacing the staff employees who formerly supported a departed CEO

can ensure the establishment of a new value system in a firm.c. Slowly conveying information on a “need-to-know basis” only to

employees ensures the firm establishment of a new value system for the firm.

d. Thoughtfully developed, well administered training programs help establish new corporate norms of behavior.

(c) Difficult/Synthesis

Section V: The Role of Ethics and National Culture

106. Studies indicate that the most influential factor in creating an ethical culture isa. the reward system.b. the training program.c. leadership.d. the attraction-selection-attrition process.

(c) Medium/Evaluation

107. Which of the following statements regarding the impact of national culture on the culture of domestic firms is correct?a. The harmonious culture of Japan is likely to lead to the establishment of

more team-oriented corporate cultures in Japanese firms.b. The hierarchical orientation in Brazilian culture is likely to produce

Brazilian companies with flat corporate structures.c. The high power distance culture of Arab countries is likely to produce

people-oriented cultures in Arab companies.d. National culture has little impact on domestic firms.

(a) Difficult/Evaluation

Page 367: Fwk Bauer Tif Organizational Behavior All Chapters

FILL IN THE BLANK

108. The system of shared assumptions, values and beliefs that show employees what is appropriate and inappropriate behavior is ______________ ___________.

(organizational culture)

109. Beliefs about human nature and reality that are below the awareness level are ___________.

(assumptions.)

110. The corner office is an example of a cultural _____________.(artifact)

111. Cultures that are flexible, adaptable and experiment with new ideas are __________ cultures.

(innovative)

112. ________ ________ value competitiveness and outperforming competitors and may not feature corporate social responsibility.

(Aggressive cultures)

113. Cultures that value fairness, supportiveness and respecting individual’s rights are ____________ _________ ____________.

(people-oriented cultures)

114. ___________ cultures are predictable, rule-oriented and bureaucratic.(Stable)

115. Firms that emphasize achievement, results and action are _______ _______ cultures.

(outcome-oriented)

116. Emphasis on precision and paying attention to details characterize a ______ _________ culture.

(detail-oriented)

117. A ___________ culture is one that is shared by organization members and is very difficult to alter in any business situations like mergers or acquisitions.

(strong)

118. Cultures that emerge in different departments, branches or geographic locations are called __________.

(subcultures)

119. A ____________ is a form of a ______________ that shares values and beliefs in direct opposition to the values of the larger organization.

(counterculture, subculture)

120. The most important factors in the creation of an organization’s culture are _________, ____________ and _____________.

Page 368: Fwk Bauer Tif Organizational Behavior All Chapters

(founder values, preferences and industry demands)

121. ___________ refers to the process where new employees learn the attitudes, knowledge, skills and behaviors required to function effectively within the organization.

(Onboarding)

122. __________ or relationship building is an important activity new employees can undertake to ensure their adjustment to a firm.

(Networking)

123. A ______ __________ program indoctrinates new employees to the company culture, as well as introducing them to their new jobs and colleagues.

(formal orientation)

124. A trusted person who provides an employee with advice and support regarding career related matters is a __________.

(mentor)

125. A statement of purpose describing who the company is and what it does is a __________ _________.

(mission statement)

126. Repetitive activities within an organization that have symbolic meaning are called __________.

(rituals)

127. __________ __________ is the process by which employees modify their own beliefs and behaviors to reflect those of the leader.

(Role modeling)

SHORT ANSWERS

128. Define organizational culture.

Organizational culture is shared assumptions, values and beliefs that show employees what is appropriate and inappropriate behavior.

129. What impact does culture have on the organization?

Culture is as important as corporate strategy for business success, according to a study.

130. What are the three levels of culture?

Innermost level is assumptions that are taken for granted, belief in human nature, and reality.

Second layer is values or shared principles, standards and goals.

Page 369: Fwk Bauer Tif Organizational Behavior All Chapters

Outermost layer is artifacts that are the visible and tangible elements of culture.

131. Name two dimensions of culture according to the organizational culture profile and provide an example for each.

The dimensions are detail-oriented, innovative, aggressive, outcome-oriented, stable, people-oriented, and team-oriented.

A detail-oriented firm is the Ritz Carlton, which keeps records of customer preferences.

An innovative firm is 3M, which provides employees time each week to think creatively to develop new products.

An aggressive firm is Microsoft, which is focused on “killing” Netscape.

An outcome-oriented firm emphasizes attaining goals set and rewards the attainment but not the goal process. Many salespeople work under such a culture if they are in a commission-only program.

A stable organization is a school district, where the product and the way to “produce” it vary little over the years.

A people-oriented culture is found at Starbucks, which pays above market wages, offers tuition reimbursement and health care to its employees.

A team-oriented culture is found on many pro sports teams. The Pittsburgh Steelers, for example, feature few Pro-Bowlers yet their record is one of the best in the NFL.

132. What is a strong culture?

A strong culture is one that is shared by all organizational members. Walt Disney Company requires its resort employees to take Traditions I & II to understand the history of Walt Disney Company and the “Disney way”.

133. Describe a subculture and provide an example.

A subculture is a culture that emerges within different departments, branches or geographic locations. An example is that of a department that is divided by age. Older employees could have certain ways they approach their jobs, maybe individually, versus the way that young members of the department approach their jobs, maybe as somewhat of a team.

134. What is a counterculture and what is an example of it?

A counterculture is one whose shared values and beliefs are in direct opposition to the values of the broader organizational culture. An example is in a static, bureaucratic organization, you might find a department that is very innovative and takes risks.

Page 370: Fwk Bauer Tif Organizational Behavior All Chapters

135. How are cultures created?

Key to the creation of a culture is the values and preferences of a founder and the demand of the industry. A founder’s values help determine the way the firm does business. The industry demands also impact a culture. If the industry, for example, is highly regulated, founder values of risk taking and innovativeness have a much less likely chance of supporting success in that firm.

136. What are key factors in maintaining a firm’s culture?

Cultures are maintained by the attraction-selection-attrition process, new employee onboarding, leadership and reward systems.

137. What is the attraction-selection-attrition process?

The attraction process is what initially draws an individual to applying for a position in a firm. What signals is the applicant picking up that suggest the firm will be some place he would want to work? From the standpoint of the firm, what types of personalities and skills will best fit the firm’s needs?

The selection process is a series of techniques where the fit between applicant and firm is further investigated.

Attrition is the process where an individual who has been brought into a firm is “weeded out” when he does not perform as successfully has he might and the selection or attraction processes did not indicate such a scenario.

The attraction-selection-attrition process self defends the organizational culture.

138. What is employee onboarding?

Onboarding is the process through which new employees learn the attitudes, knowledge, skills and behaviors required to function effectively within an organization.

139. What is the new employee’s role in onboarding?

Onboarding is not a passive practice for the new employee. An employee should seek feedback on how she is performing and should network or build relationships. In addition, she needs to gather information, practice impression management and show success early on.

140. What is a mentor?

A mentor is a trusted person who provides an employee with advice and support regarding career related matters.

141. How do leaders influence culture?

Leaders influence culture through their style, role modeling and general reactions to the actions of those around them.

Page 371: Fwk Bauer Tif Organizational Behavior All Chapters

142. What are key factors in a reward system that reflects the culture of the firm?

The key factors are: the kinds of behaviors that are rewarded, punished or ignored in the firm, whether a firm rewards behaviors or results, and whether a firm uses ranking or rating in evaluation of performance.

143. Describe two visual elements of a firm’s culture and provide an example of each.

There are five visual elements: mission statement, rituals, stories, rule and policies and physical layout.

Mission statement may focus wording on employees.Rituals: Mary Kay pink Cadillac at annual meeting to top sellers.Stories: Post-it note development.Rules: Dress codePhysical Layout: Offices versus cubicles

144. What are the six steps to changing a culture?

Create a sense of urgency.Change leaders and other key players.Role model.Train.Change the reward system.Create new stories.

145. What is key to maintaining ethics in a firm’s culture?

Recent studies indicate the key element is leadership. Using firms like Enron, Adelphia Cable and others, qualitative analysis consistently points to the need for leaders to role model ethical behavior in order for it to be part of the company culture. Further, the reward systems that the leader puts into place must reflect the ethical standards for them to be effective.

146. What impact does the national culture of a country have upon its domestic company cultures?

The impact of a country’s culture on that of the company is small. The values of the country from which a founder comes are likely to have shaped his value system somewhat, but not to such a degree that country culture dictates company culture.

ESSAY

147. Choose an organization and using the dimensions of culture, describe which dimensions are present in the firm.

Answers will vary. A major research university has an aggressive and innovative culture in that professor pay and teaching load are dictated by the amount of

Page 372: Fwk Bauer Tif Organizational Behavior All Chapters

research dollars garnered by grant. That university also has a people-oriented culture in that flexible scheduling is permitted not only for faculty, but much of the staff. Outstanding benefits are provided to all university employees and pay is generally above the average for similar universities. Finally, a service culture is supported in that many university departments have extensive outreach programs to the local and state communities that allow the application of basic research concepts in the real world.

148. Choose a firm with which you are very familiar and discuss how culture was created and is maintained in that firm. Use the culture creation-maintenance model to frame your discussion.

Answers will vary. Students should include, as much as possible given their choices, some of these elements: founder values and preferences, and industry demands. For the maintenance aspect, students should discuss the attraction-selection-attrition process, new employee onboarding, leadership and the reward system.

A popular midsize, local restaurant was created 50 years ago by a now deceased individual who believed in: quality for your money and a friendly face. The founder felt that money was always tight for most people and a restaurant had to offer a better deal and experience than a customer could secure in his own home. Industry barriers to entry are relatively low so the firm had few problems getting started.

The basic meals the firm offers, such as meatloaf, roast, pork chops, and the outstanding desserts, were a great success when the firm first opened and they continue to draw people in today. The founder value of good food has not changed over the years.

The atmosphere of the restaurant has also not changed. Maintenance activities help retain that climate. The founder of the restaurant believed in good conversation over a meal in a friendly, supportive atmosphere. The hiring process reflects this culture. Interview questions focus heavily on assessing the candidate’s personality and ability to withstand stress. Further, a knowledge of the items offered on a daily basis is a key element of the selection decision.

The founder’s son now owns the restaurant and continues the same culture, role modeling the very values that originally made the place a success. Each day and evening, the current owner chats with customers at each table. Many of the customers are such regulars that personal questions about family are answered, further maintaining the key founding values.

149. Design an onboarding program for a firm of your choice. Make sure to include what employee, leader and coworker roles will be during the onboarding.

An onboarding program teaches new employees attitudes, knowledge, skills and behaviors required to function effectively within an organization. While student answers will vary, they should mention what the key values of the organization are that need to be conveyed, how they will be conveyed and the role of the leader, new employee and coworkers will be.

Page 373: Fwk Bauer Tif Organizational Behavior All Chapters

New employees need to seek feedback and network, gather information and manage their first impressions.Current coworkers need to convey information and support new employees in general.Leaders need to be cognizant of the style they exhibit and its influence on the program, role model the firm’s values and react consistently to the actions of those around him.

150. You are going to your first full-time position in the morning. What can you do to make sure you get “on board” immediately?

Answers will vary, but some key elements that should be mentioned include:

Gather information. Find out as much as you possibly can about the company and your job as quickly as you can. Be a good observer.Manage your first impression.Network.Seek feedback.Show success early on.

151. What are some of the visual elements of the culture of your firm?

Again answers will vary but students should mention items like: mission statement, rituals, stories, physical layout, and rules and policies.

Universities are very easy organizations in which to see visual elements of the culture. As an example, a large mid-Atlantic university is physically located in a city but has a campus defined by small park islands on each of the campus’ borders with the city. Each of the buildings is clearly marked with the university emblem.

The deans of each of the university’s colleges cite the mission statement as a means of developing programming that reaches out to various groups and individuals in the city itself.

One interesting ritual each spring is the closing of a portion of a city road located in front of the student union to facilitate gurney and wheelchair races between fraternities and sororities and as a means of uniting the education and medical subdivisions of the university.

One of the campus classroom buildings is a very tall facility and is the most recognizable aspect of the campus. Stories abound, particularly around the Halloween season, of ghosts that haunt the halls. These stories are shared with incoming freshmen, especially with those who have classes in that location.

Regardless of the element, the culture is conveyed successfully.

152. You are applying for a job at a variety of different companies. How will you determine whether you will be a good fit at those firms?

Page 374: Fwk Bauer Tif Organizational Behavior All Chapters

The most important thing you can do is research all the firms. Do not just assume because you know the firm’s products, for example, that you would like to work there. Talk to people, go to job placement centers, use the Internet.

Observe the physical environment when you go to interview. How is the office laid out? Do you like to work in cubicles? What is the dress code? Are employees pleasant?

Read between the lines. Does the firm have an employee handbook? Is it long, short, typed?

How are you treated? Did they go out of their way to make you feel at home? First impressions work both ways.

Ask questions. Take an active part in the interview beyond just answering your interviewer’s questions.Listen to your gut. How do you feel about your experience?